Import all
This commit is contained in:
BIN
TES/Continuite_convexite/Etude_Graphique/1E_tableau.pdf
Normal file
BIN
TES/Continuite_convexite/Etude_Graphique/1E_tableau.pdf
Normal file
Binary file not shown.
60
TES/Continuite_convexite/Etude_Graphique/1E_tableau.tex
Normal file
60
TES/Continuite_convexite/Etude_Graphique/1E_tableau.tex
Normal file
@@ -0,0 +1,60 @@
|
||||
\documentclass[a4paper,10pt]{article}
|
||||
\usepackage{myXsim}
|
||||
|
||||
\title{Graphiques et tableaux}
|
||||
\tribe{Terminale ES}
|
||||
\date{Septembre 2019}
|
||||
|
||||
\pagestyle{empty}
|
||||
|
||||
\begin{document}
|
||||
|
||||
\begin{exercise}[subtitle={Tableaux}]
|
||||
Tracer le tableau de variation et le tableau de signe des fonctions représentées ci-dessous.
|
||||
|
||||
\begin{minipage}{0.4\textwidth}
|
||||
\begin{tikzpicture}[scale=0.6, baseline=(a.north)]
|
||||
\repere{-5}{5}{-5}{5}
|
||||
\draw[very thick, color=red] plot [smooth,tension=0.5, mark=*] coordinates{(-4, -2) (-3.5, -3) (-3, 0) (-2, 1) (-1, 0) (0, -2) (1, -1) (2, -2) (2.5,0) (3, 2) (4, 3)};
|
||||
\draw (4,3) node[above right] {$\mathcal{C}_f$};
|
||||
\end{tikzpicture}
|
||||
\end{minipage}
|
||||
\begin{minipage}{0.5\textwidth}
|
||||
\begin{tikzpicture}[baseline=(a.north), yscale=0.7, xscale=2, yscale=0.8]
|
||||
\tkzInit[xmin=-0.2,xmax=0.2,xstep=.1,
|
||||
ymin=-12,ymax=6,ystep=2]
|
||||
\tkzGrid
|
||||
\tkzAxeXY
|
||||
\draw (-1,3) node[below left] {$\mathcal{C}_g$};
|
||||
\tkzFct[domain = -.1:.2,color=red,very thick]%
|
||||
{-5+x*(0.5+4*x*x*(-2+x))}
|
||||
\end{tikzpicture}
|
||||
\end{minipage}
|
||||
\end{exercise}
|
||||
|
||||
\begin{exercise}[subtitle={Graphique}]
|
||||
Pour chaque tableau ci-dessous, tracer le graphique d'une fonction qui pourrait convenir.
|
||||
|
||||
\begin{minipage}{0.5\textwidth}
|
||||
\begin{tikzpicture}[baseline=(a.north)]
|
||||
\tkzTabInit[lgt=1,espcl=1]{$x$/1,$f(x)$/1}{-10, -3, -2 , -1, 6, 10}
|
||||
\tkzTabLine{,+, z, -, z, +, z, -, z, +, }
|
||||
\end{tikzpicture}
|
||||
\end{minipage}
|
||||
\begin{minipage}{0.5\textwidth}
|
||||
\begin{tikzpicture}[baseline=(a.north)]
|
||||
\tkzTabInit[lgt=1,espcl=1]{$x$/1,$g(x)$/1}{$-\infty$, -2, 1, 4, $+\infty$}
|
||||
\tkzTabLine{,+, z, -, z, -, z, -, }
|
||||
\end{tikzpicture}
|
||||
\end{minipage}
|
||||
|
||||
\begin{tikzpicture}[baseline=(a.north)]
|
||||
\tkzTabInit[lgt=1,espcl=2]{$x$/1,$h(x)$/1}{-5, -3, 0, 3, $+\infty$}
|
||||
\tkzTabVar{+/ 2, -/ -2, +/ -1, -/ -3, +/ }
|
||||
\end{tikzpicture}
|
||||
\end{exercise}
|
||||
|
||||
\printexercise{exercise}{1}
|
||||
\printexercise{exercise}{2}
|
||||
|
||||
\end{document}
|
||||
BIN
TES/Continuite_convexite/Etude_Graphique/2E_continuite.pdf
Normal file
BIN
TES/Continuite_convexite/Etude_Graphique/2E_continuite.pdf
Normal file
Binary file not shown.
75
TES/Continuite_convexite/Etude_Graphique/2E_continuite.tex
Normal file
75
TES/Continuite_convexite/Etude_Graphique/2E_continuite.tex
Normal file
@@ -0,0 +1,75 @@
|
||||
\documentclass[a4paper,10pt]{article}
|
||||
\usepackage{myXsim}
|
||||
|
||||
\title{Autour de la notion de continuité}
|
||||
\tribe{Terminale ES}
|
||||
\date{Septembre 2019}
|
||||
|
||||
\pagestyle{empty}
|
||||
|
||||
\begin{document}
|
||||
|
||||
\begin{exercise}[subtitle={Toujours des solutions?}]
|
||||
Ci-dessous le graphiques de 3 fonctions définies sur $\intFF{-6}{5}$.
|
||||
|
||||
\hspace{-1cm}
|
||||
\begin{minipage}{0.3\textwidth}
|
||||
\begin{tikzpicture}[baseline=(a.north), xscale=0.45, yscale=0.6]
|
||||
\tkzInit[xmin=-6,xmax=6,xstep=1,
|
||||
ymin=-3,ymax=3,ystep=1]
|
||||
\tkzGrid
|
||||
\tkzAxeXY
|
||||
\draw (4,2) node[below left] {$\mathcal{C}_g$};
|
||||
\tkzFct[domain = -6:6,color=red,very thick]%
|
||||
{0.05*(x+5)*(x+1)*(x-4)}
|
||||
\end{tikzpicture}
|
||||
\end{minipage}
|
||||
\hspace{0.5cm}
|
||||
\begin{minipage}{0.3\textwidth}
|
||||
\begin{tikzpicture}[baseline=(a.north), xscale=0.45, yscale=0.6]
|
||||
\tkzInit[xmin=-6,xmax=6,xstep=1,
|
||||
ymin=-3,ymax=3,ystep=1]
|
||||
\tkzGrid
|
||||
\tkzAxeXY
|
||||
\draw (4,2) node[above left] {$\mathcal{C}_g$};
|
||||
\tkzFct[domain = -6:6,color=red,very thick]%
|
||||
{2-exp(-0.25*x)}
|
||||
\end{tikzpicture}
|
||||
\end{minipage}
|
||||
\hspace{0.5cm}
|
||||
\begin{minipage}{0.3\textwidth}
|
||||
\begin{tikzpicture}[baseline=(a.north), xscale=0.45, yscale=0.6]
|
||||
\tkzInit[xmin=-6,xmax=6,xstep=1,
|
||||
ymin=-3,ymax=3,ystep=1]
|
||||
\tkzGrid
|
||||
\tkzAxeXY
|
||||
\draw (4,2) node[below left] {$\mathcal{C}_g$};
|
||||
\draw[very thick, color=red] plot [smooth,tension=0.5] coordinates{%
|
||||
(-6,-3) (-3,-2) (-2,-1)
|
||||
};
|
||||
\draw[very thick, color=red] plot [smooth,tension=0.5] coordinates{%
|
||||
(-2,0) (2, 0.5) (6,3)
|
||||
};
|
||||
\end{tikzpicture}
|
||||
\end{minipage}
|
||||
\begin{enumerate}
|
||||
\item Résoudre les équations suivantes pour chacune des fonctions.
|
||||
\begin{multicols}{3}
|
||||
\begin{enumerate}
|
||||
\item $g(x) = -1$ sur $\intFF{-6}{5}$
|
||||
\item $g(x) = 1$ sur $\intFF{-6}{5}$
|
||||
\item $g(x) = 1$ sur $\intFF{2}{5}$
|
||||
\end{enumerate}
|
||||
\end{multicols}
|
||||
\item Quelles conditions faut-il avoir sur une fonction $g$ et sur $a$ pour que l'équation $g(x)=a$ ait une solution?
|
||||
\item Même question mais pour que cette solution soit unique?
|
||||
\end{enumerate}
|
||||
\end{exercise}
|
||||
|
||||
\vfill
|
||||
\printexercise{exercise}{1}
|
||||
\vfill
|
||||
\printexercise{exercise}{1}
|
||||
\vfill
|
||||
|
||||
\end{document}
|
||||
BIN
TES/Continuite_convexite/Etude_Graphique/2P_TVI.pdf
Normal file
BIN
TES/Continuite_convexite/Etude_Graphique/2P_TVI.pdf
Normal file
Binary file not shown.
42
TES/Continuite_convexite/Etude_Graphique/2P_TVI.tex
Normal file
42
TES/Continuite_convexite/Etude_Graphique/2P_TVI.tex
Normal file
@@ -0,0 +1,42 @@
|
||||
\documentclass[12pt,xcolor=table]{classPres}
|
||||
%\usepackage{myXsim}
|
||||
|
||||
\title{}
|
||||
\author{}
|
||||
\date{Octobre 2019}
|
||||
|
||||
\begin{document}
|
||||
\begin{frame}{Unique solution}
|
||||
\begin{block}{}
|
||||
À l'aide de votre calculatrice
|
||||
\begin{enumerate}
|
||||
\item Tracer le tableau de variation sur $\intFF{-6}{6}$ de la fonction
|
||||
\[
|
||||
f(x) = 0.05(x+5)(x+1)(x-4)
|
||||
\]
|
||||
\item Démontrer que l'équation $f(x)=1$ a des solutions sur $\intFF{-6}{6}$.
|
||||
\item Démontrer que l'équation $f(x) = -2$ a une unique solution $\alpha$ sur $\intFF{2}{5}$ puis donner une valeur approchée à $10^{-2}$ près à l'aide de la calculatrice.
|
||||
\end{enumerate}
|
||||
\end{block}
|
||||
\end{frame}
|
||||
|
||||
\begin{frame}{On recommence}
|
||||
\begin{block}{}
|
||||
À l'aide de votre calculatrice
|
||||
\begin{enumerate}
|
||||
\item Tracer le tableau de variation sur $\intFF{0}{20}$ de la fonction
|
||||
\[
|
||||
f(x) = 1000(x+5)e^{-0.2x}
|
||||
\]
|
||||
\item Démontrer que l'équation $f(x) = 3000$ a une unique solution $\alpha$ sur $\intFF{0}{20}$ puis donner une valeur approchée à $10^{-2}$ près à l'aide de la calculatrice.
|
||||
\end{enumerate}
|
||||
\end{block}
|
||||
\end{frame}
|
||||
|
||||
\end{document}
|
||||
|
||||
%%% Local Variables:
|
||||
%%% mode: latex
|
||||
%%% TeX-master: "master"
|
||||
%%% End:
|
||||
|
||||
BIN
TES/Continuite_convexite/Etude_Graphique/3E_convexite.pdf
Normal file
BIN
TES/Continuite_convexite/Etude_Graphique/3E_convexite.pdf
Normal file
Binary file not shown.
80
TES/Continuite_convexite/Etude_Graphique/3E_convexite.tex
Normal file
80
TES/Continuite_convexite/Etude_Graphique/3E_convexite.tex
Normal file
@@ -0,0 +1,80 @@
|
||||
\documentclass[a4paper,10pt]{article}
|
||||
\usepackage{myXsim}
|
||||
|
||||
\title{Autour de la notion de convexité}
|
||||
\tribe{Terminale ES}
|
||||
\date{Octobre 2019}
|
||||
|
||||
\pagestyle{empty}
|
||||
|
||||
\newcommand\lesgraphiques{%
|
||||
\begin{minipage}{0.5\textwidth}
|
||||
\includegraphics[scale=0.3]{./fig/C_f_QCM_liban2018}
|
||||
\begin{center}
|
||||
Représentation graphique de la fonction $f$
|
||||
\end{center}
|
||||
\end{minipage}
|
||||
\hfill
|
||||
\begin{minipage}{0.5\textwidth}
|
||||
\begin{tikzpicture}[xscale=0.8, yscale=]
|
||||
\tkzInit[xmin=-5,xmax=5,xstep=1,
|
||||
ymin=-4,ymax=3,ystep=1]
|
||||
\tkzGrid
|
||||
\tkzAxeXY
|
||||
\tkzFct[domain = -5:5,color=red,very thick]%
|
||||
{0.25*x**4-1.5*x**2-x+1}
|
||||
\draw (3, 2) node [color=black, below right] {$\matcal{C}_g$}
|
||||
\end{tikzpicture}
|
||||
\begin{center}
|
||||
Représentation graphique de la fonction $g$
|
||||
\end{center}
|
||||
\end{minipage}
|
||||
|
||||
\vfill
|
||||
|
||||
\begin{minipage}{0.3\textwidth}
|
||||
\begin{tikzpicture}[xscale=0.5, yscale=1]
|
||||
\tkzInit[xmin=0,xmax=10,xstep=1,
|
||||
ymin=0,ymax=3,ystep=1]
|
||||
\tkzGrid
|
||||
\tkzAxeXY
|
||||
\tkzFct[domain = 0:10,color=red,very thick]%
|
||||
{(2*x+1)*2.7**(-0.5*x)}
|
||||
\draw (9, 1) node [color=black, below] {$\matcal{C}_h$}
|
||||
\end{tikzpicture}
|
||||
\end{minipage}
|
||||
\hfill
|
||||
\begin{minipage}{0.3\textwidth}
|
||||
\begin{tikzpicture}[xscale=0.5, yscale=1]
|
||||
\tkzInit[xmin=0,xmax=10,xstep=1,
|
||||
ymin=-1,ymax=2,ystep=1]
|
||||
\tkzGrid
|
||||
\tkzAxeXY
|
||||
\tkzFct[domain = 0:10,color=red,very thick]%
|
||||
{(-x+1.5)*2.7**(-0.5*x)}
|
||||
\draw (9, 1) node [color=black, below] {$\matcal{C}_{h'}$}
|
||||
\end{tikzpicture}
|
||||
\end{minipage}
|
||||
\hfill
|
||||
\begin{minipage}{0.3\textwidth}
|
||||
\begin{tikzpicture}[xscale=0.5, yscale=1]
|
||||
\tkzInit[xmin=0,xmax=10,xstep=1,
|
||||
ymin=-2,ymax=1,ystep=1]
|
||||
\tkzGrid
|
||||
\tkzAxeXY
|
||||
\tkzFct[domain = 0:10,color=red,very thick]%
|
||||
{(0.5*x-1.75)*2.7**(-0.5*x)}
|
||||
\draw (9, 1) node [color=black, below left] {$\matcal{C}_{h''}$}
|
||||
\end{tikzpicture}
|
||||
\end{minipage}
|
||||
\begin{center}
|
||||
Représentations graphiques de la fonction $h$ et de ses dérivées
|
||||
\end{center}
|
||||
}
|
||||
\begin{document}
|
||||
|
||||
\lesgraphiques
|
||||
\vfill
|
||||
\lesgraphiques
|
||||
|
||||
\end{document}
|
||||
BIN
TES/Continuite_convexite/Etude_Graphique/3E_derivees.pdf
Normal file
BIN
TES/Continuite_convexite/Etude_Graphique/3E_derivees.pdf
Normal file
Binary file not shown.
89
TES/Continuite_convexite/Etude_Graphique/3E_derivees.tex
Normal file
89
TES/Continuite_convexite/Etude_Graphique/3E_derivees.tex
Normal file
@@ -0,0 +1,89 @@
|
||||
\documentclass[a4paper,10pt, twocolumn, landscape]{article}
|
||||
\usepackage{myXsim}
|
||||
|
||||
\title{Autour de la notion de continuité}
|
||||
\tribe{Terminale ES}
|
||||
\date{Septembre 2019}
|
||||
|
||||
\pagestyle{empty}
|
||||
|
||||
\begin{document}
|
||||
|
||||
\begin{exercise}[subtitle={Dérivée de qui?}]
|
||||
Retrouver quel graphique peut correspondre au graphique de la dérivée d'une des fonctions représentées.
|
||||
|
||||
\noindent
|
||||
\begin{minipage}{0.22\textwidth}
|
||||
\begin{tikzpicture}[baseline=(a.north), xscale=0.5, yscale=0.6]
|
||||
\tkzInit[xmin=-6,xmax=6,xstep=1,
|
||||
ymin=-3,ymax=3,ystep=1]
|
||||
\tkzGrid
|
||||
\tkzAxeXY
|
||||
\tkzFct[domain = -6:6,color=red,very thick]%
|
||||
{x*(x+2)*(x-1)*(x-4)/10}
|
||||
\end{tikzpicture}
|
||||
\end{minipage}
|
||||
\hspace{0.5cm}
|
||||
\begin{minipage}{0.22\textwidth}
|
||||
\begin{tikzpicture}[baseline=(a.north), xscale=0.5, yscale=0.6]
|
||||
\tkzInit[xmin=-6,xmax=6,xstep=1,
|
||||
ymin=-3,ymax=3,ystep=1]
|
||||
\tkzGrid
|
||||
\tkzAxeXY
|
||||
\tkzFct[domain = -6:6,color=red,very thick]%
|
||||
{0.1*(12*x**2-18*x-12)}
|
||||
\end{tikzpicture}
|
||||
\end{minipage}
|
||||
|
||||
\vfill
|
||||
\noindent
|
||||
\begin{minipage}{0.22\textwidth}
|
||||
\begin{tikzpicture}[baseline=(a.north), xscale=0.5, yscale=0.6]
|
||||
\tkzInit[xmin=-6,xmax=6,xstep=1,
|
||||
ymin=-3,ymax=3,ystep=1]
|
||||
\tkzGrid
|
||||
\tkzAxeXY
|
||||
\tkzFct[domain = -6:6,color=red,very thick]%
|
||||
{(x+5)*(x+2)*(x-1)*(x-2)/25}
|
||||
\end{tikzpicture}
|
||||
\end{minipage}
|
||||
\hspace{0.5cm}
|
||||
\begin{minipage}{0.22\textwidth}
|
||||
\begin{tikzpicture}[baseline=(a.north), xscale=0.5, yscale=0.6]
|
||||
\tkzInit[xmin=-6,xmax=6,xstep=1,
|
||||
ymin=-3,ymax=3,ystep=1]
|
||||
\tkzGrid
|
||||
\tkzAxeXY
|
||||
\tkzFct[domain = -6:6,color=red,very thick]%
|
||||
{(4*x**3+12*x**2-18*x-16)/25}
|
||||
\end{tikzpicture}
|
||||
\end{minipage}
|
||||
|
||||
\vfill
|
||||
\noindent
|
||||
\begin{minipage}{0.22\textwidth}
|
||||
\begin{tikzpicture}[baseline=(a.north), xscale=0.5, yscale=0.6]
|
||||
\tkzInit[xmin=-6,xmax=6,xstep=1,
|
||||
ymin=-3,ymax=3,ystep=1]
|
||||
\tkzGrid
|
||||
\tkzAxeXY
|
||||
\tkzFct[domain = -6:6,color=red,very thick]%
|
||||
{(12*x**2+24*x-18)/25}
|
||||
\end{tikzpicture}
|
||||
\end{minipage}
|
||||
\hspace{0.5cm}
|
||||
\begin{minipage}{0.22\textwidth}
|
||||
\begin{tikzpicture}[baseline=(a.north), xscale=0.5, yscale=0.6]
|
||||
\tkzInit[xmin=-6,xmax=6,xstep=1,
|
||||
ymin=-3,ymax=3,ystep=1]
|
||||
\tkzGrid
|
||||
\tkzAxeXY
|
||||
\tkzFct[domain = -6:6,color=red,very thick]%
|
||||
{0.1*(4*x**3 - 9*x**2 - 12*x + 8)}
|
||||
\end{tikzpicture}
|
||||
\end{minipage}
|
||||
\end{exercise}
|
||||
|
||||
\printexercise{exercise}{1}
|
||||
|
||||
\end{document}
|
||||
BIN
TES/Continuite_convexite/Etude_Graphique/3P_questions.pdf
Normal file
BIN
TES/Continuite_convexite/Etude_Graphique/3P_questions.pdf
Normal file
Binary file not shown.
55
TES/Continuite_convexite/Etude_Graphique/3P_questions.tex
Normal file
55
TES/Continuite_convexite/Etude_Graphique/3P_questions.tex
Normal file
@@ -0,0 +1,55 @@
|
||||
\documentclass[12pt,xcolor=table]{classPres}
|
||||
%\usepackage{myXsim}
|
||||
|
||||
\title{}
|
||||
\author{}
|
||||
\date{Octobre 2019}
|
||||
|
||||
\begin{document}
|
||||
|
||||
\begin{frame}{Fonction $f$}
|
||||
\begin{center}
|
||||
\includegraphics[scale=0.3]{./fig/C_f_QCM_liban2018}
|
||||
\end{center}
|
||||
\end{frame}
|
||||
|
||||
\begin{frame}[fragile]{Fonction $g$}
|
||||
\begin{center}
|
||||
\begin{tikzpicture}[xscale=0.8, yscale=]
|
||||
\tkzInit[xmin=-5,xmax=5,xstep=1,
|
||||
ymin=-4,ymax=3,ystep=1]
|
||||
\tkzGrid
|
||||
\tkzAxeXY
|
||||
\tkzFct[domain = -5:5,color=red,very thick]%
|
||||
{0.25*x**4-1.5*x**2-x+1}
|
||||
\draw (3, 2) node [color=black, below right] {$\matcal{C}_g$}
|
||||
\end{tikzpicture}
|
||||
\end{center}
|
||||
\end{frame}
|
||||
|
||||
\begin{frame}{Convexité}
|
||||
\begin{block}{Pour les fonctions $f$ et $g$}
|
||||
\begin{itemize}
|
||||
\item Déterminer sur quels intervalles chaque fonction est convexe.
|
||||
\item Même question pour concave.
|
||||
\item En déduire le tableau de signe des dérivées secondes de ces fonctions.
|
||||
\pause
|
||||
\item Trouver les points d'inflexions de ces fonctions.
|
||||
\end{itemize}
|
||||
\end{block}
|
||||
\end{frame}
|
||||
|
||||
\begin{frame}{La fonction $h$ et ses dérivées}
|
||||
\begin{itemize}
|
||||
\item Tracer le tableau de signe de $h''$ puis en déduire les variations de $h'$ puis la convexité de $h$.
|
||||
\item La fonction $h$ a-t-elle un point d'inflexion? Quelle est son abscisse?
|
||||
\end{itemize}
|
||||
\end{frame}
|
||||
|
||||
\end{document}
|
||||
|
||||
%%% Local Variables:
|
||||
%%% mode: latex
|
||||
%%% TeX-master: "master"
|
||||
%%% End:
|
||||
|
||||
Binary file not shown.
@@ -0,0 +1,72 @@
|
||||
\documentclass[12pt,xcolor=table]{classPres}
|
||||
%\usepackage{myXsim}
|
||||
|
||||
\title{}
|
||||
\author{}
|
||||
\date{Octobre 2019}
|
||||
|
||||
\begin{document}
|
||||
|
||||
\begin{frame}{Fonction $f$}
|
||||
\begin{center}
|
||||
\includegraphics[scale=0.3]{./fig/C_f_QCM_liban2018}
|
||||
\end{center}
|
||||
\end{frame}
|
||||
|
||||
\begin{frame}{Tangentes}
|
||||
\begin{block}{Pour la fonction $f$}
|
||||
\begin{enumerate}
|
||||
\item Lire graphiquement les valeurs de
|
||||
\[
|
||||
f'(2) \qquad f'(4)
|
||||
\]
|
||||
\item Parmi les équations suivantes, quelle est l'équation de la tangente à $\matcal{C}_f$ au point d'abscisse 2?
|
||||
\[
|
||||
y = 2x \qquad y = -x + 5 \qquad y = -x + 3 \qquad y = \frac{2}{3}x
|
||||
\]
|
||||
\item En déduire l'équation de la tangente au point d'abscisse 4.
|
||||
\end{enumerate}
|
||||
\end{block}
|
||||
\end{frame}
|
||||
|
||||
\begin{frame}[fragile]{Fonction $g$}
|
||||
\begin{center}
|
||||
\begin{tikzpicture}[xscale=0.8, yscale=]
|
||||
\tkzInit[xmin=-5,xmax=5,xstep=1,
|
||||
ymin=-4,ymax=3,ystep=1]
|
||||
\tkzGrid
|
||||
\tkzAxeXY
|
||||
\tkzFct[domain = -5:5,color=red,very thick]%
|
||||
{0.25*x**4-1.5*x**2-x+1}
|
||||
\draw (3, 2) node [color=black, below right] {$\matcal{C}_g$}
|
||||
\end{tikzpicture}
|
||||
\end{center}
|
||||
\end{frame}
|
||||
|
||||
|
||||
\begin{frame}{Tangentes}
|
||||
\begin{block}{Pour la fonction $g$}
|
||||
\begin{enumerate}
|
||||
\item Lire graphiquement les valeurs de
|
||||
\[
|
||||
g'(-2) \qquad g'(0)
|
||||
\]
|
||||
\item En déduire l'équation des tangentes en $x=0$
|
||||
\end{enumerate}
|
||||
\end{block}
|
||||
\begin{block}{Pour la fonction $h$ et ses dérivées}
|
||||
\begin{enumerate}
|
||||
\item Pour quel abscisse, la tangente à $\matcal{C}_h$ est horizontale?
|
||||
\item Donner l'équation de la tangente à la courbe $\matcal{C}_h$ au point d'abscisse 0.
|
||||
\item Donner l'équation de la tangente à la courbe $\matcal{C}_{h'}$ au point d'abscisse 0.
|
||||
\end{enumerate}
|
||||
\end{block}
|
||||
\end{frame}
|
||||
|
||||
\end{document}
|
||||
|
||||
%%% Local Variables:
|
||||
%%% mode: latex
|
||||
%%% TeX-master: "master"
|
||||
%%% End:
|
||||
|
||||
Binary file not shown.
|
After Width: | Height: | Size: 48 KiB |
65
TES/Continuite_convexite/Etude_Graphique/index.rst
Normal file
65
TES/Continuite_convexite/Etude_Graphique/index.rst
Normal file
@@ -0,0 +1,65 @@
|
||||
Étude graphique de la continuité et convexité des fonctions pour l'année 2019-2020 en terminale ES
|
||||
##################################################################################################
|
||||
|
||||
:date: 2019-10-08
|
||||
:modified: 2019-10-08
|
||||
:authors: Bertrand Benjamin
|
||||
:category: TESL
|
||||
:tags: Fonctions, Graphique, Continuité, Convexité
|
||||
:summary: Approche graphique de la continuité et de la convexité pour l'année 2019-2020 en terminale ESL
|
||||
|
||||
Étape 1: Lien entre représentation graphique et tableaux
|
||||
========================================================
|
||||
|
||||
.. image:: 1E_tableau.pdf
|
||||
:height: 200px
|
||||
:alt: Retour sur les tableaux pour décrire les fonctions
|
||||
|
||||
Tracer des graphiques qui correspondent à des tableaux de signe et de variations et inversement.
|
||||
|
||||
Étape 2: Résolution graphique d'équations et d'inéquations
|
||||
==========================================================
|
||||
|
||||
.. image:: 2E_continuite.pdf
|
||||
:height: 200px
|
||||
:alt: Résolution graphique d'équations
|
||||
|
||||
Les élèves résolvent graphiquement des équations. On cherche ensuite quelles doivent être les conditions pour qu'une équation ait une solution puis pour cette dernière soit unique.
|
||||
|
||||
Cours: Continuité, TVI et convention pour le tableau de variations
|
||||
|
||||
Étape 3: Convexité, concavité
|
||||
=============================
|
||||
|
||||
.. image:: 3E_derivees.pdf
|
||||
:height: 200px
|
||||
:alt: Correspondance entre graphiques et dérivées.
|
||||
|
||||
À partir de graphiques, on va demander aux élèves retrouver quel graphique correspond à la dérivée d'un autre. Pour justifier leurs choix, on poussera les élèves à tracer les tableaux de signe et de variations pour bien mettre en évidence la correspondance.
|
||||
|
||||
Une fois terminée, on demandera aux élèves des découper les graphiques pour regrouper ceux qui vont ensembles. Puis on demandera comment se traduit le signe de la dérivée seconde sur la forme de la représentation graphique de la fonction.
|
||||
|
||||
Cours: lien entre une fonction et sa dérivée, définition graphique concave, convexe. On illustrera le cours avec les graphiques de l'activité. On introduira le point d'inflexion.
|
||||
|
||||
Exercices pour reconnaître la concavité et convexité à partir d'un graphique mais aussi du graphique de la dérivée 2nd.
|
||||
|
||||
Quelques graphiques:
|
||||
|
||||
.. image:: 3E_convexite.pdf
|
||||
:height: 200px
|
||||
:alt: Quelques graphiques pour la suite
|
||||
|
||||
Les questions de convexité associées
|
||||
|
||||
.. image:: 3P_questions.pdf
|
||||
:height: 200px
|
||||
:alt: Questions sur la convexité sur les fonctions
|
||||
|
||||
Étape 4: Tangente et équation
|
||||
=============================
|
||||
|
||||
On se base sur les graphiques données en exercices à la fin de l'étape précédente.
|
||||
|
||||
.. image:: 4P_questions_tangente.pdf
|
||||
:height: 200px
|
||||
:alt: Questions sur les tangentes
|
||||
BIN
TES/Continuite_convexite/Etude_Graphique/trame.pdf
Normal file
BIN
TES/Continuite_convexite/Etude_Graphique/trame.pdf
Normal file
Binary file not shown.
41
TES/Continuite_convexite/Etude_Graphique/trame.tex
Normal file
41
TES/Continuite_convexite/Etude_Graphique/trame.tex
Normal file
@@ -0,0 +1,41 @@
|
||||
\documentclass[a4paper,10pt]{article}
|
||||
\usepackage{myXsim}
|
||||
|
||||
\title{Continuité, convexité \hfill Trame}
|
||||
\tribe{Terminale ES}
|
||||
\date{Septembre 2019}
|
||||
|
||||
\pagestyle{empty}
|
||||
|
||||
\begin{document}
|
||||
|
||||
\section{Résumer une fonction}
|
||||
|
||||
\subsection{Tableau de signes}
|
||||
|
||||
Correspondance signe, représentation graphique
|
||||
|
||||
\subsection{Tableau de variations}
|
||||
|
||||
Correspondance variations, représentation graphique
|
||||
|
||||
\section{Continuité}
|
||||
|
||||
\paragraph{Définition (graphique et non formelle)}\\
|
||||
Une fonction est continue quand le tracé de sa courbe représentative peut se faire sans lever le crayon.
|
||||
|
||||
\paragraph{Exemples}
|
||||
\begin{itemize}
|
||||
\item Une fonction continue
|
||||
\item Une fonction non continue
|
||||
\item Fonctions classiques
|
||||
\end{itemize}
|
||||
|
||||
\paragraph{Convention}
|
||||
Par convention, dans un tableau de variation, les flèches indiquent évidemment que la fonction est strictement monotone, mais aussi qu'elle est continue.
|
||||
|
||||
\paragraph{Théorème des valeurs intermédiaires}
|
||||
|
||||
\section{Concavité et convexité}
|
||||
|
||||
\end{document}
|
||||
157
TES/DM/DM_19_10/01_DM_19_10.tex
Normal file
157
TES/DM/DM_19_10/01_DM_19_10.tex
Normal file
@@ -0,0 +1,157 @@
|
||||
\documentclass[a4paper,10pt]{article}
|
||||
\usepackage{tasks}
|
||||
\usepackage{myXsim}
|
||||
|
||||
\title{DM 1 -- AIT BEN SAID Loubna}
|
||||
\tribe{Terminale ES-L}
|
||||
\date{15 novembre 2019}
|
||||
|
||||
\xsimsetup{
|
||||
solution/print = false
|
||||
}
|
||||
|
||||
\begin{document}
|
||||
\maketitle
|
||||
|
||||
Une part importante de la note sera dédiée à la rédaction, aux explications et à l'utilisation des notations mathématiques.
|
||||
|
||||
\begin{exercise}[subtitle={Débit}]
|
||||
Une commune de \np{1700} habitants au 1\up{er} janvier 2018 voit sa population augmenter de 8\,\% tous les ans.
|
||||
|
||||
Pour tout entier naturel $n$, on note $h_n$ le nombre d'habitants de l'année $2018 + n$.
|
||||
|
||||
\smallskip
|
||||
|
||||
\begin{enumerate}
|
||||
\item Déterminer la nature de la suite $(h_n)$, préciser ses éléments caractéristiques et exprimer $h_n$ en fonction de $n$.
|
||||
\end{enumerate}
|
||||
|
||||
\smallskip
|
||||
|
||||
La municipalité de cette commune a conclu un marché avec un fournisseur d'accès internet qui engage ce dernier à fournir un débit total de \np{18700}~Mbit/s au 1\up{er} janvier 2018 et à augmenter ce débit de - 5.8400\,\% par an.
|
||||
|
||||
Pour tout entier naturel $n$, on note $d_n$ le débit total dont la commune dispose l'année $2018 + n$.
|
||||
|
||||
\begin{enumerate}
|
||||
\setcounter{enumi}{1}
|
||||
\item Déterminer la nature de la suite $(d_n)$, préciser ses éléments caractéristiques et exprimer $d_n$ en fonction de $n$.
|
||||
\end{enumerate}
|
||||
|
||||
On s'intéresse maintenant au débit par habitant en supposant que celui-ci est réparti équitablement et que toute la population bénéficie d'une connexion internet individuelle.
|
||||
|
||||
Pour tout entier naturel $n$ on note $u_n$ le débit par habitant pour l'année $2018 + n$ et on admet que $u_n = \dfrac{d_n}{h_n}$.
|
||||
\begin{enumerate}
|
||||
\setcounter{enumi}{2}
|
||||
\item Calculer $u_0$ et $u_1$.
|
||||
\item Montrer pour tout entier naturel $n$ on a $u_n = 11 \times 0.98^n$.
|
||||
\item En déduire la nature de la suite $\left(u_n\right)$ et ses caractéristiques.
|
||||
\item Déterminer le sens de variations de la $\left(u_n\right)$.
|
||||
Interpréter ce résultat dans le contexte de l'énoncé.
|
||||
\end{enumerate}
|
||||
Le marché passé avec le fournisseur d'accès internet prévoit également que si le débit passe en dessous de 5 Mbit/s par habitant alors ce dernier doit changer la technologie utilisée pour la réalisation de son réseau.
|
||||
|
||||
\begin{enumerate}
|
||||
\setcounter{enumi}{6}
|
||||
\item En quelle année le fournisseur d'accès sera-t-il dans l'obligation de changer sa technologie?
|
||||
\end{enumerate}
|
||||
\end{exercise}
|
||||
\begin{solution}
|
||||
\begin{enumerate}
|
||||
\item Augmenter de 8\% revient à multiplier pas 1.08. La suite $(h_n)$ est donc géométrique de raison 1.08 et de premier terme 1700. On en déduit $h_n$ en fonction de $n$
|
||||
\[
|
||||
h_n = 1700\times 1.08^n
|
||||
\]
|
||||
\item Augmenter de - 5.8400\% revient à multiplier pas 1.0584. La suite $(d_n)$ est donc géométrique de raison 1.0584 et de premier terme 18700. On en déduit $d_n$ en fonction de $n$
|
||||
\[
|
||||
d_n = 18700\times 1.0584^n
|
||||
\]
|
||||
\item
|
||||
\begin{enumerate}
|
||||
\item
|
||||
\[
|
||||
u_0 = \frac{d_0}{h_0} = \frac{18700}{1700} = 11
|
||||
\]
|
||||
\[
|
||||
u_1 = \frac{d_1}{h_1} = \frac{19792.0800}{1836} = 10.78
|
||||
\]
|
||||
\item Démonstration de la formule
|
||||
\begin{eqnarray*}
|
||||
u_n &=& \frac{d_n}{h_n} = \frac{18700\times1.0584^n}{1700\times1.08^n} \\
|
||||
u_n &=& \frac{18700}{1700}\times\left(\frac{1.0584}{1.08}\right)^n \\
|
||||
u_n &=& 11\times0.98^n
|
||||
\end{eqnarray*}
|
||||
\item On reconnaît la forme d'une suite géométrique de raison 0.98 et de premier terme 11.
|
||||
\item La raison, $q = 0.98$, est inférieur à 1 donc la suite est décroissante. Ce qui signifie que le débit par habitant va diminuer.
|
||||
\end{enumerate}
|
||||
\item Avec le tableau de la calculatrice, on calculer les valeurs de $u_n$ jusqu'à passer en dessous de 5. On trouve $n = 40$ avec $u_{40} = 4.902704443460461508532877644$
|
||||
|
||||
\end{enumerate}
|
||||
\end{solution}
|
||||
\begin{exercise}[subtitle={Étude de fonctions}]
|
||||
\section*{Partie A}
|
||||
Dans cette partie, on étudie la fonction
|
||||
\[
|
||||
f(x) = - 2x^2 + 8x + 10
|
||||
\]
|
||||
\begin{enumerate}
|
||||
\item Calculer la dérivé de $f$.
|
||||
\item Étudier le signe de la dérivée $f'$ puis en déduire le tableau de signe de $f$.
|
||||
\end{enumerate}
|
||||
\section*{Partie B}
|
||||
Dans cette partie, on étudie la fonction
|
||||
\[
|
||||
g(x) = 2x^3 + 5x^2 + 2x + 7
|
||||
\]
|
||||
\begin{enumerate}
|
||||
\item À l'aide d'une calculatrice ou d'un ordinateur, tracer puis reporter sur votre copie la représentation graphique de $g$ en y indiquant les informations remarquables de ce graphique.
|
||||
\item Sur quel(s) intervalle(s) la fonction est convexe? concave? Y a-t-il des points d'inflexions?
|
||||
\item Calculer la dérivé de $g$.
|
||||
\item Étudier le signe de la dérivée $g'$ puis en déduire le tableau de variations de $g$.
|
||||
\item Déterminer l'équation de la tangente en $x=0$.
|
||||
\item Dériver $g'$ pour calculer $g''$.
|
||||
\item Étudier le signe de $g''$ pour en déduire la convexité de $g$ grâce au calcul puis localiser précisément le point d'inflexion.
|
||||
\end{enumerate}
|
||||
\end{exercise}
|
||||
\begin{solution}
|
||||
\section*{Partie A}
|
||||
\begin{enumerate}
|
||||
\item $8 - 4x$
|
||||
\item Correction non disponible
|
||||
\end{enumerate}
|
||||
|
||||
\section*{Partie B}
|
||||
\begin{enumerate}
|
||||
\item Correction non disponible
|
||||
\item Correction non disponible
|
||||
\item $g'(x) = 2 + 10x + 6x^2$
|
||||
\item
|
||||
On commence par calculer le discriminant de $g'(x)=2 + 10x + 6x^2$.
|
||||
\begin{eqnarray*}
|
||||
\Delta & = & b^2-4ac \\
|
||||
\Delta & = & 10^{2} - 4 \times 6 \times 2 \\
|
||||
\Delta & = & 100 - 24 \times 2 \\
|
||||
\Delta & = & 100 - 48 \\
|
||||
\Delta & = & 52
|
||||
\end{eqnarray*}
|
||||
|
||||
comme $\Delta = 52 > 0$ donc $P$ a deux racines
|
||||
|
||||
\begin{eqnarray*}
|
||||
x_1 & = & \frac{-b - \sqrt{\Delta}}{2a} = \frac{10 - \sqrt{52}}{2 \times 6} = - 0.2324081207560018 \\
|
||||
x_2 & = & \frac{-b + \sqrt{\Delta}}{2a} = \frac{10 + \sqrt{52}}{2 \times 6} = - 1.434258545910665
|
||||
\end{eqnarray*}
|
||||
Ainsi, $g'$ est du signe de $a=6$ en dehors des racines.
|
||||
|
||||
|
||||
Le tableau de variation non disponible en correction
|
||||
\item Équation de la tangente: $y = 2x + 7$
|
||||
\item $g''(x) = 12x + 10$
|
||||
\end{enumerate}
|
||||
\end{solution}
|
||||
|
||||
\end{document}
|
||||
|
||||
%%% Local Variables:
|
||||
%%% mode: latex
|
||||
%%% TeX-master: "master"
|
||||
%%% End:
|
||||
157
TES/DM/DM_19_10/02_DM_19_10.tex
Normal file
157
TES/DM/DM_19_10/02_DM_19_10.tex
Normal file
@@ -0,0 +1,157 @@
|
||||
\documentclass[a4paper,10pt]{article}
|
||||
\usepackage{tasks}
|
||||
\usepackage{myXsim}
|
||||
|
||||
\title{DM 1 -- BATEMAN Amélie}
|
||||
\tribe{Terminale ES-L}
|
||||
\date{15 novembre 2019}
|
||||
|
||||
\xsimsetup{
|
||||
solution/print = false
|
||||
}
|
||||
|
||||
\begin{document}
|
||||
\maketitle
|
||||
|
||||
Une part importante de la note sera dédiée à la rédaction, aux explications et à l'utilisation des notations mathématiques.
|
||||
|
||||
\begin{exercise}[subtitle={Débit}]
|
||||
Une commune de \np{1600} habitants au 1\up{er} janvier 2018 voit sa population augmenter de 6\,\% tous les ans.
|
||||
|
||||
Pour tout entier naturel $n$, on note $h_n$ le nombre d'habitants de l'année $2018 + n$.
|
||||
|
||||
\smallskip
|
||||
|
||||
\begin{enumerate}
|
||||
\item Déterminer la nature de la suite $(h_n)$, préciser ses éléments caractéristiques et exprimer $h_n$ en fonction de $n$.
|
||||
\end{enumerate}
|
||||
|
||||
\smallskip
|
||||
|
||||
La municipalité de cette commune a conclu un marché avec un fournisseur d'accès internet qui engage ce dernier à fournir un débit total de \np{19200}~Mbit/s au 1\up{er} janvier 2018 et à augmenter ce débit de 1.4200\,\% par an.
|
||||
|
||||
Pour tout entier naturel $n$, on note $d_n$ le débit total dont la commune dispose l'année $2018 + n$.
|
||||
|
||||
\begin{enumerate}
|
||||
\setcounter{enumi}{1}
|
||||
\item Déterminer la nature de la suite $(d_n)$, préciser ses éléments caractéristiques et exprimer $d_n$ en fonction de $n$.
|
||||
\end{enumerate}
|
||||
|
||||
On s'intéresse maintenant au débit par habitant en supposant que celui-ci est réparti équitablement et que toute la population bénéficie d'une connexion internet individuelle.
|
||||
|
||||
Pour tout entier naturel $n$ on note $u_n$ le débit par habitant pour l'année $2018 + n$ et on admet que $u_n = \dfrac{d_n}{h_n}$.
|
||||
\begin{enumerate}
|
||||
\setcounter{enumi}{2}
|
||||
\item Calculer $u_0$ et $u_1$.
|
||||
\item Montrer pour tout entier naturel $n$ on a $u_n = 12 \times 0.93^n$.
|
||||
\item En déduire la nature de la suite $\left(u_n\right)$ et ses caractéristiques.
|
||||
\item Déterminer le sens de variations de la $\left(u_n\right)$.
|
||||
Interpréter ce résultat dans le contexte de l'énoncé.
|
||||
\end{enumerate}
|
||||
Le marché passé avec le fournisseur d'accès internet prévoit également que si le débit passe en dessous de 5 Mbit/s par habitant alors ce dernier doit changer la technologie utilisée pour la réalisation de son réseau.
|
||||
|
||||
\begin{enumerate}
|
||||
\setcounter{enumi}{6}
|
||||
\item En quelle année le fournisseur d'accès sera-t-il dans l'obligation de changer sa technologie?
|
||||
\end{enumerate}
|
||||
\end{exercise}
|
||||
\begin{solution}
|
||||
\begin{enumerate}
|
||||
\item Augmenter de 6\% revient à multiplier pas 1.06. La suite $(h_n)$ est donc géométrique de raison 1.06 et de premier terme 1600. On en déduit $h_n$ en fonction de $n$
|
||||
\[
|
||||
h_n = 1600\times 1.06^n
|
||||
\]
|
||||
\item Augmenter de 1.4200\% revient à multiplier pas 0.9858. La suite $(d_n)$ est donc géométrique de raison 0.9858 et de premier terme 19200. On en déduit $d_n$ en fonction de $n$
|
||||
\[
|
||||
d_n = 19200\times 0.9858^n
|
||||
\]
|
||||
\item
|
||||
\begin{enumerate}
|
||||
\item
|
||||
\[
|
||||
u_0 = \frac{d_0}{h_0} = \frac{19200}{1600} = 12
|
||||
\]
|
||||
\[
|
||||
u_1 = \frac{d_1}{h_1} = \frac{18927.3600}{1696} = 11.16
|
||||
\]
|
||||
\item Démonstration de la formule
|
||||
\begin{eqnarray*}
|
||||
u_n &=& \frac{d_n}{h_n} = \frac{19200\times0.9858^n}{1600\times1.06^n} \\
|
||||
u_n &=& \frac{19200}{1600}\times\left(\frac{0.9858}{1.06}\right)^n \\
|
||||
u_n &=& 12\times0.93^n
|
||||
\end{eqnarray*}
|
||||
\item On reconnaît la forme d'une suite géométrique de raison 0.93 et de premier terme 12.
|
||||
\item La raison, $q = 0.93$, est inférieur à 1 donc la suite est décroissante. Ce qui signifie que le débit par habitant va diminuer.
|
||||
\end{enumerate}
|
||||
\item Avec le tableau de la calculatrice, on calculer les valeurs de $u_n$ jusqu'à passer en dessous de 5. On trouve $n = 13$ avec $u_{13} = 4.67153467986977671665730716$
|
||||
|
||||
\end{enumerate}
|
||||
\end{solution}
|
||||
\begin{exercise}[subtitle={Étude de fonctions}]
|
||||
\section*{Partie A}
|
||||
Dans cette partie, on étudie la fonction
|
||||
\[
|
||||
f(x) = 10x^2 - 8x + 7
|
||||
\]
|
||||
\begin{enumerate}
|
||||
\item Calculer la dérivé de $f$.
|
||||
\item Étudier le signe de la dérivée $f'$ puis en déduire le tableau de signe de $f$.
|
||||
\end{enumerate}
|
||||
\section*{Partie B}
|
||||
Dans cette partie, on étudie la fonction
|
||||
\[
|
||||
g(x) = - 8x^3 + 5x^2 + 9x - 3
|
||||
\]
|
||||
\begin{enumerate}
|
||||
\item À l'aide d'une calculatrice ou d'un ordinateur, tracer puis reporter sur votre copie la représentation graphique de $g$ en y indiquant les informations remarquables de ce graphique.
|
||||
\item Sur quel(s) intervalle(s) la fonction est convexe? concave? Y a-t-il des points d'inflexions?
|
||||
\item Calculer la dérivé de $g$.
|
||||
\item Étudier le signe de la dérivée $g'$ puis en déduire le tableau de variations de $g$.
|
||||
\item Déterminer l'équation de la tangente en $x=0$.
|
||||
\item Dériver $g'$ pour calculer $g''$.
|
||||
\item Étudier le signe de $g''$ pour en déduire la convexité de $g$ grâce au calcul puis localiser précisément le point d'inflexion.
|
||||
\end{enumerate}
|
||||
\end{exercise}
|
||||
\begin{solution}
|
||||
\section*{Partie A}
|
||||
\begin{enumerate}
|
||||
\item $- 8 + 20x$
|
||||
\item Correction non disponible
|
||||
\end{enumerate}
|
||||
|
||||
\section*{Partie B}
|
||||
\begin{enumerate}
|
||||
\item Correction non disponible
|
||||
\item Correction non disponible
|
||||
\item $g'(x) = 9 + 10x - 24x^2$
|
||||
\item
|
||||
On commence par calculer le discriminant de $g'(x)=9 + 10x - 24x^2$.
|
||||
\begin{eqnarray*}
|
||||
\Delta & = & b^2-4ac \\
|
||||
\Delta & = & 10^{2} - 4 \times - 24 \times 9 \\
|
||||
\Delta & = & 100 + 96 \times 9 \\
|
||||
\Delta & = & 100 + 864 \\
|
||||
\Delta & = & 964
|
||||
\end{eqnarray*}
|
||||
|
||||
comme $\Delta = 964 > 0$ donc $P$ a deux racines
|
||||
|
||||
\begin{eqnarray*}
|
||||
x_1 & = & \frac{-b - \sqrt{\Delta}}{2a} = \frac{10 - \sqrt{964}}{2 \times - 24} = - 0.43850727901083436 \\
|
||||
x_2 & = & \frac{-b + \sqrt{\Delta}}{2a} = \frac{10 + \sqrt{964}}{2 \times - 24} = 0.855173945677501
|
||||
\end{eqnarray*}
|
||||
Ainsi, $g'$ est du signe de $a=- 24$ en dehors des racines.
|
||||
|
||||
|
||||
Le tableau de variation non disponible en correction
|
||||
\item Équation de la tangente: $y = 9x + - 3$
|
||||
\item $g''(x) = - 48x + 10$
|
||||
\end{enumerate}
|
||||
\end{solution}
|
||||
|
||||
\end{document}
|
||||
|
||||
%%% Local Variables:
|
||||
%%% mode: latex
|
||||
%%% TeX-master: "master"
|
||||
%%% End:
|
||||
157
TES/DM/DM_19_10/03_DM_19_10.tex
Normal file
157
TES/DM/DM_19_10/03_DM_19_10.tex
Normal file
@@ -0,0 +1,157 @@
|
||||
\documentclass[a4paper,10pt]{article}
|
||||
\usepackage{tasks}
|
||||
\usepackage{myXsim}
|
||||
|
||||
\title{DM 1 -- BOUNOUS Matthieu}
|
||||
\tribe{Terminale ES-L}
|
||||
\date{15 novembre 2019}
|
||||
|
||||
\xsimsetup{
|
||||
solution/print = false
|
||||
}
|
||||
|
||||
\begin{document}
|
||||
\maketitle
|
||||
|
||||
Une part importante de la note sera dédiée à la rédaction, aux explications et à l'utilisation des notations mathématiques.
|
||||
|
||||
\begin{exercise}[subtitle={Débit}]
|
||||
Une commune de \np{2000} habitants au 1\up{er} janvier 2018 voit sa population augmenter de 5\,\% tous les ans.
|
||||
|
||||
Pour tout entier naturel $n$, on note $h_n$ le nombre d'habitants de l'année $2018 + n$.
|
||||
|
||||
\smallskip
|
||||
|
||||
\begin{enumerate}
|
||||
\item Déterminer la nature de la suite $(h_n)$, préciser ses éléments caractéristiques et exprimer $h_n$ en fonction de $n$.
|
||||
\end{enumerate}
|
||||
|
||||
\smallskip
|
||||
|
||||
La municipalité de cette commune a conclu un marché avec un fournisseur d'accès internet qui engage ce dernier à fournir un débit total de \np{16000}~Mbit/s au 1\up{er} janvier 2018 et à augmenter ce débit de 1.3000\,\% par an.
|
||||
|
||||
Pour tout entier naturel $n$, on note $d_n$ le débit total dont la commune dispose l'année $2018 + n$.
|
||||
|
||||
\begin{enumerate}
|
||||
\setcounter{enumi}{1}
|
||||
\item Déterminer la nature de la suite $(d_n)$, préciser ses éléments caractéristiques et exprimer $d_n$ en fonction de $n$.
|
||||
\end{enumerate}
|
||||
|
||||
On s'intéresse maintenant au débit par habitant en supposant que celui-ci est réparti équitablement et que toute la population bénéficie d'une connexion internet individuelle.
|
||||
|
||||
Pour tout entier naturel $n$ on note $u_n$ le débit par habitant pour l'année $2018 + n$ et on admet que $u_n = \dfrac{d_n}{h_n}$.
|
||||
\begin{enumerate}
|
||||
\setcounter{enumi}{2}
|
||||
\item Calculer $u_0$ et $u_1$.
|
||||
\item Montrer pour tout entier naturel $n$ on a $u_n = 8 \times 0.94^n$.
|
||||
\item En déduire la nature de la suite $\left(u_n\right)$ et ses caractéristiques.
|
||||
\item Déterminer le sens de variations de la $\left(u_n\right)$.
|
||||
Interpréter ce résultat dans le contexte de l'énoncé.
|
||||
\end{enumerate}
|
||||
Le marché passé avec le fournisseur d'accès internet prévoit également que si le débit passe en dessous de 5 Mbit/s par habitant alors ce dernier doit changer la technologie utilisée pour la réalisation de son réseau.
|
||||
|
||||
\begin{enumerate}
|
||||
\setcounter{enumi}{6}
|
||||
\item En quelle année le fournisseur d'accès sera-t-il dans l'obligation de changer sa technologie?
|
||||
\end{enumerate}
|
||||
\end{exercise}
|
||||
\begin{solution}
|
||||
\begin{enumerate}
|
||||
\item Augmenter de 5\% revient à multiplier pas 1.05. La suite $(h_n)$ est donc géométrique de raison 1.05 et de premier terme 2000. On en déduit $h_n$ en fonction de $n$
|
||||
\[
|
||||
h_n = 2000\times 1.05^n
|
||||
\]
|
||||
\item Augmenter de 1.3000\% revient à multiplier pas 0.9870. La suite $(d_n)$ est donc géométrique de raison 0.9870 et de premier terme 16000. On en déduit $d_n$ en fonction de $n$
|
||||
\[
|
||||
d_n = 16000\times 0.9870^n
|
||||
\]
|
||||
\item
|
||||
\begin{enumerate}
|
||||
\item
|
||||
\[
|
||||
u_0 = \frac{d_0}{h_0} = \frac{16000}{2000} = 8
|
||||
\]
|
||||
\[
|
||||
u_1 = \frac{d_1}{h_1} = \frac{15792}{2100} = 7.52
|
||||
\]
|
||||
\item Démonstration de la formule
|
||||
\begin{eqnarray*}
|
||||
u_n &=& \frac{d_n}{h_n} = \frac{16000\times0.9870^n}{2000\times1.05^n} \\
|
||||
u_n &=& \frac{16000}{2000}\times\left(\frac{0.9870}{1.05}\right)^n \\
|
||||
u_n &=& 8\times0.94^n
|
||||
\end{eqnarray*}
|
||||
\item On reconnaît la forme d'une suite géométrique de raison 0.94 et de premier terme 8.
|
||||
\item La raison, $q = 0.94$, est inférieur à 1 donc la suite est décroissante. Ce qui signifie que le débit par habitant va diminuer.
|
||||
\end{enumerate}
|
||||
\item Avec le tableau de la calculatrice, on calculer les valeurs de $u_n$ jusqu'à passer en dessous de 5. On trouve $n = 8$ avec $u_{8} = 4.8765515083286528$
|
||||
|
||||
\end{enumerate}
|
||||
\end{solution}
|
||||
\begin{exercise}[subtitle={Étude de fonctions}]
|
||||
\section*{Partie A}
|
||||
Dans cette partie, on étudie la fonction
|
||||
\[
|
||||
f(x) = - 4x^2 + 10x - 2
|
||||
\]
|
||||
\begin{enumerate}
|
||||
\item Calculer la dérivé de $f$.
|
||||
\item Étudier le signe de la dérivée $f'$ puis en déduire le tableau de signe de $f$.
|
||||
\end{enumerate}
|
||||
\section*{Partie B}
|
||||
Dans cette partie, on étudie la fonction
|
||||
\[
|
||||
g(x) = 3x^3 - x^2 - 4x - 9
|
||||
\]
|
||||
\begin{enumerate}
|
||||
\item À l'aide d'une calculatrice ou d'un ordinateur, tracer puis reporter sur votre copie la représentation graphique de $g$ en y indiquant les informations remarquables de ce graphique.
|
||||
\item Sur quel(s) intervalle(s) la fonction est convexe? concave? Y a-t-il des points d'inflexions?
|
||||
\item Calculer la dérivé de $g$.
|
||||
\item Étudier le signe de la dérivée $g'$ puis en déduire le tableau de variations de $g$.
|
||||
\item Déterminer l'équation de la tangente en $x=0$.
|
||||
\item Dériver $g'$ pour calculer $g''$.
|
||||
\item Étudier le signe de $g''$ pour en déduire la convexité de $g$ grâce au calcul puis localiser précisément le point d'inflexion.
|
||||
\end{enumerate}
|
||||
\end{exercise}
|
||||
\begin{solution}
|
||||
\section*{Partie A}
|
||||
\begin{enumerate}
|
||||
\item $10 - 8x$
|
||||
\item Correction non disponible
|
||||
\end{enumerate}
|
||||
|
||||
\section*{Partie B}
|
||||
\begin{enumerate}
|
||||
\item Correction non disponible
|
||||
\item Correction non disponible
|
||||
\item $g'(x) = - 4 - 2x + 9x^2$
|
||||
\item
|
||||
On commence par calculer le discriminant de $g'(x)=- 4 - 2x + 9x^2$.
|
||||
\begin{eqnarray*}
|
||||
\Delta & = & b^2-4ac \\
|
||||
\Delta & = & - 2^{2} - 4 \times 9 \times - 4 \\
|
||||
\Delta & = & 4 - 36 \times - 4 \\
|
||||
\Delta & = & 4 + 144 \\
|
||||
\Delta & = & 148
|
||||
\end{eqnarray*}
|
||||
|
||||
comme $\Delta = 148 > 0$ donc $P$ a deux racines
|
||||
|
||||
\begin{eqnarray*}
|
||||
x_1 & = & \frac{-b - \sqrt{\Delta}}{2a} = \frac{- 2 - \sqrt{148}}{2 \times 9} = - 0.5647513922553578 \\
|
||||
x_2 & = & \frac{-b + \sqrt{\Delta}}{2a} = \frac{- 2 + \sqrt{148}}{2 \times 9} = 0.78697361447758
|
||||
\end{eqnarray*}
|
||||
Ainsi, $g'$ est du signe de $a=9$ en dehors des racines.
|
||||
|
||||
|
||||
Le tableau de variation non disponible en correction
|
||||
\item Équation de la tangente: $y = - 4x + - 9$
|
||||
\item $g''(x) = 18x - 2$
|
||||
\end{enumerate}
|
||||
\end{solution}
|
||||
|
||||
\end{document}
|
||||
|
||||
%%% Local Variables:
|
||||
%%% mode: latex
|
||||
%%% TeX-master: "master"
|
||||
%%% End:
|
||||
157
TES/DM/DM_19_10/04_DM_19_10.tex
Normal file
157
TES/DM/DM_19_10/04_DM_19_10.tex
Normal file
@@ -0,0 +1,157 @@
|
||||
\documentclass[a4paper,10pt]{article}
|
||||
\usepackage{tasks}
|
||||
\usepackage{myXsim}
|
||||
|
||||
\title{DM 1 -- CRETIN Marie}
|
||||
\tribe{Terminale ES-L}
|
||||
\date{15 novembre 2019}
|
||||
|
||||
\xsimsetup{
|
||||
solution/print = false
|
||||
}
|
||||
|
||||
\begin{document}
|
||||
\maketitle
|
||||
|
||||
Une part importante de la note sera dédiée à la rédaction, aux explications et à l'utilisation des notations mathématiques.
|
||||
|
||||
\begin{exercise}[subtitle={Débit}]
|
||||
Une commune de \np{2400} habitants au 1\up{er} janvier 2018 voit sa population augmenter de 8\,\% tous les ans.
|
||||
|
||||
Pour tout entier naturel $n$, on note $h_n$ le nombre d'habitants de l'année $2018 + n$.
|
||||
|
||||
\smallskip
|
||||
|
||||
\begin{enumerate}
|
||||
\item Déterminer la nature de la suite $(h_n)$, préciser ses éléments caractéristiques et exprimer $h_n$ en fonction de $n$.
|
||||
\end{enumerate}
|
||||
|
||||
\smallskip
|
||||
|
||||
La municipalité de cette commune a conclu un marché avec un fournisseur d'accès internet qui engage ce dernier à fournir un débit total de \np{24000}~Mbit/s au 1\up{er} janvier 2018 et à augmenter ce débit de - 0.4400\,\% par an.
|
||||
|
||||
Pour tout entier naturel $n$, on note $d_n$ le débit total dont la commune dispose l'année $2018 + n$.
|
||||
|
||||
\begin{enumerate}
|
||||
\setcounter{enumi}{1}
|
||||
\item Déterminer la nature de la suite $(d_n)$, préciser ses éléments caractéristiques et exprimer $d_n$ en fonction de $n$.
|
||||
\end{enumerate}
|
||||
|
||||
On s'intéresse maintenant au débit par habitant en supposant que celui-ci est réparti équitablement et que toute la population bénéficie d'une connexion internet individuelle.
|
||||
|
||||
Pour tout entier naturel $n$ on note $u_n$ le débit par habitant pour l'année $2018 + n$ et on admet que $u_n = \dfrac{d_n}{h_n}$.
|
||||
\begin{enumerate}
|
||||
\setcounter{enumi}{2}
|
||||
\item Calculer $u_0$ et $u_1$.
|
||||
\item Montrer pour tout entier naturel $n$ on a $u_n = 10 \times 0.93^n$.
|
||||
\item En déduire la nature de la suite $\left(u_n\right)$ et ses caractéristiques.
|
||||
\item Déterminer le sens de variations de la $\left(u_n\right)$.
|
||||
Interpréter ce résultat dans le contexte de l'énoncé.
|
||||
\end{enumerate}
|
||||
Le marché passé avec le fournisseur d'accès internet prévoit également que si le débit passe en dessous de 5 Mbit/s par habitant alors ce dernier doit changer la technologie utilisée pour la réalisation de son réseau.
|
||||
|
||||
\begin{enumerate}
|
||||
\setcounter{enumi}{6}
|
||||
\item En quelle année le fournisseur d'accès sera-t-il dans l'obligation de changer sa technologie?
|
||||
\end{enumerate}
|
||||
\end{exercise}
|
||||
\begin{solution}
|
||||
\begin{enumerate}
|
||||
\item Augmenter de 8\% revient à multiplier pas 1.08. La suite $(h_n)$ est donc géométrique de raison 1.08 et de premier terme 2400. On en déduit $h_n$ en fonction de $n$
|
||||
\[
|
||||
h_n = 2400\times 1.08^n
|
||||
\]
|
||||
\item Augmenter de - 0.4400\% revient à multiplier pas 1.0044. La suite $(d_n)$ est donc géométrique de raison 1.0044 et de premier terme 24000. On en déduit $d_n$ en fonction de $n$
|
||||
\[
|
||||
d_n = 24000\times 1.0044^n
|
||||
\]
|
||||
\item
|
||||
\begin{enumerate}
|
||||
\item
|
||||
\[
|
||||
u_0 = \frac{d_0}{h_0} = \frac{24000}{2400} = 10
|
||||
\]
|
||||
\[
|
||||
u_1 = \frac{d_1}{h_1} = \frac{24105.6000}{2592} = 9.30
|
||||
\]
|
||||
\item Démonstration de la formule
|
||||
\begin{eqnarray*}
|
||||
u_n &=& \frac{d_n}{h_n} = \frac{24000\times1.0044^n}{2400\times1.08^n} \\
|
||||
u_n &=& \frac{24000}{2400}\times\left(\frac{1.0044}{1.08}\right)^n \\
|
||||
u_n &=& 10\times0.93^n
|
||||
\end{eqnarray*}
|
||||
\item On reconnaît la forme d'une suite géométrique de raison 0.93 et de premier terme 10.
|
||||
\item La raison, $q = 0.93$, est inférieur à 1 donc la suite est décroissante. Ce qui signifie que le débit par habitant va diminuer.
|
||||
\end{enumerate}
|
||||
\item Avec le tableau de la calculatrice, on calculer les valeurs de $u_n$ jusqu'à passer en dessous de 5. On trouve $n = 10$ avec $u_{10} = 4.83982307179293182490$
|
||||
|
||||
\end{enumerate}
|
||||
\end{solution}
|
||||
\begin{exercise}[subtitle={Étude de fonctions}]
|
||||
\section*{Partie A}
|
||||
Dans cette partie, on étudie la fonction
|
||||
\[
|
||||
f(x) = - 6x^2 - 8x - 4
|
||||
\]
|
||||
\begin{enumerate}
|
||||
\item Calculer la dérivé de $f$.
|
||||
\item Étudier le signe de la dérivée $f'$ puis en déduire le tableau de signe de $f$.
|
||||
\end{enumerate}
|
||||
\section*{Partie B}
|
||||
Dans cette partie, on étudie la fonction
|
||||
\[
|
||||
g(x) = x^3 - 10x^2 + 7x + 1
|
||||
\]
|
||||
\begin{enumerate}
|
||||
\item À l'aide d'une calculatrice ou d'un ordinateur, tracer puis reporter sur votre copie la représentation graphique de $g$ en y indiquant les informations remarquables de ce graphique.
|
||||
\item Sur quel(s) intervalle(s) la fonction est convexe? concave? Y a-t-il des points d'inflexions?
|
||||
\item Calculer la dérivé de $g$.
|
||||
\item Étudier le signe de la dérivée $g'$ puis en déduire le tableau de variations de $g$.
|
||||
\item Déterminer l'équation de la tangente en $x=0$.
|
||||
\item Dériver $g'$ pour calculer $g''$.
|
||||
\item Étudier le signe de $g''$ pour en déduire la convexité de $g$ grâce au calcul puis localiser précisément le point d'inflexion.
|
||||
\end{enumerate}
|
||||
\end{exercise}
|
||||
\begin{solution}
|
||||
\section*{Partie A}
|
||||
\begin{enumerate}
|
||||
\item $- 8 - 12x$
|
||||
\item Correction non disponible
|
||||
\end{enumerate}
|
||||
|
||||
\section*{Partie B}
|
||||
\begin{enumerate}
|
||||
\item Correction non disponible
|
||||
\item Correction non disponible
|
||||
\item $g'(x) = 7 + 3x^2 - 20x$
|
||||
\item
|
||||
On commence par calculer le discriminant de $g'(x)=7 + 3x^2 - 20x$.
|
||||
\begin{eqnarray*}
|
||||
\Delta & = & b^2-4ac \\
|
||||
\Delta & = & - 20^{2} - 4 \times 3 \times 7 \\
|
||||
\Delta & = & 400 - 12 \times 7 \\
|
||||
\Delta & = & 400 - 84 \\
|
||||
\Delta & = & 316
|
||||
\end{eqnarray*}
|
||||
|
||||
comme $\Delta = 316 > 0$ donc $P$ a deux racines
|
||||
|
||||
\begin{eqnarray*}
|
||||
x_1 & = & \frac{-b - \sqrt{\Delta}}{2a} = \frac{- 20 - \sqrt{316}}{2 \times 3} = 0.3706018608948038 \\
|
||||
x_2 & = & \frac{-b + \sqrt{\Delta}}{2a} = \frac{- 20 + \sqrt{316}}{2 \times 3} = 6.296064805771863
|
||||
\end{eqnarray*}
|
||||
Ainsi, $g'$ est du signe de $a=3$ en dehors des racines.
|
||||
|
||||
|
||||
Le tableau de variation non disponible en correction
|
||||
\item Équation de la tangente: $y = 7x + 1$
|
||||
\item $g''(x) = - 20 + 6x$
|
||||
\end{enumerate}
|
||||
\end{solution}
|
||||
|
||||
\end{document}
|
||||
|
||||
%%% Local Variables:
|
||||
%%% mode: latex
|
||||
%%% TeX-master: "master"
|
||||
%%% End:
|
||||
157
TES/DM/DM_19_10/05_DM_19_10.tex
Normal file
157
TES/DM/DM_19_10/05_DM_19_10.tex
Normal file
@@ -0,0 +1,157 @@
|
||||
\documentclass[a4paper,10pt]{article}
|
||||
\usepackage{tasks}
|
||||
\usepackage{myXsim}
|
||||
|
||||
\title{DM 1 -- DENIS Clarisse}
|
||||
\tribe{Terminale ES-L}
|
||||
\date{15 novembre 2019}
|
||||
|
||||
\xsimsetup{
|
||||
solution/print = false
|
||||
}
|
||||
|
||||
\begin{document}
|
||||
\maketitle
|
||||
|
||||
Une part importante de la note sera dédiée à la rédaction, aux explications et à l'utilisation des notations mathématiques.
|
||||
|
||||
\begin{exercise}[subtitle={Débit}]
|
||||
Une commune de \np{1600} habitants au 1\up{er} janvier 2018 voit sa population augmenter de 5\,\% tous les ans.
|
||||
|
||||
Pour tout entier naturel $n$, on note $h_n$ le nombre d'habitants de l'année $2018 + n$.
|
||||
|
||||
\smallskip
|
||||
|
||||
\begin{enumerate}
|
||||
\item Déterminer la nature de la suite $(h_n)$, préciser ses éléments caractéristiques et exprimer $h_n$ en fonction de $n$.
|
||||
\end{enumerate}
|
||||
|
||||
\smallskip
|
||||
|
||||
La municipalité de cette commune a conclu un marché avec un fournisseur d'accès internet qui engage ce dernier à fournir un débit total de \np{12800}~Mbit/s au 1\up{er} janvier 2018 et à augmenter ce débit de 3.4000\,\% par an.
|
||||
|
||||
Pour tout entier naturel $n$, on note $d_n$ le débit total dont la commune dispose l'année $2018 + n$.
|
||||
|
||||
\begin{enumerate}
|
||||
\setcounter{enumi}{1}
|
||||
\item Déterminer la nature de la suite $(d_n)$, préciser ses éléments caractéristiques et exprimer $d_n$ en fonction de $n$.
|
||||
\end{enumerate}
|
||||
|
||||
On s'intéresse maintenant au débit par habitant en supposant que celui-ci est réparti équitablement et que toute la population bénéficie d'une connexion internet individuelle.
|
||||
|
||||
Pour tout entier naturel $n$ on note $u_n$ le débit par habitant pour l'année $2018 + n$ et on admet que $u_n = \dfrac{d_n}{h_n}$.
|
||||
\begin{enumerate}
|
||||
\setcounter{enumi}{2}
|
||||
\item Calculer $u_0$ et $u_1$.
|
||||
\item Montrer pour tout entier naturel $n$ on a $u_n = 8 \times 0.92^n$.
|
||||
\item En déduire la nature de la suite $\left(u_n\right)$ et ses caractéristiques.
|
||||
\item Déterminer le sens de variations de la $\left(u_n\right)$.
|
||||
Interpréter ce résultat dans le contexte de l'énoncé.
|
||||
\end{enumerate}
|
||||
Le marché passé avec le fournisseur d'accès internet prévoit également que si le débit passe en dessous de 5 Mbit/s par habitant alors ce dernier doit changer la technologie utilisée pour la réalisation de son réseau.
|
||||
|
||||
\begin{enumerate}
|
||||
\setcounter{enumi}{6}
|
||||
\item En quelle année le fournisseur d'accès sera-t-il dans l'obligation de changer sa technologie?
|
||||
\end{enumerate}
|
||||
\end{exercise}
|
||||
\begin{solution}
|
||||
\begin{enumerate}
|
||||
\item Augmenter de 5\% revient à multiplier pas 1.05. La suite $(h_n)$ est donc géométrique de raison 1.05 et de premier terme 1600. On en déduit $h_n$ en fonction de $n$
|
||||
\[
|
||||
h_n = 1600\times 1.05^n
|
||||
\]
|
||||
\item Augmenter de 3.4000\% revient à multiplier pas 0.9660. La suite $(d_n)$ est donc géométrique de raison 0.9660 et de premier terme 12800. On en déduit $d_n$ en fonction de $n$
|
||||
\[
|
||||
d_n = 12800\times 0.9660^n
|
||||
\]
|
||||
\item
|
||||
\begin{enumerate}
|
||||
\item
|
||||
\[
|
||||
u_0 = \frac{d_0}{h_0} = \frac{12800}{1600} = 8
|
||||
\]
|
||||
\[
|
||||
u_1 = \frac{d_1}{h_1} = \frac{12364.8000}{1680} = 7.36
|
||||
\]
|
||||
\item Démonstration de la formule
|
||||
\begin{eqnarray*}
|
||||
u_n &=& \frac{d_n}{h_n} = \frac{12800\times0.9660^n}{1600\times1.05^n} \\
|
||||
u_n &=& \frac{12800}{1600}\times\left(\frac{0.9660}{1.05}\right)^n \\
|
||||
u_n &=& 8\times0.92^n
|
||||
\end{eqnarray*}
|
||||
\item On reconnaît la forme d'une suite géométrique de raison 0.92 et de premier terme 8.
|
||||
\item La raison, $q = 0.92$, est inférieur à 1 donc la suite est décroissante. Ce qui signifie que le débit par habitant va diminuer.
|
||||
\end{enumerate}
|
||||
\item Avec le tableau de la calculatrice, on calculer les valeurs de $u_n$ jusqu'à passer en dessous de 5. On trouve $n = 6$ avec $u_{6} = 4.850840010752$
|
||||
|
||||
\end{enumerate}
|
||||
\end{solution}
|
||||
\begin{exercise}[subtitle={Étude de fonctions}]
|
||||
\section*{Partie A}
|
||||
Dans cette partie, on étudie la fonction
|
||||
\[
|
||||
f(x) = - 7x^2 + 6x - 5
|
||||
\]
|
||||
\begin{enumerate}
|
||||
\item Calculer la dérivé de $f$.
|
||||
\item Étudier le signe de la dérivée $f'$ puis en déduire le tableau de signe de $f$.
|
||||
\end{enumerate}
|
||||
\section*{Partie B}
|
||||
Dans cette partie, on étudie la fonction
|
||||
\[
|
||||
g(x) = - 7x^3 + 2x^2 + 2x - 6
|
||||
\]
|
||||
\begin{enumerate}
|
||||
\item À l'aide d'une calculatrice ou d'un ordinateur, tracer puis reporter sur votre copie la représentation graphique de $g$ en y indiquant les informations remarquables de ce graphique.
|
||||
\item Sur quel(s) intervalle(s) la fonction est convexe? concave? Y a-t-il des points d'inflexions?
|
||||
\item Calculer la dérivé de $g$.
|
||||
\item Étudier le signe de la dérivée $g'$ puis en déduire le tableau de variations de $g$.
|
||||
\item Déterminer l'équation de la tangente en $x=0$.
|
||||
\item Dériver $g'$ pour calculer $g''$.
|
||||
\item Étudier le signe de $g''$ pour en déduire la convexité de $g$ grâce au calcul puis localiser précisément le point d'inflexion.
|
||||
\end{enumerate}
|
||||
\end{exercise}
|
||||
\begin{solution}
|
||||
\section*{Partie A}
|
||||
\begin{enumerate}
|
||||
\item $6 - 14x$
|
||||
\item Correction non disponible
|
||||
\end{enumerate}
|
||||
|
||||
\section*{Partie B}
|
||||
\begin{enumerate}
|
||||
\item Correction non disponible
|
||||
\item Correction non disponible
|
||||
\item $g'(x) = 2 + 4x - 21x^2$
|
||||
\item
|
||||
On commence par calculer le discriminant de $g'(x)=2 + 4x - 21x^2$.
|
||||
\begin{eqnarray*}
|
||||
\Delta & = & b^2-4ac \\
|
||||
\Delta & = & 4^{2} - 4 \times - 21 \times 2 \\
|
||||
\Delta & = & 16 + 84 \times 2 \\
|
||||
\Delta & = & 16 + 168 \\
|
||||
\Delta & = & 184
|
||||
\end{eqnarray*}
|
||||
|
||||
comme $\Delta = 184 > 0$ donc $P$ a deux racines
|
||||
|
||||
\begin{eqnarray*}
|
||||
x_1 & = & \frac{-b - \sqrt{\Delta}}{2a} = \frac{4 - \sqrt{184}}{2 \times - 21} = - 0.22772999919644135 \\
|
||||
x_2 & = & \frac{-b + \sqrt{\Delta}}{2a} = \frac{4 + \sqrt{184}}{2 \times - 21} = 0.41820618967263185
|
||||
\end{eqnarray*}
|
||||
Ainsi, $g'$ est du signe de $a=- 21$ en dehors des racines.
|
||||
|
||||
|
||||
Le tableau de variation non disponible en correction
|
||||
\item Équation de la tangente: $y = 2x + - 6$
|
||||
\item $g''(x) = - 42x + 4$
|
||||
\end{enumerate}
|
||||
\end{solution}
|
||||
|
||||
\end{document}
|
||||
|
||||
%%% Local Variables:
|
||||
%%% mode: latex
|
||||
%%% TeX-master: "master"
|
||||
%%% End:
|
||||
157
TES/DM/DM_19_10/06_DM_19_10.tex
Normal file
157
TES/DM/DM_19_10/06_DM_19_10.tex
Normal file
@@ -0,0 +1,157 @@
|
||||
\documentclass[a4paper,10pt]{article}
|
||||
\usepackage{tasks}
|
||||
\usepackage{myXsim}
|
||||
|
||||
\title{DM 1 -- DOS SANTOS Théo}
|
||||
\tribe{Terminale ES-L}
|
||||
\date{15 novembre 2019}
|
||||
|
||||
\xsimsetup{
|
||||
solution/print = false
|
||||
}
|
||||
|
||||
\begin{document}
|
||||
\maketitle
|
||||
|
||||
Une part importante de la note sera dédiée à la rédaction, aux explications et à l'utilisation des notations mathématiques.
|
||||
|
||||
\begin{exercise}[subtitle={Débit}]
|
||||
Une commune de \np{1500} habitants au 1\up{er} janvier 2018 voit sa population augmenter de 8\,\% tous les ans.
|
||||
|
||||
Pour tout entier naturel $n$, on note $h_n$ le nombre d'habitants de l'année $2018 + n$.
|
||||
|
||||
\smallskip
|
||||
|
||||
\begin{enumerate}
|
||||
\item Déterminer la nature de la suite $(h_n)$, préciser ses éléments caractéristiques et exprimer $h_n$ en fonction de $n$.
|
||||
\end{enumerate}
|
||||
|
||||
\smallskip
|
||||
|
||||
La municipalité de cette commune a conclu un marché avec un fournisseur d'accès internet qui engage ce dernier à fournir un débit total de \np{18000}~Mbit/s au 1\up{er} janvier 2018 et à augmenter ce débit de 0.6400\,\% par an.
|
||||
|
||||
Pour tout entier naturel $n$, on note $d_n$ le débit total dont la commune dispose l'année $2018 + n$.
|
||||
|
||||
\begin{enumerate}
|
||||
\setcounter{enumi}{1}
|
||||
\item Déterminer la nature de la suite $(d_n)$, préciser ses éléments caractéristiques et exprimer $d_n$ en fonction de $n$.
|
||||
\end{enumerate}
|
||||
|
||||
On s'intéresse maintenant au débit par habitant en supposant que celui-ci est réparti équitablement et que toute la population bénéficie d'une connexion internet individuelle.
|
||||
|
||||
Pour tout entier naturel $n$ on note $u_n$ le débit par habitant pour l'année $2018 + n$ et on admet que $u_n = \dfrac{d_n}{h_n}$.
|
||||
\begin{enumerate}
|
||||
\setcounter{enumi}{2}
|
||||
\item Calculer $u_0$ et $u_1$.
|
||||
\item Montrer pour tout entier naturel $n$ on a $u_n = 12 \times 0.92^n$.
|
||||
\item En déduire la nature de la suite $\left(u_n\right)$ et ses caractéristiques.
|
||||
\item Déterminer le sens de variations de la $\left(u_n\right)$.
|
||||
Interpréter ce résultat dans le contexte de l'énoncé.
|
||||
\end{enumerate}
|
||||
Le marché passé avec le fournisseur d'accès internet prévoit également que si le débit passe en dessous de 5 Mbit/s par habitant alors ce dernier doit changer la technologie utilisée pour la réalisation de son réseau.
|
||||
|
||||
\begin{enumerate}
|
||||
\setcounter{enumi}{6}
|
||||
\item En quelle année le fournisseur d'accès sera-t-il dans l'obligation de changer sa technologie?
|
||||
\end{enumerate}
|
||||
\end{exercise}
|
||||
\begin{solution}
|
||||
\begin{enumerate}
|
||||
\item Augmenter de 8\% revient à multiplier pas 1.08. La suite $(h_n)$ est donc géométrique de raison 1.08 et de premier terme 1500. On en déduit $h_n$ en fonction de $n$
|
||||
\[
|
||||
h_n = 1500\times 1.08^n
|
||||
\]
|
||||
\item Augmenter de 0.6400\% revient à multiplier pas 0.9936. La suite $(d_n)$ est donc géométrique de raison 0.9936 et de premier terme 18000. On en déduit $d_n$ en fonction de $n$
|
||||
\[
|
||||
d_n = 18000\times 0.9936^n
|
||||
\]
|
||||
\item
|
||||
\begin{enumerate}
|
||||
\item
|
||||
\[
|
||||
u_0 = \frac{d_0}{h_0} = \frac{18000}{1500} = 12
|
||||
\]
|
||||
\[
|
||||
u_1 = \frac{d_1}{h_1} = \frac{17884.8000}{1620} = 11.04
|
||||
\]
|
||||
\item Démonstration de la formule
|
||||
\begin{eqnarray*}
|
||||
u_n &=& \frac{d_n}{h_n} = \frac{18000\times0.9936^n}{1500\times1.08^n} \\
|
||||
u_n &=& \frac{18000}{1500}\times\left(\frac{0.9936}{1.08}\right)^n \\
|
||||
u_n &=& 12\times0.92^n
|
||||
\end{eqnarray*}
|
||||
\item On reconnaît la forme d'une suite géométrique de raison 0.92 et de premier terme 12.
|
||||
\item La raison, $q = 0.92$, est inférieur à 1 donc la suite est décroissante. Ce qui signifie que le débit par habitant va diminuer.
|
||||
\end{enumerate}
|
||||
\item Avec le tableau de la calculatrice, on calculer les valeurs de $u_n$ jusqu'à passer en dessous de 5. On trouve $n = 11$ avec $u_{11} = 4.7956485346288988061696$
|
||||
|
||||
\end{enumerate}
|
||||
\end{solution}
|
||||
\begin{exercise}[subtitle={Étude de fonctions}]
|
||||
\section*{Partie A}
|
||||
Dans cette partie, on étudie la fonction
|
||||
\[
|
||||
f(x) = 10x^2 - 5x - 6
|
||||
\]
|
||||
\begin{enumerate}
|
||||
\item Calculer la dérivé de $f$.
|
||||
\item Étudier le signe de la dérivée $f'$ puis en déduire le tableau de signe de $f$.
|
||||
\end{enumerate}
|
||||
\section*{Partie B}
|
||||
Dans cette partie, on étudie la fonction
|
||||
\[
|
||||
g(x) = 8x^3 - 4x^2 - 8x + 9
|
||||
\]
|
||||
\begin{enumerate}
|
||||
\item À l'aide d'une calculatrice ou d'un ordinateur, tracer puis reporter sur votre copie la représentation graphique de $g$ en y indiquant les informations remarquables de ce graphique.
|
||||
\item Sur quel(s) intervalle(s) la fonction est convexe? concave? Y a-t-il des points d'inflexions?
|
||||
\item Calculer la dérivé de $g$.
|
||||
\item Étudier le signe de la dérivée $g'$ puis en déduire le tableau de variations de $g$.
|
||||
\item Déterminer l'équation de la tangente en $x=0$.
|
||||
\item Dériver $g'$ pour calculer $g''$.
|
||||
\item Étudier le signe de $g''$ pour en déduire la convexité de $g$ grâce au calcul puis localiser précisément le point d'inflexion.
|
||||
\end{enumerate}
|
||||
\end{exercise}
|
||||
\begin{solution}
|
||||
\section*{Partie A}
|
||||
\begin{enumerate}
|
||||
\item $- 5 + 20x$
|
||||
\item Correction non disponible
|
||||
\end{enumerate}
|
||||
|
||||
\section*{Partie B}
|
||||
\begin{enumerate}
|
||||
\item Correction non disponible
|
||||
\item Correction non disponible
|
||||
\item $g'(x) = - 8 - 8x + 24x^2$
|
||||
\item
|
||||
On commence par calculer le discriminant de $g'(x)=- 8 - 8x + 24x^2$.
|
||||
\begin{eqnarray*}
|
||||
\Delta & = & b^2-4ac \\
|
||||
\Delta & = & - 8^{2} - 4 \times 24 \times - 8 \\
|
||||
\Delta & = & 64 - 96 \times - 8 \\
|
||||
\Delta & = & 64 + 768 \\
|
||||
\Delta & = & 832
|
||||
\end{eqnarray*}
|
||||
|
||||
comme $\Delta = 832 > 0$ donc $P$ a deux racines
|
||||
|
||||
\begin{eqnarray*}
|
||||
x_1 & = & \frac{-b - \sqrt{\Delta}}{2a} = \frac{- 8 - \sqrt{832}}{2 \times 24} = - 0.4342585459106649 \\
|
||||
x_2 & = & \frac{-b + \sqrt{\Delta}}{2a} = \frac{- 8 + \sqrt{832}}{2 \times 24} = 0.7675918792439983
|
||||
\end{eqnarray*}
|
||||
Ainsi, $g'$ est du signe de $a=24$ en dehors des racines.
|
||||
|
||||
|
||||
Le tableau de variation non disponible en correction
|
||||
\item Équation de la tangente: $y = - 8x + 9$
|
||||
\item $g''(x) = 48x - 8$
|
||||
\end{enumerate}
|
||||
\end{solution}
|
||||
|
||||
\end{document}
|
||||
|
||||
%%% Local Variables:
|
||||
%%% mode: latex
|
||||
%%% TeX-master: "master"
|
||||
%%% End:
|
||||
157
TES/DM/DM_19_10/07_DM_19_10.tex
Normal file
157
TES/DM/DM_19_10/07_DM_19_10.tex
Normal file
@@ -0,0 +1,157 @@
|
||||
\documentclass[a4paper,10pt]{article}
|
||||
\usepackage{tasks}
|
||||
\usepackage{myXsim}
|
||||
|
||||
\title{DM 1 -- FERREIRA Tina}
|
||||
\tribe{Terminale ES-L}
|
||||
\date{15 novembre 2019}
|
||||
|
||||
\xsimsetup{
|
||||
solution/print = false
|
||||
}
|
||||
|
||||
\begin{document}
|
||||
\maketitle
|
||||
|
||||
Une part importante de la note sera dédiée à la rédaction, aux explications et à l'utilisation des notations mathématiques.
|
||||
|
||||
\begin{exercise}[subtitle={Débit}]
|
||||
Une commune de \np{1700} habitants au 1\up{er} janvier 2018 voit sa population augmenter de 8\,\% tous les ans.
|
||||
|
||||
Pour tout entier naturel $n$, on note $h_n$ le nombre d'habitants de l'année $2018 + n$.
|
||||
|
||||
\smallskip
|
||||
|
||||
\begin{enumerate}
|
||||
\item Déterminer la nature de la suite $(h_n)$, préciser ses éléments caractéristiques et exprimer $h_n$ en fonction de $n$.
|
||||
\end{enumerate}
|
||||
|
||||
\smallskip
|
||||
|
||||
La municipalité de cette commune a conclu un marché avec un fournisseur d'accès internet qui engage ce dernier à fournir un débit total de \np{20400}~Mbit/s au 1\up{er} janvier 2018 et à augmenter ce débit de - 5.8400\,\% par an.
|
||||
|
||||
Pour tout entier naturel $n$, on note $d_n$ le débit total dont la commune dispose l'année $2018 + n$.
|
||||
|
||||
\begin{enumerate}
|
||||
\setcounter{enumi}{1}
|
||||
\item Déterminer la nature de la suite $(d_n)$, préciser ses éléments caractéristiques et exprimer $d_n$ en fonction de $n$.
|
||||
\end{enumerate}
|
||||
|
||||
On s'intéresse maintenant au débit par habitant en supposant que celui-ci est réparti équitablement et que toute la population bénéficie d'une connexion internet individuelle.
|
||||
|
||||
Pour tout entier naturel $n$ on note $u_n$ le débit par habitant pour l'année $2018 + n$ et on admet que $u_n = \dfrac{d_n}{h_n}$.
|
||||
\begin{enumerate}
|
||||
\setcounter{enumi}{2}
|
||||
\item Calculer $u_0$ et $u_1$.
|
||||
\item Montrer pour tout entier naturel $n$ on a $u_n = 12 \times 0.98^n$.
|
||||
\item En déduire la nature de la suite $\left(u_n\right)$ et ses caractéristiques.
|
||||
\item Déterminer le sens de variations de la $\left(u_n\right)$.
|
||||
Interpréter ce résultat dans le contexte de l'énoncé.
|
||||
\end{enumerate}
|
||||
Le marché passé avec le fournisseur d'accès internet prévoit également que si le débit passe en dessous de 5 Mbit/s par habitant alors ce dernier doit changer la technologie utilisée pour la réalisation de son réseau.
|
||||
|
||||
\begin{enumerate}
|
||||
\setcounter{enumi}{6}
|
||||
\item En quelle année le fournisseur d'accès sera-t-il dans l'obligation de changer sa technologie?
|
||||
\end{enumerate}
|
||||
\end{exercise}
|
||||
\begin{solution}
|
||||
\begin{enumerate}
|
||||
\item Augmenter de 8\% revient à multiplier pas 1.08. La suite $(h_n)$ est donc géométrique de raison 1.08 et de premier terme 1700. On en déduit $h_n$ en fonction de $n$
|
||||
\[
|
||||
h_n = 1700\times 1.08^n
|
||||
\]
|
||||
\item Augmenter de - 5.8400\% revient à multiplier pas 1.0584. La suite $(d_n)$ est donc géométrique de raison 1.0584 et de premier terme 20400. On en déduit $d_n$ en fonction de $n$
|
||||
\[
|
||||
d_n = 20400\times 1.0584^n
|
||||
\]
|
||||
\item
|
||||
\begin{enumerate}
|
||||
\item
|
||||
\[
|
||||
u_0 = \frac{d_0}{h_0} = \frac{20400}{1700} = 12
|
||||
\]
|
||||
\[
|
||||
u_1 = \frac{d_1}{h_1} = \frac{21591.3600}{1836} = 11.76
|
||||
\]
|
||||
\item Démonstration de la formule
|
||||
\begin{eqnarray*}
|
||||
u_n &=& \frac{d_n}{h_n} = \frac{20400\times1.0584^n}{1700\times1.08^n} \\
|
||||
u_n &=& \frac{20400}{1700}\times\left(\frac{1.0584}{1.08}\right)^n \\
|
||||
u_n &=& 12\times0.98^n
|
||||
\end{eqnarray*}
|
||||
\item On reconnaît la forme d'une suite géométrique de raison 0.98 et de premier terme 12.
|
||||
\item La raison, $q = 0.98$, est inférieur à 1 donc la suite est décroissante. Ce qui signifie que le débit par habitant va diminuer.
|
||||
\end{enumerate}
|
||||
\item Avec le tableau de la calculatrice, on calculer les valeurs de $u_n$ jusqu'à passer en dessous de 5. On trouve $n = 44$ avec $u_{44} = 4.933198338041945361137775984$
|
||||
|
||||
\end{enumerate}
|
||||
\end{solution}
|
||||
\begin{exercise}[subtitle={Étude de fonctions}]
|
||||
\section*{Partie A}
|
||||
Dans cette partie, on étudie la fonction
|
||||
\[
|
||||
f(x) = 3x^2 + 7x - 9
|
||||
\]
|
||||
\begin{enumerate}
|
||||
\item Calculer la dérivé de $f$.
|
||||
\item Étudier le signe de la dérivée $f'$ puis en déduire le tableau de signe de $f$.
|
||||
\end{enumerate}
|
||||
\section*{Partie B}
|
||||
Dans cette partie, on étudie la fonction
|
||||
\[
|
||||
g(x) = - 4x^3 - 9x^2 - 4x - 2
|
||||
\]
|
||||
\begin{enumerate}
|
||||
\item À l'aide d'une calculatrice ou d'un ordinateur, tracer puis reporter sur votre copie la représentation graphique de $g$ en y indiquant les informations remarquables de ce graphique.
|
||||
\item Sur quel(s) intervalle(s) la fonction est convexe? concave? Y a-t-il des points d'inflexions?
|
||||
\item Calculer la dérivé de $g$.
|
||||
\item Étudier le signe de la dérivée $g'$ puis en déduire le tableau de variations de $g$.
|
||||
\item Déterminer l'équation de la tangente en $x=0$.
|
||||
\item Dériver $g'$ pour calculer $g''$.
|
||||
\item Étudier le signe de $g''$ pour en déduire la convexité de $g$ grâce au calcul puis localiser précisément le point d'inflexion.
|
||||
\end{enumerate}
|
||||
\end{exercise}
|
||||
\begin{solution}
|
||||
\section*{Partie A}
|
||||
\begin{enumerate}
|
||||
\item $7 + 6x$
|
||||
\item Correction non disponible
|
||||
\end{enumerate}
|
||||
|
||||
\section*{Partie B}
|
||||
\begin{enumerate}
|
||||
\item Correction non disponible
|
||||
\item Correction non disponible
|
||||
\item $g'(x) = - 4 - 18x - 12x^2$
|
||||
\item
|
||||
On commence par calculer le discriminant de $g'(x)=- 4 - 18x - 12x^2$.
|
||||
\begin{eqnarray*}
|
||||
\Delta & = & b^2-4ac \\
|
||||
\Delta & = & - 18^{2} - 4 \times - 12 \times - 4 \\
|
||||
\Delta & = & 324 + 48 \times - 4 \\
|
||||
\Delta & = & 324 - 192 \\
|
||||
\Delta & = & 132
|
||||
\end{eqnarray*}
|
||||
|
||||
comme $\Delta = 132 > 0$ donc $P$ a deux racines
|
||||
|
||||
\begin{eqnarray*}
|
||||
x_1 & = & \frac{-b - \sqrt{\Delta}}{2a} = \frac{- 18 - \sqrt{132}}{2 \times - 12} = - 0.27128644612183095 \\
|
||||
x_2 & = & \frac{-b + \sqrt{\Delta}}{2a} = \frac{- 18 + \sqrt{132}}{2 \times - 12} = - 1.228713553878169
|
||||
\end{eqnarray*}
|
||||
Ainsi, $g'$ est du signe de $a=- 12$ en dehors des racines.
|
||||
|
||||
|
||||
Le tableau de variation non disponible en correction
|
||||
\item Équation de la tangente: $y = - 4x + - 2$
|
||||
\item $g''(x) = - 24x - 18$
|
||||
\end{enumerate}
|
||||
\end{solution}
|
||||
|
||||
\end{document}
|
||||
|
||||
%%% Local Variables:
|
||||
%%% mode: latex
|
||||
%%% TeX-master: "master"
|
||||
%%% End:
|
||||
157
TES/DM/DM_19_10/08_DM_19_10.tex
Normal file
157
TES/DM/DM_19_10/08_DM_19_10.tex
Normal file
@@ -0,0 +1,157 @@
|
||||
\documentclass[a4paper,10pt]{article}
|
||||
\usepackage{tasks}
|
||||
\usepackage{myXsim}
|
||||
|
||||
\title{DM 1 -- GAUDARD Camille}
|
||||
\tribe{Terminale ES-L}
|
||||
\date{15 novembre 2019}
|
||||
|
||||
\xsimsetup{
|
||||
solution/print = false
|
||||
}
|
||||
|
||||
\begin{document}
|
||||
\maketitle
|
||||
|
||||
Une part importante de la note sera dédiée à la rédaction, aux explications et à l'utilisation des notations mathématiques.
|
||||
|
||||
\begin{exercise}[subtitle={Débit}]
|
||||
Une commune de \np{1600} habitants au 1\up{er} janvier 2018 voit sa population augmenter de 7\,\% tous les ans.
|
||||
|
||||
Pour tout entier naturel $n$, on note $h_n$ le nombre d'habitants de l'année $2018 + n$.
|
||||
|
||||
\smallskip
|
||||
|
||||
\begin{enumerate}
|
||||
\item Déterminer la nature de la suite $(h_n)$, préciser ses éléments caractéristiques et exprimer $h_n$ en fonction de $n$.
|
||||
\end{enumerate}
|
||||
|
||||
\smallskip
|
||||
|
||||
La municipalité de cette commune a conclu un marché avec un fournisseur d'accès internet qui engage ce dernier à fournir un débit total de \np{11200}~Mbit/s au 1\up{er} janvier 2018 et à augmenter ce débit de 0.4900\,\% par an.
|
||||
|
||||
Pour tout entier naturel $n$, on note $d_n$ le débit total dont la commune dispose l'année $2018 + n$.
|
||||
|
||||
\begin{enumerate}
|
||||
\setcounter{enumi}{1}
|
||||
\item Déterminer la nature de la suite $(d_n)$, préciser ses éléments caractéristiques et exprimer $d_n$ en fonction de $n$.
|
||||
\end{enumerate}
|
||||
|
||||
On s'intéresse maintenant au débit par habitant en supposant que celui-ci est réparti équitablement et que toute la population bénéficie d'une connexion internet individuelle.
|
||||
|
||||
Pour tout entier naturel $n$ on note $u_n$ le débit par habitant pour l'année $2018 + n$ et on admet que $u_n = \dfrac{d_n}{h_n}$.
|
||||
\begin{enumerate}
|
||||
\setcounter{enumi}{2}
|
||||
\item Calculer $u_0$ et $u_1$.
|
||||
\item Montrer pour tout entier naturel $n$ on a $u_n = 7 \times 0.93^n$.
|
||||
\item En déduire la nature de la suite $\left(u_n\right)$ et ses caractéristiques.
|
||||
\item Déterminer le sens de variations de la $\left(u_n\right)$.
|
||||
Interpréter ce résultat dans le contexte de l'énoncé.
|
||||
\end{enumerate}
|
||||
Le marché passé avec le fournisseur d'accès internet prévoit également que si le débit passe en dessous de 5 Mbit/s par habitant alors ce dernier doit changer la technologie utilisée pour la réalisation de son réseau.
|
||||
|
||||
\begin{enumerate}
|
||||
\setcounter{enumi}{6}
|
||||
\item En quelle année le fournisseur d'accès sera-t-il dans l'obligation de changer sa technologie?
|
||||
\end{enumerate}
|
||||
\end{exercise}
|
||||
\begin{solution}
|
||||
\begin{enumerate}
|
||||
\item Augmenter de 7\% revient à multiplier pas 1.07. La suite $(h_n)$ est donc géométrique de raison 1.07 et de premier terme 1600. On en déduit $h_n$ en fonction de $n$
|
||||
\[
|
||||
h_n = 1600\times 1.07^n
|
||||
\]
|
||||
\item Augmenter de 0.4900\% revient à multiplier pas 0.9951. La suite $(d_n)$ est donc géométrique de raison 0.9951 et de premier terme 11200. On en déduit $d_n$ en fonction de $n$
|
||||
\[
|
||||
d_n = 11200\times 0.9951^n
|
||||
\]
|
||||
\item
|
||||
\begin{enumerate}
|
||||
\item
|
||||
\[
|
||||
u_0 = \frac{d_0}{h_0} = \frac{11200}{1600} = 7
|
||||
\]
|
||||
\[
|
||||
u_1 = \frac{d_1}{h_1} = \frac{11145.1200}{1712} = 6.51
|
||||
\]
|
||||
\item Démonstration de la formule
|
||||
\begin{eqnarray*}
|
||||
u_n &=& \frac{d_n}{h_n} = \frac{11200\times0.9951^n}{1600\times1.07^n} \\
|
||||
u_n &=& \frac{11200}{1600}\times\left(\frac{0.9951}{1.07}\right)^n \\
|
||||
u_n &=& 7\times0.93^n
|
||||
\end{eqnarray*}
|
||||
\item On reconnaît la forme d'une suite géométrique de raison 0.93 et de premier terme 7.
|
||||
\item La raison, $q = 0.93$, est inférieur à 1 donc la suite est décroissante. Ce qui signifie que le débit par habitant va diminuer.
|
||||
\end{enumerate}
|
||||
\item Avec le tableau de la calculatrice, on calculer les valeurs de $u_n$ jusqu'à passer en dessous de 5. On trouve $n = 5$ avec $u_{5} = 4.8698185851$
|
||||
|
||||
\end{enumerate}
|
||||
\end{solution}
|
||||
\begin{exercise}[subtitle={Étude de fonctions}]
|
||||
\section*{Partie A}
|
||||
Dans cette partie, on étudie la fonction
|
||||
\[
|
||||
f(x) = - 4x^2 - 10x - 3
|
||||
\]
|
||||
\begin{enumerate}
|
||||
\item Calculer la dérivé de $f$.
|
||||
\item Étudier le signe de la dérivée $f'$ puis en déduire le tableau de signe de $f$.
|
||||
\end{enumerate}
|
||||
\section*{Partie B}
|
||||
Dans cette partie, on étudie la fonction
|
||||
\[
|
||||
g(x) = - x^3 - 8x^2 - 5x - 9
|
||||
\]
|
||||
\begin{enumerate}
|
||||
\item À l'aide d'une calculatrice ou d'un ordinateur, tracer puis reporter sur votre copie la représentation graphique de $g$ en y indiquant les informations remarquables de ce graphique.
|
||||
\item Sur quel(s) intervalle(s) la fonction est convexe? concave? Y a-t-il des points d'inflexions?
|
||||
\item Calculer la dérivé de $g$.
|
||||
\item Étudier le signe de la dérivée $g'$ puis en déduire le tableau de variations de $g$.
|
||||
\item Déterminer l'équation de la tangente en $x=0$.
|
||||
\item Dériver $g'$ pour calculer $g''$.
|
||||
\item Étudier le signe de $g''$ pour en déduire la convexité de $g$ grâce au calcul puis localiser précisément le point d'inflexion.
|
||||
\end{enumerate}
|
||||
\end{exercise}
|
||||
\begin{solution}
|
||||
\section*{Partie A}
|
||||
\begin{enumerate}
|
||||
\item $- 10 - 8x$
|
||||
\item Correction non disponible
|
||||
\end{enumerate}
|
||||
|
||||
\section*{Partie B}
|
||||
\begin{enumerate}
|
||||
\item Correction non disponible
|
||||
\item Correction non disponible
|
||||
\item $g'(x) = - 5 - 16x - 3x^2$
|
||||
\item
|
||||
On commence par calculer le discriminant de $g'(x)=- 5 - 16x - 3x^2$.
|
||||
\begin{eqnarray*}
|
||||
\Delta & = & b^2-4ac \\
|
||||
\Delta & = & - 16^{2} - 4 \times - 3 \times - 5 \\
|
||||
\Delta & = & 256 + 12 \times - 5 \\
|
||||
\Delta & = & 256 - 60 \\
|
||||
\Delta & = & 196
|
||||
\end{eqnarray*}
|
||||
|
||||
comme $\Delta = 196 > 0$ donc $P$ a deux racines
|
||||
|
||||
\begin{eqnarray*}
|
||||
x_1 & = & \frac{-b - \sqrt{\Delta}}{2a} = \frac{- 16 - \sqrt{196}}{2 \times - 3} = - 0.3333333333333333 \\
|
||||
x_2 & = & \frac{-b + \sqrt{\Delta}}{2a} = \frac{- 16 + \sqrt{196}}{2 \times - 3} = - 5
|
||||
\end{eqnarray*}
|
||||
Ainsi, $g'$ est du signe de $a=- 3$ en dehors des racines.
|
||||
|
||||
|
||||
Le tableau de variation non disponible en correction
|
||||
\item Équation de la tangente: $y = - 5x + - 9$
|
||||
\item $g''(x) = - 6x - 16$
|
||||
\end{enumerate}
|
||||
\end{solution}
|
||||
|
||||
\end{document}
|
||||
|
||||
%%% Local Variables:
|
||||
%%% mode: latex
|
||||
%%% TeX-master: "master"
|
||||
%%% End:
|
||||
157
TES/DM/DM_19_10/09_DM_19_10.tex
Normal file
157
TES/DM/DM_19_10/09_DM_19_10.tex
Normal file
@@ -0,0 +1,157 @@
|
||||
\documentclass[a4paper,10pt]{article}
|
||||
\usepackage{tasks}
|
||||
\usepackage{myXsim}
|
||||
|
||||
\title{DM 1 -- GUVERCIN Dilara Melisa}
|
||||
\tribe{Terminale ES-L}
|
||||
\date{15 novembre 2019}
|
||||
|
||||
\xsimsetup{
|
||||
solution/print = false
|
||||
}
|
||||
|
||||
\begin{document}
|
||||
\maketitle
|
||||
|
||||
Une part importante de la note sera dédiée à la rédaction, aux explications et à l'utilisation des notations mathématiques.
|
||||
|
||||
\begin{exercise}[subtitle={Débit}]
|
||||
Une commune de \np{2500} habitants au 1\up{er} janvier 2018 voit sa population augmenter de 7\,\% tous les ans.
|
||||
|
||||
Pour tout entier naturel $n$, on note $h_n$ le nombre d'habitants de l'année $2018 + n$.
|
||||
|
||||
\smallskip
|
||||
|
||||
\begin{enumerate}
|
||||
\item Déterminer la nature de la suite $(h_n)$, préciser ses éléments caractéristiques et exprimer $h_n$ en fonction de $n$.
|
||||
\end{enumerate}
|
||||
|
||||
\smallskip
|
||||
|
||||
La municipalité de cette commune a conclu un marché avec un fournisseur d'accès internet qui engage ce dernier à fournir un débit total de \np{17500}~Mbit/s au 1\up{er} janvier 2018 et à augmenter ce débit de - 4.8600\,\% par an.
|
||||
|
||||
Pour tout entier naturel $n$, on note $d_n$ le débit total dont la commune dispose l'année $2018 + n$.
|
||||
|
||||
\begin{enumerate}
|
||||
\setcounter{enumi}{1}
|
||||
\item Déterminer la nature de la suite $(d_n)$, préciser ses éléments caractéristiques et exprimer $d_n$ en fonction de $n$.
|
||||
\end{enumerate}
|
||||
|
||||
On s'intéresse maintenant au débit par habitant en supposant que celui-ci est réparti équitablement et que toute la population bénéficie d'une connexion internet individuelle.
|
||||
|
||||
Pour tout entier naturel $n$ on note $u_n$ le débit par habitant pour l'année $2018 + n$ et on admet que $u_n = \dfrac{d_n}{h_n}$.
|
||||
\begin{enumerate}
|
||||
\setcounter{enumi}{2}
|
||||
\item Calculer $u_0$ et $u_1$.
|
||||
\item Montrer pour tout entier naturel $n$ on a $u_n = 7 \times 0.98^n$.
|
||||
\item En déduire la nature de la suite $\left(u_n\right)$ et ses caractéristiques.
|
||||
\item Déterminer le sens de variations de la $\left(u_n\right)$.
|
||||
Interpréter ce résultat dans le contexte de l'énoncé.
|
||||
\end{enumerate}
|
||||
Le marché passé avec le fournisseur d'accès internet prévoit également que si le débit passe en dessous de 5 Mbit/s par habitant alors ce dernier doit changer la technologie utilisée pour la réalisation de son réseau.
|
||||
|
||||
\begin{enumerate}
|
||||
\setcounter{enumi}{6}
|
||||
\item En quelle année le fournisseur d'accès sera-t-il dans l'obligation de changer sa technologie?
|
||||
\end{enumerate}
|
||||
\end{exercise}
|
||||
\begin{solution}
|
||||
\begin{enumerate}
|
||||
\item Augmenter de 7\% revient à multiplier pas 1.07. La suite $(h_n)$ est donc géométrique de raison 1.07 et de premier terme 2500. On en déduit $h_n$ en fonction de $n$
|
||||
\[
|
||||
h_n = 2500\times 1.07^n
|
||||
\]
|
||||
\item Augmenter de - 4.8600\% revient à multiplier pas 1.0486. La suite $(d_n)$ est donc géométrique de raison 1.0486 et de premier terme 17500. On en déduit $d_n$ en fonction de $n$
|
||||
\[
|
||||
d_n = 17500\times 1.0486^n
|
||||
\]
|
||||
\item
|
||||
\begin{enumerate}
|
||||
\item
|
||||
\[
|
||||
u_0 = \frac{d_0}{h_0} = \frac{17500}{2500} = 7
|
||||
\]
|
||||
\[
|
||||
u_1 = \frac{d_1}{h_1} = \frac{18350.5000}{2675} = 6.86
|
||||
\]
|
||||
\item Démonstration de la formule
|
||||
\begin{eqnarray*}
|
||||
u_n &=& \frac{d_n}{h_n} = \frac{17500\times1.0486^n}{2500\times1.07^n} \\
|
||||
u_n &=& \frac{17500}{2500}\times\left(\frac{1.0486}{1.07}\right)^n \\
|
||||
u_n &=& 7\times0.98^n
|
||||
\end{eqnarray*}
|
||||
\item On reconnaît la forme d'une suite géométrique de raison 0.98 et de premier terme 7.
|
||||
\item La raison, $q = 0.98$, est inférieur à 1 donc la suite est décroissante. Ce qui signifie que le débit par habitant va diminuer.
|
||||
\end{enumerate}
|
||||
\item Avec le tableau de la calculatrice, on calculer les valeurs de $u_n$ jusqu'à passer en dessous de 5. On trouve $n = 17$ avec $u_{17} = 4.965252363264521426471716467$
|
||||
|
||||
\end{enumerate}
|
||||
\end{solution}
|
||||
\begin{exercise}[subtitle={Étude de fonctions}]
|
||||
\section*{Partie A}
|
||||
Dans cette partie, on étudie la fonction
|
||||
\[
|
||||
f(x) = - 7x^2 + 2x - 9
|
||||
\]
|
||||
\begin{enumerate}
|
||||
\item Calculer la dérivé de $f$.
|
||||
\item Étudier le signe de la dérivée $f'$ puis en déduire le tableau de signe de $f$.
|
||||
\end{enumerate}
|
||||
\section*{Partie B}
|
||||
Dans cette partie, on étudie la fonction
|
||||
\[
|
||||
g(x) = - 7x^3 - 2x^2 + 10x - 10
|
||||
\]
|
||||
\begin{enumerate}
|
||||
\item À l'aide d'une calculatrice ou d'un ordinateur, tracer puis reporter sur votre copie la représentation graphique de $g$ en y indiquant les informations remarquables de ce graphique.
|
||||
\item Sur quel(s) intervalle(s) la fonction est convexe? concave? Y a-t-il des points d'inflexions?
|
||||
\item Calculer la dérivé de $g$.
|
||||
\item Étudier le signe de la dérivée $g'$ puis en déduire le tableau de variations de $g$.
|
||||
\item Déterminer l'équation de la tangente en $x=0$.
|
||||
\item Dériver $g'$ pour calculer $g''$.
|
||||
\item Étudier le signe de $g''$ pour en déduire la convexité de $g$ grâce au calcul puis localiser précisément le point d'inflexion.
|
||||
\end{enumerate}
|
||||
\end{exercise}
|
||||
\begin{solution}
|
||||
\section*{Partie A}
|
||||
\begin{enumerate}
|
||||
\item $2 - 14x$
|
||||
\item Correction non disponible
|
||||
\end{enumerate}
|
||||
|
||||
\section*{Partie B}
|
||||
\begin{enumerate}
|
||||
\item Correction non disponible
|
||||
\item Correction non disponible
|
||||
\item $g'(x) = 10 - 4x - 21x^2$
|
||||
\item
|
||||
On commence par calculer le discriminant de $g'(x)=10 - 4x - 21x^2$.
|
||||
\begin{eqnarray*}
|
||||
\Delta & = & b^2-4ac \\
|
||||
\Delta & = & - 4^{2} - 4 \times - 21 \times 10 \\
|
||||
\Delta & = & 16 + 84 \times 10 \\
|
||||
\Delta & = & 16 + 840 \\
|
||||
\Delta & = & 856
|
||||
\end{eqnarray*}
|
||||
|
||||
comme $\Delta = 856 > 0$ donc $P$ a deux racines
|
||||
|
||||
\begin{eqnarray*}
|
||||
x_1 & = & \frac{-b - \sqrt{\Delta}}{2a} = \frac{- 4 - \sqrt{856}}{2 \times - 21} = - 0.7918447065870377 \\
|
||||
x_2 & = & \frac{-b + \sqrt{\Delta}}{2a} = \frac{- 4 + \sqrt{856}}{2 \times - 21} = 0.6013685161108473
|
||||
\end{eqnarray*}
|
||||
Ainsi, $g'$ est du signe de $a=- 21$ en dehors des racines.
|
||||
|
||||
|
||||
Le tableau de variation non disponible en correction
|
||||
\item Équation de la tangente: $y = 10x + - 10$
|
||||
\item $g''(x) = - 42x - 4$
|
||||
\end{enumerate}
|
||||
\end{solution}
|
||||
|
||||
\end{document}
|
||||
|
||||
%%% Local Variables:
|
||||
%%% mode: latex
|
||||
%%% TeX-master: "master"
|
||||
%%% End:
|
||||
157
TES/DM/DM_19_10/10_DM_19_10.tex
Normal file
157
TES/DM/DM_19_10/10_DM_19_10.tex
Normal file
@@ -0,0 +1,157 @@
|
||||
\documentclass[a4paper,10pt]{article}
|
||||
\usepackage{tasks}
|
||||
\usepackage{myXsim}
|
||||
|
||||
\title{DM 1 -- HALEGOI Agathe}
|
||||
\tribe{Terminale ES-L}
|
||||
\date{15 novembre 2019}
|
||||
|
||||
\xsimsetup{
|
||||
solution/print = false
|
||||
}
|
||||
|
||||
\begin{document}
|
||||
\maketitle
|
||||
|
||||
Une part importante de la note sera dédiée à la rédaction, aux explications et à l'utilisation des notations mathématiques.
|
||||
|
||||
\begin{exercise}[subtitle={Débit}]
|
||||
Une commune de \np{2200} habitants au 1\up{er} janvier 2018 voit sa population augmenter de 5\,\% tous les ans.
|
||||
|
||||
Pour tout entier naturel $n$, on note $h_n$ le nombre d'habitants de l'année $2018 + n$.
|
||||
|
||||
\smallskip
|
||||
|
||||
\begin{enumerate}
|
||||
\item Déterminer la nature de la suite $(h_n)$, préciser ses éléments caractéristiques et exprimer $h_n$ en fonction de $n$.
|
||||
\end{enumerate}
|
||||
|
||||
\smallskip
|
||||
|
||||
La municipalité de cette commune a conclu un marché avec un fournisseur d'accès internet qui engage ce dernier à fournir un débit total de \np{26400}~Mbit/s au 1\up{er} janvier 2018 et à augmenter ce débit de 2.3500\,\% par an.
|
||||
|
||||
Pour tout entier naturel $n$, on note $d_n$ le débit total dont la commune dispose l'année $2018 + n$.
|
||||
|
||||
\begin{enumerate}
|
||||
\setcounter{enumi}{1}
|
||||
\item Déterminer la nature de la suite $(d_n)$, préciser ses éléments caractéristiques et exprimer $d_n$ en fonction de $n$.
|
||||
\end{enumerate}
|
||||
|
||||
On s'intéresse maintenant au débit par habitant en supposant que celui-ci est réparti équitablement et que toute la population bénéficie d'une connexion internet individuelle.
|
||||
|
||||
Pour tout entier naturel $n$ on note $u_n$ le débit par habitant pour l'année $2018 + n$ et on admet que $u_n = \dfrac{d_n}{h_n}$.
|
||||
\begin{enumerate}
|
||||
\setcounter{enumi}{2}
|
||||
\item Calculer $u_0$ et $u_1$.
|
||||
\item Montrer pour tout entier naturel $n$ on a $u_n = 12 \times 0.93^n$.
|
||||
\item En déduire la nature de la suite $\left(u_n\right)$ et ses caractéristiques.
|
||||
\item Déterminer le sens de variations de la $\left(u_n\right)$.
|
||||
Interpréter ce résultat dans le contexte de l'énoncé.
|
||||
\end{enumerate}
|
||||
Le marché passé avec le fournisseur d'accès internet prévoit également que si le débit passe en dessous de 5 Mbit/s par habitant alors ce dernier doit changer la technologie utilisée pour la réalisation de son réseau.
|
||||
|
||||
\begin{enumerate}
|
||||
\setcounter{enumi}{6}
|
||||
\item En quelle année le fournisseur d'accès sera-t-il dans l'obligation de changer sa technologie?
|
||||
\end{enumerate}
|
||||
\end{exercise}
|
||||
\begin{solution}
|
||||
\begin{enumerate}
|
||||
\item Augmenter de 5\% revient à multiplier pas 1.05. La suite $(h_n)$ est donc géométrique de raison 1.05 et de premier terme 2200. On en déduit $h_n$ en fonction de $n$
|
||||
\[
|
||||
h_n = 2200\times 1.05^n
|
||||
\]
|
||||
\item Augmenter de 2.3500\% revient à multiplier pas 0.9765. La suite $(d_n)$ est donc géométrique de raison 0.9765 et de premier terme 26400. On en déduit $d_n$ en fonction de $n$
|
||||
\[
|
||||
d_n = 26400\times 0.9765^n
|
||||
\]
|
||||
\item
|
||||
\begin{enumerate}
|
||||
\item
|
||||
\[
|
||||
u_0 = \frac{d_0}{h_0} = \frac{26400}{2200} = 12
|
||||
\]
|
||||
\[
|
||||
u_1 = \frac{d_1}{h_1} = \frac{25779.6000}{2310} = 11.16
|
||||
\]
|
||||
\item Démonstration de la formule
|
||||
\begin{eqnarray*}
|
||||
u_n &=& \frac{d_n}{h_n} = \frac{26400\times0.9765^n}{2200\times1.05^n} \\
|
||||
u_n &=& \frac{26400}{2200}\times\left(\frac{0.9765}{1.05}\right)^n \\
|
||||
u_n &=& 12\times0.93^n
|
||||
\end{eqnarray*}
|
||||
\item On reconnaît la forme d'une suite géométrique de raison 0.93 et de premier terme 12.
|
||||
\item La raison, $q = 0.93$, est inférieur à 1 donc la suite est décroissante. Ce qui signifie que le débit par habitant va diminuer.
|
||||
\end{enumerate}
|
||||
\item Avec le tableau de la calculatrice, on calculer les valeurs de $u_n$ jusqu'à passer en dessous de 5. On trouve $n = 13$ avec $u_{13} = 4.67153467986977671665730716$
|
||||
|
||||
\end{enumerate}
|
||||
\end{solution}
|
||||
\begin{exercise}[subtitle={Étude de fonctions}]
|
||||
\section*{Partie A}
|
||||
Dans cette partie, on étudie la fonction
|
||||
\[
|
||||
f(x) = 7x^2 - 5x + 7
|
||||
\]
|
||||
\begin{enumerate}
|
||||
\item Calculer la dérivé de $f$.
|
||||
\item Étudier le signe de la dérivée $f'$ puis en déduire le tableau de signe de $f$.
|
||||
\end{enumerate}
|
||||
\section*{Partie B}
|
||||
Dans cette partie, on étudie la fonction
|
||||
\[
|
||||
g(x) = - 6x^3 + 5x^2 + 4x - 5
|
||||
\]
|
||||
\begin{enumerate}
|
||||
\item À l'aide d'une calculatrice ou d'un ordinateur, tracer puis reporter sur votre copie la représentation graphique de $g$ en y indiquant les informations remarquables de ce graphique.
|
||||
\item Sur quel(s) intervalle(s) la fonction est convexe? concave? Y a-t-il des points d'inflexions?
|
||||
\item Calculer la dérivé de $g$.
|
||||
\item Étudier le signe de la dérivée $g'$ puis en déduire le tableau de variations de $g$.
|
||||
\item Déterminer l'équation de la tangente en $x=0$.
|
||||
\item Dériver $g'$ pour calculer $g''$.
|
||||
\item Étudier le signe de $g''$ pour en déduire la convexité de $g$ grâce au calcul puis localiser précisément le point d'inflexion.
|
||||
\end{enumerate}
|
||||
\end{exercise}
|
||||
\begin{solution}
|
||||
\section*{Partie A}
|
||||
\begin{enumerate}
|
||||
\item $- 5 + 14x$
|
||||
\item Correction non disponible
|
||||
\end{enumerate}
|
||||
|
||||
\section*{Partie B}
|
||||
\begin{enumerate}
|
||||
\item Correction non disponible
|
||||
\item Correction non disponible
|
||||
\item $g'(x) = 4 + 10x - 18x^2$
|
||||
\item
|
||||
On commence par calculer le discriminant de $g'(x)=4 + 10x - 18x^2$.
|
||||
\begin{eqnarray*}
|
||||
\Delta & = & b^2-4ac \\
|
||||
\Delta & = & 10^{2} - 4 \times - 18 \times 4 \\
|
||||
\Delta & = & 100 + 72 \times 4 \\
|
||||
\Delta & = & 100 + 288 \\
|
||||
\Delta & = & 388
|
||||
\end{eqnarray*}
|
||||
|
||||
comme $\Delta = 388 > 0$ donc $P$ a deux racines
|
||||
|
||||
\begin{eqnarray*}
|
||||
x_1 & = & \frac{-b - \sqrt{\Delta}}{2a} = \frac{10 - \sqrt{388}}{2 \times - 18} = - 0.26938098898867247 \\
|
||||
x_2 & = & \frac{-b + \sqrt{\Delta}}{2a} = \frac{10 + \sqrt{388}}{2 \times - 18} = 0.824936544544228
|
||||
\end{eqnarray*}
|
||||
Ainsi, $g'$ est du signe de $a=- 18$ en dehors des racines.
|
||||
|
||||
|
||||
Le tableau de variation non disponible en correction
|
||||
\item Équation de la tangente: $y = 4x + - 5$
|
||||
\item $g''(x) = - 36x + 10$
|
||||
\end{enumerate}
|
||||
\end{solution}
|
||||
|
||||
\end{document}
|
||||
|
||||
%%% Local Variables:
|
||||
%%% mode: latex
|
||||
%%% TeX-master: "master"
|
||||
%%% End:
|
||||
157
TES/DM/DM_19_10/11_DM_19_10.tex
Normal file
157
TES/DM/DM_19_10/11_DM_19_10.tex
Normal file
@@ -0,0 +1,157 @@
|
||||
\documentclass[a4paper,10pt]{article}
|
||||
\usepackage{tasks}
|
||||
\usepackage{myXsim}
|
||||
|
||||
\title{DM 1 -- JOURDAN Alice}
|
||||
\tribe{Terminale ES-L}
|
||||
\date{15 novembre 2019}
|
||||
|
||||
\xsimsetup{
|
||||
solution/print = false
|
||||
}
|
||||
|
||||
\begin{document}
|
||||
\maketitle
|
||||
|
||||
Une part importante de la note sera dédiée à la rédaction, aux explications et à l'utilisation des notations mathématiques.
|
||||
|
||||
\begin{exercise}[subtitle={Débit}]
|
||||
Une commune de \np{1500} habitants au 1\up{er} janvier 2018 voit sa population augmenter de 8\,\% tous les ans.
|
||||
|
||||
Pour tout entier naturel $n$, on note $h_n$ le nombre d'habitants de l'année $2018 + n$.
|
||||
|
||||
\smallskip
|
||||
|
||||
\begin{enumerate}
|
||||
\item Déterminer la nature de la suite $(h_n)$, préciser ses éléments caractéristiques et exprimer $h_n$ en fonction de $n$.
|
||||
\end{enumerate}
|
||||
|
||||
\smallskip
|
||||
|
||||
La municipalité de cette commune a conclu un marché avec un fournisseur d'accès internet qui engage ce dernier à fournir un débit total de \np{15000}~Mbit/s au 1\up{er} janvier 2018 et à augmenter ce débit de - 6.9200\,\% par an.
|
||||
|
||||
Pour tout entier naturel $n$, on note $d_n$ le débit total dont la commune dispose l'année $2018 + n$.
|
||||
|
||||
\begin{enumerate}
|
||||
\setcounter{enumi}{1}
|
||||
\item Déterminer la nature de la suite $(d_n)$, préciser ses éléments caractéristiques et exprimer $d_n$ en fonction de $n$.
|
||||
\end{enumerate}
|
||||
|
||||
On s'intéresse maintenant au débit par habitant en supposant que celui-ci est réparti équitablement et que toute la population bénéficie d'une connexion internet individuelle.
|
||||
|
||||
Pour tout entier naturel $n$ on note $u_n$ le débit par habitant pour l'année $2018 + n$ et on admet que $u_n = \dfrac{d_n}{h_n}$.
|
||||
\begin{enumerate}
|
||||
\setcounter{enumi}{2}
|
||||
\item Calculer $u_0$ et $u_1$.
|
||||
\item Montrer pour tout entier naturel $n$ on a $u_n = 10 \times 0.99^n$.
|
||||
\item En déduire la nature de la suite $\left(u_n\right)$ et ses caractéristiques.
|
||||
\item Déterminer le sens de variations de la $\left(u_n\right)$.
|
||||
Interpréter ce résultat dans le contexte de l'énoncé.
|
||||
\end{enumerate}
|
||||
Le marché passé avec le fournisseur d'accès internet prévoit également que si le débit passe en dessous de 5 Mbit/s par habitant alors ce dernier doit changer la technologie utilisée pour la réalisation de son réseau.
|
||||
|
||||
\begin{enumerate}
|
||||
\setcounter{enumi}{6}
|
||||
\item En quelle année le fournisseur d'accès sera-t-il dans l'obligation de changer sa technologie?
|
||||
\end{enumerate}
|
||||
\end{exercise}
|
||||
\begin{solution}
|
||||
\begin{enumerate}
|
||||
\item Augmenter de 8\% revient à multiplier pas 1.08. La suite $(h_n)$ est donc géométrique de raison 1.08 et de premier terme 1500. On en déduit $h_n$ en fonction de $n$
|
||||
\[
|
||||
h_n = 1500\times 1.08^n
|
||||
\]
|
||||
\item Augmenter de - 6.9200\% revient à multiplier pas 1.0692. La suite $(d_n)$ est donc géométrique de raison 1.0692 et de premier terme 15000. On en déduit $d_n$ en fonction de $n$
|
||||
\[
|
||||
d_n = 15000\times 1.0692^n
|
||||
\]
|
||||
\item
|
||||
\begin{enumerate}
|
||||
\item
|
||||
\[
|
||||
u_0 = \frac{d_0}{h_0} = \frac{15000}{1500} = 10
|
||||
\]
|
||||
\[
|
||||
u_1 = \frac{d_1}{h_1} = \frac{16038}{1620} = 9.90
|
||||
\]
|
||||
\item Démonstration de la formule
|
||||
\begin{eqnarray*}
|
||||
u_n &=& \frac{d_n}{h_n} = \frac{15000\times1.0692^n}{1500\times1.08^n} \\
|
||||
u_n &=& \frac{15000}{1500}\times\left(\frac{1.0692}{1.08}\right)^n \\
|
||||
u_n &=& 10\times0.99^n
|
||||
\end{eqnarray*}
|
||||
\item On reconnaît la forme d'une suite géométrique de raison 0.99 et de premier terme 10.
|
||||
\item La raison, $q = 0.99$, est inférieur à 1 donc la suite est décroissante. Ce qui signifie que le débit par habitant va diminuer.
|
||||
\end{enumerate}
|
||||
\item Avec le tableau de la calculatrice, on calculer les valeurs de $u_n$ jusqu'à passer en dessous de 5. On trouve $n = 69$ avec $u_{69} = 4.998370298991992523867655626$
|
||||
|
||||
\end{enumerate}
|
||||
\end{solution}
|
||||
\begin{exercise}[subtitle={Étude de fonctions}]
|
||||
\section*{Partie A}
|
||||
Dans cette partie, on étudie la fonction
|
||||
\[
|
||||
f(x) = - 6x^2 - 7x + 4
|
||||
\]
|
||||
\begin{enumerate}
|
||||
\item Calculer la dérivé de $f$.
|
||||
\item Étudier le signe de la dérivée $f'$ puis en déduire le tableau de signe de $f$.
|
||||
\end{enumerate}
|
||||
\section*{Partie B}
|
||||
Dans cette partie, on étudie la fonction
|
||||
\[
|
||||
g(x) = 2x^3 - 8x^2 + 2x - 10
|
||||
\]
|
||||
\begin{enumerate}
|
||||
\item À l'aide d'une calculatrice ou d'un ordinateur, tracer puis reporter sur votre copie la représentation graphique de $g$ en y indiquant les informations remarquables de ce graphique.
|
||||
\item Sur quel(s) intervalle(s) la fonction est convexe? concave? Y a-t-il des points d'inflexions?
|
||||
\item Calculer la dérivé de $g$.
|
||||
\item Étudier le signe de la dérivée $g'$ puis en déduire le tableau de variations de $g$.
|
||||
\item Déterminer l'équation de la tangente en $x=0$.
|
||||
\item Dériver $g'$ pour calculer $g''$.
|
||||
\item Étudier le signe de $g''$ pour en déduire la convexité de $g$ grâce au calcul puis localiser précisément le point d'inflexion.
|
||||
\end{enumerate}
|
||||
\end{exercise}
|
||||
\begin{solution}
|
||||
\section*{Partie A}
|
||||
\begin{enumerate}
|
||||
\item $- 7 - 12x$
|
||||
\item Correction non disponible
|
||||
\end{enumerate}
|
||||
|
||||
\section*{Partie B}
|
||||
\begin{enumerate}
|
||||
\item Correction non disponible
|
||||
\item Correction non disponible
|
||||
\item $g'(x) = 2 - 16x + 6x^2$
|
||||
\item
|
||||
On commence par calculer le discriminant de $g'(x)=2 - 16x + 6x^2$.
|
||||
\begin{eqnarray*}
|
||||
\Delta & = & b^2-4ac \\
|
||||
\Delta & = & - 16^{2} - 4 \times 6 \times 2 \\
|
||||
\Delta & = & 256 - 24 \times 2 \\
|
||||
\Delta & = & 256 - 48 \\
|
||||
\Delta & = & 208
|
||||
\end{eqnarray*}
|
||||
|
||||
comme $\Delta = 208 > 0$ donc $P$ a deux racines
|
||||
|
||||
\begin{eqnarray*}
|
||||
x_1 & = & \frac{-b - \sqrt{\Delta}}{2a} = \frac{- 16 - \sqrt{208}}{2 \times 6} = 0.13148290817867028 \\
|
||||
x_2 & = & \frac{-b + \sqrt{\Delta}}{2a} = \frac{- 16 + \sqrt{208}}{2 \times 6} = 2.5351837584879964
|
||||
\end{eqnarray*}
|
||||
Ainsi, $g'$ est du signe de $a=6$ en dehors des racines.
|
||||
|
||||
|
||||
Le tableau de variation non disponible en correction
|
||||
\item Équation de la tangente: $y = 2x + - 10$
|
||||
\item $g''(x) = 12x - 16$
|
||||
\end{enumerate}
|
||||
\end{solution}
|
||||
|
||||
\end{document}
|
||||
|
||||
%%% Local Variables:
|
||||
%%% mode: latex
|
||||
%%% TeX-master: "master"
|
||||
%%% End:
|
||||
157
TES/DM/DM_19_10/12_DM_19_10.tex
Normal file
157
TES/DM/DM_19_10/12_DM_19_10.tex
Normal file
@@ -0,0 +1,157 @@
|
||||
\documentclass[a4paper,10pt]{article}
|
||||
\usepackage{tasks}
|
||||
\usepackage{myXsim}
|
||||
|
||||
\title{DM 1 -- LIANDRAT Léa}
|
||||
\tribe{Terminale ES-L}
|
||||
\date{15 novembre 2019}
|
||||
|
||||
\xsimsetup{
|
||||
solution/print = false
|
||||
}
|
||||
|
||||
\begin{document}
|
||||
\maketitle
|
||||
|
||||
Une part importante de la note sera dédiée à la rédaction, aux explications et à l'utilisation des notations mathématiques.
|
||||
|
||||
\begin{exercise}[subtitle={Débit}]
|
||||
Une commune de \np{2400} habitants au 1\up{er} janvier 2018 voit sa population augmenter de 7\,\% tous les ans.
|
||||
|
||||
Pour tout entier naturel $n$, on note $h_n$ le nombre d'habitants de l'année $2018 + n$.
|
||||
|
||||
\smallskip
|
||||
|
||||
\begin{enumerate}
|
||||
\item Déterminer la nature de la suite $(h_n)$, préciser ses éléments caractéristiques et exprimer $h_n$ en fonction de $n$.
|
||||
\end{enumerate}
|
||||
|
||||
\smallskip
|
||||
|
||||
La municipalité de cette commune a conclu un marché avec un fournisseur d'accès internet qui engage ce dernier à fournir un débit total de \np{21600}~Mbit/s au 1\up{er} janvier 2018 et à augmenter ce débit de - 2.7200\,\% par an.
|
||||
|
||||
Pour tout entier naturel $n$, on note $d_n$ le débit total dont la commune dispose l'année $2018 + n$.
|
||||
|
||||
\begin{enumerate}
|
||||
\setcounter{enumi}{1}
|
||||
\item Déterminer la nature de la suite $(d_n)$, préciser ses éléments caractéristiques et exprimer $d_n$ en fonction de $n$.
|
||||
\end{enumerate}
|
||||
|
||||
On s'intéresse maintenant au débit par habitant en supposant que celui-ci est réparti équitablement et que toute la population bénéficie d'une connexion internet individuelle.
|
||||
|
||||
Pour tout entier naturel $n$ on note $u_n$ le débit par habitant pour l'année $2018 + n$ et on admet que $u_n = \dfrac{d_n}{h_n}$.
|
||||
\begin{enumerate}
|
||||
\setcounter{enumi}{2}
|
||||
\item Calculer $u_0$ et $u_1$.
|
||||
\item Montrer pour tout entier naturel $n$ on a $u_n = 9 \times 0.96^n$.
|
||||
\item En déduire la nature de la suite $\left(u_n\right)$ et ses caractéristiques.
|
||||
\item Déterminer le sens de variations de la $\left(u_n\right)$.
|
||||
Interpréter ce résultat dans le contexte de l'énoncé.
|
||||
\end{enumerate}
|
||||
Le marché passé avec le fournisseur d'accès internet prévoit également que si le débit passe en dessous de 5 Mbit/s par habitant alors ce dernier doit changer la technologie utilisée pour la réalisation de son réseau.
|
||||
|
||||
\begin{enumerate}
|
||||
\setcounter{enumi}{6}
|
||||
\item En quelle année le fournisseur d'accès sera-t-il dans l'obligation de changer sa technologie?
|
||||
\end{enumerate}
|
||||
\end{exercise}
|
||||
\begin{solution}
|
||||
\begin{enumerate}
|
||||
\item Augmenter de 7\% revient à multiplier pas 1.07. La suite $(h_n)$ est donc géométrique de raison 1.07 et de premier terme 2400. On en déduit $h_n$ en fonction de $n$
|
||||
\[
|
||||
h_n = 2400\times 1.07^n
|
||||
\]
|
||||
\item Augmenter de - 2.7200\% revient à multiplier pas 1.0272. La suite $(d_n)$ est donc géométrique de raison 1.0272 et de premier terme 21600. On en déduit $d_n$ en fonction de $n$
|
||||
\[
|
||||
d_n = 21600\times 1.0272^n
|
||||
\]
|
||||
\item
|
||||
\begin{enumerate}
|
||||
\item
|
||||
\[
|
||||
u_0 = \frac{d_0}{h_0} = \frac{21600}{2400} = 9
|
||||
\]
|
||||
\[
|
||||
u_1 = \frac{d_1}{h_1} = \frac{22187.5200}{2568} = 8.64
|
||||
\]
|
||||
\item Démonstration de la formule
|
||||
\begin{eqnarray*}
|
||||
u_n &=& \frac{d_n}{h_n} = \frac{21600\times1.0272^n}{2400\times1.07^n} \\
|
||||
u_n &=& \frac{21600}{2400}\times\left(\frac{1.0272}{1.07}\right)^n \\
|
||||
u_n &=& 9\times0.96^n
|
||||
\end{eqnarray*}
|
||||
\item On reconnaît la forme d'une suite géométrique de raison 0.96 et de premier terme 9.
|
||||
\item La raison, $q = 0.96$, est inférieur à 1 donc la suite est décroissante. Ce qui signifie que le débit par habitant va diminuer.
|
||||
\end{enumerate}
|
||||
\item Avec le tableau de la calculatrice, on calculer les valeurs de $u_n$ jusqu'à passer en dessous de 5. On trouve $n = 15$ avec $u_{15} = 4.878777418748181525190970180$
|
||||
|
||||
\end{enumerate}
|
||||
\end{solution}
|
||||
\begin{exercise}[subtitle={Étude de fonctions}]
|
||||
\section*{Partie A}
|
||||
Dans cette partie, on étudie la fonction
|
||||
\[
|
||||
f(x) = 4x^2 - 4x - 8
|
||||
\]
|
||||
\begin{enumerate}
|
||||
\item Calculer la dérivé de $f$.
|
||||
\item Étudier le signe de la dérivée $f'$ puis en déduire le tableau de signe de $f$.
|
||||
\end{enumerate}
|
||||
\section*{Partie B}
|
||||
Dans cette partie, on étudie la fonction
|
||||
\[
|
||||
g(x) = - x^3 - 4x^2 - x + 2
|
||||
\]
|
||||
\begin{enumerate}
|
||||
\item À l'aide d'une calculatrice ou d'un ordinateur, tracer puis reporter sur votre copie la représentation graphique de $g$ en y indiquant les informations remarquables de ce graphique.
|
||||
\item Sur quel(s) intervalle(s) la fonction est convexe? concave? Y a-t-il des points d'inflexions?
|
||||
\item Calculer la dérivé de $g$.
|
||||
\item Étudier le signe de la dérivée $g'$ puis en déduire le tableau de variations de $g$.
|
||||
\item Déterminer l'équation de la tangente en $x=0$.
|
||||
\item Dériver $g'$ pour calculer $g''$.
|
||||
\item Étudier le signe de $g''$ pour en déduire la convexité de $g$ grâce au calcul puis localiser précisément le point d'inflexion.
|
||||
\end{enumerate}
|
||||
\end{exercise}
|
||||
\begin{solution}
|
||||
\section*{Partie A}
|
||||
\begin{enumerate}
|
||||
\item $- 4 + 8x$
|
||||
\item Correction non disponible
|
||||
\end{enumerate}
|
||||
|
||||
\section*{Partie B}
|
||||
\begin{enumerate}
|
||||
\item Correction non disponible
|
||||
\item Correction non disponible
|
||||
\item $g'(x) = - 1 - 8x - 3x^2$
|
||||
\item
|
||||
On commence par calculer le discriminant de $g'(x)=- 1 - 8x - 3x^2$.
|
||||
\begin{eqnarray*}
|
||||
\Delta & = & b^2-4ac \\
|
||||
\Delta & = & - 8^{2} - 4 \times - 3 \times - 1 \\
|
||||
\Delta & = & 64 + 12 \times - 1 \\
|
||||
\Delta & = & 64 - 12 \\
|
||||
\Delta & = & 52
|
||||
\end{eqnarray*}
|
||||
|
||||
comme $\Delta = 52 > 0$ donc $P$ a deux racines
|
||||
|
||||
\begin{eqnarray*}
|
||||
x_1 & = & \frac{-b - \sqrt{\Delta}}{2a} = \frac{- 8 - \sqrt{52}}{2 \times - 3} = - 0.13148290817867028 \\
|
||||
x_2 & = & \frac{-b + \sqrt{\Delta}}{2a} = \frac{- 8 + \sqrt{52}}{2 \times - 3} = - 2.5351837584879964
|
||||
\end{eqnarray*}
|
||||
Ainsi, $g'$ est du signe de $a=- 3$ en dehors des racines.
|
||||
|
||||
|
||||
Le tableau de variation non disponible en correction
|
||||
\item Équation de la tangente: $y = - 1x + 2$
|
||||
\item $g''(x) = - 6x - 8$
|
||||
\end{enumerate}
|
||||
\end{solution}
|
||||
|
||||
\end{document}
|
||||
|
||||
%%% Local Variables:
|
||||
%%% mode: latex
|
||||
%%% TeX-master: "master"
|
||||
%%% End:
|
||||
157
TES/DM/DM_19_10/13_DM_19_10.tex
Normal file
157
TES/DM/DM_19_10/13_DM_19_10.tex
Normal file
@@ -0,0 +1,157 @@
|
||||
\documentclass[a4paper,10pt]{article}
|
||||
\usepackage{tasks}
|
||||
\usepackage{myXsim}
|
||||
|
||||
\title{DM 1 -- LOULID Manar}
|
||||
\tribe{Terminale ES-L}
|
||||
\date{15 novembre 2019}
|
||||
|
||||
\xsimsetup{
|
||||
solution/print = false
|
||||
}
|
||||
|
||||
\begin{document}
|
||||
\maketitle
|
||||
|
||||
Une part importante de la note sera dédiée à la rédaction, aux explications et à l'utilisation des notations mathématiques.
|
||||
|
||||
\begin{exercise}[subtitle={Débit}]
|
||||
Une commune de \np{1600} habitants au 1\up{er} janvier 2018 voit sa population augmenter de 7\,\% tous les ans.
|
||||
|
||||
Pour tout entier naturel $n$, on note $h_n$ le nombre d'habitants de l'année $2018 + n$.
|
||||
|
||||
\smallskip
|
||||
|
||||
\begin{enumerate}
|
||||
\item Déterminer la nature de la suite $(h_n)$, préciser ses éléments caractéristiques et exprimer $h_n$ en fonction de $n$.
|
||||
\end{enumerate}
|
||||
|
||||
\smallskip
|
||||
|
||||
La municipalité de cette commune a conclu un marché avec un fournisseur d'accès internet qui engage ce dernier à fournir un débit total de \np{12800}~Mbit/s au 1\up{er} janvier 2018 et à augmenter ce débit de 0.4900\,\% par an.
|
||||
|
||||
Pour tout entier naturel $n$, on note $d_n$ le débit total dont la commune dispose l'année $2018 + n$.
|
||||
|
||||
\begin{enumerate}
|
||||
\setcounter{enumi}{1}
|
||||
\item Déterminer la nature de la suite $(d_n)$, préciser ses éléments caractéristiques et exprimer $d_n$ en fonction de $n$.
|
||||
\end{enumerate}
|
||||
|
||||
On s'intéresse maintenant au débit par habitant en supposant que celui-ci est réparti équitablement et que toute la population bénéficie d'une connexion internet individuelle.
|
||||
|
||||
Pour tout entier naturel $n$ on note $u_n$ le débit par habitant pour l'année $2018 + n$ et on admet que $u_n = \dfrac{d_n}{h_n}$.
|
||||
\begin{enumerate}
|
||||
\setcounter{enumi}{2}
|
||||
\item Calculer $u_0$ et $u_1$.
|
||||
\item Montrer pour tout entier naturel $n$ on a $u_n = 8 \times 0.93^n$.
|
||||
\item En déduire la nature de la suite $\left(u_n\right)$ et ses caractéristiques.
|
||||
\item Déterminer le sens de variations de la $\left(u_n\right)$.
|
||||
Interpréter ce résultat dans le contexte de l'énoncé.
|
||||
\end{enumerate}
|
||||
Le marché passé avec le fournisseur d'accès internet prévoit également que si le débit passe en dessous de 5 Mbit/s par habitant alors ce dernier doit changer la technologie utilisée pour la réalisation de son réseau.
|
||||
|
||||
\begin{enumerate}
|
||||
\setcounter{enumi}{6}
|
||||
\item En quelle année le fournisseur d'accès sera-t-il dans l'obligation de changer sa technologie?
|
||||
\end{enumerate}
|
||||
\end{exercise}
|
||||
\begin{solution}
|
||||
\begin{enumerate}
|
||||
\item Augmenter de 7\% revient à multiplier pas 1.07. La suite $(h_n)$ est donc géométrique de raison 1.07 et de premier terme 1600. On en déduit $h_n$ en fonction de $n$
|
||||
\[
|
||||
h_n = 1600\times 1.07^n
|
||||
\]
|
||||
\item Augmenter de 0.4900\% revient à multiplier pas 0.9951. La suite $(d_n)$ est donc géométrique de raison 0.9951 et de premier terme 12800. On en déduit $d_n$ en fonction de $n$
|
||||
\[
|
||||
d_n = 12800\times 0.9951^n
|
||||
\]
|
||||
\item
|
||||
\begin{enumerate}
|
||||
\item
|
||||
\[
|
||||
u_0 = \frac{d_0}{h_0} = \frac{12800}{1600} = 8
|
||||
\]
|
||||
\[
|
||||
u_1 = \frac{d_1}{h_1} = \frac{12737.2800}{1712} = 7.44
|
||||
\]
|
||||
\item Démonstration de la formule
|
||||
\begin{eqnarray*}
|
||||
u_n &=& \frac{d_n}{h_n} = \frac{12800\times0.9951^n}{1600\times1.07^n} \\
|
||||
u_n &=& \frac{12800}{1600}\times\left(\frac{0.9951}{1.07}\right)^n \\
|
||||
u_n &=& 8\times0.93^n
|
||||
\end{eqnarray*}
|
||||
\item On reconnaît la forme d'une suite géométrique de raison 0.93 et de premier terme 8.
|
||||
\item La raison, $q = 0.93$, est inférieur à 1 donc la suite est décroissante. Ce qui signifie que le débit par habitant va diminuer.
|
||||
\end{enumerate}
|
||||
\item Avec le tableau de la calculatrice, on calculer les valeurs de $u_n$ jusqu'à passer en dessous de 5. On trouve $n = 7$ avec $u_{7} = 4.81360696486056$
|
||||
|
||||
\end{enumerate}
|
||||
\end{solution}
|
||||
\begin{exercise}[subtitle={Étude de fonctions}]
|
||||
\section*{Partie A}
|
||||
Dans cette partie, on étudie la fonction
|
||||
\[
|
||||
f(x) = - 7x^2 + 6x - 5
|
||||
\]
|
||||
\begin{enumerate}
|
||||
\item Calculer la dérivé de $f$.
|
||||
\item Étudier le signe de la dérivée $f'$ puis en déduire le tableau de signe de $f$.
|
||||
\end{enumerate}
|
||||
\section*{Partie B}
|
||||
Dans cette partie, on étudie la fonction
|
||||
\[
|
||||
g(x) = 3x^3 - 9x^2 + 7x - 9
|
||||
\]
|
||||
\begin{enumerate}
|
||||
\item À l'aide d'une calculatrice ou d'un ordinateur, tracer puis reporter sur votre copie la représentation graphique de $g$ en y indiquant les informations remarquables de ce graphique.
|
||||
\item Sur quel(s) intervalle(s) la fonction est convexe? concave? Y a-t-il des points d'inflexions?
|
||||
\item Calculer la dérivé de $g$.
|
||||
\item Étudier le signe de la dérivée $g'$ puis en déduire le tableau de variations de $g$.
|
||||
\item Déterminer l'équation de la tangente en $x=0$.
|
||||
\item Dériver $g'$ pour calculer $g''$.
|
||||
\item Étudier le signe de $g''$ pour en déduire la convexité de $g$ grâce au calcul puis localiser précisément le point d'inflexion.
|
||||
\end{enumerate}
|
||||
\end{exercise}
|
||||
\begin{solution}
|
||||
\section*{Partie A}
|
||||
\begin{enumerate}
|
||||
\item $6 - 14x$
|
||||
\item Correction non disponible
|
||||
\end{enumerate}
|
||||
|
||||
\section*{Partie B}
|
||||
\begin{enumerate}
|
||||
\item Correction non disponible
|
||||
\item Correction non disponible
|
||||
\item $g'(x) = 7 - 18x + 9x^2$
|
||||
\item
|
||||
On commence par calculer le discriminant de $g'(x)=7 - 18x + 9x^2$.
|
||||
\begin{eqnarray*}
|
||||
\Delta & = & b^2-4ac \\
|
||||
\Delta & = & - 18^{2} - 4 \times 9 \times 7 \\
|
||||
\Delta & = & 324 - 36 \times 7 \\
|
||||
\Delta & = & 324 - 252 \\
|
||||
\Delta & = & 72
|
||||
\end{eqnarray*}
|
||||
|
||||
comme $\Delta = 72 > 0$ donc $P$ a deux racines
|
||||
|
||||
\begin{eqnarray*}
|
||||
x_1 & = & \frac{-b - \sqrt{\Delta}}{2a} = \frac{- 18 - \sqrt{72}}{2 \times 9} = 0.5285954792089683 \\
|
||||
x_2 & = & \frac{-b + \sqrt{\Delta}}{2a} = \frac{- 18 + \sqrt{72}}{2 \times 9} = 1.4714045207910316
|
||||
\end{eqnarray*}
|
||||
Ainsi, $g'$ est du signe de $a=9$ en dehors des racines.
|
||||
|
||||
|
||||
Le tableau de variation non disponible en correction
|
||||
\item Équation de la tangente: $y = 7x + - 9$
|
||||
\item $g''(x) = 18x - 18$
|
||||
\end{enumerate}
|
||||
\end{solution}
|
||||
|
||||
\end{document}
|
||||
|
||||
%%% Local Variables:
|
||||
%%% mode: latex
|
||||
%%% TeX-master: "master"
|
||||
%%% End:
|
||||
157
TES/DM/DM_19_10/14_DM_19_10.tex
Normal file
157
TES/DM/DM_19_10/14_DM_19_10.tex
Normal file
@@ -0,0 +1,157 @@
|
||||
\documentclass[a4paper,10pt]{article}
|
||||
\usepackage{tasks}
|
||||
\usepackage{myXsim}
|
||||
|
||||
\title{DM 1 -- MARQUET Elisa}
|
||||
\tribe{Terminale ES-L}
|
||||
\date{15 novembre 2019}
|
||||
|
||||
\xsimsetup{
|
||||
solution/print = false
|
||||
}
|
||||
|
||||
\begin{document}
|
||||
\maketitle
|
||||
|
||||
Une part importante de la note sera dédiée à la rédaction, aux explications et à l'utilisation des notations mathématiques.
|
||||
|
||||
\begin{exercise}[subtitle={Débit}]
|
||||
Une commune de \np{2000} habitants au 1\up{er} janvier 2018 voit sa population augmenter de 9\,\% tous les ans.
|
||||
|
||||
Pour tout entier naturel $n$, on note $h_n$ le nombre d'habitants de l'année $2018 + n$.
|
||||
|
||||
\smallskip
|
||||
|
||||
\begin{enumerate}
|
||||
\item Déterminer la nature de la suite $(h_n)$, préciser ses éléments caractéristiques et exprimer $h_n$ en fonction de $n$.
|
||||
\end{enumerate}
|
||||
|
||||
\smallskip
|
||||
|
||||
La municipalité de cette commune a conclu un marché avec un fournisseur d'accès internet qui engage ce dernier à fournir un débit total de \np{18000}~Mbit/s au 1\up{er} janvier 2018 et à augmenter ce débit de - 1.3700\,\% par an.
|
||||
|
||||
Pour tout entier naturel $n$, on note $d_n$ le débit total dont la commune dispose l'année $2018 + n$.
|
||||
|
||||
\begin{enumerate}
|
||||
\setcounter{enumi}{1}
|
||||
\item Déterminer la nature de la suite $(d_n)$, préciser ses éléments caractéristiques et exprimer $d_n$ en fonction de $n$.
|
||||
\end{enumerate}
|
||||
|
||||
On s'intéresse maintenant au débit par habitant en supposant que celui-ci est réparti équitablement et que toute la population bénéficie d'une connexion internet individuelle.
|
||||
|
||||
Pour tout entier naturel $n$ on note $u_n$ le débit par habitant pour l'année $2018 + n$ et on admet que $u_n = \dfrac{d_n}{h_n}$.
|
||||
\begin{enumerate}
|
||||
\setcounter{enumi}{2}
|
||||
\item Calculer $u_0$ et $u_1$.
|
||||
\item Montrer pour tout entier naturel $n$ on a $u_n = 9 \times 0.93^n$.
|
||||
\item En déduire la nature de la suite $\left(u_n\right)$ et ses caractéristiques.
|
||||
\item Déterminer le sens de variations de la $\left(u_n\right)$.
|
||||
Interpréter ce résultat dans le contexte de l'énoncé.
|
||||
\end{enumerate}
|
||||
Le marché passé avec le fournisseur d'accès internet prévoit également que si le débit passe en dessous de 5 Mbit/s par habitant alors ce dernier doit changer la technologie utilisée pour la réalisation de son réseau.
|
||||
|
||||
\begin{enumerate}
|
||||
\setcounter{enumi}{6}
|
||||
\item En quelle année le fournisseur d'accès sera-t-il dans l'obligation de changer sa technologie?
|
||||
\end{enumerate}
|
||||
\end{exercise}
|
||||
\begin{solution}
|
||||
\begin{enumerate}
|
||||
\item Augmenter de 9\% revient à multiplier pas 1.09. La suite $(h_n)$ est donc géométrique de raison 1.09 et de premier terme 2000. On en déduit $h_n$ en fonction de $n$
|
||||
\[
|
||||
h_n = 2000\times 1.09^n
|
||||
\]
|
||||
\item Augmenter de - 1.3700\% revient à multiplier pas 1.0137. La suite $(d_n)$ est donc géométrique de raison 1.0137 et de premier terme 18000. On en déduit $d_n$ en fonction de $n$
|
||||
\[
|
||||
d_n = 18000\times 1.0137^n
|
||||
\]
|
||||
\item
|
||||
\begin{enumerate}
|
||||
\item
|
||||
\[
|
||||
u_0 = \frac{d_0}{h_0} = \frac{18000}{2000} = 9
|
||||
\]
|
||||
\[
|
||||
u_1 = \frac{d_1}{h_1} = \frac{18246.6000}{2180} = 8.37
|
||||
\]
|
||||
\item Démonstration de la formule
|
||||
\begin{eqnarray*}
|
||||
u_n &=& \frac{d_n}{h_n} = \frac{18000\times1.0137^n}{2000\times1.09^n} \\
|
||||
u_n &=& \frac{18000}{2000}\times\left(\frac{1.0137}{1.09}\right)^n \\
|
||||
u_n &=& 9\times0.93^n
|
||||
\end{eqnarray*}
|
||||
\item On reconnaît la forme d'une suite géométrique de raison 0.93 et de premier terme 9.
|
||||
\item La raison, $q = 0.93$, est inférieur à 1 donc la suite est décroissante. Ce qui signifie que le débit par habitant va diminuer.
|
||||
\end{enumerate}
|
||||
\item Avec le tableau de la calculatrice, on calculer les valeurs de $u_n$ jusqu'à passer en dessous de 5. On trouve $n = 9$ avec $u_{9} = 4.683699746896385637$
|
||||
|
||||
\end{enumerate}
|
||||
\end{solution}
|
||||
\begin{exercise}[subtitle={Étude de fonctions}]
|
||||
\section*{Partie A}
|
||||
Dans cette partie, on étudie la fonction
|
||||
\[
|
||||
f(x) = - 8x^2 - 9x - 1
|
||||
\]
|
||||
\begin{enumerate}
|
||||
\item Calculer la dérivé de $f$.
|
||||
\item Étudier le signe de la dérivée $f'$ puis en déduire le tableau de signe de $f$.
|
||||
\end{enumerate}
|
||||
\section*{Partie B}
|
||||
Dans cette partie, on étudie la fonction
|
||||
\[
|
||||
g(x) = 6x^3 + 5x^2 - x + 5
|
||||
\]
|
||||
\begin{enumerate}
|
||||
\item À l'aide d'une calculatrice ou d'un ordinateur, tracer puis reporter sur votre copie la représentation graphique de $g$ en y indiquant les informations remarquables de ce graphique.
|
||||
\item Sur quel(s) intervalle(s) la fonction est convexe? concave? Y a-t-il des points d'inflexions?
|
||||
\item Calculer la dérivé de $g$.
|
||||
\item Étudier le signe de la dérivée $g'$ puis en déduire le tableau de variations de $g$.
|
||||
\item Déterminer l'équation de la tangente en $x=0$.
|
||||
\item Dériver $g'$ pour calculer $g''$.
|
||||
\item Étudier le signe de $g''$ pour en déduire la convexité de $g$ grâce au calcul puis localiser précisément le point d'inflexion.
|
||||
\end{enumerate}
|
||||
\end{exercise}
|
||||
\begin{solution}
|
||||
\section*{Partie A}
|
||||
\begin{enumerate}
|
||||
\item $- 9 - 16x$
|
||||
\item Correction non disponible
|
||||
\end{enumerate}
|
||||
|
||||
\section*{Partie B}
|
||||
\begin{enumerate}
|
||||
\item Correction non disponible
|
||||
\item Correction non disponible
|
||||
\item $g'(x) = - 1 + 10x + 18x^2$
|
||||
\item
|
||||
On commence par calculer le discriminant de $g'(x)=- 1 + 10x + 18x^2$.
|
||||
\begin{eqnarray*}
|
||||
\Delta & = & b^2-4ac \\
|
||||
\Delta & = & 10^{2} - 4 \times 18 \times - 1 \\
|
||||
\Delta & = & 100 - 72 \times - 1 \\
|
||||
\Delta & = & 100 + 72 \\
|
||||
\Delta & = & 172
|
||||
\end{eqnarray*}
|
||||
|
||||
comme $\Delta = 172 > 0$ donc $P$ a deux racines
|
||||
|
||||
\begin{eqnarray*}
|
||||
x_1 & = & \frac{-b - \sqrt{\Delta}}{2a} = \frac{10 - \sqrt{172}}{2 \times 18} = - 0.6420799180167778 \\
|
||||
x_2 & = & \frac{-b + \sqrt{\Delta}}{2a} = \frac{10 + \sqrt{172}}{2 \times 18} = 0.08652436246122225
|
||||
\end{eqnarray*}
|
||||
Ainsi, $g'$ est du signe de $a=18$ en dehors des racines.
|
||||
|
||||
|
||||
Le tableau de variation non disponible en correction
|
||||
\item Équation de la tangente: $y = - 1x + 5$
|
||||
\item $g''(x) = 36x + 10$
|
||||
\end{enumerate}
|
||||
\end{solution}
|
||||
|
||||
\end{document}
|
||||
|
||||
%%% Local Variables:
|
||||
%%% mode: latex
|
||||
%%% TeX-master: "master"
|
||||
%%% End:
|
||||
157
TES/DM/DM_19_10/15_DM_19_10.tex
Normal file
157
TES/DM/DM_19_10/15_DM_19_10.tex
Normal file
@@ -0,0 +1,157 @@
|
||||
\documentclass[a4paper,10pt]{article}
|
||||
\usepackage{tasks}
|
||||
\usepackage{myXsim}
|
||||
|
||||
\title{DM 1 -- MENARD Cassandre}
|
||||
\tribe{Terminale ES-L}
|
||||
\date{15 novembre 2019}
|
||||
|
||||
\xsimsetup{
|
||||
solution/print = false
|
||||
}
|
||||
|
||||
\begin{document}
|
||||
\maketitle
|
||||
|
||||
Une part importante de la note sera dédiée à la rédaction, aux explications et à l'utilisation des notations mathématiques.
|
||||
|
||||
\begin{exercise}[subtitle={Débit}]
|
||||
Une commune de \np{1700} habitants au 1\up{er} janvier 2018 voit sa population augmenter de 5\,\% tous les ans.
|
||||
|
||||
Pour tout entier naturel $n$, on note $h_n$ le nombre d'habitants de l'année $2018 + n$.
|
||||
|
||||
\smallskip
|
||||
|
||||
\begin{enumerate}
|
||||
\item Déterminer la nature de la suite $(h_n)$, préciser ses éléments caractéristiques et exprimer $h_n$ en fonction de $n$.
|
||||
\end{enumerate}
|
||||
|
||||
\smallskip
|
||||
|
||||
La municipalité de cette commune a conclu un marché avec un fournisseur d'accès internet qui engage ce dernier à fournir un débit total de \np{13600}~Mbit/s au 1\up{er} janvier 2018 et à augmenter ce débit de - 0.8000\,\% par an.
|
||||
|
||||
Pour tout entier naturel $n$, on note $d_n$ le débit total dont la commune dispose l'année $2018 + n$.
|
||||
|
||||
\begin{enumerate}
|
||||
\setcounter{enumi}{1}
|
||||
\item Déterminer la nature de la suite $(d_n)$, préciser ses éléments caractéristiques et exprimer $d_n$ en fonction de $n$.
|
||||
\end{enumerate}
|
||||
|
||||
On s'intéresse maintenant au débit par habitant en supposant que celui-ci est réparti équitablement et que toute la population bénéficie d'une connexion internet individuelle.
|
||||
|
||||
Pour tout entier naturel $n$ on note $u_n$ le débit par habitant pour l'année $2018 + n$ et on admet que $u_n = \dfrac{d_n}{h_n}$.
|
||||
\begin{enumerate}
|
||||
\setcounter{enumi}{2}
|
||||
\item Calculer $u_0$ et $u_1$.
|
||||
\item Montrer pour tout entier naturel $n$ on a $u_n = 8 \times 0.96^n$.
|
||||
\item En déduire la nature de la suite $\left(u_n\right)$ et ses caractéristiques.
|
||||
\item Déterminer le sens de variations de la $\left(u_n\right)$.
|
||||
Interpréter ce résultat dans le contexte de l'énoncé.
|
||||
\end{enumerate}
|
||||
Le marché passé avec le fournisseur d'accès internet prévoit également que si le débit passe en dessous de 5 Mbit/s par habitant alors ce dernier doit changer la technologie utilisée pour la réalisation de son réseau.
|
||||
|
||||
\begin{enumerate}
|
||||
\setcounter{enumi}{6}
|
||||
\item En quelle année le fournisseur d'accès sera-t-il dans l'obligation de changer sa technologie?
|
||||
\end{enumerate}
|
||||
\end{exercise}
|
||||
\begin{solution}
|
||||
\begin{enumerate}
|
||||
\item Augmenter de 5\% revient à multiplier pas 1.05. La suite $(h_n)$ est donc géométrique de raison 1.05 et de premier terme 1700. On en déduit $h_n$ en fonction de $n$
|
||||
\[
|
||||
h_n = 1700\times 1.05^n
|
||||
\]
|
||||
\item Augmenter de - 0.8000\% revient à multiplier pas 1.0080. La suite $(d_n)$ est donc géométrique de raison 1.0080 et de premier terme 13600. On en déduit $d_n$ en fonction de $n$
|
||||
\[
|
||||
d_n = 13600\times 1.0080^n
|
||||
\]
|
||||
\item
|
||||
\begin{enumerate}
|
||||
\item
|
||||
\[
|
||||
u_0 = \frac{d_0}{h_0} = \frac{13600}{1700} = 8
|
||||
\]
|
||||
\[
|
||||
u_1 = \frac{d_1}{h_1} = \frac{13708.8000}{1785} = 7.68
|
||||
\]
|
||||
\item Démonstration de la formule
|
||||
\begin{eqnarray*}
|
||||
u_n &=& \frac{d_n}{h_n} = \frac{13600\times1.0080^n}{1700\times1.05^n} \\
|
||||
u_n &=& \frac{13600}{1700}\times\left(\frac{1.0080}{1.05}\right)^n \\
|
||||
u_n &=& 8\times0.96^n
|
||||
\end{eqnarray*}
|
||||
\item On reconnaît la forme d'une suite géométrique de raison 0.96 et de premier terme 8.
|
||||
\item La raison, $q = 0.96$, est inférieur à 1 donc la suite est décroissante. Ce qui signifie que le débit par habitant va diminuer.
|
||||
\end{enumerate}
|
||||
\item Avec le tableau de la calculatrice, on calculer les valeurs de $u_n$ jusqu'à passer en dessous de 5. On trouve $n = 12$ avec $u_{12} = 4.901678058638138910179328$
|
||||
|
||||
\end{enumerate}
|
||||
\end{solution}
|
||||
\begin{exercise}[subtitle={Étude de fonctions}]
|
||||
\section*{Partie A}
|
||||
Dans cette partie, on étudie la fonction
|
||||
\[
|
||||
f(x) = - 6x^2 + 9x + 10
|
||||
\]
|
||||
\begin{enumerate}
|
||||
\item Calculer la dérivé de $f$.
|
||||
\item Étudier le signe de la dérivée $f'$ puis en déduire le tableau de signe de $f$.
|
||||
\end{enumerate}
|
||||
\section*{Partie B}
|
||||
Dans cette partie, on étudie la fonction
|
||||
\[
|
||||
g(x) = 9x^3 - 6x^2 - 2x + 7
|
||||
\]
|
||||
\begin{enumerate}
|
||||
\item À l'aide d'une calculatrice ou d'un ordinateur, tracer puis reporter sur votre copie la représentation graphique de $g$ en y indiquant les informations remarquables de ce graphique.
|
||||
\item Sur quel(s) intervalle(s) la fonction est convexe? concave? Y a-t-il des points d'inflexions?
|
||||
\item Calculer la dérivé de $g$.
|
||||
\item Étudier le signe de la dérivée $g'$ puis en déduire le tableau de variations de $g$.
|
||||
\item Déterminer l'équation de la tangente en $x=0$.
|
||||
\item Dériver $g'$ pour calculer $g''$.
|
||||
\item Étudier le signe de $g''$ pour en déduire la convexité de $g$ grâce au calcul puis localiser précisément le point d'inflexion.
|
||||
\end{enumerate}
|
||||
\end{exercise}
|
||||
\begin{solution}
|
||||
\section*{Partie A}
|
||||
\begin{enumerate}
|
||||
\item $9 - 12x$
|
||||
\item Correction non disponible
|
||||
\end{enumerate}
|
||||
|
||||
\section*{Partie B}
|
||||
\begin{enumerate}
|
||||
\item Correction non disponible
|
||||
\item Correction non disponible
|
||||
\item $g'(x) = - 2 - 12x + 27x^2$
|
||||
\item
|
||||
On commence par calculer le discriminant de $g'(x)=- 2 - 12x + 27x^2$.
|
||||
\begin{eqnarray*}
|
||||
\Delta & = & b^2-4ac \\
|
||||
\Delta & = & - 12^{2} - 4 \times 27 \times - 2 \\
|
||||
\Delta & = & 144 - 108 \times - 2 \\
|
||||
\Delta & = & 144 + 216 \\
|
||||
\Delta & = & 360
|
||||
\end{eqnarray*}
|
||||
|
||||
comme $\Delta = 360 > 0$ donc $P$ a deux racines
|
||||
|
||||
\begin{eqnarray*}
|
||||
x_1 & = & \frac{-b - \sqrt{\Delta}}{2a} = \frac{- 12 - \sqrt{360}}{2 \times 27} = - 0.12914196224093105 \\
|
||||
x_2 & = & \frac{-b + \sqrt{\Delta}}{2a} = \frac{- 12 + \sqrt{360}}{2 \times 27} = 0.5735864066853755
|
||||
\end{eqnarray*}
|
||||
Ainsi, $g'$ est du signe de $a=27$ en dehors des racines.
|
||||
|
||||
|
||||
Le tableau de variation non disponible en correction
|
||||
\item Équation de la tangente: $y = - 2x + 7$
|
||||
\item $g''(x) = 54x - 12$
|
||||
\end{enumerate}
|
||||
\end{solution}
|
||||
|
||||
\end{document}
|
||||
|
||||
%%% Local Variables:
|
||||
%%% mode: latex
|
||||
%%% TeX-master: "master"
|
||||
%%% End:
|
||||
157
TES/DM/DM_19_10/16_DM_19_10.tex
Normal file
157
TES/DM/DM_19_10/16_DM_19_10.tex
Normal file
@@ -0,0 +1,157 @@
|
||||
\documentclass[a4paper,10pt]{article}
|
||||
\usepackage{tasks}
|
||||
\usepackage{myXsim}
|
||||
|
||||
\title{DM 1 -- MICHEL-PROST Lauryne}
|
||||
\tribe{Terminale ES-L}
|
||||
\date{15 novembre 2019}
|
||||
|
||||
\xsimsetup{
|
||||
solution/print = false
|
||||
}
|
||||
|
||||
\begin{document}
|
||||
\maketitle
|
||||
|
||||
Une part importante de la note sera dédiée à la rédaction, aux explications et à l'utilisation des notations mathématiques.
|
||||
|
||||
\begin{exercise}[subtitle={Débit}]
|
||||
Une commune de \np{2500} habitants au 1\up{er} janvier 2018 voit sa population augmenter de 7\,\% tous les ans.
|
||||
|
||||
Pour tout entier naturel $n$, on note $h_n$ le nombre d'habitants de l'année $2018 + n$.
|
||||
|
||||
\smallskip
|
||||
|
||||
\begin{enumerate}
|
||||
\item Déterminer la nature de la suite $(h_n)$, préciser ses éléments caractéristiques et exprimer $h_n$ en fonction de $n$.
|
||||
\end{enumerate}
|
||||
|
||||
\smallskip
|
||||
|
||||
La municipalité de cette commune a conclu un marché avec un fournisseur d'accès internet qui engage ce dernier à fournir un débit total de \np{17500}~Mbit/s au 1\up{er} janvier 2018 et à augmenter ce débit de - 4.8600\,\% par an.
|
||||
|
||||
Pour tout entier naturel $n$, on note $d_n$ le débit total dont la commune dispose l'année $2018 + n$.
|
||||
|
||||
\begin{enumerate}
|
||||
\setcounter{enumi}{1}
|
||||
\item Déterminer la nature de la suite $(d_n)$, préciser ses éléments caractéristiques et exprimer $d_n$ en fonction de $n$.
|
||||
\end{enumerate}
|
||||
|
||||
On s'intéresse maintenant au débit par habitant en supposant que celui-ci est réparti équitablement et que toute la population bénéficie d'une connexion internet individuelle.
|
||||
|
||||
Pour tout entier naturel $n$ on note $u_n$ le débit par habitant pour l'année $2018 + n$ et on admet que $u_n = \dfrac{d_n}{h_n}$.
|
||||
\begin{enumerate}
|
||||
\setcounter{enumi}{2}
|
||||
\item Calculer $u_0$ et $u_1$.
|
||||
\item Montrer pour tout entier naturel $n$ on a $u_n = 7 \times 0.98^n$.
|
||||
\item En déduire la nature de la suite $\left(u_n\right)$ et ses caractéristiques.
|
||||
\item Déterminer le sens de variations de la $\left(u_n\right)$.
|
||||
Interpréter ce résultat dans le contexte de l'énoncé.
|
||||
\end{enumerate}
|
||||
Le marché passé avec le fournisseur d'accès internet prévoit également que si le débit passe en dessous de 5 Mbit/s par habitant alors ce dernier doit changer la technologie utilisée pour la réalisation de son réseau.
|
||||
|
||||
\begin{enumerate}
|
||||
\setcounter{enumi}{6}
|
||||
\item En quelle année le fournisseur d'accès sera-t-il dans l'obligation de changer sa technologie?
|
||||
\end{enumerate}
|
||||
\end{exercise}
|
||||
\begin{solution}
|
||||
\begin{enumerate}
|
||||
\item Augmenter de 7\% revient à multiplier pas 1.07. La suite $(h_n)$ est donc géométrique de raison 1.07 et de premier terme 2500. On en déduit $h_n$ en fonction de $n$
|
||||
\[
|
||||
h_n = 2500\times 1.07^n
|
||||
\]
|
||||
\item Augmenter de - 4.8600\% revient à multiplier pas 1.0486. La suite $(d_n)$ est donc géométrique de raison 1.0486 et de premier terme 17500. On en déduit $d_n$ en fonction de $n$
|
||||
\[
|
||||
d_n = 17500\times 1.0486^n
|
||||
\]
|
||||
\item
|
||||
\begin{enumerate}
|
||||
\item
|
||||
\[
|
||||
u_0 = \frac{d_0}{h_0} = \frac{17500}{2500} = 7
|
||||
\]
|
||||
\[
|
||||
u_1 = \frac{d_1}{h_1} = \frac{18350.5000}{2675} = 6.86
|
||||
\]
|
||||
\item Démonstration de la formule
|
||||
\begin{eqnarray*}
|
||||
u_n &=& \frac{d_n}{h_n} = \frac{17500\times1.0486^n}{2500\times1.07^n} \\
|
||||
u_n &=& \frac{17500}{2500}\times\left(\frac{1.0486}{1.07}\right)^n \\
|
||||
u_n &=& 7\times0.98^n
|
||||
\end{eqnarray*}
|
||||
\item On reconnaît la forme d'une suite géométrique de raison 0.98 et de premier terme 7.
|
||||
\item La raison, $q = 0.98$, est inférieur à 1 donc la suite est décroissante. Ce qui signifie que le débit par habitant va diminuer.
|
||||
\end{enumerate}
|
||||
\item Avec le tableau de la calculatrice, on calculer les valeurs de $u_n$ jusqu'à passer en dessous de 5. On trouve $n = 17$ avec $u_{17} = 4.965252363264521426471716467$
|
||||
|
||||
\end{enumerate}
|
||||
\end{solution}
|
||||
\begin{exercise}[subtitle={Étude de fonctions}]
|
||||
\section*{Partie A}
|
||||
Dans cette partie, on étudie la fonction
|
||||
\[
|
||||
f(x) = - 5x^2 + 4x - 10
|
||||
\]
|
||||
\begin{enumerate}
|
||||
\item Calculer la dérivé de $f$.
|
||||
\item Étudier le signe de la dérivée $f'$ puis en déduire le tableau de signe de $f$.
|
||||
\end{enumerate}
|
||||
\section*{Partie B}
|
||||
Dans cette partie, on étudie la fonction
|
||||
\[
|
||||
g(x) = x^3 - 7x^2 - 6x + 10
|
||||
\]
|
||||
\begin{enumerate}
|
||||
\item À l'aide d'une calculatrice ou d'un ordinateur, tracer puis reporter sur votre copie la représentation graphique de $g$ en y indiquant les informations remarquables de ce graphique.
|
||||
\item Sur quel(s) intervalle(s) la fonction est convexe? concave? Y a-t-il des points d'inflexions?
|
||||
\item Calculer la dérivé de $g$.
|
||||
\item Étudier le signe de la dérivée $g'$ puis en déduire le tableau de variations de $g$.
|
||||
\item Déterminer l'équation de la tangente en $x=0$.
|
||||
\item Dériver $g'$ pour calculer $g''$.
|
||||
\item Étudier le signe de $g''$ pour en déduire la convexité de $g$ grâce au calcul puis localiser précisément le point d'inflexion.
|
||||
\end{enumerate}
|
||||
\end{exercise}
|
||||
\begin{solution}
|
||||
\section*{Partie A}
|
||||
\begin{enumerate}
|
||||
\item $4 - 10x$
|
||||
\item Correction non disponible
|
||||
\end{enumerate}
|
||||
|
||||
\section*{Partie B}
|
||||
\begin{enumerate}
|
||||
\item Correction non disponible
|
||||
\item Correction non disponible
|
||||
\item $g'(x) = - 6 + 3x^2 - 14x$
|
||||
\item
|
||||
On commence par calculer le discriminant de $g'(x)=- 6 + 3x^2 - 14x$.
|
||||
\begin{eqnarray*}
|
||||
\Delta & = & b^2-4ac \\
|
||||
\Delta & = & - 14^{2} - 4 \times 3 \times - 6 \\
|
||||
\Delta & = & 196 - 12 \times - 6 \\
|
||||
\Delta & = & 196 + 72 \\
|
||||
\Delta & = & 268
|
||||
\end{eqnarray*}
|
||||
|
||||
comme $\Delta = 268 > 0$ donc $P$ a deux racines
|
||||
|
||||
\begin{eqnarray*}
|
||||
x_1 & = & \frac{-b - \sqrt{\Delta}}{2a} = \frac{- 14 - \sqrt{268}}{2 \times 3} = - 0.39511759062415014 \\
|
||||
x_2 & = & \frac{-b + \sqrt{\Delta}}{2a} = \frac{- 14 + \sqrt{268}}{2 \times 3} = 5.0617842572908165
|
||||
\end{eqnarray*}
|
||||
Ainsi, $g'$ est du signe de $a=3$ en dehors des racines.
|
||||
|
||||
|
||||
Le tableau de variation non disponible en correction
|
||||
\item Équation de la tangente: $y = - 6x + 10$
|
||||
\item $g''(x) = - 14 + 6x$
|
||||
\end{enumerate}
|
||||
\end{solution}
|
||||
|
||||
\end{document}
|
||||
|
||||
%%% Local Variables:
|
||||
%%% mode: latex
|
||||
%%% TeX-master: "master"
|
||||
%%% End:
|
||||
157
TES/DM/DM_19_10/17_DM_19_10.tex
Normal file
157
TES/DM/DM_19_10/17_DM_19_10.tex
Normal file
@@ -0,0 +1,157 @@
|
||||
\documentclass[a4paper,10pt]{article}
|
||||
\usepackage{tasks}
|
||||
\usepackage{myXsim}
|
||||
|
||||
\title{DM 1 -- MOUBARIK Ines}
|
||||
\tribe{Terminale ES-L}
|
||||
\date{15 novembre 2019}
|
||||
|
||||
\xsimsetup{
|
||||
solution/print = false
|
||||
}
|
||||
|
||||
\begin{document}
|
||||
\maketitle
|
||||
|
||||
Une part importante de la note sera dédiée à la rédaction, aux explications et à l'utilisation des notations mathématiques.
|
||||
|
||||
\begin{exercise}[subtitle={Débit}]
|
||||
Une commune de \np{2200} habitants au 1\up{er} janvier 2018 voit sa population augmenter de 5\,\% tous les ans.
|
||||
|
||||
Pour tout entier naturel $n$, on note $h_n$ le nombre d'habitants de l'année $2018 + n$.
|
||||
|
||||
\smallskip
|
||||
|
||||
\begin{enumerate}
|
||||
\item Déterminer la nature de la suite $(h_n)$, préciser ses éléments caractéristiques et exprimer $h_n$ en fonction de $n$.
|
||||
\end{enumerate}
|
||||
|
||||
\smallskip
|
||||
|
||||
La municipalité de cette commune a conclu un marché avec un fournisseur d'accès internet qui engage ce dernier à fournir un débit total de \np{22000}~Mbit/s au 1\up{er} janvier 2018 et à augmenter ce débit de - 1.8500\,\% par an.
|
||||
|
||||
Pour tout entier naturel $n$, on note $d_n$ le débit total dont la commune dispose l'année $2018 + n$.
|
||||
|
||||
\begin{enumerate}
|
||||
\setcounter{enumi}{1}
|
||||
\item Déterminer la nature de la suite $(d_n)$, préciser ses éléments caractéristiques et exprimer $d_n$ en fonction de $n$.
|
||||
\end{enumerate}
|
||||
|
||||
On s'intéresse maintenant au débit par habitant en supposant que celui-ci est réparti équitablement et que toute la population bénéficie d'une connexion internet individuelle.
|
||||
|
||||
Pour tout entier naturel $n$ on note $u_n$ le débit par habitant pour l'année $2018 + n$ et on admet que $u_n = \dfrac{d_n}{h_n}$.
|
||||
\begin{enumerate}
|
||||
\setcounter{enumi}{2}
|
||||
\item Calculer $u_0$ et $u_1$.
|
||||
\item Montrer pour tout entier naturel $n$ on a $u_n = 10 \times 0.97^n$.
|
||||
\item En déduire la nature de la suite $\left(u_n\right)$ et ses caractéristiques.
|
||||
\item Déterminer le sens de variations de la $\left(u_n\right)$.
|
||||
Interpréter ce résultat dans le contexte de l'énoncé.
|
||||
\end{enumerate}
|
||||
Le marché passé avec le fournisseur d'accès internet prévoit également que si le débit passe en dessous de 5 Mbit/s par habitant alors ce dernier doit changer la technologie utilisée pour la réalisation de son réseau.
|
||||
|
||||
\begin{enumerate}
|
||||
\setcounter{enumi}{6}
|
||||
\item En quelle année le fournisseur d'accès sera-t-il dans l'obligation de changer sa technologie?
|
||||
\end{enumerate}
|
||||
\end{exercise}
|
||||
\begin{solution}
|
||||
\begin{enumerate}
|
||||
\item Augmenter de 5\% revient à multiplier pas 1.05. La suite $(h_n)$ est donc géométrique de raison 1.05 et de premier terme 2200. On en déduit $h_n$ en fonction de $n$
|
||||
\[
|
||||
h_n = 2200\times 1.05^n
|
||||
\]
|
||||
\item Augmenter de - 1.8500\% revient à multiplier pas 1.0185. La suite $(d_n)$ est donc géométrique de raison 1.0185 et de premier terme 22000. On en déduit $d_n$ en fonction de $n$
|
||||
\[
|
||||
d_n = 22000\times 1.0185^n
|
||||
\]
|
||||
\item
|
||||
\begin{enumerate}
|
||||
\item
|
||||
\[
|
||||
u_0 = \frac{d_0}{h_0} = \frac{22000}{2200} = 10
|
||||
\]
|
||||
\[
|
||||
u_1 = \frac{d_1}{h_1} = \frac{22407}{2310} = 9.70
|
||||
\]
|
||||
\item Démonstration de la formule
|
||||
\begin{eqnarray*}
|
||||
u_n &=& \frac{d_n}{h_n} = \frac{22000\times1.0185^n}{2200\times1.05^n} \\
|
||||
u_n &=& \frac{22000}{2200}\times\left(\frac{1.0185}{1.05}\right)^n \\
|
||||
u_n &=& 10\times0.97^n
|
||||
\end{eqnarray*}
|
||||
\item On reconnaît la forme d'une suite géométrique de raison 0.97 et de premier terme 10.
|
||||
\item La raison, $q = 0.97$, est inférieur à 1 donc la suite est décroissante. Ce qui signifie que le débit par habitant va diminuer.
|
||||
\end{enumerate}
|
||||
\item Avec le tableau de la calculatrice, on calculer les valeurs de $u_n$ jusqu'à passer en dessous de 5. On trouve $n = 23$ avec $u_{23} = 4.963064143419831996986398968$
|
||||
|
||||
\end{enumerate}
|
||||
\end{solution}
|
||||
\begin{exercise}[subtitle={Étude de fonctions}]
|
||||
\section*{Partie A}
|
||||
Dans cette partie, on étudie la fonction
|
||||
\[
|
||||
f(x) = - 4x^2 - 6x - 7
|
||||
\]
|
||||
\begin{enumerate}
|
||||
\item Calculer la dérivé de $f$.
|
||||
\item Étudier le signe de la dérivée $f'$ puis en déduire le tableau de signe de $f$.
|
||||
\end{enumerate}
|
||||
\section*{Partie B}
|
||||
Dans cette partie, on étudie la fonction
|
||||
\[
|
||||
g(x) = 9x^3 - 8x^2 - 10x - 3
|
||||
\]
|
||||
\begin{enumerate}
|
||||
\item À l'aide d'une calculatrice ou d'un ordinateur, tracer puis reporter sur votre copie la représentation graphique de $g$ en y indiquant les informations remarquables de ce graphique.
|
||||
\item Sur quel(s) intervalle(s) la fonction est convexe? concave? Y a-t-il des points d'inflexions?
|
||||
\item Calculer la dérivé de $g$.
|
||||
\item Étudier le signe de la dérivée $g'$ puis en déduire le tableau de variations de $g$.
|
||||
\item Déterminer l'équation de la tangente en $x=0$.
|
||||
\item Dériver $g'$ pour calculer $g''$.
|
||||
\item Étudier le signe de $g''$ pour en déduire la convexité de $g$ grâce au calcul puis localiser précisément le point d'inflexion.
|
||||
\end{enumerate}
|
||||
\end{exercise}
|
||||
\begin{solution}
|
||||
\section*{Partie A}
|
||||
\begin{enumerate}
|
||||
\item $- 6 - 8x$
|
||||
\item Correction non disponible
|
||||
\end{enumerate}
|
||||
|
||||
\section*{Partie B}
|
||||
\begin{enumerate}
|
||||
\item Correction non disponible
|
||||
\item Correction non disponible
|
||||
\item $g'(x) = - 10 - 16x + 27x^2$
|
||||
\item
|
||||
On commence par calculer le discriminant de $g'(x)=- 10 - 16x + 27x^2$.
|
||||
\begin{eqnarray*}
|
||||
\Delta & = & b^2-4ac \\
|
||||
\Delta & = & - 16^{2} - 4 \times 27 \times - 10 \\
|
||||
\Delta & = & 256 - 108 \times - 10 \\
|
||||
\Delta & = & 256 + 1080 \\
|
||||
\Delta & = & 1336
|
||||
\end{eqnarray*}
|
||||
|
||||
comme $\Delta = 1336 > 0$ donc $P$ a deux racines
|
||||
|
||||
\begin{eqnarray*}
|
||||
x_1 & = & \frac{-b - \sqrt{\Delta}}{2a} = \frac{- 16 - \sqrt{1336}}{2 \times 27} = - 0.38058025490729874 \\
|
||||
x_2 & = & \frac{-b + \sqrt{\Delta}}{2a} = \frac{- 16 + \sqrt{1336}}{2 \times 27} = 0.9731728474998914
|
||||
\end{eqnarray*}
|
||||
Ainsi, $g'$ est du signe de $a=27$ en dehors des racines.
|
||||
|
||||
|
||||
Le tableau de variation non disponible en correction
|
||||
\item Équation de la tangente: $y = - 10x + - 3$
|
||||
\item $g''(x) = 54x - 16$
|
||||
\end{enumerate}
|
||||
\end{solution}
|
||||
|
||||
\end{document}
|
||||
|
||||
%%% Local Variables:
|
||||
%%% mode: latex
|
||||
%%% TeX-master: "master"
|
||||
%%% End:
|
||||
157
TES/DM/DM_19_10/18_DM_19_10.tex
Normal file
157
TES/DM/DM_19_10/18_DM_19_10.tex
Normal file
@@ -0,0 +1,157 @@
|
||||
\documentclass[a4paper,10pt]{article}
|
||||
\usepackage{tasks}
|
||||
\usepackage{myXsim}
|
||||
|
||||
\title{DM 1 -- MOUBARIK Sarah}
|
||||
\tribe{Terminale ES-L}
|
||||
\date{15 novembre 2019}
|
||||
|
||||
\xsimsetup{
|
||||
solution/print = false
|
||||
}
|
||||
|
||||
\begin{document}
|
||||
\maketitle
|
||||
|
||||
Une part importante de la note sera dédiée à la rédaction, aux explications et à l'utilisation des notations mathématiques.
|
||||
|
||||
\begin{exercise}[subtitle={Débit}]
|
||||
Une commune de \np{1700} habitants au 1\up{er} janvier 2018 voit sa population augmenter de 9\,\% tous les ans.
|
||||
|
||||
Pour tout entier naturel $n$, on note $h_n$ le nombre d'habitants de l'année $2018 + n$.
|
||||
|
||||
\smallskip
|
||||
|
||||
\begin{enumerate}
|
||||
\item Déterminer la nature de la suite $(h_n)$, préciser ses éléments caractéristiques et exprimer $h_n$ en fonction de $n$.
|
||||
\end{enumerate}
|
||||
|
||||
\smallskip
|
||||
|
||||
La municipalité de cette commune a conclu un marché avec un fournisseur d'accès internet qui engage ce dernier à fournir un débit total de \np{20400}~Mbit/s au 1\up{er} janvier 2018 et à augmenter ce débit de - 5.7300\,\% par an.
|
||||
|
||||
Pour tout entier naturel $n$, on note $d_n$ le débit total dont la commune dispose l'année $2018 + n$.
|
||||
|
||||
\begin{enumerate}
|
||||
\setcounter{enumi}{1}
|
||||
\item Déterminer la nature de la suite $(d_n)$, préciser ses éléments caractéristiques et exprimer $d_n$ en fonction de $n$.
|
||||
\end{enumerate}
|
||||
|
||||
On s'intéresse maintenant au débit par habitant en supposant que celui-ci est réparti équitablement et que toute la population bénéficie d'une connexion internet individuelle.
|
||||
|
||||
Pour tout entier naturel $n$ on note $u_n$ le débit par habitant pour l'année $2018 + n$ et on admet que $u_n = \dfrac{d_n}{h_n}$.
|
||||
\begin{enumerate}
|
||||
\setcounter{enumi}{2}
|
||||
\item Calculer $u_0$ et $u_1$.
|
||||
\item Montrer pour tout entier naturel $n$ on a $u_n = 12 \times 0.97^n$.
|
||||
\item En déduire la nature de la suite $\left(u_n\right)$ et ses caractéristiques.
|
||||
\item Déterminer le sens de variations de la $\left(u_n\right)$.
|
||||
Interpréter ce résultat dans le contexte de l'énoncé.
|
||||
\end{enumerate}
|
||||
Le marché passé avec le fournisseur d'accès internet prévoit également que si le débit passe en dessous de 5 Mbit/s par habitant alors ce dernier doit changer la technologie utilisée pour la réalisation de son réseau.
|
||||
|
||||
\begin{enumerate}
|
||||
\setcounter{enumi}{6}
|
||||
\item En quelle année le fournisseur d'accès sera-t-il dans l'obligation de changer sa technologie?
|
||||
\end{enumerate}
|
||||
\end{exercise}
|
||||
\begin{solution}
|
||||
\begin{enumerate}
|
||||
\item Augmenter de 9\% revient à multiplier pas 1.09. La suite $(h_n)$ est donc géométrique de raison 1.09 et de premier terme 1700. On en déduit $h_n$ en fonction de $n$
|
||||
\[
|
||||
h_n = 1700\times 1.09^n
|
||||
\]
|
||||
\item Augmenter de - 5.7300\% revient à multiplier pas 1.0573. La suite $(d_n)$ est donc géométrique de raison 1.0573 et de premier terme 20400. On en déduit $d_n$ en fonction de $n$
|
||||
\[
|
||||
d_n = 20400\times 1.0573^n
|
||||
\]
|
||||
\item
|
||||
\begin{enumerate}
|
||||
\item
|
||||
\[
|
||||
u_0 = \frac{d_0}{h_0} = \frac{20400}{1700} = 12
|
||||
\]
|
||||
\[
|
||||
u_1 = \frac{d_1}{h_1} = \frac{21568.9200}{1853} = 11.64
|
||||
\]
|
||||
\item Démonstration de la formule
|
||||
\begin{eqnarray*}
|
||||
u_n &=& \frac{d_n}{h_n} = \frac{20400\times1.0573^n}{1700\times1.09^n} \\
|
||||
u_n &=& \frac{20400}{1700}\times\left(\frac{1.0573}{1.09}\right)^n \\
|
||||
u_n &=& 12\times0.97^n
|
||||
\end{eqnarray*}
|
||||
\item On reconnaît la forme d'une suite géométrique de raison 0.97 et de premier terme 12.
|
||||
\item La raison, $q = 0.97$, est inférieur à 1 donc la suite est décroissante. Ce qui signifie que le débit par habitant va diminuer.
|
||||
\end{enumerate}
|
||||
\item Avec le tableau de la calculatrice, on calculer les valeurs de $u_n$ jusqu'à passer en dessous de 5. On trouve $n = 29$ avec $u_{29} = 4.960912188162776953332127961$
|
||||
|
||||
\end{enumerate}
|
||||
\end{solution}
|
||||
\begin{exercise}[subtitle={Étude de fonctions}]
|
||||
\section*{Partie A}
|
||||
Dans cette partie, on étudie la fonction
|
||||
\[
|
||||
f(x) = - 4x^2 + x + 7
|
||||
\]
|
||||
\begin{enumerate}
|
||||
\item Calculer la dérivé de $f$.
|
||||
\item Étudier le signe de la dérivée $f'$ puis en déduire le tableau de signe de $f$.
|
||||
\end{enumerate}
|
||||
\section*{Partie B}
|
||||
Dans cette partie, on étudie la fonction
|
||||
\[
|
||||
g(x) = 8x^3 - 9x^2 + 2x - 7
|
||||
\]
|
||||
\begin{enumerate}
|
||||
\item À l'aide d'une calculatrice ou d'un ordinateur, tracer puis reporter sur votre copie la représentation graphique de $g$ en y indiquant les informations remarquables de ce graphique.
|
||||
\item Sur quel(s) intervalle(s) la fonction est convexe? concave? Y a-t-il des points d'inflexions?
|
||||
\item Calculer la dérivé de $g$.
|
||||
\item Étudier le signe de la dérivée $g'$ puis en déduire le tableau de variations de $g$.
|
||||
\item Déterminer l'équation de la tangente en $x=0$.
|
||||
\item Dériver $g'$ pour calculer $g''$.
|
||||
\item Étudier le signe de $g''$ pour en déduire la convexité de $g$ grâce au calcul puis localiser précisément le point d'inflexion.
|
||||
\end{enumerate}
|
||||
\end{exercise}
|
||||
\begin{solution}
|
||||
\section*{Partie A}
|
||||
\begin{enumerate}
|
||||
\item $1 - 8x$
|
||||
\item Correction non disponible
|
||||
\end{enumerate}
|
||||
|
||||
\section*{Partie B}
|
||||
\begin{enumerate}
|
||||
\item Correction non disponible
|
||||
\item Correction non disponible
|
||||
\item $g'(x) = 2 - 18x + 24x^2$
|
||||
\item
|
||||
On commence par calculer le discriminant de $g'(x)=2 - 18x + 24x^2$.
|
||||
\begin{eqnarray*}
|
||||
\Delta & = & b^2-4ac \\
|
||||
\Delta & = & - 18^{2} - 4 \times 24 \times 2 \\
|
||||
\Delta & = & 324 - 96 \times 2 \\
|
||||
\Delta & = & 324 - 192 \\
|
||||
\Delta & = & 132
|
||||
\end{eqnarray*}
|
||||
|
||||
comme $\Delta = 132 > 0$ donc $P$ a deux racines
|
||||
|
||||
\begin{eqnarray*}
|
||||
x_1 & = & \frac{-b - \sqrt{\Delta}}{2a} = \frac{- 18 - \sqrt{132}}{2 \times 24} = 0.13564322306091547 \\
|
||||
x_2 & = & \frac{-b + \sqrt{\Delta}}{2a} = \frac{- 18 + \sqrt{132}}{2 \times 24} = 0.6143567769390845
|
||||
\end{eqnarray*}
|
||||
Ainsi, $g'$ est du signe de $a=24$ en dehors des racines.
|
||||
|
||||
|
||||
Le tableau de variation non disponible en correction
|
||||
\item Équation de la tangente: $y = 2x + - 7$
|
||||
\item $g''(x) = 48x - 18$
|
||||
\end{enumerate}
|
||||
\end{solution}
|
||||
|
||||
\end{document}
|
||||
|
||||
%%% Local Variables:
|
||||
%%% mode: latex
|
||||
%%% TeX-master: "master"
|
||||
%%% End:
|
||||
157
TES/DM/DM_19_10/19_DM_19_10.tex
Normal file
157
TES/DM/DM_19_10/19_DM_19_10.tex
Normal file
@@ -0,0 +1,157 @@
|
||||
\documentclass[a4paper,10pt]{article}
|
||||
\usepackage{tasks}
|
||||
\usepackage{myXsim}
|
||||
|
||||
\title{DM 1 -- PERREARD Noémie}
|
||||
\tribe{Terminale ES-L}
|
||||
\date{15 novembre 2019}
|
||||
|
||||
\xsimsetup{
|
||||
solution/print = false
|
||||
}
|
||||
|
||||
\begin{document}
|
||||
\maketitle
|
||||
|
||||
Une part importante de la note sera dédiée à la rédaction, aux explications et à l'utilisation des notations mathématiques.
|
||||
|
||||
\begin{exercise}[subtitle={Débit}]
|
||||
Une commune de \np{1900} habitants au 1\up{er} janvier 2018 voit sa population augmenter de 7\,\% tous les ans.
|
||||
|
||||
Pour tout entier naturel $n$, on note $h_n$ le nombre d'habitants de l'année $2018 + n$.
|
||||
|
||||
\smallskip
|
||||
|
||||
\begin{enumerate}
|
||||
\item Déterminer la nature de la suite $(h_n)$, préciser ses éléments caractéristiques et exprimer $h_n$ en fonction de $n$.
|
||||
\end{enumerate}
|
||||
|
||||
\smallskip
|
||||
|
||||
La municipalité de cette commune a conclu un marché avec un fournisseur d'accès internet qui engage ce dernier à fournir un débit total de \np{17100}~Mbit/s au 1\up{er} janvier 2018 et à augmenter ce débit de - 0.5800\,\% par an.
|
||||
|
||||
Pour tout entier naturel $n$, on note $d_n$ le débit total dont la commune dispose l'année $2018 + n$.
|
||||
|
||||
\begin{enumerate}
|
||||
\setcounter{enumi}{1}
|
||||
\item Déterminer la nature de la suite $(d_n)$, préciser ses éléments caractéristiques et exprimer $d_n$ en fonction de $n$.
|
||||
\end{enumerate}
|
||||
|
||||
On s'intéresse maintenant au débit par habitant en supposant que celui-ci est réparti équitablement et que toute la population bénéficie d'une connexion internet individuelle.
|
||||
|
||||
Pour tout entier naturel $n$ on note $u_n$ le débit par habitant pour l'année $2018 + n$ et on admet que $u_n = \dfrac{d_n}{h_n}$.
|
||||
\begin{enumerate}
|
||||
\setcounter{enumi}{2}
|
||||
\item Calculer $u_0$ et $u_1$.
|
||||
\item Montrer pour tout entier naturel $n$ on a $u_n = 9 \times 0.94^n$.
|
||||
\item En déduire la nature de la suite $\left(u_n\right)$ et ses caractéristiques.
|
||||
\item Déterminer le sens de variations de la $\left(u_n\right)$.
|
||||
Interpréter ce résultat dans le contexte de l'énoncé.
|
||||
\end{enumerate}
|
||||
Le marché passé avec le fournisseur d'accès internet prévoit également que si le débit passe en dessous de 5 Mbit/s par habitant alors ce dernier doit changer la technologie utilisée pour la réalisation de son réseau.
|
||||
|
||||
\begin{enumerate}
|
||||
\setcounter{enumi}{6}
|
||||
\item En quelle année le fournisseur d'accès sera-t-il dans l'obligation de changer sa technologie?
|
||||
\end{enumerate}
|
||||
\end{exercise}
|
||||
\begin{solution}
|
||||
\begin{enumerate}
|
||||
\item Augmenter de 7\% revient à multiplier pas 1.07. La suite $(h_n)$ est donc géométrique de raison 1.07 et de premier terme 1900. On en déduit $h_n$ en fonction de $n$
|
||||
\[
|
||||
h_n = 1900\times 1.07^n
|
||||
\]
|
||||
\item Augmenter de - 0.5800\% revient à multiplier pas 1.0058. La suite $(d_n)$ est donc géométrique de raison 1.0058 et de premier terme 17100. On en déduit $d_n$ en fonction de $n$
|
||||
\[
|
||||
d_n = 17100\times 1.0058^n
|
||||
\]
|
||||
\item
|
||||
\begin{enumerate}
|
||||
\item
|
||||
\[
|
||||
u_0 = \frac{d_0}{h_0} = \frac{17100}{1900} = 9
|
||||
\]
|
||||
\[
|
||||
u_1 = \frac{d_1}{h_1} = \frac{17199.1800}{2033} = 8.46
|
||||
\]
|
||||
\item Démonstration de la formule
|
||||
\begin{eqnarray*}
|
||||
u_n &=& \frac{d_n}{h_n} = \frac{17100\times1.0058^n}{1900\times1.07^n} \\
|
||||
u_n &=& \frac{17100}{1900}\times\left(\frac{1.0058}{1.07}\right)^n \\
|
||||
u_n &=& 9\times0.94^n
|
||||
\end{eqnarray*}
|
||||
\item On reconnaît la forme d'une suite géométrique de raison 0.94 et de premier terme 9.
|
||||
\item La raison, $q = 0.94$, est inférieur à 1 donc la suite est décroissante. Ce qui signifie que le débit par habitant va diminuer.
|
||||
\end{enumerate}
|
||||
\item Avec le tableau de la calculatrice, on calculer les valeurs de $u_n$ jusqu'à passer en dessous de 5. On trouve $n = 10$ avec $u_{10} = 4.84753602685409731584$
|
||||
|
||||
\end{enumerate}
|
||||
\end{solution}
|
||||
\begin{exercise}[subtitle={Étude de fonctions}]
|
||||
\section*{Partie A}
|
||||
Dans cette partie, on étudie la fonction
|
||||
\[
|
||||
f(x) = - 4x^2 - 2x + 9
|
||||
\]
|
||||
\begin{enumerate}
|
||||
\item Calculer la dérivé de $f$.
|
||||
\item Étudier le signe de la dérivée $f'$ puis en déduire le tableau de signe de $f$.
|
||||
\end{enumerate}
|
||||
\section*{Partie B}
|
||||
Dans cette partie, on étudie la fonction
|
||||
\[
|
||||
g(x) = - 6x^3 - 9x^2 - 4x - 3
|
||||
\]
|
||||
\begin{enumerate}
|
||||
\item À l'aide d'une calculatrice ou d'un ordinateur, tracer puis reporter sur votre copie la représentation graphique de $g$ en y indiquant les informations remarquables de ce graphique.
|
||||
\item Sur quel(s) intervalle(s) la fonction est convexe? concave? Y a-t-il des points d'inflexions?
|
||||
\item Calculer la dérivé de $g$.
|
||||
\item Étudier le signe de la dérivée $g'$ puis en déduire le tableau de variations de $g$.
|
||||
\item Déterminer l'équation de la tangente en $x=0$.
|
||||
\item Dériver $g'$ pour calculer $g''$.
|
||||
\item Étudier le signe de $g''$ pour en déduire la convexité de $g$ grâce au calcul puis localiser précisément le point d'inflexion.
|
||||
\end{enumerate}
|
||||
\end{exercise}
|
||||
\begin{solution}
|
||||
\section*{Partie A}
|
||||
\begin{enumerate}
|
||||
\item $- 2 - 8x$
|
||||
\item Correction non disponible
|
||||
\end{enumerate}
|
||||
|
||||
\section*{Partie B}
|
||||
\begin{enumerate}
|
||||
\item Correction non disponible
|
||||
\item Correction non disponible
|
||||
\item $g'(x) = - 4 - 18x - 18x^2$
|
||||
\item
|
||||
On commence par calculer le discriminant de $g'(x)=- 4 - 18x - 18x^2$.
|
||||
\begin{eqnarray*}
|
||||
\Delta & = & b^2-4ac \\
|
||||
\Delta & = & - 18^{2} - 4 \times - 18 \times - 4 \\
|
||||
\Delta & = & 324 + 72 \times - 4 \\
|
||||
\Delta & = & 324 - 288 \\
|
||||
\Delta & = & 36
|
||||
\end{eqnarray*}
|
||||
|
||||
comme $\Delta = 36 > 0$ donc $P$ a deux racines
|
||||
|
||||
\begin{eqnarray*}
|
||||
x_1 & = & \frac{-b - \sqrt{\Delta}}{2a} = \frac{- 18 - \sqrt{36}}{2 \times - 18} = - 0.3333333333333333 \\
|
||||
x_2 & = & \frac{-b + \sqrt{\Delta}}{2a} = \frac{- 18 + \sqrt{36}}{2 \times - 18} = - 0.6666666666666666
|
||||
\end{eqnarray*}
|
||||
Ainsi, $g'$ est du signe de $a=- 18$ en dehors des racines.
|
||||
|
||||
|
||||
Le tableau de variation non disponible en correction
|
||||
\item Équation de la tangente: $y = - 4x + - 3$
|
||||
\item $g''(x) = - 36x - 18$
|
||||
\end{enumerate}
|
||||
\end{solution}
|
||||
|
||||
\end{document}
|
||||
|
||||
%%% Local Variables:
|
||||
%%% mode: latex
|
||||
%%% TeX-master: "master"
|
||||
%%% End:
|
||||
157
TES/DM/DM_19_10/20_DM_19_10.tex
Normal file
157
TES/DM/DM_19_10/20_DM_19_10.tex
Normal file
@@ -0,0 +1,157 @@
|
||||
\documentclass[a4paper,10pt]{article}
|
||||
\usepackage{tasks}
|
||||
\usepackage{myXsim}
|
||||
|
||||
\title{DM 1 -- URPIN Flora}
|
||||
\tribe{Terminale ES-L}
|
||||
\date{15 novembre 2019}
|
||||
|
||||
\xsimsetup{
|
||||
solution/print = false
|
||||
}
|
||||
|
||||
\begin{document}
|
||||
\maketitle
|
||||
|
||||
Une part importante de la note sera dédiée à la rédaction, aux explications et à l'utilisation des notations mathématiques.
|
||||
|
||||
\begin{exercise}[subtitle={Débit}]
|
||||
Une commune de \np{2500} habitants au 1\up{er} janvier 2018 voit sa population augmenter de 9\,\% tous les ans.
|
||||
|
||||
Pour tout entier naturel $n$, on note $h_n$ le nombre d'habitants de l'année $2018 + n$.
|
||||
|
||||
\smallskip
|
||||
|
||||
\begin{enumerate}
|
||||
\item Déterminer la nature de la suite $(h_n)$, préciser ses éléments caractéristiques et exprimer $h_n$ en fonction de $n$.
|
||||
\end{enumerate}
|
||||
|
||||
\smallskip
|
||||
|
||||
La municipalité de cette commune a conclu un marché avec un fournisseur d'accès internet qui engage ce dernier à fournir un débit total de \np{17500}~Mbit/s au 1\up{er} janvier 2018 et à augmenter ce débit de - 6.8200\,\% par an.
|
||||
|
||||
Pour tout entier naturel $n$, on note $d_n$ le débit total dont la commune dispose l'année $2018 + n$.
|
||||
|
||||
\begin{enumerate}
|
||||
\setcounter{enumi}{1}
|
||||
\item Déterminer la nature de la suite $(d_n)$, préciser ses éléments caractéristiques et exprimer $d_n$ en fonction de $n$.
|
||||
\end{enumerate}
|
||||
|
||||
On s'intéresse maintenant au débit par habitant en supposant que celui-ci est réparti équitablement et que toute la population bénéficie d'une connexion internet individuelle.
|
||||
|
||||
Pour tout entier naturel $n$ on note $u_n$ le débit par habitant pour l'année $2018 + n$ et on admet que $u_n = \dfrac{d_n}{h_n}$.
|
||||
\begin{enumerate}
|
||||
\setcounter{enumi}{2}
|
||||
\item Calculer $u_0$ et $u_1$.
|
||||
\item Montrer pour tout entier naturel $n$ on a $u_n = 7 \times 0.98^n$.
|
||||
\item En déduire la nature de la suite $\left(u_n\right)$ et ses caractéristiques.
|
||||
\item Déterminer le sens de variations de la $\left(u_n\right)$.
|
||||
Interpréter ce résultat dans le contexte de l'énoncé.
|
||||
\end{enumerate}
|
||||
Le marché passé avec le fournisseur d'accès internet prévoit également que si le débit passe en dessous de 5 Mbit/s par habitant alors ce dernier doit changer la technologie utilisée pour la réalisation de son réseau.
|
||||
|
||||
\begin{enumerate}
|
||||
\setcounter{enumi}{6}
|
||||
\item En quelle année le fournisseur d'accès sera-t-il dans l'obligation de changer sa technologie?
|
||||
\end{enumerate}
|
||||
\end{exercise}
|
||||
\begin{solution}
|
||||
\begin{enumerate}
|
||||
\item Augmenter de 9\% revient à multiplier pas 1.09. La suite $(h_n)$ est donc géométrique de raison 1.09 et de premier terme 2500. On en déduit $h_n$ en fonction de $n$
|
||||
\[
|
||||
h_n = 2500\times 1.09^n
|
||||
\]
|
||||
\item Augmenter de - 6.8200\% revient à multiplier pas 1.0682. La suite $(d_n)$ est donc géométrique de raison 1.0682 et de premier terme 17500. On en déduit $d_n$ en fonction de $n$
|
||||
\[
|
||||
d_n = 17500\times 1.0682^n
|
||||
\]
|
||||
\item
|
||||
\begin{enumerate}
|
||||
\item
|
||||
\[
|
||||
u_0 = \frac{d_0}{h_0} = \frac{17500}{2500} = 7
|
||||
\]
|
||||
\[
|
||||
u_1 = \frac{d_1}{h_1} = \frac{18693.5000}{2725} = 6.86
|
||||
\]
|
||||
\item Démonstration de la formule
|
||||
\begin{eqnarray*}
|
||||
u_n &=& \frac{d_n}{h_n} = \frac{17500\times1.0682^n}{2500\times1.09^n} \\
|
||||
u_n &=& \frac{17500}{2500}\times\left(\frac{1.0682}{1.09}\right)^n \\
|
||||
u_n &=& 7\times0.98^n
|
||||
\end{eqnarray*}
|
||||
\item On reconnaît la forme d'une suite géométrique de raison 0.98 et de premier terme 7.
|
||||
\item La raison, $q = 0.98$, est inférieur à 1 donc la suite est décroissante. Ce qui signifie que le débit par habitant va diminuer.
|
||||
\end{enumerate}
|
||||
\item Avec le tableau de la calculatrice, on calculer les valeurs de $u_n$ jusqu'à passer en dessous de 5. On trouve $n = 17$ avec $u_{17} = 4.965252363264521426471716467$
|
||||
|
||||
\end{enumerate}
|
||||
\end{solution}
|
||||
\begin{exercise}[subtitle={Étude de fonctions}]
|
||||
\section*{Partie A}
|
||||
Dans cette partie, on étudie la fonction
|
||||
\[
|
||||
f(x) = - 2x^2 + 3x - 6
|
||||
\]
|
||||
\begin{enumerate}
|
||||
\item Calculer la dérivé de $f$.
|
||||
\item Étudier le signe de la dérivée $f'$ puis en déduire le tableau de signe de $f$.
|
||||
\end{enumerate}
|
||||
\section*{Partie B}
|
||||
Dans cette partie, on étudie la fonction
|
||||
\[
|
||||
g(x) = - 9x^3 + 3x^2 + 2x + 2
|
||||
\]
|
||||
\begin{enumerate}
|
||||
\item À l'aide d'une calculatrice ou d'un ordinateur, tracer puis reporter sur votre copie la représentation graphique de $g$ en y indiquant les informations remarquables de ce graphique.
|
||||
\item Sur quel(s) intervalle(s) la fonction est convexe? concave? Y a-t-il des points d'inflexions?
|
||||
\item Calculer la dérivé de $g$.
|
||||
\item Étudier le signe de la dérivée $g'$ puis en déduire le tableau de variations de $g$.
|
||||
\item Déterminer l'équation de la tangente en $x=0$.
|
||||
\item Dériver $g'$ pour calculer $g''$.
|
||||
\item Étudier le signe de $g''$ pour en déduire la convexité de $g$ grâce au calcul puis localiser précisément le point d'inflexion.
|
||||
\end{enumerate}
|
||||
\end{exercise}
|
||||
\begin{solution}
|
||||
\section*{Partie A}
|
||||
\begin{enumerate}
|
||||
\item $3 - 4x$
|
||||
\item Correction non disponible
|
||||
\end{enumerate}
|
||||
|
||||
\section*{Partie B}
|
||||
\begin{enumerate}
|
||||
\item Correction non disponible
|
||||
\item Correction non disponible
|
||||
\item $g'(x) = 2 + 6x - 27x^2$
|
||||
\item
|
||||
On commence par calculer le discriminant de $g'(x)=2 + 6x - 27x^2$.
|
||||
\begin{eqnarray*}
|
||||
\Delta & = & b^2-4ac \\
|
||||
\Delta & = & 6^{2} - 4 \times - 27 \times 2 \\
|
||||
\Delta & = & 36 + 108 \times 2 \\
|
||||
\Delta & = & 36 + 216 \\
|
||||
\Delta & = & 252
|
||||
\end{eqnarray*}
|
||||
|
||||
comme $\Delta = 252 > 0$ donc $P$ a deux racines
|
||||
|
||||
\begin{eqnarray*}
|
||||
x_1 & = & \frac{-b - \sqrt{\Delta}}{2a} = \frac{6 - \sqrt{252}}{2 \times - 27} = - 0.18286125678495452 \\
|
||||
x_2 & = & \frac{-b + \sqrt{\Delta}}{2a} = \frac{6 + \sqrt{252}}{2 \times - 27} = 0.4050834790071768
|
||||
\end{eqnarray*}
|
||||
Ainsi, $g'$ est du signe de $a=- 27$ en dehors des racines.
|
||||
|
||||
|
||||
Le tableau de variation non disponible en correction
|
||||
\item Équation de la tangente: $y = 2x + 2$
|
||||
\item $g''(x) = - 54x + 6$
|
||||
\end{enumerate}
|
||||
\end{solution}
|
||||
|
||||
\end{document}
|
||||
|
||||
%%% Local Variables:
|
||||
%%% mode: latex
|
||||
%%% TeX-master: "master"
|
||||
%%% End:
|
||||
157
TES/DM/DM_19_10/21_DM_19_10.tex
Normal file
157
TES/DM/DM_19_10/21_DM_19_10.tex
Normal file
@@ -0,0 +1,157 @@
|
||||
\documentclass[a4paper,10pt]{article}
|
||||
\usepackage{tasks}
|
||||
\usepackage{myXsim}
|
||||
|
||||
\title{DM 1 -- VISENTIN Aurélie}
|
||||
\tribe{Terminale ES-L}
|
||||
\date{15 novembre 2019}
|
||||
|
||||
\xsimsetup{
|
||||
solution/print = false
|
||||
}
|
||||
|
||||
\begin{document}
|
||||
\maketitle
|
||||
|
||||
Une part importante de la note sera dédiée à la rédaction, aux explications et à l'utilisation des notations mathématiques.
|
||||
|
||||
\begin{exercise}[subtitle={Débit}]
|
||||
Une commune de \np{2200} habitants au 1\up{er} janvier 2018 voit sa population augmenter de 7\,\% tous les ans.
|
||||
|
||||
Pour tout entier naturel $n$, on note $h_n$ le nombre d'habitants de l'année $2018 + n$.
|
||||
|
||||
\smallskip
|
||||
|
||||
\begin{enumerate}
|
||||
\item Déterminer la nature de la suite $(h_n)$, préciser ses éléments caractéristiques et exprimer $h_n$ en fonction de $n$.
|
||||
\end{enumerate}
|
||||
|
||||
\smallskip
|
||||
|
||||
La municipalité de cette commune a conclu un marché avec un fournisseur d'accès internet qui engage ce dernier à fournir un débit total de \np{15400}~Mbit/s au 1\up{er} janvier 2018 et à augmenter ce débit de - 5.9300\,\% par an.
|
||||
|
||||
Pour tout entier naturel $n$, on note $d_n$ le débit total dont la commune dispose l'année $2018 + n$.
|
||||
|
||||
\begin{enumerate}
|
||||
\setcounter{enumi}{1}
|
||||
\item Déterminer la nature de la suite $(d_n)$, préciser ses éléments caractéristiques et exprimer $d_n$ en fonction de $n$.
|
||||
\end{enumerate}
|
||||
|
||||
On s'intéresse maintenant au débit par habitant en supposant que celui-ci est réparti équitablement et que toute la population bénéficie d'une connexion internet individuelle.
|
||||
|
||||
Pour tout entier naturel $n$ on note $u_n$ le débit par habitant pour l'année $2018 + n$ et on admet que $u_n = \dfrac{d_n}{h_n}$.
|
||||
\begin{enumerate}
|
||||
\setcounter{enumi}{2}
|
||||
\item Calculer $u_0$ et $u_1$.
|
||||
\item Montrer pour tout entier naturel $n$ on a $u_n = 7 \times 0.99^n$.
|
||||
\item En déduire la nature de la suite $\left(u_n\right)$ et ses caractéristiques.
|
||||
\item Déterminer le sens de variations de la $\left(u_n\right)$.
|
||||
Interpréter ce résultat dans le contexte de l'énoncé.
|
||||
\end{enumerate}
|
||||
Le marché passé avec le fournisseur d'accès internet prévoit également que si le débit passe en dessous de 5 Mbit/s par habitant alors ce dernier doit changer la technologie utilisée pour la réalisation de son réseau.
|
||||
|
||||
\begin{enumerate}
|
||||
\setcounter{enumi}{6}
|
||||
\item En quelle année le fournisseur d'accès sera-t-il dans l'obligation de changer sa technologie?
|
||||
\end{enumerate}
|
||||
\end{exercise}
|
||||
\begin{solution}
|
||||
\begin{enumerate}
|
||||
\item Augmenter de 7\% revient à multiplier pas 1.07. La suite $(h_n)$ est donc géométrique de raison 1.07 et de premier terme 2200. On en déduit $h_n$ en fonction de $n$
|
||||
\[
|
||||
h_n = 2200\times 1.07^n
|
||||
\]
|
||||
\item Augmenter de - 5.9300\% revient à multiplier pas 1.0593. La suite $(d_n)$ est donc géométrique de raison 1.0593 et de premier terme 15400. On en déduit $d_n$ en fonction de $n$
|
||||
\[
|
||||
d_n = 15400\times 1.0593^n
|
||||
\]
|
||||
\item
|
||||
\begin{enumerate}
|
||||
\item
|
||||
\[
|
||||
u_0 = \frac{d_0}{h_0} = \frac{15400}{2200} = 7
|
||||
\]
|
||||
\[
|
||||
u_1 = \frac{d_1}{h_1} = \frac{16313.2200}{2354} = 6.93
|
||||
\]
|
||||
\item Démonstration de la formule
|
||||
\begin{eqnarray*}
|
||||
u_n &=& \frac{d_n}{h_n} = \frac{15400\times1.0593^n}{2200\times1.07^n} \\
|
||||
u_n &=& \frac{15400}{2200}\times\left(\frac{1.0593}{1.07}\right)^n \\
|
||||
u_n &=& 7\times0.99^n
|
||||
\end{eqnarray*}
|
||||
\item On reconnaît la forme d'une suite géométrique de raison 0.99 et de premier terme 7.
|
||||
\item La raison, $q = 0.99$, est inférieur à 1 donc la suite est décroissante. Ce qui signifie que le débit par habitant va diminuer.
|
||||
\end{enumerate}
|
||||
\item Avec le tableau de la calculatrice, on calculer les valeurs de $u_n$ jusqu'à passer en dessous de 5. On trouve $n = 34$ avec $u_{34} = 4.973872590906046483848960634$
|
||||
|
||||
\end{enumerate}
|
||||
\end{solution}
|
||||
\begin{exercise}[subtitle={Étude de fonctions}]
|
||||
\section*{Partie A}
|
||||
Dans cette partie, on étudie la fonction
|
||||
\[
|
||||
f(x) = - 5x^2 + 9x + 10
|
||||
\]
|
||||
\begin{enumerate}
|
||||
\item Calculer la dérivé de $f$.
|
||||
\item Étudier le signe de la dérivée $f'$ puis en déduire le tableau de signe de $f$.
|
||||
\end{enumerate}
|
||||
\section*{Partie B}
|
||||
Dans cette partie, on étudie la fonction
|
||||
\[
|
||||
g(x) = - 2x^3 - x^2 + 4x - 7
|
||||
\]
|
||||
\begin{enumerate}
|
||||
\item À l'aide d'une calculatrice ou d'un ordinateur, tracer puis reporter sur votre copie la représentation graphique de $g$ en y indiquant les informations remarquables de ce graphique.
|
||||
\item Sur quel(s) intervalle(s) la fonction est convexe? concave? Y a-t-il des points d'inflexions?
|
||||
\item Calculer la dérivé de $g$.
|
||||
\item Étudier le signe de la dérivée $g'$ puis en déduire le tableau de variations de $g$.
|
||||
\item Déterminer l'équation de la tangente en $x=0$.
|
||||
\item Dériver $g'$ pour calculer $g''$.
|
||||
\item Étudier le signe de $g''$ pour en déduire la convexité de $g$ grâce au calcul puis localiser précisément le point d'inflexion.
|
||||
\end{enumerate}
|
||||
\end{exercise}
|
||||
\begin{solution}
|
||||
\section*{Partie A}
|
||||
\begin{enumerate}
|
||||
\item $9 - 10x$
|
||||
\item Correction non disponible
|
||||
\end{enumerate}
|
||||
|
||||
\section*{Partie B}
|
||||
\begin{enumerate}
|
||||
\item Correction non disponible
|
||||
\item Correction non disponible
|
||||
\item $g'(x) = 4 - 2x - 6x^2$
|
||||
\item
|
||||
On commence par calculer le discriminant de $g'(x)=4 - 2x - 6x^2$.
|
||||
\begin{eqnarray*}
|
||||
\Delta & = & b^2-4ac \\
|
||||
\Delta & = & - 2^{2} - 4 \times - 6 \times 4 \\
|
||||
\Delta & = & 4 + 24 \times 4 \\
|
||||
\Delta & = & 4 + 96 \\
|
||||
\Delta & = & 100
|
||||
\end{eqnarray*}
|
||||
|
||||
comme $\Delta = 100 > 0$ donc $P$ a deux racines
|
||||
|
||||
\begin{eqnarray*}
|
||||
x_1 & = & \frac{-b - \sqrt{\Delta}}{2a} = \frac{- 2 - \sqrt{100}}{2 \times - 6} = - 1 \\
|
||||
x_2 & = & \frac{-b + \sqrt{\Delta}}{2a} = \frac{- 2 + \sqrt{100}}{2 \times - 6} = 0.6666666666666666
|
||||
\end{eqnarray*}
|
||||
Ainsi, $g'$ est du signe de $a=- 6$ en dehors des racines.
|
||||
|
||||
|
||||
Le tableau de variation non disponible en correction
|
||||
\item Équation de la tangente: $y = 4x + - 7$
|
||||
\item $g''(x) = - 12x - 2$
|
||||
\end{enumerate}
|
||||
\end{solution}
|
||||
|
||||
\end{document}
|
||||
|
||||
%%% Local Variables:
|
||||
%%% mode: latex
|
||||
%%% TeX-master: "master"
|
||||
%%% End:
|
||||
BIN
TES/DM/DM_19_10/all_DM_19_10.pdf
Normal file
BIN
TES/DM/DM_19_10/all_DM_19_10.pdf
Normal file
Binary file not shown.
BIN
TES/DM/DM_19_10/corr_DM_19_10.pdf
Normal file
BIN
TES/DM/DM_19_10/corr_DM_19_10.pdf
Normal file
Binary file not shown.
27
TES/DM/DM_19_10/tpl_DM_19_10.tex
Normal file
27
TES/DM/DM_19_10/tpl_DM_19_10.tex
Normal file
@@ -0,0 +1,27 @@
|
||||
\documentclass[a4paper,10pt]{article}
|
||||
\usepackage{tasks}
|
||||
\usepackage{myXsim}
|
||||
|
||||
\title{DM 1 -- \Var{infos["name"]}}
|
||||
\tribe{Terminale ES-L}
|
||||
\date{15 novembre 2019}
|
||||
|
||||
\xsimsetup{
|
||||
solution/print = false
|
||||
}
|
||||
|
||||
\begin{document}
|
||||
\maketitle
|
||||
|
||||
Une part importante de la note sera dédiée à la rédaction, aux explications et à l'utilisation des notations mathématiques.
|
||||
|
||||
\Block{include "./tpl_debit.tex"}
|
||||
\Block{include "./tpl_variation.tex"}
|
||||
|
||||
\end{document}
|
||||
|
||||
%%% Local Variables:
|
||||
%%% mode: latex
|
||||
%%% TeX-master: "master"
|
||||
%%% End:
|
||||
|
||||
82
TES/DM/DM_19_10/tpl_debit.tex
Normal file
82
TES/DM/DM_19_10/tpl_debit.tex
Normal file
@@ -0,0 +1,82 @@
|
||||
%- set u0 = Integer.random(min_value=7, max_value=12)
|
||||
%- set uq = Decimal.random(min_value=0.9, max_value=1, digits=2)
|
||||
%- set h0 = Integer.random(min_value=15, max_value=25)*100
|
||||
%- set hp = Integer.random(min_value=5, max_value=9)
|
||||
%- set hq = 1 + hp*Decimal("0.01")
|
||||
%- set d0 = u0*h0
|
||||
%- set dq = uq*hq
|
||||
%- set dp = (1-dq)*100
|
||||
\begin{exercise}[subtitle={Débit}]
|
||||
Une commune de \np{\Var{h0}} habitants au 1\up{er} janvier 2018 voit sa population augmenter de \Var{hp}\,\% tous les ans.
|
||||
|
||||
Pour tout entier naturel $n$, on note $h_n$ le nombre d'habitants de l'année $2018 + n$.
|
||||
|
||||
\smallskip
|
||||
|
||||
\begin{enumerate}
|
||||
\item Déterminer la nature de la suite $(h_n)$, préciser ses éléments caractéristiques et exprimer $h_n$ en fonction de $n$.
|
||||
\end{enumerate}
|
||||
|
||||
\smallskip
|
||||
|
||||
La municipalité de cette commune a conclu un marché avec un fournisseur d'accès internet qui engage ce dernier à fournir un débit total de \np{\Var{d0}}~Mbit/s au 1\up{er} janvier 2018 et à augmenter ce débit de \Var{dp}\,\% par an.
|
||||
|
||||
Pour tout entier naturel $n$, on note $d_n$ le débit total dont la commune dispose l'année $2018 + n$.
|
||||
|
||||
\begin{enumerate}
|
||||
\setcounter{enumi}{1}
|
||||
\item Déterminer la nature de la suite $(d_n)$, préciser ses éléments caractéristiques et exprimer $d_n$ en fonction de $n$.
|
||||
\end{enumerate}
|
||||
|
||||
On s'intéresse maintenant au débit par habitant en supposant que celui-ci est réparti équitablement et que toute la population bénéficie d'une connexion internet individuelle.
|
||||
|
||||
Pour tout entier naturel $n$ on note $u_n$ le débit par habitant pour l'année $2018 + n$ et on admet que $u_n = \dfrac{d_n}{h_n}$.
|
||||
\begin{enumerate}
|
||||
\setcounter{enumi}{2}
|
||||
\item Calculer $u_0$ et $u_1$.
|
||||
\item Montrer pour tout entier naturel $n$ on a $u_n = \Var{u0} \times \Var{uq}^n$.
|
||||
\item En déduire la nature de la suite $\left(u_n\right)$ et ses caractéristiques.
|
||||
\item Déterminer le sens de variations de la $\left(u_n\right)$.
|
||||
Interpréter ce résultat dans le contexte de l'énoncé.
|
||||
\end{enumerate}
|
||||
Le marché passé avec le fournisseur d'accès internet prévoit également que si le débit passe en dessous de 5 Mbit/s par habitant alors ce dernier doit changer la technologie utilisée pour la réalisation de son réseau.
|
||||
|
||||
\begin{enumerate}
|
||||
\setcounter{enumi}{6}
|
||||
\item En quelle année le fournisseur d'accès sera-t-il dans l'obligation de changer sa technologie?
|
||||
\end{enumerate}
|
||||
\end{exercise}
|
||||
\begin{solution}
|
||||
\begin{enumerate}
|
||||
\item Augmenter de \Var{hp}\% revient à multiplier pas \Var{hq}. La suite $(h_n)$ est donc géométrique de raison \Var{hq} et de premier terme \Var{h0}. On en déduit $h_n$ en fonction de $n$
|
||||
\[
|
||||
h_n = \Var{h0}\times \Var{hq}^n
|
||||
\]
|
||||
\item Augmenter de \Var{dp}\% revient à multiplier pas \Var{dq}. La suite $(d_n)$ est donc géométrique de raison \Var{dq} et de premier terme \Var{d0}. On en déduit $d_n$ en fonction de $n$
|
||||
\[
|
||||
d_n = \Var{d0}\times \Var{dq}^n
|
||||
\]
|
||||
\item
|
||||
\begin{enumerate}
|
||||
\item
|
||||
\[
|
||||
u_0 = \frac{d_0}{h_0} = \frac{\Var{d0}}{\Var{h0}} = \Var{u0}
|
||||
\]
|
||||
\[
|
||||
u_1 = \frac{d_1}{h_1} = \frac{\Var{d0*dq}}{\Var{h0*hq}} = \Var{u0*uq}
|
||||
\]
|
||||
\item Démonstration de la formule
|
||||
\begin{eqnarray*}
|
||||
u_n &=& \frac{d_n}{h_n} = \frac{\Var{d0}\times\Var{dq}^n}{\Var{h0}\times\Var{hq}^n} \\
|
||||
u_n &=& \frac{\Var{d0}}{\Var{h0}}\times\left(\frac{\Var{dq}}{\Var{hq}}\right)^n \\
|
||||
u_n &=& \Var{u0}\times\Var{uq}^n
|
||||
\end{eqnarray*}
|
||||
\item On reconnaît la forme d'une suite géométrique de raison \Var{uq} et de premier terme \Var{u0}.
|
||||
\item La raison, $q = \Var{uq}$, est inférieur à 1 donc la suite est décroissante. Ce qui signifie que le débit par habitant va diminuer.
|
||||
\end{enumerate}
|
||||
%- set n = int(log(5/u0.raw)/log(uq.raw))
|
||||
%- set N = ceil(log(5/u0.raw)/log(uq.raw))
|
||||
\item Avec le tableau de la calculatrice, on calculer les valeurs de $u_n$ jusqu'à passer en dessous de 5. On trouve $n = \Var{N}$ avec $u_{\Var{N}} = \Var{u0.raw*uq.raw**N}$
|
||||
|
||||
\end{enumerate}
|
||||
\end{solution}
|
||||
66
TES/DM/DM_19_10/tpl_variation.tex
Normal file
66
TES/DM/DM_19_10/tpl_variation.tex
Normal file
@@ -0,0 +1,66 @@
|
||||
%- macro solveEquation(P)
|
||||
On commence par calculer le discriminant de $g'(x)=\Var{P}$.
|
||||
\begin{eqnarray*}
|
||||
\Delta & = & b^2-4ac \\
|
||||
\Var{P.delta.explain()|calculus(name="\\Delta")}
|
||||
\end{eqnarray*}
|
||||
|
||||
comme $\Delta = \Var{P.delta} > 0$ donc $P$ a deux racines
|
||||
|
||||
\begin{eqnarray*}
|
||||
x_1 & = & \frac{-b - \sqrt{\Delta}}{2a} = \frac{\Var{-P.b} - \sqrt{\Var{P.delta}}}{2 \times \Var{P.a}} = \Var{P.roots[0] } \\
|
||||
x_2 & = & \frac{-b + \sqrt{\Delta}}{2a} = \frac{\Var{-P.b} + \sqrt{\Var{P.delta}}}{2 \times \Var{P.a}} = \Var{P.roots[1] }
|
||||
\end{eqnarray*}
|
||||
Ainsi, $g'$ est du signe de $a=\Var{P.a}$ en dehors des racines.
|
||||
%- endmacro
|
||||
|
||||
\begin{exercise}[subtitle={Étude de fonctions}]
|
||||
\section*{Partie A}
|
||||
Dans cette partie, on étudie la fonction
|
||||
%- set f = Expression.random("{a}*x^2+{b}*x+{c}")
|
||||
%- set Df = f.differentiate()
|
||||
\[
|
||||
f(x) = \Var{f}
|
||||
\]
|
||||
\begin{enumerate}
|
||||
\item Calculer la dérivé de $f$.
|
||||
\item Étudier le signe de la dérivée $f'$ puis en déduire le tableau de signe de $f$.
|
||||
\end{enumerate}
|
||||
\section*{Partie B}
|
||||
Dans cette partie, on étudie la fonction
|
||||
%- set g = Expression.random("{a}*x^3+{b}*x^2+{c}*x+{d}", conditions=["4*b**2-4*3*a*c>0"])
|
||||
%- set Dg = g.differentiate()
|
||||
%- set maxg = round(max(abs(g(Dg.roots[0]).raw),abs(g(Dg.roots[0]).raw)),1)
|
||||
\[
|
||||
g(x) = \Var{g}
|
||||
\]
|
||||
\begin{enumerate}
|
||||
\item À l'aide d'une calculatrice ou d'un ordinateur, tracer puis reporter sur votre copie la représentation graphique de $g$ en y indiquant les informations remarquables de ce graphique.
|
||||
\item Sur quel(s) intervalle(s) la fonction est convexe? concave? Y a-t-il des points d'inflexions?
|
||||
\item Calculer la dérivé de $g$.
|
||||
\item Étudier le signe de la dérivée $g'$ puis en déduire le tableau de variations de $g$.
|
||||
\item Déterminer l'équation de la tangente en $x=0$.
|
||||
\item Dériver $g'$ pour calculer $g''$.
|
||||
\item Étudier le signe de $g''$ pour en déduire la convexité de $g$ grâce au calcul puis localiser précisément le point d'inflexion.
|
||||
\end{enumerate}
|
||||
\end{exercise}
|
||||
\begin{solution}
|
||||
\section*{Partie A}
|
||||
\begin{enumerate}
|
||||
\item $\Var{Df}$
|
||||
\item Correction non disponible
|
||||
\end{enumerate}
|
||||
|
||||
\section*{Partie B}
|
||||
\begin{enumerate}
|
||||
\item Correction non disponible
|
||||
\item Correction non disponible
|
||||
\item $g'(x) = \Var{Dg}$
|
||||
\item
|
||||
\Var{solveEquation(Dg)}
|
||||
|
||||
Le tableau de variation non disponible en correction
|
||||
\item Équation de la tangente: $y = \Var{Dg(0)}x + \Var{g(0)}$
|
||||
\item $g''(x) = \Var{Dg.differentiate()}$
|
||||
\end{enumerate}
|
||||
\end{solution}
|
||||
121
TES/DM/DM_20_02/01_DM_20_02.tex
Normal file
121
TES/DM/DM_20_02/01_DM_20_02.tex
Normal file
@@ -0,0 +1,121 @@
|
||||
\documentclass[a4paper,10pt]{article}
|
||||
\usepackage{tasks}
|
||||
\usepackage{myXsim}
|
||||
|
||||
\title{DM 2 -- AIT BEN SAID Loubna}
|
||||
\tribe{Terminale ES-L}
|
||||
\date{9 mars 2020}
|
||||
|
||||
\xsimsetup{
|
||||
solution/print = false
|
||||
}
|
||||
|
||||
\begin{document}
|
||||
\maketitle
|
||||
|
||||
\begin{exercise}[subtitle={Étude de fonction}]
|
||||
On considère la fonction dérivable $f$ définie sur $I = [0~;~64]$ par:
|
||||
|
||||
\[f(x) = \np{3000}(x + 16)\text{e}^{- 0.0625x}.\]
|
||||
|
||||
\textbf{Partie A - Étude graphique}
|
||||
|
||||
On a représenté sur le graphique en annexe, la courbe représentative de la fonction $f$.
|
||||
|
||||
\begin{enumerate}
|
||||
\item Avec un tableur tracer et imprimer la courbe représentative de $f$ sur $I$
|
||||
\end{enumerate}
|
||||
\emph{Répondre aux questions suivantes par lecture graphique.}
|
||||
\begin{enumerate}
|
||||
\setcounter{enumi}{1}
|
||||
\item Résoudre graphiquement et de façon approchée l'équation $f(x) = \np{24000}$.
|
||||
\item Donner un encadrement de la quantité
|
||||
\[
|
||||
\int_{2}^{32} f(x) \; dx
|
||||
\]
|
||||
Vous expliquerez votre démarche en utilisant le graphique.
|
||||
\end{enumerate}
|
||||
|
||||
\medskip
|
||||
|
||||
\textbf{Partie B - Étude théorique}
|
||||
|
||||
\begin{enumerate}
|
||||
\setcounter{enumi}{4}
|
||||
\item Étude des variations.
|
||||
\begin{enumerate}
|
||||
\item On note $f'$ la dérivée de la fonction $f$ sur $\intFF{0}{64}$.
|
||||
|
||||
Démontrer que pour tout $x$ de $\intFF{0}{20}$ , $f'(x) = - 187.5000x\text{e}^{-0.0625x}$.
|
||||
|
||||
\item En déduire le sens de variation de $f$ et dresser son tableau des variations sur l'intervalle $\intFF{0}{64}$. Si nécessaire, arrondir à l'unité les valeurs présentes dans le tableau.
|
||||
|
||||
\item Démontrer que l'équation $f(x) = \np{24000}$ admet une unique solution $\alpha$ sur $\intFF{0}{64}$, puis donner une valeur approchée de $\alpha$ à $10^{-2}$ près à l'aide de la calculatrice.
|
||||
\end{enumerate}
|
||||
|
||||
\item Étude de la convexité
|
||||
\begin{enumerate}
|
||||
\item On note $f''$ la dérivée seconde de $f$ sur $\intFF{0}{64}$.
|
||||
|
||||
Démontrer que pour tout $x$ de $\intFF{0}{20}$ , $f''(x) = (11.71875000x-187.5000)\text{e}^{-0.0625x}$.
|
||||
|
||||
\item Démontrer que $f$ admet un point d'inflexion dont on donnera son abscisse.
|
||||
\end{enumerate}
|
||||
|
||||
\item Aire sous la courbe
|
||||
\begin{enumerate}
|
||||
\item On souhaite approximer la fonction $f$ sur l'intervalle $I$ par la droite $D$ qui relie les points $(0;f(0))$ et $(64, f(64))$. Tracer cette droite sur le graphique.
|
||||
\item On note $g$ la fonction affine qui décrit cette droite $D$. Détermine l'expression de $g$
|
||||
\item Calculer $\displaystyle \int_0^{64} g(x)\; dx$
|
||||
\item Avec la calculatrice, calculer une valeur approchée de $\displaystyle\int_{0}^{64} f(x) \; dx$
|
||||
\item Comparer les valeurs trouvées aux deux questions précédentes. Comment s'explique l'écart entre ces deux valeurs?
|
||||
\end{enumerate}
|
||||
\end{enumerate}
|
||||
|
||||
\medskip
|
||||
|
||||
\textbf{Partie C - Application économique}
|
||||
|
||||
Une entreprise a pris la décision de fermer son usine de production de smartphones en 64 mois.
|
||||
|
||||
La fonction capacité de production de cette usine est modélisée sur l'intervalle $\intFF{0}{64}$ par la fonction $f$ étudiée dans les parties A et B.
|
||||
|
||||
Le nombre $x$ représente le temps en mois après la décision de la fermeture du site et le nombre $f(x)$ représente capacité production de smartphone au moment $x$.
|
||||
|
||||
Utiliser les résultats de la partie B afin de répondre aux questions suivantes:
|
||||
|
||||
\begin{enumerate}
|
||||
\setcounter{enumi}{7}
|
||||
\item Combien de smartphones pouvaient être produit à la fermeture de l'usine?
|
||||
\item Pendant combien de temps la capacité de production de l'usine a réussi à se maintenir au dessus de \np{24000}?
|
||||
\item Combien de smartphones ont pu être produit entre la prise de décision et la fermeture de l'usine?
|
||||
\end{enumerate}
|
||||
|
||||
\end{exercise}
|
||||
|
||||
\begin{solution}
|
||||
\begin{enumerate}
|
||||
\item
|
||||
\begin{tikzpicture}[yscale=1, xscale=0.234375]
|
||||
\tkzInit[xmin=0,xmax=65,xstep=1,
|
||||
ymin=0,ymax=51000,ystep=5000]
|
||||
\tkzGrid
|
||||
\tkzGrid[sub, subxstep=0.2, subystep=1000.0]
|
||||
\tkzAxeXY[up space=0.5,right space=.2]
|
||||
\tkzFct[domain = 0:64, line width=1pt]{3000*(x+16)*exp(-0.0625*x)}
|
||||
%M\tkzFct[domain = 0:64, line width=1pt, blue]{24000}
|
||||
\end{tikzpicture}
|
||||
\item Tracer la droite $y=24000$. C'est l'abscisse de l'intersection entre cette droite et la courbe
|
||||
\item
|
||||
\item
|
||||
|
||||
\end{enumerate}
|
||||
|
||||
\end{solution}
|
||||
|
||||
\end{document}
|
||||
|
||||
%%% Local Variables:
|
||||
%%% mode: latex
|
||||
%%% TeX-master: "master"
|
||||
%%% End:
|
||||
121
TES/DM/DM_20_02/02_DM_20_02.tex
Normal file
121
TES/DM/DM_20_02/02_DM_20_02.tex
Normal file
@@ -0,0 +1,121 @@
|
||||
\documentclass[a4paper,10pt]{article}
|
||||
\usepackage{tasks}
|
||||
\usepackage{myXsim}
|
||||
|
||||
\title{DM 2 -- BATEMAN Amélie}
|
||||
\tribe{Terminale ES-L}
|
||||
\date{9 mars 2020}
|
||||
|
||||
\xsimsetup{
|
||||
solution/print = false
|
||||
}
|
||||
|
||||
\begin{document}
|
||||
\maketitle
|
||||
|
||||
\begin{exercise}[subtitle={Étude de fonction}]
|
||||
On considère la fonction dérivable $f$ définie sur $I = [0~;~32]$ par:
|
||||
|
||||
\[f(x) = \np{10000}(x + 8)\text{e}^{- 0.125x}.\]
|
||||
|
||||
\textbf{Partie A - Étude graphique}
|
||||
|
||||
On a représenté sur le graphique en annexe, la courbe représentative de la fonction $f$.
|
||||
|
||||
\begin{enumerate}
|
||||
\item Avec un tableur tracer et imprimer la courbe représentative de $f$ sur $I$
|
||||
\end{enumerate}
|
||||
\emph{Répondre aux questions suivantes par lecture graphique.}
|
||||
\begin{enumerate}
|
||||
\setcounter{enumi}{1}
|
||||
\item Résoudre graphiquement et de façon approchée l'équation $f(x) = \np{40000}$.
|
||||
\item Donner un encadrement de la quantité
|
||||
\[
|
||||
\int_{2}^{16} f(x) \; dx
|
||||
\]
|
||||
Vous expliquerez votre démarche en utilisant le graphique.
|
||||
\end{enumerate}
|
||||
|
||||
\medskip
|
||||
|
||||
\textbf{Partie B - Étude théorique}
|
||||
|
||||
\begin{enumerate}
|
||||
\setcounter{enumi}{4}
|
||||
\item Étude des variations.
|
||||
\begin{enumerate}
|
||||
\item On note $f'$ la dérivée de la fonction $f$ sur $\intFF{0}{32}$.
|
||||
|
||||
Démontrer que pour tout $x$ de $\intFF{0}{20}$ , $f'(x) = - 1250x\text{e}^{-0.125x}$.
|
||||
|
||||
\item En déduire le sens de variation de $f$ et dresser son tableau des variations sur l'intervalle $\intFF{0}{32}$. Si nécessaire, arrondir à l'unité les valeurs présentes dans le tableau.
|
||||
|
||||
\item Démontrer que l'équation $f(x) = \np{40000}$ admet une unique solution $\alpha$ sur $\intFF{0}{32}$, puis donner une valeur approchée de $\alpha$ à $10^{-2}$ près à l'aide de la calculatrice.
|
||||
\end{enumerate}
|
||||
|
||||
\item Étude de la convexité
|
||||
\begin{enumerate}
|
||||
\item On note $f''$ la dérivée seconde de $f$ sur $\intFF{0}{32}$.
|
||||
|
||||
Démontrer que pour tout $x$ de $\intFF{0}{20}$ , $f''(x) = (156.250x-1250)\text{e}^{-0.125x}$.
|
||||
|
||||
\item Démontrer que $f$ admet un point d'inflexion dont on donnera son abscisse.
|
||||
\end{enumerate}
|
||||
|
||||
\item Aire sous la courbe
|
||||
\begin{enumerate}
|
||||
\item On souhaite approximer la fonction $f$ sur l'intervalle $I$ par la droite $D$ qui relie les points $(0;f(0))$ et $(32, f(32))$. Tracer cette droite sur le graphique.
|
||||
\item On note $g$ la fonction affine qui décrit cette droite $D$. Détermine l'expression de $g$
|
||||
\item Calculer $\displaystyle \int_0^{32} g(x)\; dx$
|
||||
\item Avec la calculatrice, calculer une valeur approchée de $\displaystyle\int_{0}^{32} f(x) \; dx$
|
||||
\item Comparer les valeurs trouvées aux deux questions précédentes. Comment s'explique l'écart entre ces deux valeurs?
|
||||
\end{enumerate}
|
||||
\end{enumerate}
|
||||
|
||||
\medskip
|
||||
|
||||
\textbf{Partie C - Application économique}
|
||||
|
||||
Une entreprise a pris la décision de fermer son usine de production de smartphones en 32 mois.
|
||||
|
||||
La fonction capacité de production de cette usine est modélisée sur l'intervalle $\intFF{0}{32}$ par la fonction $f$ étudiée dans les parties A et B.
|
||||
|
||||
Le nombre $x$ représente le temps en mois après la décision de la fermeture du site et le nombre $f(x)$ représente capacité production de smartphone au moment $x$.
|
||||
|
||||
Utiliser les résultats de la partie B afin de répondre aux questions suivantes:
|
||||
|
||||
\begin{enumerate}
|
||||
\setcounter{enumi}{7}
|
||||
\item Combien de smartphones pouvaient être produit à la fermeture de l'usine?
|
||||
\item Pendant combien de temps la capacité de production de l'usine a réussi à se maintenir au dessus de \np{40000}?
|
||||
\item Combien de smartphones ont pu être produit entre la prise de décision et la fermeture de l'usine?
|
||||
\end{enumerate}
|
||||
|
||||
\end{exercise}
|
||||
|
||||
\begin{solution}
|
||||
\begin{enumerate}
|
||||
\item
|
||||
\begin{tikzpicture}[yscale=1, xscale=0.46875]
|
||||
\tkzInit[xmin=0,xmax=33,xstep=1,
|
||||
ymin=0,ymax=90000,ystep=9000]
|
||||
\tkzGrid
|
||||
\tkzGrid[sub, subxstep=0.2, subystep=1800.0]
|
||||
\tkzAxeXY[up space=0.5,right space=.2]
|
||||
\tkzFct[domain = 0:32, line width=1pt]{10000*(x+8)*exp(-0.125*x)}
|
||||
%M\tkzFct[domain = 0:32, line width=1pt, blue]{40000}
|
||||
\end{tikzpicture}
|
||||
\item Tracer la droite $y=40000$. C'est l'abscisse de l'intersection entre cette droite et la courbe
|
||||
\item
|
||||
\item
|
||||
|
||||
\end{enumerate}
|
||||
|
||||
\end{solution}
|
||||
|
||||
\end{document}
|
||||
|
||||
%%% Local Variables:
|
||||
%%% mode: latex
|
||||
%%% TeX-master: "master"
|
||||
%%% End:
|
||||
121
TES/DM/DM_20_02/03_DM_20_02.tex
Normal file
121
TES/DM/DM_20_02/03_DM_20_02.tex
Normal file
@@ -0,0 +1,121 @@
|
||||
\documentclass[a4paper,10pt]{article}
|
||||
\usepackage{tasks}
|
||||
\usepackage{myXsim}
|
||||
|
||||
\title{DM 2 -- BOUNOUS Matthieu}
|
||||
\tribe{Terminale ES-L}
|
||||
\date{9 mars 2020}
|
||||
|
||||
\xsimsetup{
|
||||
solution/print = false
|
||||
}
|
||||
|
||||
\begin{document}
|
||||
\maketitle
|
||||
|
||||
\begin{exercise}[subtitle={Étude de fonction}]
|
||||
On considère la fonction dérivable $f$ définie sur $I = [0~;~64]$ par:
|
||||
|
||||
\[f(x) = \np{10000}(x + 16)\text{e}^{- 0.0625x}.\]
|
||||
|
||||
\textbf{Partie A - Étude graphique}
|
||||
|
||||
On a représenté sur le graphique en annexe, la courbe représentative de la fonction $f$.
|
||||
|
||||
\begin{enumerate}
|
||||
\item Avec un tableur tracer et imprimer la courbe représentative de $f$ sur $I$
|
||||
\end{enumerate}
|
||||
\emph{Répondre aux questions suivantes par lecture graphique.}
|
||||
\begin{enumerate}
|
||||
\setcounter{enumi}{1}
|
||||
\item Résoudre graphiquement et de façon approchée l'équation $f(x) = \np{80000}$.
|
||||
\item Donner un encadrement de la quantité
|
||||
\[
|
||||
\int_{2}^{32} f(x) \; dx
|
||||
\]
|
||||
Vous expliquerez votre démarche en utilisant le graphique.
|
||||
\end{enumerate}
|
||||
|
||||
\medskip
|
||||
|
||||
\textbf{Partie B - Étude théorique}
|
||||
|
||||
\begin{enumerate}
|
||||
\setcounter{enumi}{4}
|
||||
\item Étude des variations.
|
||||
\begin{enumerate}
|
||||
\item On note $f'$ la dérivée de la fonction $f$ sur $\intFF{0}{64}$.
|
||||
|
||||
Démontrer que pour tout $x$ de $\intFF{0}{20}$ , $f'(x) = - 625x\text{e}^{-0.0625x}$.
|
||||
|
||||
\item En déduire le sens de variation de $f$ et dresser son tableau des variations sur l'intervalle $\intFF{0}{64}$. Si nécessaire, arrondir à l'unité les valeurs présentes dans le tableau.
|
||||
|
||||
\item Démontrer que l'équation $f(x) = \np{80000}$ admet une unique solution $\alpha$ sur $\intFF{0}{64}$, puis donner une valeur approchée de $\alpha$ à $10^{-2}$ près à l'aide de la calculatrice.
|
||||
\end{enumerate}
|
||||
|
||||
\item Étude de la convexité
|
||||
\begin{enumerate}
|
||||
\item On note $f''$ la dérivée seconde de $f$ sur $\intFF{0}{64}$.
|
||||
|
||||
Démontrer que pour tout $x$ de $\intFF{0}{20}$ , $f''(x) = (39.0625x-625)\text{e}^{-0.0625x}$.
|
||||
|
||||
\item Démontrer que $f$ admet un point d'inflexion dont on donnera son abscisse.
|
||||
\end{enumerate}
|
||||
|
||||
\item Aire sous la courbe
|
||||
\begin{enumerate}
|
||||
\item On souhaite approximer la fonction $f$ sur l'intervalle $I$ par la droite $D$ qui relie les points $(0;f(0))$ et $(64, f(64))$. Tracer cette droite sur le graphique.
|
||||
\item On note $g$ la fonction affine qui décrit cette droite $D$. Détermine l'expression de $g$
|
||||
\item Calculer $\displaystyle \int_0^{64} g(x)\; dx$
|
||||
\item Avec la calculatrice, calculer une valeur approchée de $\displaystyle\int_{0}^{64} f(x) \; dx$
|
||||
\item Comparer les valeurs trouvées aux deux questions précédentes. Comment s'explique l'écart entre ces deux valeurs?
|
||||
\end{enumerate}
|
||||
\end{enumerate}
|
||||
|
||||
\medskip
|
||||
|
||||
\textbf{Partie C - Application économique}
|
||||
|
||||
Une entreprise a pris la décision de fermer son usine de production de smartphones en 64 mois.
|
||||
|
||||
La fonction capacité de production de cette usine est modélisée sur l'intervalle $\intFF{0}{64}$ par la fonction $f$ étudiée dans les parties A et B.
|
||||
|
||||
Le nombre $x$ représente le temps en mois après la décision de la fermeture du site et le nombre $f(x)$ représente capacité production de smartphone au moment $x$.
|
||||
|
||||
Utiliser les résultats de la partie B afin de répondre aux questions suivantes:
|
||||
|
||||
\begin{enumerate}
|
||||
\setcounter{enumi}{7}
|
||||
\item Combien de smartphones pouvaient être produit à la fermeture de l'usine?
|
||||
\item Pendant combien de temps la capacité de production de l'usine a réussi à se maintenir au dessus de \np{80000}?
|
||||
\item Combien de smartphones ont pu être produit entre la prise de décision et la fermeture de l'usine?
|
||||
\end{enumerate}
|
||||
|
||||
\end{exercise}
|
||||
|
||||
\begin{solution}
|
||||
\begin{enumerate}
|
||||
\item
|
||||
\begin{tikzpicture}[yscale=1, xscale=0.234375]
|
||||
\tkzInit[xmin=0,xmax=65,xstep=1,
|
||||
ymin=0,ymax=170000,ystep=17000]
|
||||
\tkzGrid
|
||||
\tkzGrid[sub, subxstep=0.2, subystep=3400.0]
|
||||
\tkzAxeXY[up space=0.5,right space=.2]
|
||||
\tkzFct[domain = 0:64, line width=1pt]{10000*(x+16)*exp(-0.0625*x)}
|
||||
%M\tkzFct[domain = 0:64, line width=1pt, blue]{80000}
|
||||
\end{tikzpicture}
|
||||
\item Tracer la droite $y=80000$. C'est l'abscisse de l'intersection entre cette droite et la courbe
|
||||
\item
|
||||
\item
|
||||
|
||||
\end{enumerate}
|
||||
|
||||
\end{solution}
|
||||
|
||||
\end{document}
|
||||
|
||||
%%% Local Variables:
|
||||
%%% mode: latex
|
||||
%%% TeX-master: "master"
|
||||
%%% End:
|
||||
121
TES/DM/DM_20_02/04_DM_20_02.tex
Normal file
121
TES/DM/DM_20_02/04_DM_20_02.tex
Normal file
@@ -0,0 +1,121 @@
|
||||
\documentclass[a4paper,10pt]{article}
|
||||
\usepackage{tasks}
|
||||
\usepackage{myXsim}
|
||||
|
||||
\title{DM 2 -- CRETIN Marie}
|
||||
\tribe{Terminale ES-L}
|
||||
\date{9 mars 2020}
|
||||
|
||||
\xsimsetup{
|
||||
solution/print = false
|
||||
}
|
||||
|
||||
\begin{document}
|
||||
\maketitle
|
||||
|
||||
\begin{exercise}[subtitle={Étude de fonction}]
|
||||
On considère la fonction dérivable $f$ définie sur $I = [0~;~80]$ par:
|
||||
|
||||
\[f(x) = \np{10000}(x + 20)\text{e}^{- 0.05x}.\]
|
||||
|
||||
\textbf{Partie A - Étude graphique}
|
||||
|
||||
On a représenté sur le graphique en annexe, la courbe représentative de la fonction $f$.
|
||||
|
||||
\begin{enumerate}
|
||||
\item Avec un tableur tracer et imprimer la courbe représentative de $f$ sur $I$
|
||||
\end{enumerate}
|
||||
\emph{Répondre aux questions suivantes par lecture graphique.}
|
||||
\begin{enumerate}
|
||||
\setcounter{enumi}{1}
|
||||
\item Résoudre graphiquement et de façon approchée l'équation $f(x) = \np{100000}$.
|
||||
\item Donner un encadrement de la quantité
|
||||
\[
|
||||
\int_{2}^{40} f(x) \; dx
|
||||
\]
|
||||
Vous expliquerez votre démarche en utilisant le graphique.
|
||||
\end{enumerate}
|
||||
|
||||
\medskip
|
||||
|
||||
\textbf{Partie B - Étude théorique}
|
||||
|
||||
\begin{enumerate}
|
||||
\setcounter{enumi}{4}
|
||||
\item Étude des variations.
|
||||
\begin{enumerate}
|
||||
\item On note $f'$ la dérivée de la fonction $f$ sur $\intFF{0}{80}$.
|
||||
|
||||
Démontrer que pour tout $x$ de $\intFF{0}{20}$ , $f'(x) = - 500x\text{e}^{-0.05x}$.
|
||||
|
||||
\item En déduire le sens de variation de $f$ et dresser son tableau des variations sur l'intervalle $\intFF{0}{80}$. Si nécessaire, arrondir à l'unité les valeurs présentes dans le tableau.
|
||||
|
||||
\item Démontrer que l'équation $f(x) = \np{100000}$ admet une unique solution $\alpha$ sur $\intFF{0}{80}$, puis donner une valeur approchée de $\alpha$ à $10^{-2}$ près à l'aide de la calculatrice.
|
||||
\end{enumerate}
|
||||
|
||||
\item Étude de la convexité
|
||||
\begin{enumerate}
|
||||
\item On note $f''$ la dérivée seconde de $f$ sur $\intFF{0}{80}$.
|
||||
|
||||
Démontrer que pour tout $x$ de $\intFF{0}{20}$ , $f''(x) = (25x-500)\text{e}^{-0.05x}$.
|
||||
|
||||
\item Démontrer que $f$ admet un point d'inflexion dont on donnera son abscisse.
|
||||
\end{enumerate}
|
||||
|
||||
\item Aire sous la courbe
|
||||
\begin{enumerate}
|
||||
\item On souhaite approximer la fonction $f$ sur l'intervalle $I$ par la droite $D$ qui relie les points $(0;f(0))$ et $(80, f(80))$. Tracer cette droite sur le graphique.
|
||||
\item On note $g$ la fonction affine qui décrit cette droite $D$. Détermine l'expression de $g$
|
||||
\item Calculer $\displaystyle \int_0^{80} g(x)\; dx$
|
||||
\item Avec la calculatrice, calculer une valeur approchée de $\displaystyle\int_{0}^{80} f(x) \; dx$
|
||||
\item Comparer les valeurs trouvées aux deux questions précédentes. Comment s'explique l'écart entre ces deux valeurs?
|
||||
\end{enumerate}
|
||||
\end{enumerate}
|
||||
|
||||
\medskip
|
||||
|
||||
\textbf{Partie C - Application économique}
|
||||
|
||||
Une entreprise a pris la décision de fermer son usine de production de smartphones en 80 mois.
|
||||
|
||||
La fonction capacité de production de cette usine est modélisée sur l'intervalle $\intFF{0}{80}$ par la fonction $f$ étudiée dans les parties A et B.
|
||||
|
||||
Le nombre $x$ représente le temps en mois après la décision de la fermeture du site et le nombre $f(x)$ représente capacité production de smartphone au moment $x$.
|
||||
|
||||
Utiliser les résultats de la partie B afin de répondre aux questions suivantes:
|
||||
|
||||
\begin{enumerate}
|
||||
\setcounter{enumi}{7}
|
||||
\item Combien de smartphones pouvaient être produit à la fermeture de l'usine?
|
||||
\item Pendant combien de temps la capacité de production de l'usine a réussi à se maintenir au dessus de \np{100000}?
|
||||
\item Combien de smartphones ont pu être produit entre la prise de décision et la fermeture de l'usine?
|
||||
\end{enumerate}
|
||||
|
||||
\end{exercise}
|
||||
|
||||
\begin{solution}
|
||||
\begin{enumerate}
|
||||
\item
|
||||
\begin{tikzpicture}[yscale=1, xscale=0.1875]
|
||||
\tkzInit[xmin=0,xmax=81,xstep=1,
|
||||
ymin=0,ymax=210000,ystep=21000]
|
||||
\tkzGrid
|
||||
\tkzGrid[sub, subxstep=0.2, subystep=4200.0]
|
||||
\tkzAxeXY[up space=0.5,right space=.2]
|
||||
\tkzFct[domain = 0:80, line width=1pt]{10000*(x+20)*exp(-0.05*x)}
|
||||
%M\tkzFct[domain = 0:80, line width=1pt, blue]{100000}
|
||||
\end{tikzpicture}
|
||||
\item Tracer la droite $y=100000$. C'est l'abscisse de l'intersection entre cette droite et la courbe
|
||||
\item
|
||||
\item
|
||||
|
||||
\end{enumerate}
|
||||
|
||||
\end{solution}
|
||||
|
||||
\end{document}
|
||||
|
||||
%%% Local Variables:
|
||||
%%% mode: latex
|
||||
%%% TeX-master: "master"
|
||||
%%% End:
|
||||
121
TES/DM/DM_20_02/05_DM_20_02.tex
Normal file
121
TES/DM/DM_20_02/05_DM_20_02.tex
Normal file
@@ -0,0 +1,121 @@
|
||||
\documentclass[a4paper,10pt]{article}
|
||||
\usepackage{tasks}
|
||||
\usepackage{myXsim}
|
||||
|
||||
\title{DM 2 -- DENIS Clarisse}
|
||||
\tribe{Terminale ES-L}
|
||||
\date{9 mars 2020}
|
||||
|
||||
\xsimsetup{
|
||||
solution/print = false
|
||||
}
|
||||
|
||||
\begin{document}
|
||||
\maketitle
|
||||
|
||||
\begin{exercise}[subtitle={Étude de fonction}]
|
||||
On considère la fonction dérivable $f$ définie sur $I = [0~;~64]$ par:
|
||||
|
||||
\[f(x) = \np{9000}(x + 16)\text{e}^{- 0.0625x}.\]
|
||||
|
||||
\textbf{Partie A - Étude graphique}
|
||||
|
||||
On a représenté sur le graphique en annexe, la courbe représentative de la fonction $f$.
|
||||
|
||||
\begin{enumerate}
|
||||
\item Avec un tableur tracer et imprimer la courbe représentative de $f$ sur $I$
|
||||
\end{enumerate}
|
||||
\emph{Répondre aux questions suivantes par lecture graphique.}
|
||||
\begin{enumerate}
|
||||
\setcounter{enumi}{1}
|
||||
\item Résoudre graphiquement et de façon approchée l'équation $f(x) = \np{72000}$.
|
||||
\item Donner un encadrement de la quantité
|
||||
\[
|
||||
\int_{2}^{32} f(x) \; dx
|
||||
\]
|
||||
Vous expliquerez votre démarche en utilisant le graphique.
|
||||
\end{enumerate}
|
||||
|
||||
\medskip
|
||||
|
||||
\textbf{Partie B - Étude théorique}
|
||||
|
||||
\begin{enumerate}
|
||||
\setcounter{enumi}{4}
|
||||
\item Étude des variations.
|
||||
\begin{enumerate}
|
||||
\item On note $f'$ la dérivée de la fonction $f$ sur $\intFF{0}{64}$.
|
||||
|
||||
Démontrer que pour tout $x$ de $\intFF{0}{20}$ , $f'(x) = - 562.5000x\text{e}^{-0.0625x}$.
|
||||
|
||||
\item En déduire le sens de variation de $f$ et dresser son tableau des variations sur l'intervalle $\intFF{0}{64}$. Si nécessaire, arrondir à l'unité les valeurs présentes dans le tableau.
|
||||
|
||||
\item Démontrer que l'équation $f(x) = \np{72000}$ admet une unique solution $\alpha$ sur $\intFF{0}{64}$, puis donner une valeur approchée de $\alpha$ à $10^{-2}$ près à l'aide de la calculatrice.
|
||||
\end{enumerate}
|
||||
|
||||
\item Étude de la convexité
|
||||
\begin{enumerate}
|
||||
\item On note $f''$ la dérivée seconde de $f$ sur $\intFF{0}{64}$.
|
||||
|
||||
Démontrer que pour tout $x$ de $\intFF{0}{20}$ , $f''(x) = (35.15625000x-562.5000)\text{e}^{-0.0625x}$.
|
||||
|
||||
\item Démontrer que $f$ admet un point d'inflexion dont on donnera son abscisse.
|
||||
\end{enumerate}
|
||||
|
||||
\item Aire sous la courbe
|
||||
\begin{enumerate}
|
||||
\item On souhaite approximer la fonction $f$ sur l'intervalle $I$ par la droite $D$ qui relie les points $(0;f(0))$ et $(64, f(64))$. Tracer cette droite sur le graphique.
|
||||
\item On note $g$ la fonction affine qui décrit cette droite $D$. Détermine l'expression de $g$
|
||||
\item Calculer $\displaystyle \int_0^{64} g(x)\; dx$
|
||||
\item Avec la calculatrice, calculer une valeur approchée de $\displaystyle\int_{0}^{64} f(x) \; dx$
|
||||
\item Comparer les valeurs trouvées aux deux questions précédentes. Comment s'explique l'écart entre ces deux valeurs?
|
||||
\end{enumerate}
|
||||
\end{enumerate}
|
||||
|
||||
\medskip
|
||||
|
||||
\textbf{Partie C - Application économique}
|
||||
|
||||
Une entreprise a pris la décision de fermer son usine de production de smartphones en 64 mois.
|
||||
|
||||
La fonction capacité de production de cette usine est modélisée sur l'intervalle $\intFF{0}{64}$ par la fonction $f$ étudiée dans les parties A et B.
|
||||
|
||||
Le nombre $x$ représente le temps en mois après la décision de la fermeture du site et le nombre $f(x)$ représente capacité production de smartphone au moment $x$.
|
||||
|
||||
Utiliser les résultats de la partie B afin de répondre aux questions suivantes:
|
||||
|
||||
\begin{enumerate}
|
||||
\setcounter{enumi}{7}
|
||||
\item Combien de smartphones pouvaient être produit à la fermeture de l'usine?
|
||||
\item Pendant combien de temps la capacité de production de l'usine a réussi à se maintenir au dessus de \np{72000}?
|
||||
\item Combien de smartphones ont pu être produit entre la prise de décision et la fermeture de l'usine?
|
||||
\end{enumerate}
|
||||
|
||||
\end{exercise}
|
||||
|
||||
\begin{solution}
|
||||
\begin{enumerate}
|
||||
\item
|
||||
\begin{tikzpicture}[yscale=1, xscale=0.234375]
|
||||
\tkzInit[xmin=0,xmax=65,xstep=1,
|
||||
ymin=0,ymax=153000,ystep=15000]
|
||||
\tkzGrid
|
||||
\tkzGrid[sub, subxstep=0.2, subystep=3000.0]
|
||||
\tkzAxeXY[up space=0.5,right space=.2]
|
||||
\tkzFct[domain = 0:64, line width=1pt]{9000*(x+16)*exp(-0.0625*x)}
|
||||
%M\tkzFct[domain = 0:64, line width=1pt, blue]{72000}
|
||||
\end{tikzpicture}
|
||||
\item Tracer la droite $y=72000$. C'est l'abscisse de l'intersection entre cette droite et la courbe
|
||||
\item
|
||||
\item
|
||||
|
||||
\end{enumerate}
|
||||
|
||||
\end{solution}
|
||||
|
||||
\end{document}
|
||||
|
||||
%%% Local Variables:
|
||||
%%% mode: latex
|
||||
%%% TeX-master: "master"
|
||||
%%% End:
|
||||
121
TES/DM/DM_20_02/06_DM_20_02.tex
Normal file
121
TES/DM/DM_20_02/06_DM_20_02.tex
Normal file
@@ -0,0 +1,121 @@
|
||||
\documentclass[a4paper,10pt]{article}
|
||||
\usepackage{tasks}
|
||||
\usepackage{myXsim}
|
||||
|
||||
\title{DM 2 -- DOS SANTOS Théo}
|
||||
\tribe{Terminale ES-L}
|
||||
\date{9 mars 2020}
|
||||
|
||||
\xsimsetup{
|
||||
solution/print = false
|
||||
}
|
||||
|
||||
\begin{document}
|
||||
\maketitle
|
||||
|
||||
\begin{exercise}[subtitle={Étude de fonction}]
|
||||
On considère la fonction dérivable $f$ définie sur $I = [0~;~64]$ par:
|
||||
|
||||
\[f(x) = \np{2000}(x + 16)\text{e}^{- 0.0625x}.\]
|
||||
|
||||
\textbf{Partie A - Étude graphique}
|
||||
|
||||
On a représenté sur le graphique en annexe, la courbe représentative de la fonction $f$.
|
||||
|
||||
\begin{enumerate}
|
||||
\item Avec un tableur tracer et imprimer la courbe représentative de $f$ sur $I$
|
||||
\end{enumerate}
|
||||
\emph{Répondre aux questions suivantes par lecture graphique.}
|
||||
\begin{enumerate}
|
||||
\setcounter{enumi}{1}
|
||||
\item Résoudre graphiquement et de façon approchée l'équation $f(x) = \np{16000}$.
|
||||
\item Donner un encadrement de la quantité
|
||||
\[
|
||||
\int_{2}^{32} f(x) \; dx
|
||||
\]
|
||||
Vous expliquerez votre démarche en utilisant le graphique.
|
||||
\end{enumerate}
|
||||
|
||||
\medskip
|
||||
|
||||
\textbf{Partie B - Étude théorique}
|
||||
|
||||
\begin{enumerate}
|
||||
\setcounter{enumi}{4}
|
||||
\item Étude des variations.
|
||||
\begin{enumerate}
|
||||
\item On note $f'$ la dérivée de la fonction $f$ sur $\intFF{0}{64}$.
|
||||
|
||||
Démontrer que pour tout $x$ de $\intFF{0}{20}$ , $f'(x) = - 125x\text{e}^{-0.0625x}$.
|
||||
|
||||
\item En déduire le sens de variation de $f$ et dresser son tableau des variations sur l'intervalle $\intFF{0}{64}$. Si nécessaire, arrondir à l'unité les valeurs présentes dans le tableau.
|
||||
|
||||
\item Démontrer que l'équation $f(x) = \np{16000}$ admet une unique solution $\alpha$ sur $\intFF{0}{64}$, puis donner une valeur approchée de $\alpha$ à $10^{-2}$ près à l'aide de la calculatrice.
|
||||
\end{enumerate}
|
||||
|
||||
\item Étude de la convexité
|
||||
\begin{enumerate}
|
||||
\item On note $f''$ la dérivée seconde de $f$ sur $\intFF{0}{64}$.
|
||||
|
||||
Démontrer que pour tout $x$ de $\intFF{0}{20}$ , $f''(x) = (7.8125x-125)\text{e}^{-0.0625x}$.
|
||||
|
||||
\item Démontrer que $f$ admet un point d'inflexion dont on donnera son abscisse.
|
||||
\end{enumerate}
|
||||
|
||||
\item Aire sous la courbe
|
||||
\begin{enumerate}
|
||||
\item On souhaite approximer la fonction $f$ sur l'intervalle $I$ par la droite $D$ qui relie les points $(0;f(0))$ et $(64, f(64))$. Tracer cette droite sur le graphique.
|
||||
\item On note $g$ la fonction affine qui décrit cette droite $D$. Détermine l'expression de $g$
|
||||
\item Calculer $\displaystyle \int_0^{64} g(x)\; dx$
|
||||
\item Avec la calculatrice, calculer une valeur approchée de $\displaystyle\int_{0}^{64} f(x) \; dx$
|
||||
\item Comparer les valeurs trouvées aux deux questions précédentes. Comment s'explique l'écart entre ces deux valeurs?
|
||||
\end{enumerate}
|
||||
\end{enumerate}
|
||||
|
||||
\medskip
|
||||
|
||||
\textbf{Partie C - Application économique}
|
||||
|
||||
Une entreprise a pris la décision de fermer son usine de production de smartphones en 64 mois.
|
||||
|
||||
La fonction capacité de production de cette usine est modélisée sur l'intervalle $\intFF{0}{64}$ par la fonction $f$ étudiée dans les parties A et B.
|
||||
|
||||
Le nombre $x$ représente le temps en mois après la décision de la fermeture du site et le nombre $f(x)$ représente capacité production de smartphone au moment $x$.
|
||||
|
||||
Utiliser les résultats de la partie B afin de répondre aux questions suivantes:
|
||||
|
||||
\begin{enumerate}
|
||||
\setcounter{enumi}{7}
|
||||
\item Combien de smartphones pouvaient être produit à la fermeture de l'usine?
|
||||
\item Pendant combien de temps la capacité de production de l'usine a réussi à se maintenir au dessus de \np{16000}?
|
||||
\item Combien de smartphones ont pu être produit entre la prise de décision et la fermeture de l'usine?
|
||||
\end{enumerate}
|
||||
|
||||
\end{exercise}
|
||||
|
||||
\begin{solution}
|
||||
\begin{enumerate}
|
||||
\item
|
||||
\begin{tikzpicture}[yscale=1, xscale=0.234375]
|
||||
\tkzInit[xmin=0,xmax=65,xstep=1,
|
||||
ymin=0,ymax=34000,ystep=3000]
|
||||
\tkzGrid
|
||||
\tkzGrid[sub, subxstep=0.2, subystep=600.0]
|
||||
\tkzAxeXY[up space=0.5,right space=.2]
|
||||
\tkzFct[domain = 0:64, line width=1pt]{2000*(x+16)*exp(-0.0625*x)}
|
||||
%M\tkzFct[domain = 0:64, line width=1pt, blue]{16000}
|
||||
\end{tikzpicture}
|
||||
\item Tracer la droite $y=16000$. C'est l'abscisse de l'intersection entre cette droite et la courbe
|
||||
\item
|
||||
\item
|
||||
|
||||
\end{enumerate}
|
||||
|
||||
\end{solution}
|
||||
|
||||
\end{document}
|
||||
|
||||
%%% Local Variables:
|
||||
%%% mode: latex
|
||||
%%% TeX-master: "master"
|
||||
%%% End:
|
||||
121
TES/DM/DM_20_02/07_DM_20_02.tex
Normal file
121
TES/DM/DM_20_02/07_DM_20_02.tex
Normal file
@@ -0,0 +1,121 @@
|
||||
\documentclass[a4paper,10pt]{article}
|
||||
\usepackage{tasks}
|
||||
\usepackage{myXsim}
|
||||
|
||||
\title{DM 2 -- FERREIRA Tina}
|
||||
\tribe{Terminale ES-L}
|
||||
\date{9 mars 2020}
|
||||
|
||||
\xsimsetup{
|
||||
solution/print = false
|
||||
}
|
||||
|
||||
\begin{document}
|
||||
\maketitle
|
||||
|
||||
\begin{exercise}[subtitle={Étude de fonction}]
|
||||
On considère la fonction dérivable $f$ définie sur $I = [0~;~80]$ par:
|
||||
|
||||
\[f(x) = \np{8000}(x + 20)\text{e}^{- 0.05x}.\]
|
||||
|
||||
\textbf{Partie A - Étude graphique}
|
||||
|
||||
On a représenté sur le graphique en annexe, la courbe représentative de la fonction $f$.
|
||||
|
||||
\begin{enumerate}
|
||||
\item Avec un tableur tracer et imprimer la courbe représentative de $f$ sur $I$
|
||||
\end{enumerate}
|
||||
\emph{Répondre aux questions suivantes par lecture graphique.}
|
||||
\begin{enumerate}
|
||||
\setcounter{enumi}{1}
|
||||
\item Résoudre graphiquement et de façon approchée l'équation $f(x) = \np{80000}$.
|
||||
\item Donner un encadrement de la quantité
|
||||
\[
|
||||
\int_{2}^{40} f(x) \; dx
|
||||
\]
|
||||
Vous expliquerez votre démarche en utilisant le graphique.
|
||||
\end{enumerate}
|
||||
|
||||
\medskip
|
||||
|
||||
\textbf{Partie B - Étude théorique}
|
||||
|
||||
\begin{enumerate}
|
||||
\setcounter{enumi}{4}
|
||||
\item Étude des variations.
|
||||
\begin{enumerate}
|
||||
\item On note $f'$ la dérivée de la fonction $f$ sur $\intFF{0}{80}$.
|
||||
|
||||
Démontrer que pour tout $x$ de $\intFF{0}{20}$ , $f'(x) = - 400x\text{e}^{-0.05x}$.
|
||||
|
||||
\item En déduire le sens de variation de $f$ et dresser son tableau des variations sur l'intervalle $\intFF{0}{80}$. Si nécessaire, arrondir à l'unité les valeurs présentes dans le tableau.
|
||||
|
||||
\item Démontrer que l'équation $f(x) = \np{80000}$ admet une unique solution $\alpha$ sur $\intFF{0}{80}$, puis donner une valeur approchée de $\alpha$ à $10^{-2}$ près à l'aide de la calculatrice.
|
||||
\end{enumerate}
|
||||
|
||||
\item Étude de la convexité
|
||||
\begin{enumerate}
|
||||
\item On note $f''$ la dérivée seconde de $f$ sur $\intFF{0}{80}$.
|
||||
|
||||
Démontrer que pour tout $x$ de $\intFF{0}{20}$ , $f''(x) = (20x-400)\text{e}^{-0.05x}$.
|
||||
|
||||
\item Démontrer que $f$ admet un point d'inflexion dont on donnera son abscisse.
|
||||
\end{enumerate}
|
||||
|
||||
\item Aire sous la courbe
|
||||
\begin{enumerate}
|
||||
\item On souhaite approximer la fonction $f$ sur l'intervalle $I$ par la droite $D$ qui relie les points $(0;f(0))$ et $(80, f(80))$. Tracer cette droite sur le graphique.
|
||||
\item On note $g$ la fonction affine qui décrit cette droite $D$. Détermine l'expression de $g$
|
||||
\item Calculer $\displaystyle \int_0^{80} g(x)\; dx$
|
||||
\item Avec la calculatrice, calculer une valeur approchée de $\displaystyle\int_{0}^{80} f(x) \; dx$
|
||||
\item Comparer les valeurs trouvées aux deux questions précédentes. Comment s'explique l'écart entre ces deux valeurs?
|
||||
\end{enumerate}
|
||||
\end{enumerate}
|
||||
|
||||
\medskip
|
||||
|
||||
\textbf{Partie C - Application économique}
|
||||
|
||||
Une entreprise a pris la décision de fermer son usine de production de smartphones en 80 mois.
|
||||
|
||||
La fonction capacité de production de cette usine est modélisée sur l'intervalle $\intFF{0}{80}$ par la fonction $f$ étudiée dans les parties A et B.
|
||||
|
||||
Le nombre $x$ représente le temps en mois après la décision de la fermeture du site et le nombre $f(x)$ représente capacité production de smartphone au moment $x$.
|
||||
|
||||
Utiliser les résultats de la partie B afin de répondre aux questions suivantes:
|
||||
|
||||
\begin{enumerate}
|
||||
\setcounter{enumi}{7}
|
||||
\item Combien de smartphones pouvaient être produit à la fermeture de l'usine?
|
||||
\item Pendant combien de temps la capacité de production de l'usine a réussi à se maintenir au dessus de \np{80000}?
|
||||
\item Combien de smartphones ont pu être produit entre la prise de décision et la fermeture de l'usine?
|
||||
\end{enumerate}
|
||||
|
||||
\end{exercise}
|
||||
|
||||
\begin{solution}
|
||||
\begin{enumerate}
|
||||
\item
|
||||
\begin{tikzpicture}[yscale=1, xscale=0.1875]
|
||||
\tkzInit[xmin=0,xmax=81,xstep=1,
|
||||
ymin=0,ymax=168000,ystep=16000]
|
||||
\tkzGrid
|
||||
\tkzGrid[sub, subxstep=0.2, subystep=3200.0]
|
||||
\tkzAxeXY[up space=0.5,right space=.2]
|
||||
\tkzFct[domain = 0:80, line width=1pt]{8000*(x+20)*exp(-0.05*x)}
|
||||
%M\tkzFct[domain = 0:80, line width=1pt, blue]{80000}
|
||||
\end{tikzpicture}
|
||||
\item Tracer la droite $y=80000$. C'est l'abscisse de l'intersection entre cette droite et la courbe
|
||||
\item
|
||||
\item
|
||||
|
||||
\end{enumerate}
|
||||
|
||||
\end{solution}
|
||||
|
||||
\end{document}
|
||||
|
||||
%%% Local Variables:
|
||||
%%% mode: latex
|
||||
%%% TeX-master: "master"
|
||||
%%% End:
|
||||
121
TES/DM/DM_20_02/08_DM_20_02.tex
Normal file
121
TES/DM/DM_20_02/08_DM_20_02.tex
Normal file
@@ -0,0 +1,121 @@
|
||||
\documentclass[a4paper,10pt]{article}
|
||||
\usepackage{tasks}
|
||||
\usepackage{myXsim}
|
||||
|
||||
\title{DM 2 -- GAUDARD Camille}
|
||||
\tribe{Terminale ES-L}
|
||||
\date{9 mars 2020}
|
||||
|
||||
\xsimsetup{
|
||||
solution/print = false
|
||||
}
|
||||
|
||||
\begin{document}
|
||||
\maketitle
|
||||
|
||||
\begin{exercise}[subtitle={Étude de fonction}]
|
||||
On considère la fonction dérivable $f$ définie sur $I = [0~;~80]$ par:
|
||||
|
||||
\[f(x) = \np{7000}(x + 20)\text{e}^{- 0.05x}.\]
|
||||
|
||||
\textbf{Partie A - Étude graphique}
|
||||
|
||||
On a représenté sur le graphique en annexe, la courbe représentative de la fonction $f$.
|
||||
|
||||
\begin{enumerate}
|
||||
\item Avec un tableur tracer et imprimer la courbe représentative de $f$ sur $I$
|
||||
\end{enumerate}
|
||||
\emph{Répondre aux questions suivantes par lecture graphique.}
|
||||
\begin{enumerate}
|
||||
\setcounter{enumi}{1}
|
||||
\item Résoudre graphiquement et de façon approchée l'équation $f(x) = \np{70000}$.
|
||||
\item Donner un encadrement de la quantité
|
||||
\[
|
||||
\int_{2}^{40} f(x) \; dx
|
||||
\]
|
||||
Vous expliquerez votre démarche en utilisant le graphique.
|
||||
\end{enumerate}
|
||||
|
||||
\medskip
|
||||
|
||||
\textbf{Partie B - Étude théorique}
|
||||
|
||||
\begin{enumerate}
|
||||
\setcounter{enumi}{4}
|
||||
\item Étude des variations.
|
||||
\begin{enumerate}
|
||||
\item On note $f'$ la dérivée de la fonction $f$ sur $\intFF{0}{80}$.
|
||||
|
||||
Démontrer que pour tout $x$ de $\intFF{0}{20}$ , $f'(x) = - 350x\text{e}^{-0.05x}$.
|
||||
|
||||
\item En déduire le sens de variation de $f$ et dresser son tableau des variations sur l'intervalle $\intFF{0}{80}$. Si nécessaire, arrondir à l'unité les valeurs présentes dans le tableau.
|
||||
|
||||
\item Démontrer que l'équation $f(x) = \np{70000}$ admet une unique solution $\alpha$ sur $\intFF{0}{80}$, puis donner une valeur approchée de $\alpha$ à $10^{-2}$ près à l'aide de la calculatrice.
|
||||
\end{enumerate}
|
||||
|
||||
\item Étude de la convexité
|
||||
\begin{enumerate}
|
||||
\item On note $f''$ la dérivée seconde de $f$ sur $\intFF{0}{80}$.
|
||||
|
||||
Démontrer que pour tout $x$ de $\intFF{0}{20}$ , $f''(x) = (17.50x-350)\text{e}^{-0.05x}$.
|
||||
|
||||
\item Démontrer que $f$ admet un point d'inflexion dont on donnera son abscisse.
|
||||
\end{enumerate}
|
||||
|
||||
\item Aire sous la courbe
|
||||
\begin{enumerate}
|
||||
\item On souhaite approximer la fonction $f$ sur l'intervalle $I$ par la droite $D$ qui relie les points $(0;f(0))$ et $(80, f(80))$. Tracer cette droite sur le graphique.
|
||||
\item On note $g$ la fonction affine qui décrit cette droite $D$. Détermine l'expression de $g$
|
||||
\item Calculer $\displaystyle \int_0^{80} g(x)\; dx$
|
||||
\item Avec la calculatrice, calculer une valeur approchée de $\displaystyle\int_{0}^{80} f(x) \; dx$
|
||||
\item Comparer les valeurs trouvées aux deux questions précédentes. Comment s'explique l'écart entre ces deux valeurs?
|
||||
\end{enumerate}
|
||||
\end{enumerate}
|
||||
|
||||
\medskip
|
||||
|
||||
\textbf{Partie C - Application économique}
|
||||
|
||||
Une entreprise a pris la décision de fermer son usine de production de smartphones en 80 mois.
|
||||
|
||||
La fonction capacité de production de cette usine est modélisée sur l'intervalle $\intFF{0}{80}$ par la fonction $f$ étudiée dans les parties A et B.
|
||||
|
||||
Le nombre $x$ représente le temps en mois après la décision de la fermeture du site et le nombre $f(x)$ représente capacité production de smartphone au moment $x$.
|
||||
|
||||
Utiliser les résultats de la partie B afin de répondre aux questions suivantes:
|
||||
|
||||
\begin{enumerate}
|
||||
\setcounter{enumi}{7}
|
||||
\item Combien de smartphones pouvaient être produit à la fermeture de l'usine?
|
||||
\item Pendant combien de temps la capacité de production de l'usine a réussi à se maintenir au dessus de \np{70000}?
|
||||
\item Combien de smartphones ont pu être produit entre la prise de décision et la fermeture de l'usine?
|
||||
\end{enumerate}
|
||||
|
||||
\end{exercise}
|
||||
|
||||
\begin{solution}
|
||||
\begin{enumerate}
|
||||
\item
|
||||
\begin{tikzpicture}[yscale=1, xscale=0.1875]
|
||||
\tkzInit[xmin=0,xmax=81,xstep=1,
|
||||
ymin=0,ymax=147000,ystep=14000]
|
||||
\tkzGrid
|
||||
\tkzGrid[sub, subxstep=0.2, subystep=2800.0]
|
||||
\tkzAxeXY[up space=0.5,right space=.2]
|
||||
\tkzFct[domain = 0:80, line width=1pt]{7000*(x+20)*exp(-0.05*x)}
|
||||
%M\tkzFct[domain = 0:80, line width=1pt, blue]{70000}
|
||||
\end{tikzpicture}
|
||||
\item Tracer la droite $y=70000$. C'est l'abscisse de l'intersection entre cette droite et la courbe
|
||||
\item
|
||||
\item
|
||||
|
||||
\end{enumerate}
|
||||
|
||||
\end{solution}
|
||||
|
||||
\end{document}
|
||||
|
||||
%%% Local Variables:
|
||||
%%% mode: latex
|
||||
%%% TeX-master: "master"
|
||||
%%% End:
|
||||
121
TES/DM/DM_20_02/09_DM_20_02.tex
Normal file
121
TES/DM/DM_20_02/09_DM_20_02.tex
Normal file
@@ -0,0 +1,121 @@
|
||||
\documentclass[a4paper,10pt]{article}
|
||||
\usepackage{tasks}
|
||||
\usepackage{myXsim}
|
||||
|
||||
\title{DM 2 -- GUVERCIN Dilara Melisa}
|
||||
\tribe{Terminale ES-L}
|
||||
\date{9 mars 2020}
|
||||
|
||||
\xsimsetup{
|
||||
solution/print = false
|
||||
}
|
||||
|
||||
\begin{document}
|
||||
\maketitle
|
||||
|
||||
\begin{exercise}[subtitle={Étude de fonction}]
|
||||
On considère la fonction dérivable $f$ définie sur $I = [0~;~80]$ par:
|
||||
|
||||
\[f(x) = \np{4000}(x + 20)\text{e}^{- 0.05x}.\]
|
||||
|
||||
\textbf{Partie A - Étude graphique}
|
||||
|
||||
On a représenté sur le graphique en annexe, la courbe représentative de la fonction $f$.
|
||||
|
||||
\begin{enumerate}
|
||||
\item Avec un tableur tracer et imprimer la courbe représentative de $f$ sur $I$
|
||||
\end{enumerate}
|
||||
\emph{Répondre aux questions suivantes par lecture graphique.}
|
||||
\begin{enumerate}
|
||||
\setcounter{enumi}{1}
|
||||
\item Résoudre graphiquement et de façon approchée l'équation $f(x) = \np{40000}$.
|
||||
\item Donner un encadrement de la quantité
|
||||
\[
|
||||
\int_{2}^{40} f(x) \; dx
|
||||
\]
|
||||
Vous expliquerez votre démarche en utilisant le graphique.
|
||||
\end{enumerate}
|
||||
|
||||
\medskip
|
||||
|
||||
\textbf{Partie B - Étude théorique}
|
||||
|
||||
\begin{enumerate}
|
||||
\setcounter{enumi}{4}
|
||||
\item Étude des variations.
|
||||
\begin{enumerate}
|
||||
\item On note $f'$ la dérivée de la fonction $f$ sur $\intFF{0}{80}$.
|
||||
|
||||
Démontrer que pour tout $x$ de $\intFF{0}{20}$ , $f'(x) = - 200x\text{e}^{-0.05x}$.
|
||||
|
||||
\item En déduire le sens de variation de $f$ et dresser son tableau des variations sur l'intervalle $\intFF{0}{80}$. Si nécessaire, arrondir à l'unité les valeurs présentes dans le tableau.
|
||||
|
||||
\item Démontrer que l'équation $f(x) = \np{40000}$ admet une unique solution $\alpha$ sur $\intFF{0}{80}$, puis donner une valeur approchée de $\alpha$ à $10^{-2}$ près à l'aide de la calculatrice.
|
||||
\end{enumerate}
|
||||
|
||||
\item Étude de la convexité
|
||||
\begin{enumerate}
|
||||
\item On note $f''$ la dérivée seconde de $f$ sur $\intFF{0}{80}$.
|
||||
|
||||
Démontrer que pour tout $x$ de $\intFF{0}{20}$ , $f''(x) = (10x-200)\text{e}^{-0.05x}$.
|
||||
|
||||
\item Démontrer que $f$ admet un point d'inflexion dont on donnera son abscisse.
|
||||
\end{enumerate}
|
||||
|
||||
\item Aire sous la courbe
|
||||
\begin{enumerate}
|
||||
\item On souhaite approximer la fonction $f$ sur l'intervalle $I$ par la droite $D$ qui relie les points $(0;f(0))$ et $(80, f(80))$. Tracer cette droite sur le graphique.
|
||||
\item On note $g$ la fonction affine qui décrit cette droite $D$. Détermine l'expression de $g$
|
||||
\item Calculer $\displaystyle \int_0^{80} g(x)\; dx$
|
||||
\item Avec la calculatrice, calculer une valeur approchée de $\displaystyle\int_{0}^{80} f(x) \; dx$
|
||||
\item Comparer les valeurs trouvées aux deux questions précédentes. Comment s'explique l'écart entre ces deux valeurs?
|
||||
\end{enumerate}
|
||||
\end{enumerate}
|
||||
|
||||
\medskip
|
||||
|
||||
\textbf{Partie C - Application économique}
|
||||
|
||||
Une entreprise a pris la décision de fermer son usine de production de smartphones en 80 mois.
|
||||
|
||||
La fonction capacité de production de cette usine est modélisée sur l'intervalle $\intFF{0}{80}$ par la fonction $f$ étudiée dans les parties A et B.
|
||||
|
||||
Le nombre $x$ représente le temps en mois après la décision de la fermeture du site et le nombre $f(x)$ représente capacité production de smartphone au moment $x$.
|
||||
|
||||
Utiliser les résultats de la partie B afin de répondre aux questions suivantes:
|
||||
|
||||
\begin{enumerate}
|
||||
\setcounter{enumi}{7}
|
||||
\item Combien de smartphones pouvaient être produit à la fermeture de l'usine?
|
||||
\item Pendant combien de temps la capacité de production de l'usine a réussi à se maintenir au dessus de \np{40000}?
|
||||
\item Combien de smartphones ont pu être produit entre la prise de décision et la fermeture de l'usine?
|
||||
\end{enumerate}
|
||||
|
||||
\end{exercise}
|
||||
|
||||
\begin{solution}
|
||||
\begin{enumerate}
|
||||
\item
|
||||
\begin{tikzpicture}[yscale=1, xscale=0.1875]
|
||||
\tkzInit[xmin=0,xmax=81,xstep=1,
|
||||
ymin=0,ymax=84000,ystep=8000]
|
||||
\tkzGrid
|
||||
\tkzGrid[sub, subxstep=0.2, subystep=1600.0]
|
||||
\tkzAxeXY[up space=0.5,right space=.2]
|
||||
\tkzFct[domain = 0:80, line width=1pt]{4000*(x+20)*exp(-0.05*x)}
|
||||
%M\tkzFct[domain = 0:80, line width=1pt, blue]{40000}
|
||||
\end{tikzpicture}
|
||||
\item Tracer la droite $y=40000$. C'est l'abscisse de l'intersection entre cette droite et la courbe
|
||||
\item
|
||||
\item
|
||||
|
||||
\end{enumerate}
|
||||
|
||||
\end{solution}
|
||||
|
||||
\end{document}
|
||||
|
||||
%%% Local Variables:
|
||||
%%% mode: latex
|
||||
%%% TeX-master: "master"
|
||||
%%% End:
|
||||
121
TES/DM/DM_20_02/10_DM_20_02.tex
Normal file
121
TES/DM/DM_20_02/10_DM_20_02.tex
Normal file
@@ -0,0 +1,121 @@
|
||||
\documentclass[a4paper,10pt]{article}
|
||||
\usepackage{tasks}
|
||||
\usepackage{myXsim}
|
||||
|
||||
\title{DM 2 -- HALEGOI Agathe}
|
||||
\tribe{Terminale ES-L}
|
||||
\date{9 mars 2020}
|
||||
|
||||
\xsimsetup{
|
||||
solution/print = false
|
||||
}
|
||||
|
||||
\begin{document}
|
||||
\maketitle
|
||||
|
||||
\begin{exercise}[subtitle={Étude de fonction}]
|
||||
On considère la fonction dérivable $f$ définie sur $I = [0~;~64]$ par:
|
||||
|
||||
\[f(x) = \np{6000}(x + 16)\text{e}^{- 0.0625x}.\]
|
||||
|
||||
\textbf{Partie A - Étude graphique}
|
||||
|
||||
On a représenté sur le graphique en annexe, la courbe représentative de la fonction $f$.
|
||||
|
||||
\begin{enumerate}
|
||||
\item Avec un tableur tracer et imprimer la courbe représentative de $f$ sur $I$
|
||||
\end{enumerate}
|
||||
\emph{Répondre aux questions suivantes par lecture graphique.}
|
||||
\begin{enumerate}
|
||||
\setcounter{enumi}{1}
|
||||
\item Résoudre graphiquement et de façon approchée l'équation $f(x) = \np{48000}$.
|
||||
\item Donner un encadrement de la quantité
|
||||
\[
|
||||
\int_{2}^{32} f(x) \; dx
|
||||
\]
|
||||
Vous expliquerez votre démarche en utilisant le graphique.
|
||||
\end{enumerate}
|
||||
|
||||
\medskip
|
||||
|
||||
\textbf{Partie B - Étude théorique}
|
||||
|
||||
\begin{enumerate}
|
||||
\setcounter{enumi}{4}
|
||||
\item Étude des variations.
|
||||
\begin{enumerate}
|
||||
\item On note $f'$ la dérivée de la fonction $f$ sur $\intFF{0}{64}$.
|
||||
|
||||
Démontrer que pour tout $x$ de $\intFF{0}{20}$ , $f'(x) = - 375x\text{e}^{-0.0625x}$.
|
||||
|
||||
\item En déduire le sens de variation de $f$ et dresser son tableau des variations sur l'intervalle $\intFF{0}{64}$. Si nécessaire, arrondir à l'unité les valeurs présentes dans le tableau.
|
||||
|
||||
\item Démontrer que l'équation $f(x) = \np{48000}$ admet une unique solution $\alpha$ sur $\intFF{0}{64}$, puis donner une valeur approchée de $\alpha$ à $10^{-2}$ près à l'aide de la calculatrice.
|
||||
\end{enumerate}
|
||||
|
||||
\item Étude de la convexité
|
||||
\begin{enumerate}
|
||||
\item On note $f''$ la dérivée seconde de $f$ sur $\intFF{0}{64}$.
|
||||
|
||||
Démontrer que pour tout $x$ de $\intFF{0}{20}$ , $f''(x) = (23.4375x-375)\text{e}^{-0.0625x}$.
|
||||
|
||||
\item Démontrer que $f$ admet un point d'inflexion dont on donnera son abscisse.
|
||||
\end{enumerate}
|
||||
|
||||
\item Aire sous la courbe
|
||||
\begin{enumerate}
|
||||
\item On souhaite approximer la fonction $f$ sur l'intervalle $I$ par la droite $D$ qui relie les points $(0;f(0))$ et $(64, f(64))$. Tracer cette droite sur le graphique.
|
||||
\item On note $g$ la fonction affine qui décrit cette droite $D$. Détermine l'expression de $g$
|
||||
\item Calculer $\displaystyle \int_0^{64} g(x)\; dx$
|
||||
\item Avec la calculatrice, calculer une valeur approchée de $\displaystyle\int_{0}^{64} f(x) \; dx$
|
||||
\item Comparer les valeurs trouvées aux deux questions précédentes. Comment s'explique l'écart entre ces deux valeurs?
|
||||
\end{enumerate}
|
||||
\end{enumerate}
|
||||
|
||||
\medskip
|
||||
|
||||
\textbf{Partie C - Application économique}
|
||||
|
||||
Une entreprise a pris la décision de fermer son usine de production de smartphones en 64 mois.
|
||||
|
||||
La fonction capacité de production de cette usine est modélisée sur l'intervalle $\intFF{0}{64}$ par la fonction $f$ étudiée dans les parties A et B.
|
||||
|
||||
Le nombre $x$ représente le temps en mois après la décision de la fermeture du site et le nombre $f(x)$ représente capacité production de smartphone au moment $x$.
|
||||
|
||||
Utiliser les résultats de la partie B afin de répondre aux questions suivantes:
|
||||
|
||||
\begin{enumerate}
|
||||
\setcounter{enumi}{7}
|
||||
\item Combien de smartphones pouvaient être produit à la fermeture de l'usine?
|
||||
\item Pendant combien de temps la capacité de production de l'usine a réussi à se maintenir au dessus de \np{48000}?
|
||||
\item Combien de smartphones ont pu être produit entre la prise de décision et la fermeture de l'usine?
|
||||
\end{enumerate}
|
||||
|
||||
\end{exercise}
|
||||
|
||||
\begin{solution}
|
||||
\begin{enumerate}
|
||||
\item
|
||||
\begin{tikzpicture}[yscale=1, xscale=0.234375]
|
||||
\tkzInit[xmin=0,xmax=65,xstep=1,
|
||||
ymin=0,ymax=102000,ystep=10000]
|
||||
\tkzGrid
|
||||
\tkzGrid[sub, subxstep=0.2, subystep=2000.0]
|
||||
\tkzAxeXY[up space=0.5,right space=.2]
|
||||
\tkzFct[domain = 0:64, line width=1pt]{6000*(x+16)*exp(-0.0625*x)}
|
||||
%M\tkzFct[domain = 0:64, line width=1pt, blue]{48000}
|
||||
\end{tikzpicture}
|
||||
\item Tracer la droite $y=48000$. C'est l'abscisse de l'intersection entre cette droite et la courbe
|
||||
\item
|
||||
\item
|
||||
|
||||
\end{enumerate}
|
||||
|
||||
\end{solution}
|
||||
|
||||
\end{document}
|
||||
|
||||
%%% Local Variables:
|
||||
%%% mode: latex
|
||||
%%% TeX-master: "master"
|
||||
%%% End:
|
||||
121
TES/DM/DM_20_02/11_DM_20_02.tex
Normal file
121
TES/DM/DM_20_02/11_DM_20_02.tex
Normal file
@@ -0,0 +1,121 @@
|
||||
\documentclass[a4paper,10pt]{article}
|
||||
\usepackage{tasks}
|
||||
\usepackage{myXsim}
|
||||
|
||||
\title{DM 2 -- JOURDAN Alice}
|
||||
\tribe{Terminale ES-L}
|
||||
\date{9 mars 2020}
|
||||
|
||||
\xsimsetup{
|
||||
solution/print = false
|
||||
}
|
||||
|
||||
\begin{document}
|
||||
\maketitle
|
||||
|
||||
\begin{exercise}[subtitle={Étude de fonction}]
|
||||
On considère la fonction dérivable $f$ définie sur $I = [0~;~32]$ par:
|
||||
|
||||
\[f(x) = \np{3000}(x + 8)\text{e}^{- 0.125x}.\]
|
||||
|
||||
\textbf{Partie A - Étude graphique}
|
||||
|
||||
On a représenté sur le graphique en annexe, la courbe représentative de la fonction $f$.
|
||||
|
||||
\begin{enumerate}
|
||||
\item Avec un tableur tracer et imprimer la courbe représentative de $f$ sur $I$
|
||||
\end{enumerate}
|
||||
\emph{Répondre aux questions suivantes par lecture graphique.}
|
||||
\begin{enumerate}
|
||||
\setcounter{enumi}{1}
|
||||
\item Résoudre graphiquement et de façon approchée l'équation $f(x) = \np{12000}$.
|
||||
\item Donner un encadrement de la quantité
|
||||
\[
|
||||
\int_{2}^{16} f(x) \; dx
|
||||
\]
|
||||
Vous expliquerez votre démarche en utilisant le graphique.
|
||||
\end{enumerate}
|
||||
|
||||
\medskip
|
||||
|
||||
\textbf{Partie B - Étude théorique}
|
||||
|
||||
\begin{enumerate}
|
||||
\setcounter{enumi}{4}
|
||||
\item Étude des variations.
|
||||
\begin{enumerate}
|
||||
\item On note $f'$ la dérivée de la fonction $f$ sur $\intFF{0}{32}$.
|
||||
|
||||
Démontrer que pour tout $x$ de $\intFF{0}{20}$ , $f'(x) = - 375x\text{e}^{-0.125x}$.
|
||||
|
||||
\item En déduire le sens de variation de $f$ et dresser son tableau des variations sur l'intervalle $\intFF{0}{32}$. Si nécessaire, arrondir à l'unité les valeurs présentes dans le tableau.
|
||||
|
||||
\item Démontrer que l'équation $f(x) = \np{12000}$ admet une unique solution $\alpha$ sur $\intFF{0}{32}$, puis donner une valeur approchée de $\alpha$ à $10^{-2}$ près à l'aide de la calculatrice.
|
||||
\end{enumerate}
|
||||
|
||||
\item Étude de la convexité
|
||||
\begin{enumerate}
|
||||
\item On note $f''$ la dérivée seconde de $f$ sur $\intFF{0}{32}$.
|
||||
|
||||
Démontrer que pour tout $x$ de $\intFF{0}{20}$ , $f''(x) = (46.875x-375)\text{e}^{-0.125x}$.
|
||||
|
||||
\item Démontrer que $f$ admet un point d'inflexion dont on donnera son abscisse.
|
||||
\end{enumerate}
|
||||
|
||||
\item Aire sous la courbe
|
||||
\begin{enumerate}
|
||||
\item On souhaite approximer la fonction $f$ sur l'intervalle $I$ par la droite $D$ qui relie les points $(0;f(0))$ et $(32, f(32))$. Tracer cette droite sur le graphique.
|
||||
\item On note $g$ la fonction affine qui décrit cette droite $D$. Détermine l'expression de $g$
|
||||
\item Calculer $\displaystyle \int_0^{32} g(x)\; dx$
|
||||
\item Avec la calculatrice, calculer une valeur approchée de $\displaystyle\int_{0}^{32} f(x) \; dx$
|
||||
\item Comparer les valeurs trouvées aux deux questions précédentes. Comment s'explique l'écart entre ces deux valeurs?
|
||||
\end{enumerate}
|
||||
\end{enumerate}
|
||||
|
||||
\medskip
|
||||
|
||||
\textbf{Partie C - Application économique}
|
||||
|
||||
Une entreprise a pris la décision de fermer son usine de production de smartphones en 32 mois.
|
||||
|
||||
La fonction capacité de production de cette usine est modélisée sur l'intervalle $\intFF{0}{32}$ par la fonction $f$ étudiée dans les parties A et B.
|
||||
|
||||
Le nombre $x$ représente le temps en mois après la décision de la fermeture du site et le nombre $f(x)$ représente capacité production de smartphone au moment $x$.
|
||||
|
||||
Utiliser les résultats de la partie B afin de répondre aux questions suivantes:
|
||||
|
||||
\begin{enumerate}
|
||||
\setcounter{enumi}{7}
|
||||
\item Combien de smartphones pouvaient être produit à la fermeture de l'usine?
|
||||
\item Pendant combien de temps la capacité de production de l'usine a réussi à se maintenir au dessus de \np{12000}?
|
||||
\item Combien de smartphones ont pu être produit entre la prise de décision et la fermeture de l'usine?
|
||||
\end{enumerate}
|
||||
|
||||
\end{exercise}
|
||||
|
||||
\begin{solution}
|
||||
\begin{enumerate}
|
||||
\item
|
||||
\begin{tikzpicture}[yscale=1, xscale=0.46875]
|
||||
\tkzInit[xmin=0,xmax=33,xstep=1,
|
||||
ymin=0,ymax=27000,ystep=2000]
|
||||
\tkzGrid
|
||||
\tkzGrid[sub, subxstep=0.2, subystep=400.0]
|
||||
\tkzAxeXY[up space=0.5,right space=.2]
|
||||
\tkzFct[domain = 0:32, line width=1pt]{3000*(x+8)*exp(-0.125*x)}
|
||||
%M\tkzFct[domain = 0:32, line width=1pt, blue]{12000}
|
||||
\end{tikzpicture}
|
||||
\item Tracer la droite $y=12000$. C'est l'abscisse de l'intersection entre cette droite et la courbe
|
||||
\item
|
||||
\item
|
||||
|
||||
\end{enumerate}
|
||||
|
||||
\end{solution}
|
||||
|
||||
\end{document}
|
||||
|
||||
%%% Local Variables:
|
||||
%%% mode: latex
|
||||
%%% TeX-master: "master"
|
||||
%%% End:
|
||||
121
TES/DM/DM_20_02/12_DM_20_02.tex
Normal file
121
TES/DM/DM_20_02/12_DM_20_02.tex
Normal file
@@ -0,0 +1,121 @@
|
||||
\documentclass[a4paper,10pt]{article}
|
||||
\usepackage{tasks}
|
||||
\usepackage{myXsim}
|
||||
|
||||
\title{DM 2 -- LIANDRAT Léa}
|
||||
\tribe{Terminale ES-L}
|
||||
\date{9 mars 2020}
|
||||
|
||||
\xsimsetup{
|
||||
solution/print = false
|
||||
}
|
||||
|
||||
\begin{document}
|
||||
\maketitle
|
||||
|
||||
\begin{exercise}[subtitle={Étude de fonction}]
|
||||
On considère la fonction dérivable $f$ définie sur $I = [0~;~32]$ par:
|
||||
|
||||
\[f(x) = \np{3000}(x + 8)\text{e}^{- 0.125x}.\]
|
||||
|
||||
\textbf{Partie A - Étude graphique}
|
||||
|
||||
On a représenté sur le graphique en annexe, la courbe représentative de la fonction $f$.
|
||||
|
||||
\begin{enumerate}
|
||||
\item Avec un tableur tracer et imprimer la courbe représentative de $f$ sur $I$
|
||||
\end{enumerate}
|
||||
\emph{Répondre aux questions suivantes par lecture graphique.}
|
||||
\begin{enumerate}
|
||||
\setcounter{enumi}{1}
|
||||
\item Résoudre graphiquement et de façon approchée l'équation $f(x) = \np{12000}$.
|
||||
\item Donner un encadrement de la quantité
|
||||
\[
|
||||
\int_{2}^{16} f(x) \; dx
|
||||
\]
|
||||
Vous expliquerez votre démarche en utilisant le graphique.
|
||||
\end{enumerate}
|
||||
|
||||
\medskip
|
||||
|
||||
\textbf{Partie B - Étude théorique}
|
||||
|
||||
\begin{enumerate}
|
||||
\setcounter{enumi}{4}
|
||||
\item Étude des variations.
|
||||
\begin{enumerate}
|
||||
\item On note $f'$ la dérivée de la fonction $f$ sur $\intFF{0}{32}$.
|
||||
|
||||
Démontrer que pour tout $x$ de $\intFF{0}{20}$ , $f'(x) = - 375x\text{e}^{-0.125x}$.
|
||||
|
||||
\item En déduire le sens de variation de $f$ et dresser son tableau des variations sur l'intervalle $\intFF{0}{32}$. Si nécessaire, arrondir à l'unité les valeurs présentes dans le tableau.
|
||||
|
||||
\item Démontrer que l'équation $f(x) = \np{12000}$ admet une unique solution $\alpha$ sur $\intFF{0}{32}$, puis donner une valeur approchée de $\alpha$ à $10^{-2}$ près à l'aide de la calculatrice.
|
||||
\end{enumerate}
|
||||
|
||||
\item Étude de la convexité
|
||||
\begin{enumerate}
|
||||
\item On note $f''$ la dérivée seconde de $f$ sur $\intFF{0}{32}$.
|
||||
|
||||
Démontrer que pour tout $x$ de $\intFF{0}{20}$ , $f''(x) = (46.875x-375)\text{e}^{-0.125x}$.
|
||||
|
||||
\item Démontrer que $f$ admet un point d'inflexion dont on donnera son abscisse.
|
||||
\end{enumerate}
|
||||
|
||||
\item Aire sous la courbe
|
||||
\begin{enumerate}
|
||||
\item On souhaite approximer la fonction $f$ sur l'intervalle $I$ par la droite $D$ qui relie les points $(0;f(0))$ et $(32, f(32))$. Tracer cette droite sur le graphique.
|
||||
\item On note $g$ la fonction affine qui décrit cette droite $D$. Détermine l'expression de $g$
|
||||
\item Calculer $\displaystyle \int_0^{32} g(x)\; dx$
|
||||
\item Avec la calculatrice, calculer une valeur approchée de $\displaystyle\int_{0}^{32} f(x) \; dx$
|
||||
\item Comparer les valeurs trouvées aux deux questions précédentes. Comment s'explique l'écart entre ces deux valeurs?
|
||||
\end{enumerate}
|
||||
\end{enumerate}
|
||||
|
||||
\medskip
|
||||
|
||||
\textbf{Partie C - Application économique}
|
||||
|
||||
Une entreprise a pris la décision de fermer son usine de production de smartphones en 32 mois.
|
||||
|
||||
La fonction capacité de production de cette usine est modélisée sur l'intervalle $\intFF{0}{32}$ par la fonction $f$ étudiée dans les parties A et B.
|
||||
|
||||
Le nombre $x$ représente le temps en mois après la décision de la fermeture du site et le nombre $f(x)$ représente capacité production de smartphone au moment $x$.
|
||||
|
||||
Utiliser les résultats de la partie B afin de répondre aux questions suivantes:
|
||||
|
||||
\begin{enumerate}
|
||||
\setcounter{enumi}{7}
|
||||
\item Combien de smartphones pouvaient être produit à la fermeture de l'usine?
|
||||
\item Pendant combien de temps la capacité de production de l'usine a réussi à se maintenir au dessus de \np{12000}?
|
||||
\item Combien de smartphones ont pu être produit entre la prise de décision et la fermeture de l'usine?
|
||||
\end{enumerate}
|
||||
|
||||
\end{exercise}
|
||||
|
||||
\begin{solution}
|
||||
\begin{enumerate}
|
||||
\item
|
||||
\begin{tikzpicture}[yscale=1, xscale=0.46875]
|
||||
\tkzInit[xmin=0,xmax=33,xstep=1,
|
||||
ymin=0,ymax=27000,ystep=2000]
|
||||
\tkzGrid
|
||||
\tkzGrid[sub, subxstep=0.2, subystep=400.0]
|
||||
\tkzAxeXY[up space=0.5,right space=.2]
|
||||
\tkzFct[domain = 0:32, line width=1pt]{3000*(x+8)*exp(-0.125*x)}
|
||||
%M\tkzFct[domain = 0:32, line width=1pt, blue]{12000}
|
||||
\end{tikzpicture}
|
||||
\item Tracer la droite $y=12000$. C'est l'abscisse de l'intersection entre cette droite et la courbe
|
||||
\item
|
||||
\item
|
||||
|
||||
\end{enumerate}
|
||||
|
||||
\end{solution}
|
||||
|
||||
\end{document}
|
||||
|
||||
%%% Local Variables:
|
||||
%%% mode: latex
|
||||
%%% TeX-master: "master"
|
||||
%%% End:
|
||||
121
TES/DM/DM_20_02/13_DM_20_02.tex
Normal file
121
TES/DM/DM_20_02/13_DM_20_02.tex
Normal file
@@ -0,0 +1,121 @@
|
||||
\documentclass[a4paper,10pt]{article}
|
||||
\usepackage{tasks}
|
||||
\usepackage{myXsim}
|
||||
|
||||
\title{DM 2 -- LOULID Manar}
|
||||
\tribe{Terminale ES-L}
|
||||
\date{9 mars 2020}
|
||||
|
||||
\xsimsetup{
|
||||
solution/print = false
|
||||
}
|
||||
|
||||
\begin{document}
|
||||
\maketitle
|
||||
|
||||
\begin{exercise}[subtitle={Étude de fonction}]
|
||||
On considère la fonction dérivable $f$ définie sur $I = [0~;~16]$ par:
|
||||
|
||||
\[f(x) = \np{7000}(x + 4)\text{e}^{- 0.25x}.\]
|
||||
|
||||
\textbf{Partie A - Étude graphique}
|
||||
|
||||
On a représenté sur le graphique en annexe, la courbe représentative de la fonction $f$.
|
||||
|
||||
\begin{enumerate}
|
||||
\item Avec un tableur tracer et imprimer la courbe représentative de $f$ sur $I$
|
||||
\end{enumerate}
|
||||
\emph{Répondre aux questions suivantes par lecture graphique.}
|
||||
\begin{enumerate}
|
||||
\setcounter{enumi}{1}
|
||||
\item Résoudre graphiquement et de façon approchée l'équation $f(x) = \np{14000}$.
|
||||
\item Donner un encadrement de la quantité
|
||||
\[
|
||||
\int_{2}^{8} f(x) \; dx
|
||||
\]
|
||||
Vous expliquerez votre démarche en utilisant le graphique.
|
||||
\end{enumerate}
|
||||
|
||||
\medskip
|
||||
|
||||
\textbf{Partie B - Étude théorique}
|
||||
|
||||
\begin{enumerate}
|
||||
\setcounter{enumi}{4}
|
||||
\item Étude des variations.
|
||||
\begin{enumerate}
|
||||
\item On note $f'$ la dérivée de la fonction $f$ sur $\intFF{0}{16}$.
|
||||
|
||||
Démontrer que pour tout $x$ de $\intFF{0}{20}$ , $f'(x) = - 1750x\text{e}^{-0.25x}$.
|
||||
|
||||
\item En déduire le sens de variation de $f$ et dresser son tableau des variations sur l'intervalle $\intFF{0}{16}$. Si nécessaire, arrondir à l'unité les valeurs présentes dans le tableau.
|
||||
|
||||
\item Démontrer que l'équation $f(x) = \np{14000}$ admet une unique solution $\alpha$ sur $\intFF{0}{16}$, puis donner une valeur approchée de $\alpha$ à $10^{-2}$ près à l'aide de la calculatrice.
|
||||
\end{enumerate}
|
||||
|
||||
\item Étude de la convexité
|
||||
\begin{enumerate}
|
||||
\item On note $f''$ la dérivée seconde de $f$ sur $\intFF{0}{16}$.
|
||||
|
||||
Démontrer que pour tout $x$ de $\intFF{0}{20}$ , $f''(x) = (437.50x-1750)\text{e}^{-0.25x}$.
|
||||
|
||||
\item Démontrer que $f$ admet un point d'inflexion dont on donnera son abscisse.
|
||||
\end{enumerate}
|
||||
|
||||
\item Aire sous la courbe
|
||||
\begin{enumerate}
|
||||
\item On souhaite approximer la fonction $f$ sur l'intervalle $I$ par la droite $D$ qui relie les points $(0;f(0))$ et $(16, f(16))$. Tracer cette droite sur le graphique.
|
||||
\item On note $g$ la fonction affine qui décrit cette droite $D$. Détermine l'expression de $g$
|
||||
\item Calculer $\displaystyle \int_0^{16} g(x)\; dx$
|
||||
\item Avec la calculatrice, calculer une valeur approchée de $\displaystyle\int_{0}^{16} f(x) \; dx$
|
||||
\item Comparer les valeurs trouvées aux deux questions précédentes. Comment s'explique l'écart entre ces deux valeurs?
|
||||
\end{enumerate}
|
||||
\end{enumerate}
|
||||
|
||||
\medskip
|
||||
|
||||
\textbf{Partie C - Application économique}
|
||||
|
||||
Une entreprise a pris la décision de fermer son usine de production de smartphones en 16 mois.
|
||||
|
||||
La fonction capacité de production de cette usine est modélisée sur l'intervalle $\intFF{0}{16}$ par la fonction $f$ étudiée dans les parties A et B.
|
||||
|
||||
Le nombre $x$ représente le temps en mois après la décision de la fermeture du site et le nombre $f(x)$ représente capacité production de smartphone au moment $x$.
|
||||
|
||||
Utiliser les résultats de la partie B afin de répondre aux questions suivantes:
|
||||
|
||||
\begin{enumerate}
|
||||
\setcounter{enumi}{7}
|
||||
\item Combien de smartphones pouvaient être produit à la fermeture de l'usine?
|
||||
\item Pendant combien de temps la capacité de production de l'usine a réussi à se maintenir au dessus de \np{14000}?
|
||||
\item Combien de smartphones ont pu être produit entre la prise de décision et la fermeture de l'usine?
|
||||
\end{enumerate}
|
||||
|
||||
\end{exercise}
|
||||
|
||||
\begin{solution}
|
||||
\begin{enumerate}
|
||||
\item
|
||||
\begin{tikzpicture}[yscale=1, xscale=0.9375]
|
||||
\tkzInit[xmin=0,xmax=17,xstep=1,
|
||||
ymin=0,ymax=35000,ystep=3000]
|
||||
\tkzGrid
|
||||
\tkzGrid[sub, subxstep=0.2, subystep=600.0]
|
||||
\tkzAxeXY[up space=0.5,right space=.2]
|
||||
\tkzFct[domain = 0:16, line width=1pt]{7000*(x+4)*exp(-0.25*x)}
|
||||
%M\tkzFct[domain = 0:16, line width=1pt, blue]{14000}
|
||||
\end{tikzpicture}
|
||||
\item Tracer la droite $y=14000$. C'est l'abscisse de l'intersection entre cette droite et la courbe
|
||||
\item
|
||||
\item
|
||||
|
||||
\end{enumerate}
|
||||
|
||||
\end{solution}
|
||||
|
||||
\end{document}
|
||||
|
||||
%%% Local Variables:
|
||||
%%% mode: latex
|
||||
%%% TeX-master: "master"
|
||||
%%% End:
|
||||
121
TES/DM/DM_20_02/14_DM_20_02.tex
Normal file
121
TES/DM/DM_20_02/14_DM_20_02.tex
Normal file
@@ -0,0 +1,121 @@
|
||||
\documentclass[a4paper,10pt]{article}
|
||||
\usepackage{tasks}
|
||||
\usepackage{myXsim}
|
||||
|
||||
\title{DM 2 -- MARQUET Elisa}
|
||||
\tribe{Terminale ES-L}
|
||||
\date{9 mars 2020}
|
||||
|
||||
\xsimsetup{
|
||||
solution/print = false
|
||||
}
|
||||
|
||||
\begin{document}
|
||||
\maketitle
|
||||
|
||||
\begin{exercise}[subtitle={Étude de fonction}]
|
||||
On considère la fonction dérivable $f$ définie sur $I = [0~;~32]$ par:
|
||||
|
||||
\[f(x) = \np{8000}(x + 8)\text{e}^{- 0.125x}.\]
|
||||
|
||||
\textbf{Partie A - Étude graphique}
|
||||
|
||||
On a représenté sur le graphique en annexe, la courbe représentative de la fonction $f$.
|
||||
|
||||
\begin{enumerate}
|
||||
\item Avec un tableur tracer et imprimer la courbe représentative de $f$ sur $I$
|
||||
\end{enumerate}
|
||||
\emph{Répondre aux questions suivantes par lecture graphique.}
|
||||
\begin{enumerate}
|
||||
\setcounter{enumi}{1}
|
||||
\item Résoudre graphiquement et de façon approchée l'équation $f(x) = \np{32000}$.
|
||||
\item Donner un encadrement de la quantité
|
||||
\[
|
||||
\int_{2}^{16} f(x) \; dx
|
||||
\]
|
||||
Vous expliquerez votre démarche en utilisant le graphique.
|
||||
\end{enumerate}
|
||||
|
||||
\medskip
|
||||
|
||||
\textbf{Partie B - Étude théorique}
|
||||
|
||||
\begin{enumerate}
|
||||
\setcounter{enumi}{4}
|
||||
\item Étude des variations.
|
||||
\begin{enumerate}
|
||||
\item On note $f'$ la dérivée de la fonction $f$ sur $\intFF{0}{32}$.
|
||||
|
||||
Démontrer que pour tout $x$ de $\intFF{0}{20}$ , $f'(x) = - 1000x\text{e}^{-0.125x}$.
|
||||
|
||||
\item En déduire le sens de variation de $f$ et dresser son tableau des variations sur l'intervalle $\intFF{0}{32}$. Si nécessaire, arrondir à l'unité les valeurs présentes dans le tableau.
|
||||
|
||||
\item Démontrer que l'équation $f(x) = \np{32000}$ admet une unique solution $\alpha$ sur $\intFF{0}{32}$, puis donner une valeur approchée de $\alpha$ à $10^{-2}$ près à l'aide de la calculatrice.
|
||||
\end{enumerate}
|
||||
|
||||
\item Étude de la convexité
|
||||
\begin{enumerate}
|
||||
\item On note $f''$ la dérivée seconde de $f$ sur $\intFF{0}{32}$.
|
||||
|
||||
Démontrer que pour tout $x$ de $\intFF{0}{20}$ , $f''(x) = (125x-1000)\text{e}^{-0.125x}$.
|
||||
|
||||
\item Démontrer que $f$ admet un point d'inflexion dont on donnera son abscisse.
|
||||
\end{enumerate}
|
||||
|
||||
\item Aire sous la courbe
|
||||
\begin{enumerate}
|
||||
\item On souhaite approximer la fonction $f$ sur l'intervalle $I$ par la droite $D$ qui relie les points $(0;f(0))$ et $(32, f(32))$. Tracer cette droite sur le graphique.
|
||||
\item On note $g$ la fonction affine qui décrit cette droite $D$. Détermine l'expression de $g$
|
||||
\item Calculer $\displaystyle \int_0^{32} g(x)\; dx$
|
||||
\item Avec la calculatrice, calculer une valeur approchée de $\displaystyle\int_{0}^{32} f(x) \; dx$
|
||||
\item Comparer les valeurs trouvées aux deux questions précédentes. Comment s'explique l'écart entre ces deux valeurs?
|
||||
\end{enumerate}
|
||||
\end{enumerate}
|
||||
|
||||
\medskip
|
||||
|
||||
\textbf{Partie C - Application économique}
|
||||
|
||||
Une entreprise a pris la décision de fermer son usine de production de smartphones en 32 mois.
|
||||
|
||||
La fonction capacité de production de cette usine est modélisée sur l'intervalle $\intFF{0}{32}$ par la fonction $f$ étudiée dans les parties A et B.
|
||||
|
||||
Le nombre $x$ représente le temps en mois après la décision de la fermeture du site et le nombre $f(x)$ représente capacité production de smartphone au moment $x$.
|
||||
|
||||
Utiliser les résultats de la partie B afin de répondre aux questions suivantes:
|
||||
|
||||
\begin{enumerate}
|
||||
\setcounter{enumi}{7}
|
||||
\item Combien de smartphones pouvaient être produit à la fermeture de l'usine?
|
||||
\item Pendant combien de temps la capacité de production de l'usine a réussi à se maintenir au dessus de \np{32000}?
|
||||
\item Combien de smartphones ont pu être produit entre la prise de décision et la fermeture de l'usine?
|
||||
\end{enumerate}
|
||||
|
||||
\end{exercise}
|
||||
|
||||
\begin{solution}
|
||||
\begin{enumerate}
|
||||
\item
|
||||
\begin{tikzpicture}[yscale=1, xscale=0.46875]
|
||||
\tkzInit[xmin=0,xmax=33,xstep=1,
|
||||
ymin=0,ymax=72000,ystep=7000]
|
||||
\tkzGrid
|
||||
\tkzGrid[sub, subxstep=0.2, subystep=1400.0]
|
||||
\tkzAxeXY[up space=0.5,right space=.2]
|
||||
\tkzFct[domain = 0:32, line width=1pt]{8000*(x+8)*exp(-0.125*x)}
|
||||
%M\tkzFct[domain = 0:32, line width=1pt, blue]{32000}
|
||||
\end{tikzpicture}
|
||||
\item Tracer la droite $y=32000$. C'est l'abscisse de l'intersection entre cette droite et la courbe
|
||||
\item
|
||||
\item
|
||||
|
||||
\end{enumerate}
|
||||
|
||||
\end{solution}
|
||||
|
||||
\end{document}
|
||||
|
||||
%%% Local Variables:
|
||||
%%% mode: latex
|
||||
%%% TeX-master: "master"
|
||||
%%% End:
|
||||
121
TES/DM/DM_20_02/15_DM_20_02.tex
Normal file
121
TES/DM/DM_20_02/15_DM_20_02.tex
Normal file
@@ -0,0 +1,121 @@
|
||||
\documentclass[a4paper,10pt]{article}
|
||||
\usepackage{tasks}
|
||||
\usepackage{myXsim}
|
||||
|
||||
\title{DM 2 -- MENARD Cassandre}
|
||||
\tribe{Terminale ES-L}
|
||||
\date{9 mars 2020}
|
||||
|
||||
\xsimsetup{
|
||||
solution/print = false
|
||||
}
|
||||
|
||||
\begin{document}
|
||||
\maketitle
|
||||
|
||||
\begin{exercise}[subtitle={Étude de fonction}]
|
||||
On considère la fonction dérivable $f$ définie sur $I = [0~;~80]$ par:
|
||||
|
||||
\[f(x) = \np{6000}(x + 20)\text{e}^{- 0.05x}.\]
|
||||
|
||||
\textbf{Partie A - Étude graphique}
|
||||
|
||||
On a représenté sur le graphique en annexe, la courbe représentative de la fonction $f$.
|
||||
|
||||
\begin{enumerate}
|
||||
\item Avec un tableur tracer et imprimer la courbe représentative de $f$ sur $I$
|
||||
\end{enumerate}
|
||||
\emph{Répondre aux questions suivantes par lecture graphique.}
|
||||
\begin{enumerate}
|
||||
\setcounter{enumi}{1}
|
||||
\item Résoudre graphiquement et de façon approchée l'équation $f(x) = \np{60000}$.
|
||||
\item Donner un encadrement de la quantité
|
||||
\[
|
||||
\int_{2}^{40} f(x) \; dx
|
||||
\]
|
||||
Vous expliquerez votre démarche en utilisant le graphique.
|
||||
\end{enumerate}
|
||||
|
||||
\medskip
|
||||
|
||||
\textbf{Partie B - Étude théorique}
|
||||
|
||||
\begin{enumerate}
|
||||
\setcounter{enumi}{4}
|
||||
\item Étude des variations.
|
||||
\begin{enumerate}
|
||||
\item On note $f'$ la dérivée de la fonction $f$ sur $\intFF{0}{80}$.
|
||||
|
||||
Démontrer que pour tout $x$ de $\intFF{0}{20}$ , $f'(x) = - 300x\text{e}^{-0.05x}$.
|
||||
|
||||
\item En déduire le sens de variation de $f$ et dresser son tableau des variations sur l'intervalle $\intFF{0}{80}$. Si nécessaire, arrondir à l'unité les valeurs présentes dans le tableau.
|
||||
|
||||
\item Démontrer que l'équation $f(x) = \np{60000}$ admet une unique solution $\alpha$ sur $\intFF{0}{80}$, puis donner une valeur approchée de $\alpha$ à $10^{-2}$ près à l'aide de la calculatrice.
|
||||
\end{enumerate}
|
||||
|
||||
\item Étude de la convexité
|
||||
\begin{enumerate}
|
||||
\item On note $f''$ la dérivée seconde de $f$ sur $\intFF{0}{80}$.
|
||||
|
||||
Démontrer que pour tout $x$ de $\intFF{0}{20}$ , $f''(x) = (15x-300)\text{e}^{-0.05x}$.
|
||||
|
||||
\item Démontrer que $f$ admet un point d'inflexion dont on donnera son abscisse.
|
||||
\end{enumerate}
|
||||
|
||||
\item Aire sous la courbe
|
||||
\begin{enumerate}
|
||||
\item On souhaite approximer la fonction $f$ sur l'intervalle $I$ par la droite $D$ qui relie les points $(0;f(0))$ et $(80, f(80))$. Tracer cette droite sur le graphique.
|
||||
\item On note $g$ la fonction affine qui décrit cette droite $D$. Détermine l'expression de $g$
|
||||
\item Calculer $\displaystyle \int_0^{80} g(x)\; dx$
|
||||
\item Avec la calculatrice, calculer une valeur approchée de $\displaystyle\int_{0}^{80} f(x) \; dx$
|
||||
\item Comparer les valeurs trouvées aux deux questions précédentes. Comment s'explique l'écart entre ces deux valeurs?
|
||||
\end{enumerate}
|
||||
\end{enumerate}
|
||||
|
||||
\medskip
|
||||
|
||||
\textbf{Partie C - Application économique}
|
||||
|
||||
Une entreprise a pris la décision de fermer son usine de production de smartphones en 80 mois.
|
||||
|
||||
La fonction capacité de production de cette usine est modélisée sur l'intervalle $\intFF{0}{80}$ par la fonction $f$ étudiée dans les parties A et B.
|
||||
|
||||
Le nombre $x$ représente le temps en mois après la décision de la fermeture du site et le nombre $f(x)$ représente capacité production de smartphone au moment $x$.
|
||||
|
||||
Utiliser les résultats de la partie B afin de répondre aux questions suivantes:
|
||||
|
||||
\begin{enumerate}
|
||||
\setcounter{enumi}{7}
|
||||
\item Combien de smartphones pouvaient être produit à la fermeture de l'usine?
|
||||
\item Pendant combien de temps la capacité de production de l'usine a réussi à se maintenir au dessus de \np{60000}?
|
||||
\item Combien de smartphones ont pu être produit entre la prise de décision et la fermeture de l'usine?
|
||||
\end{enumerate}
|
||||
|
||||
\end{exercise}
|
||||
|
||||
\begin{solution}
|
||||
\begin{enumerate}
|
||||
\item
|
||||
\begin{tikzpicture}[yscale=1, xscale=0.1875]
|
||||
\tkzInit[xmin=0,xmax=81,xstep=1,
|
||||
ymin=0,ymax=126000,ystep=12000]
|
||||
\tkzGrid
|
||||
\tkzGrid[sub, subxstep=0.2, subystep=2400.0]
|
||||
\tkzAxeXY[up space=0.5,right space=.2]
|
||||
\tkzFct[domain = 0:80, line width=1pt]{6000*(x+20)*exp(-0.05*x)}
|
||||
%M\tkzFct[domain = 0:80, line width=1pt, blue]{60000}
|
||||
\end{tikzpicture}
|
||||
\item Tracer la droite $y=60000$. C'est l'abscisse de l'intersection entre cette droite et la courbe
|
||||
\item
|
||||
\item
|
||||
|
||||
\end{enumerate}
|
||||
|
||||
\end{solution}
|
||||
|
||||
\end{document}
|
||||
|
||||
%%% Local Variables:
|
||||
%%% mode: latex
|
||||
%%% TeX-master: "master"
|
||||
%%% End:
|
||||
121
TES/DM/DM_20_02/16_DM_20_02.tex
Normal file
121
TES/DM/DM_20_02/16_DM_20_02.tex
Normal file
@@ -0,0 +1,121 @@
|
||||
\documentclass[a4paper,10pt]{article}
|
||||
\usepackage{tasks}
|
||||
\usepackage{myXsim}
|
||||
|
||||
\title{DM 2 -- MICHEL-PROST Lauryne}
|
||||
\tribe{Terminale ES-L}
|
||||
\date{9 mars 2020}
|
||||
|
||||
\xsimsetup{
|
||||
solution/print = false
|
||||
}
|
||||
|
||||
\begin{document}
|
||||
\maketitle
|
||||
|
||||
\begin{exercise}[subtitle={Étude de fonction}]
|
||||
On considère la fonction dérivable $f$ définie sur $I = [0~;~80]$ par:
|
||||
|
||||
\[f(x) = \np{10000}(x + 20)\text{e}^{- 0.05x}.\]
|
||||
|
||||
\textbf{Partie A - Étude graphique}
|
||||
|
||||
On a représenté sur le graphique en annexe, la courbe représentative de la fonction $f$.
|
||||
|
||||
\begin{enumerate}
|
||||
\item Avec un tableur tracer et imprimer la courbe représentative de $f$ sur $I$
|
||||
\end{enumerate}
|
||||
\emph{Répondre aux questions suivantes par lecture graphique.}
|
||||
\begin{enumerate}
|
||||
\setcounter{enumi}{1}
|
||||
\item Résoudre graphiquement et de façon approchée l'équation $f(x) = \np{100000}$.
|
||||
\item Donner un encadrement de la quantité
|
||||
\[
|
||||
\int_{2}^{40} f(x) \; dx
|
||||
\]
|
||||
Vous expliquerez votre démarche en utilisant le graphique.
|
||||
\end{enumerate}
|
||||
|
||||
\medskip
|
||||
|
||||
\textbf{Partie B - Étude théorique}
|
||||
|
||||
\begin{enumerate}
|
||||
\setcounter{enumi}{4}
|
||||
\item Étude des variations.
|
||||
\begin{enumerate}
|
||||
\item On note $f'$ la dérivée de la fonction $f$ sur $\intFF{0}{80}$.
|
||||
|
||||
Démontrer que pour tout $x$ de $\intFF{0}{20}$ , $f'(x) = - 500x\text{e}^{-0.05x}$.
|
||||
|
||||
\item En déduire le sens de variation de $f$ et dresser son tableau des variations sur l'intervalle $\intFF{0}{80}$. Si nécessaire, arrondir à l'unité les valeurs présentes dans le tableau.
|
||||
|
||||
\item Démontrer que l'équation $f(x) = \np{100000}$ admet une unique solution $\alpha$ sur $\intFF{0}{80}$, puis donner une valeur approchée de $\alpha$ à $10^{-2}$ près à l'aide de la calculatrice.
|
||||
\end{enumerate}
|
||||
|
||||
\item Étude de la convexité
|
||||
\begin{enumerate}
|
||||
\item On note $f''$ la dérivée seconde de $f$ sur $\intFF{0}{80}$.
|
||||
|
||||
Démontrer que pour tout $x$ de $\intFF{0}{20}$ , $f''(x) = (25x-500)\text{e}^{-0.05x}$.
|
||||
|
||||
\item Démontrer que $f$ admet un point d'inflexion dont on donnera son abscisse.
|
||||
\end{enumerate}
|
||||
|
||||
\item Aire sous la courbe
|
||||
\begin{enumerate}
|
||||
\item On souhaite approximer la fonction $f$ sur l'intervalle $I$ par la droite $D$ qui relie les points $(0;f(0))$ et $(80, f(80))$. Tracer cette droite sur le graphique.
|
||||
\item On note $g$ la fonction affine qui décrit cette droite $D$. Détermine l'expression de $g$
|
||||
\item Calculer $\displaystyle \int_0^{80} g(x)\; dx$
|
||||
\item Avec la calculatrice, calculer une valeur approchée de $\displaystyle\int_{0}^{80} f(x) \; dx$
|
||||
\item Comparer les valeurs trouvées aux deux questions précédentes. Comment s'explique l'écart entre ces deux valeurs?
|
||||
\end{enumerate}
|
||||
\end{enumerate}
|
||||
|
||||
\medskip
|
||||
|
||||
\textbf{Partie C - Application économique}
|
||||
|
||||
Une entreprise a pris la décision de fermer son usine de production de smartphones en 80 mois.
|
||||
|
||||
La fonction capacité de production de cette usine est modélisée sur l'intervalle $\intFF{0}{80}$ par la fonction $f$ étudiée dans les parties A et B.
|
||||
|
||||
Le nombre $x$ représente le temps en mois après la décision de la fermeture du site et le nombre $f(x)$ représente capacité production de smartphone au moment $x$.
|
||||
|
||||
Utiliser les résultats de la partie B afin de répondre aux questions suivantes:
|
||||
|
||||
\begin{enumerate}
|
||||
\setcounter{enumi}{7}
|
||||
\item Combien de smartphones pouvaient être produit à la fermeture de l'usine?
|
||||
\item Pendant combien de temps la capacité de production de l'usine a réussi à se maintenir au dessus de \np{100000}?
|
||||
\item Combien de smartphones ont pu être produit entre la prise de décision et la fermeture de l'usine?
|
||||
\end{enumerate}
|
||||
|
||||
\end{exercise}
|
||||
|
||||
\begin{solution}
|
||||
\begin{enumerate}
|
||||
\item
|
||||
\begin{tikzpicture}[yscale=1, xscale=0.1875]
|
||||
\tkzInit[xmin=0,xmax=81,xstep=1,
|
||||
ymin=0,ymax=210000,ystep=21000]
|
||||
\tkzGrid
|
||||
\tkzGrid[sub, subxstep=0.2, subystep=4200.0]
|
||||
\tkzAxeXY[up space=0.5,right space=.2]
|
||||
\tkzFct[domain = 0:80, line width=1pt]{10000*(x+20)*exp(-0.05*x)}
|
||||
%M\tkzFct[domain = 0:80, line width=1pt, blue]{100000}
|
||||
\end{tikzpicture}
|
||||
\item Tracer la droite $y=100000$. C'est l'abscisse de l'intersection entre cette droite et la courbe
|
||||
\item
|
||||
\item
|
||||
|
||||
\end{enumerate}
|
||||
|
||||
\end{solution}
|
||||
|
||||
\end{document}
|
||||
|
||||
%%% Local Variables:
|
||||
%%% mode: latex
|
||||
%%% TeX-master: "master"
|
||||
%%% End:
|
||||
121
TES/DM/DM_20_02/17_DM_20_02.tex
Normal file
121
TES/DM/DM_20_02/17_DM_20_02.tex
Normal file
@@ -0,0 +1,121 @@
|
||||
\documentclass[a4paper,10pt]{article}
|
||||
\usepackage{tasks}
|
||||
\usepackage{myXsim}
|
||||
|
||||
\title{DM 2 -- MOUBARIK Ines}
|
||||
\tribe{Terminale ES-L}
|
||||
\date{9 mars 2020}
|
||||
|
||||
\xsimsetup{
|
||||
solution/print = false
|
||||
}
|
||||
|
||||
\begin{document}
|
||||
\maketitle
|
||||
|
||||
\begin{exercise}[subtitle={Étude de fonction}]
|
||||
On considère la fonction dérivable $f$ définie sur $I = [0~;~32]$ par:
|
||||
|
||||
\[f(x) = \np{3000}(x + 8)\text{e}^{- 0.125x}.\]
|
||||
|
||||
\textbf{Partie A - Étude graphique}
|
||||
|
||||
On a représenté sur le graphique en annexe, la courbe représentative de la fonction $f$.
|
||||
|
||||
\begin{enumerate}
|
||||
\item Avec un tableur tracer et imprimer la courbe représentative de $f$ sur $I$
|
||||
\end{enumerate}
|
||||
\emph{Répondre aux questions suivantes par lecture graphique.}
|
||||
\begin{enumerate}
|
||||
\setcounter{enumi}{1}
|
||||
\item Résoudre graphiquement et de façon approchée l'équation $f(x) = \np{12000}$.
|
||||
\item Donner un encadrement de la quantité
|
||||
\[
|
||||
\int_{2}^{16} f(x) \; dx
|
||||
\]
|
||||
Vous expliquerez votre démarche en utilisant le graphique.
|
||||
\end{enumerate}
|
||||
|
||||
\medskip
|
||||
|
||||
\textbf{Partie B - Étude théorique}
|
||||
|
||||
\begin{enumerate}
|
||||
\setcounter{enumi}{4}
|
||||
\item Étude des variations.
|
||||
\begin{enumerate}
|
||||
\item On note $f'$ la dérivée de la fonction $f$ sur $\intFF{0}{32}$.
|
||||
|
||||
Démontrer que pour tout $x$ de $\intFF{0}{20}$ , $f'(x) = - 375x\text{e}^{-0.125x}$.
|
||||
|
||||
\item En déduire le sens de variation de $f$ et dresser son tableau des variations sur l'intervalle $\intFF{0}{32}$. Si nécessaire, arrondir à l'unité les valeurs présentes dans le tableau.
|
||||
|
||||
\item Démontrer que l'équation $f(x) = \np{12000}$ admet une unique solution $\alpha$ sur $\intFF{0}{32}$, puis donner une valeur approchée de $\alpha$ à $10^{-2}$ près à l'aide de la calculatrice.
|
||||
\end{enumerate}
|
||||
|
||||
\item Étude de la convexité
|
||||
\begin{enumerate}
|
||||
\item On note $f''$ la dérivée seconde de $f$ sur $\intFF{0}{32}$.
|
||||
|
||||
Démontrer que pour tout $x$ de $\intFF{0}{20}$ , $f''(x) = (46.875x-375)\text{e}^{-0.125x}$.
|
||||
|
||||
\item Démontrer que $f$ admet un point d'inflexion dont on donnera son abscisse.
|
||||
\end{enumerate}
|
||||
|
||||
\item Aire sous la courbe
|
||||
\begin{enumerate}
|
||||
\item On souhaite approximer la fonction $f$ sur l'intervalle $I$ par la droite $D$ qui relie les points $(0;f(0))$ et $(32, f(32))$. Tracer cette droite sur le graphique.
|
||||
\item On note $g$ la fonction affine qui décrit cette droite $D$. Détermine l'expression de $g$
|
||||
\item Calculer $\displaystyle \int_0^{32} g(x)\; dx$
|
||||
\item Avec la calculatrice, calculer une valeur approchée de $\displaystyle\int_{0}^{32} f(x) \; dx$
|
||||
\item Comparer les valeurs trouvées aux deux questions précédentes. Comment s'explique l'écart entre ces deux valeurs?
|
||||
\end{enumerate}
|
||||
\end{enumerate}
|
||||
|
||||
\medskip
|
||||
|
||||
\textbf{Partie C - Application économique}
|
||||
|
||||
Une entreprise a pris la décision de fermer son usine de production de smartphones en 32 mois.
|
||||
|
||||
La fonction capacité de production de cette usine est modélisée sur l'intervalle $\intFF{0}{32}$ par la fonction $f$ étudiée dans les parties A et B.
|
||||
|
||||
Le nombre $x$ représente le temps en mois après la décision de la fermeture du site et le nombre $f(x)$ représente capacité production de smartphone au moment $x$.
|
||||
|
||||
Utiliser les résultats de la partie B afin de répondre aux questions suivantes:
|
||||
|
||||
\begin{enumerate}
|
||||
\setcounter{enumi}{7}
|
||||
\item Combien de smartphones pouvaient être produit à la fermeture de l'usine?
|
||||
\item Pendant combien de temps la capacité de production de l'usine a réussi à se maintenir au dessus de \np{12000}?
|
||||
\item Combien de smartphones ont pu être produit entre la prise de décision et la fermeture de l'usine?
|
||||
\end{enumerate}
|
||||
|
||||
\end{exercise}
|
||||
|
||||
\begin{solution}
|
||||
\begin{enumerate}
|
||||
\item
|
||||
\begin{tikzpicture}[yscale=1, xscale=0.46875]
|
||||
\tkzInit[xmin=0,xmax=33,xstep=1,
|
||||
ymin=0,ymax=27000,ystep=2000]
|
||||
\tkzGrid
|
||||
\tkzGrid[sub, subxstep=0.2, subystep=400.0]
|
||||
\tkzAxeXY[up space=0.5,right space=.2]
|
||||
\tkzFct[domain = 0:32, line width=1pt]{3000*(x+8)*exp(-0.125*x)}
|
||||
%M\tkzFct[domain = 0:32, line width=1pt, blue]{12000}
|
||||
\end{tikzpicture}
|
||||
\item Tracer la droite $y=12000$. C'est l'abscisse de l'intersection entre cette droite et la courbe
|
||||
\item
|
||||
\item
|
||||
|
||||
\end{enumerate}
|
||||
|
||||
\end{solution}
|
||||
|
||||
\end{document}
|
||||
|
||||
%%% Local Variables:
|
||||
%%% mode: latex
|
||||
%%% TeX-master: "master"
|
||||
%%% End:
|
||||
121
TES/DM/DM_20_02/18_DM_20_02.tex
Normal file
121
TES/DM/DM_20_02/18_DM_20_02.tex
Normal file
@@ -0,0 +1,121 @@
|
||||
\documentclass[a4paper,10pt]{article}
|
||||
\usepackage{tasks}
|
||||
\usepackage{myXsim}
|
||||
|
||||
\title{DM 2 -- MOUBARIK Sarah}
|
||||
\tribe{Terminale ES-L}
|
||||
\date{9 mars 2020}
|
||||
|
||||
\xsimsetup{
|
||||
solution/print = false
|
||||
}
|
||||
|
||||
\begin{document}
|
||||
\maketitle
|
||||
|
||||
\begin{exercise}[subtitle={Étude de fonction}]
|
||||
On considère la fonction dérivable $f$ définie sur $I = [0~;~32]$ par:
|
||||
|
||||
\[f(x) = \np{2000}(x + 8)\text{e}^{- 0.125x}.\]
|
||||
|
||||
\textbf{Partie A - Étude graphique}
|
||||
|
||||
On a représenté sur le graphique en annexe, la courbe représentative de la fonction $f$.
|
||||
|
||||
\begin{enumerate}
|
||||
\item Avec un tableur tracer et imprimer la courbe représentative de $f$ sur $I$
|
||||
\end{enumerate}
|
||||
\emph{Répondre aux questions suivantes par lecture graphique.}
|
||||
\begin{enumerate}
|
||||
\setcounter{enumi}{1}
|
||||
\item Résoudre graphiquement et de façon approchée l'équation $f(x) = \np{8000}$.
|
||||
\item Donner un encadrement de la quantité
|
||||
\[
|
||||
\int_{2}^{16} f(x) \; dx
|
||||
\]
|
||||
Vous expliquerez votre démarche en utilisant le graphique.
|
||||
\end{enumerate}
|
||||
|
||||
\medskip
|
||||
|
||||
\textbf{Partie B - Étude théorique}
|
||||
|
||||
\begin{enumerate}
|
||||
\setcounter{enumi}{4}
|
||||
\item Étude des variations.
|
||||
\begin{enumerate}
|
||||
\item On note $f'$ la dérivée de la fonction $f$ sur $\intFF{0}{32}$.
|
||||
|
||||
Démontrer que pour tout $x$ de $\intFF{0}{20}$ , $f'(x) = - 250x\text{e}^{-0.125x}$.
|
||||
|
||||
\item En déduire le sens de variation de $f$ et dresser son tableau des variations sur l'intervalle $\intFF{0}{32}$. Si nécessaire, arrondir à l'unité les valeurs présentes dans le tableau.
|
||||
|
||||
\item Démontrer que l'équation $f(x) = \np{8000}$ admet une unique solution $\alpha$ sur $\intFF{0}{32}$, puis donner une valeur approchée de $\alpha$ à $10^{-2}$ près à l'aide de la calculatrice.
|
||||
\end{enumerate}
|
||||
|
||||
\item Étude de la convexité
|
||||
\begin{enumerate}
|
||||
\item On note $f''$ la dérivée seconde de $f$ sur $\intFF{0}{32}$.
|
||||
|
||||
Démontrer que pour tout $x$ de $\intFF{0}{20}$ , $f''(x) = (31.250x-250)\text{e}^{-0.125x}$.
|
||||
|
||||
\item Démontrer que $f$ admet un point d'inflexion dont on donnera son abscisse.
|
||||
\end{enumerate}
|
||||
|
||||
\item Aire sous la courbe
|
||||
\begin{enumerate}
|
||||
\item On souhaite approximer la fonction $f$ sur l'intervalle $I$ par la droite $D$ qui relie les points $(0;f(0))$ et $(32, f(32))$. Tracer cette droite sur le graphique.
|
||||
\item On note $g$ la fonction affine qui décrit cette droite $D$. Détermine l'expression de $g$
|
||||
\item Calculer $\displaystyle \int_0^{32} g(x)\; dx$
|
||||
\item Avec la calculatrice, calculer une valeur approchée de $\displaystyle\int_{0}^{32} f(x) \; dx$
|
||||
\item Comparer les valeurs trouvées aux deux questions précédentes. Comment s'explique l'écart entre ces deux valeurs?
|
||||
\end{enumerate}
|
||||
\end{enumerate}
|
||||
|
||||
\medskip
|
||||
|
||||
\textbf{Partie C - Application économique}
|
||||
|
||||
Une entreprise a pris la décision de fermer son usine de production de smartphones en 32 mois.
|
||||
|
||||
La fonction capacité de production de cette usine est modélisée sur l'intervalle $\intFF{0}{32}$ par la fonction $f$ étudiée dans les parties A et B.
|
||||
|
||||
Le nombre $x$ représente le temps en mois après la décision de la fermeture du site et le nombre $f(x)$ représente capacité production de smartphone au moment $x$.
|
||||
|
||||
Utiliser les résultats de la partie B afin de répondre aux questions suivantes:
|
||||
|
||||
\begin{enumerate}
|
||||
\setcounter{enumi}{7}
|
||||
\item Combien de smartphones pouvaient être produit à la fermeture de l'usine?
|
||||
\item Pendant combien de temps la capacité de production de l'usine a réussi à se maintenir au dessus de \np{8000}?
|
||||
\item Combien de smartphones ont pu être produit entre la prise de décision et la fermeture de l'usine?
|
||||
\end{enumerate}
|
||||
|
||||
\end{exercise}
|
||||
|
||||
\begin{solution}
|
||||
\begin{enumerate}
|
||||
\item
|
||||
\begin{tikzpicture}[yscale=1, xscale=0.46875]
|
||||
\tkzInit[xmin=0,xmax=33,xstep=1,
|
||||
ymin=0,ymax=18000,ystep=1000]
|
||||
\tkzGrid
|
||||
\tkzGrid[sub, subxstep=0.2, subystep=200.0]
|
||||
\tkzAxeXY[up space=0.5,right space=.2]
|
||||
\tkzFct[domain = 0:32, line width=1pt]{2000*(x+8)*exp(-0.125*x)}
|
||||
%M\tkzFct[domain = 0:32, line width=1pt, blue]{8000}
|
||||
\end{tikzpicture}
|
||||
\item Tracer la droite $y=8000$. C'est l'abscisse de l'intersection entre cette droite et la courbe
|
||||
\item
|
||||
\item
|
||||
|
||||
\end{enumerate}
|
||||
|
||||
\end{solution}
|
||||
|
||||
\end{document}
|
||||
|
||||
%%% Local Variables:
|
||||
%%% mode: latex
|
||||
%%% TeX-master: "master"
|
||||
%%% End:
|
||||
121
TES/DM/DM_20_02/19_DM_20_02.tex
Normal file
121
TES/DM/DM_20_02/19_DM_20_02.tex
Normal file
@@ -0,0 +1,121 @@
|
||||
\documentclass[a4paper,10pt]{article}
|
||||
\usepackage{tasks}
|
||||
\usepackage{myXsim}
|
||||
|
||||
\title{DM 2 -- PERREARD Noémie}
|
||||
\tribe{Terminale ES-L}
|
||||
\date{9 mars 2020}
|
||||
|
||||
\xsimsetup{
|
||||
solution/print = false
|
||||
}
|
||||
|
||||
\begin{document}
|
||||
\maketitle
|
||||
|
||||
\begin{exercise}[subtitle={Étude de fonction}]
|
||||
On considère la fonction dérivable $f$ définie sur $I = [0~;~32]$ par:
|
||||
|
||||
\[f(x) = \np{10000}(x + 8)\text{e}^{- 0.125x}.\]
|
||||
|
||||
\textbf{Partie A - Étude graphique}
|
||||
|
||||
On a représenté sur le graphique en annexe, la courbe représentative de la fonction $f$.
|
||||
|
||||
\begin{enumerate}
|
||||
\item Avec un tableur tracer et imprimer la courbe représentative de $f$ sur $I$
|
||||
\end{enumerate}
|
||||
\emph{Répondre aux questions suivantes par lecture graphique.}
|
||||
\begin{enumerate}
|
||||
\setcounter{enumi}{1}
|
||||
\item Résoudre graphiquement et de façon approchée l'équation $f(x) = \np{40000}$.
|
||||
\item Donner un encadrement de la quantité
|
||||
\[
|
||||
\int_{2}^{16} f(x) \; dx
|
||||
\]
|
||||
Vous expliquerez votre démarche en utilisant le graphique.
|
||||
\end{enumerate}
|
||||
|
||||
\medskip
|
||||
|
||||
\textbf{Partie B - Étude théorique}
|
||||
|
||||
\begin{enumerate}
|
||||
\setcounter{enumi}{4}
|
||||
\item Étude des variations.
|
||||
\begin{enumerate}
|
||||
\item On note $f'$ la dérivée de la fonction $f$ sur $\intFF{0}{32}$.
|
||||
|
||||
Démontrer que pour tout $x$ de $\intFF{0}{20}$ , $f'(x) = - 1250x\text{e}^{-0.125x}$.
|
||||
|
||||
\item En déduire le sens de variation de $f$ et dresser son tableau des variations sur l'intervalle $\intFF{0}{32}$. Si nécessaire, arrondir à l'unité les valeurs présentes dans le tableau.
|
||||
|
||||
\item Démontrer que l'équation $f(x) = \np{40000}$ admet une unique solution $\alpha$ sur $\intFF{0}{32}$, puis donner une valeur approchée de $\alpha$ à $10^{-2}$ près à l'aide de la calculatrice.
|
||||
\end{enumerate}
|
||||
|
||||
\item Étude de la convexité
|
||||
\begin{enumerate}
|
||||
\item On note $f''$ la dérivée seconde de $f$ sur $\intFF{0}{32}$.
|
||||
|
||||
Démontrer que pour tout $x$ de $\intFF{0}{20}$ , $f''(x) = (156.250x-1250)\text{e}^{-0.125x}$.
|
||||
|
||||
\item Démontrer que $f$ admet un point d'inflexion dont on donnera son abscisse.
|
||||
\end{enumerate}
|
||||
|
||||
\item Aire sous la courbe
|
||||
\begin{enumerate}
|
||||
\item On souhaite approximer la fonction $f$ sur l'intervalle $I$ par la droite $D$ qui relie les points $(0;f(0))$ et $(32, f(32))$. Tracer cette droite sur le graphique.
|
||||
\item On note $g$ la fonction affine qui décrit cette droite $D$. Détermine l'expression de $g$
|
||||
\item Calculer $\displaystyle \int_0^{32} g(x)\; dx$
|
||||
\item Avec la calculatrice, calculer une valeur approchée de $\displaystyle\int_{0}^{32} f(x) \; dx$
|
||||
\item Comparer les valeurs trouvées aux deux questions précédentes. Comment s'explique l'écart entre ces deux valeurs?
|
||||
\end{enumerate}
|
||||
\end{enumerate}
|
||||
|
||||
\medskip
|
||||
|
||||
\textbf{Partie C - Application économique}
|
||||
|
||||
Une entreprise a pris la décision de fermer son usine de production de smartphones en 32 mois.
|
||||
|
||||
La fonction capacité de production de cette usine est modélisée sur l'intervalle $\intFF{0}{32}$ par la fonction $f$ étudiée dans les parties A et B.
|
||||
|
||||
Le nombre $x$ représente le temps en mois après la décision de la fermeture du site et le nombre $f(x)$ représente capacité production de smartphone au moment $x$.
|
||||
|
||||
Utiliser les résultats de la partie B afin de répondre aux questions suivantes:
|
||||
|
||||
\begin{enumerate}
|
||||
\setcounter{enumi}{7}
|
||||
\item Combien de smartphones pouvaient être produit à la fermeture de l'usine?
|
||||
\item Pendant combien de temps la capacité de production de l'usine a réussi à se maintenir au dessus de \np{40000}?
|
||||
\item Combien de smartphones ont pu être produit entre la prise de décision et la fermeture de l'usine?
|
||||
\end{enumerate}
|
||||
|
||||
\end{exercise}
|
||||
|
||||
\begin{solution}
|
||||
\begin{enumerate}
|
||||
\item
|
||||
\begin{tikzpicture}[yscale=1, xscale=0.46875]
|
||||
\tkzInit[xmin=0,xmax=33,xstep=1,
|
||||
ymin=0,ymax=90000,ystep=9000]
|
||||
\tkzGrid
|
||||
\tkzGrid[sub, subxstep=0.2, subystep=1800.0]
|
||||
\tkzAxeXY[up space=0.5,right space=.2]
|
||||
\tkzFct[domain = 0:32, line width=1pt]{10000*(x+8)*exp(-0.125*x)}
|
||||
%M\tkzFct[domain = 0:32, line width=1pt, blue]{40000}
|
||||
\end{tikzpicture}
|
||||
\item Tracer la droite $y=40000$. C'est l'abscisse de l'intersection entre cette droite et la courbe
|
||||
\item
|
||||
\item
|
||||
|
||||
\end{enumerate}
|
||||
|
||||
\end{solution}
|
||||
|
||||
\end{document}
|
||||
|
||||
%%% Local Variables:
|
||||
%%% mode: latex
|
||||
%%% TeX-master: "master"
|
||||
%%% End:
|
||||
121
TES/DM/DM_20_02/20_DM_20_02.tex
Normal file
121
TES/DM/DM_20_02/20_DM_20_02.tex
Normal file
@@ -0,0 +1,121 @@
|
||||
\documentclass[a4paper,10pt]{article}
|
||||
\usepackage{tasks}
|
||||
\usepackage{myXsim}
|
||||
|
||||
\title{DM 2 -- URPIN Flora}
|
||||
\tribe{Terminale ES-L}
|
||||
\date{9 mars 2020}
|
||||
|
||||
\xsimsetup{
|
||||
solution/print = false
|
||||
}
|
||||
|
||||
\begin{document}
|
||||
\maketitle
|
||||
|
||||
\begin{exercise}[subtitle={Étude de fonction}]
|
||||
On considère la fonction dérivable $f$ définie sur $I = [0~;~16]$ par:
|
||||
|
||||
\[f(x) = \np{3000}(x + 4)\text{e}^{- 0.25x}.\]
|
||||
|
||||
\textbf{Partie A - Étude graphique}
|
||||
|
||||
On a représenté sur le graphique en annexe, la courbe représentative de la fonction $f$.
|
||||
|
||||
\begin{enumerate}
|
||||
\item Avec un tableur tracer et imprimer la courbe représentative de $f$ sur $I$
|
||||
\end{enumerate}
|
||||
\emph{Répondre aux questions suivantes par lecture graphique.}
|
||||
\begin{enumerate}
|
||||
\setcounter{enumi}{1}
|
||||
\item Résoudre graphiquement et de façon approchée l'équation $f(x) = \np{6000}$.
|
||||
\item Donner un encadrement de la quantité
|
||||
\[
|
||||
\int_{2}^{8} f(x) \; dx
|
||||
\]
|
||||
Vous expliquerez votre démarche en utilisant le graphique.
|
||||
\end{enumerate}
|
||||
|
||||
\medskip
|
||||
|
||||
\textbf{Partie B - Étude théorique}
|
||||
|
||||
\begin{enumerate}
|
||||
\setcounter{enumi}{4}
|
||||
\item Étude des variations.
|
||||
\begin{enumerate}
|
||||
\item On note $f'$ la dérivée de la fonction $f$ sur $\intFF{0}{16}$.
|
||||
|
||||
Démontrer que pour tout $x$ de $\intFF{0}{20}$ , $f'(x) = - 750x\text{e}^{-0.25x}$.
|
||||
|
||||
\item En déduire le sens de variation de $f$ et dresser son tableau des variations sur l'intervalle $\intFF{0}{16}$. Si nécessaire, arrondir à l'unité les valeurs présentes dans le tableau.
|
||||
|
||||
\item Démontrer que l'équation $f(x) = \np{6000}$ admet une unique solution $\alpha$ sur $\intFF{0}{16}$, puis donner une valeur approchée de $\alpha$ à $10^{-2}$ près à l'aide de la calculatrice.
|
||||
\end{enumerate}
|
||||
|
||||
\item Étude de la convexité
|
||||
\begin{enumerate}
|
||||
\item On note $f''$ la dérivée seconde de $f$ sur $\intFF{0}{16}$.
|
||||
|
||||
Démontrer que pour tout $x$ de $\intFF{0}{20}$ , $f''(x) = (187.50x-750)\text{e}^{-0.25x}$.
|
||||
|
||||
\item Démontrer que $f$ admet un point d'inflexion dont on donnera son abscisse.
|
||||
\end{enumerate}
|
||||
|
||||
\item Aire sous la courbe
|
||||
\begin{enumerate}
|
||||
\item On souhaite approximer la fonction $f$ sur l'intervalle $I$ par la droite $D$ qui relie les points $(0;f(0))$ et $(16, f(16))$. Tracer cette droite sur le graphique.
|
||||
\item On note $g$ la fonction affine qui décrit cette droite $D$. Détermine l'expression de $g$
|
||||
\item Calculer $\displaystyle \int_0^{16} g(x)\; dx$
|
||||
\item Avec la calculatrice, calculer une valeur approchée de $\displaystyle\int_{0}^{16} f(x) \; dx$
|
||||
\item Comparer les valeurs trouvées aux deux questions précédentes. Comment s'explique l'écart entre ces deux valeurs?
|
||||
\end{enumerate}
|
||||
\end{enumerate}
|
||||
|
||||
\medskip
|
||||
|
||||
\textbf{Partie C - Application économique}
|
||||
|
||||
Une entreprise a pris la décision de fermer son usine de production de smartphones en 16 mois.
|
||||
|
||||
La fonction capacité de production de cette usine est modélisée sur l'intervalle $\intFF{0}{16}$ par la fonction $f$ étudiée dans les parties A et B.
|
||||
|
||||
Le nombre $x$ représente le temps en mois après la décision de la fermeture du site et le nombre $f(x)$ représente capacité production de smartphone au moment $x$.
|
||||
|
||||
Utiliser les résultats de la partie B afin de répondre aux questions suivantes:
|
||||
|
||||
\begin{enumerate}
|
||||
\setcounter{enumi}{7}
|
||||
\item Combien de smartphones pouvaient être produit à la fermeture de l'usine?
|
||||
\item Pendant combien de temps la capacité de production de l'usine a réussi à se maintenir au dessus de \np{6000}?
|
||||
\item Combien de smartphones ont pu être produit entre la prise de décision et la fermeture de l'usine?
|
||||
\end{enumerate}
|
||||
|
||||
\end{exercise}
|
||||
|
||||
\begin{solution}
|
||||
\begin{enumerate}
|
||||
\item
|
||||
\begin{tikzpicture}[yscale=1, xscale=0.9375]
|
||||
\tkzInit[xmin=0,xmax=17,xstep=1,
|
||||
ymin=0,ymax=15000,ystep=1000]
|
||||
\tkzGrid
|
||||
\tkzGrid[sub, subxstep=0.2, subystep=200.0]
|
||||
\tkzAxeXY[up space=0.5,right space=.2]
|
||||
\tkzFct[domain = 0:16, line width=1pt]{3000*(x+4)*exp(-0.25*x)}
|
||||
%M\tkzFct[domain = 0:16, line width=1pt, blue]{6000}
|
||||
\end{tikzpicture}
|
||||
\item Tracer la droite $y=6000$. C'est l'abscisse de l'intersection entre cette droite et la courbe
|
||||
\item
|
||||
\item
|
||||
|
||||
\end{enumerate}
|
||||
|
||||
\end{solution}
|
||||
|
||||
\end{document}
|
||||
|
||||
%%% Local Variables:
|
||||
%%% mode: latex
|
||||
%%% TeX-master: "master"
|
||||
%%% End:
|
||||
121
TES/DM/DM_20_02/21_DM_20_02.tex
Normal file
121
TES/DM/DM_20_02/21_DM_20_02.tex
Normal file
@@ -0,0 +1,121 @@
|
||||
\documentclass[a4paper,10pt]{article}
|
||||
\usepackage{tasks}
|
||||
\usepackage{myXsim}
|
||||
|
||||
\title{DM 2 -- VISENTIN Aurélie}
|
||||
\tribe{Terminale ES-L}
|
||||
\date{9 mars 2020}
|
||||
|
||||
\xsimsetup{
|
||||
solution/print = false
|
||||
}
|
||||
|
||||
\begin{document}
|
||||
\maketitle
|
||||
|
||||
\begin{exercise}[subtitle={Étude de fonction}]
|
||||
On considère la fonction dérivable $f$ définie sur $I = [0~;~32]$ par:
|
||||
|
||||
\[f(x) = \np{10000}(x + 8)\text{e}^{- 0.125x}.\]
|
||||
|
||||
\textbf{Partie A - Étude graphique}
|
||||
|
||||
On a représenté sur le graphique en annexe, la courbe représentative de la fonction $f$.
|
||||
|
||||
\begin{enumerate}
|
||||
\item Avec un tableur tracer et imprimer la courbe représentative de $f$ sur $I$
|
||||
\end{enumerate}
|
||||
\emph{Répondre aux questions suivantes par lecture graphique.}
|
||||
\begin{enumerate}
|
||||
\setcounter{enumi}{1}
|
||||
\item Résoudre graphiquement et de façon approchée l'équation $f(x) = \np{40000}$.
|
||||
\item Donner un encadrement de la quantité
|
||||
\[
|
||||
\int_{2}^{16} f(x) \; dx
|
||||
\]
|
||||
Vous expliquerez votre démarche en utilisant le graphique.
|
||||
\end{enumerate}
|
||||
|
||||
\medskip
|
||||
|
||||
\textbf{Partie B - Étude théorique}
|
||||
|
||||
\begin{enumerate}
|
||||
\setcounter{enumi}{4}
|
||||
\item Étude des variations.
|
||||
\begin{enumerate}
|
||||
\item On note $f'$ la dérivée de la fonction $f$ sur $\intFF{0}{32}$.
|
||||
|
||||
Démontrer que pour tout $x$ de $\intFF{0}{20}$ , $f'(x) = - 1250x\text{e}^{-0.125x}$.
|
||||
|
||||
\item En déduire le sens de variation de $f$ et dresser son tableau des variations sur l'intervalle $\intFF{0}{32}$. Si nécessaire, arrondir à l'unité les valeurs présentes dans le tableau.
|
||||
|
||||
\item Démontrer que l'équation $f(x) = \np{40000}$ admet une unique solution $\alpha$ sur $\intFF{0}{32}$, puis donner une valeur approchée de $\alpha$ à $10^{-2}$ près à l'aide de la calculatrice.
|
||||
\end{enumerate}
|
||||
|
||||
\item Étude de la convexité
|
||||
\begin{enumerate}
|
||||
\item On note $f''$ la dérivée seconde de $f$ sur $\intFF{0}{32}$.
|
||||
|
||||
Démontrer que pour tout $x$ de $\intFF{0}{20}$ , $f''(x) = (156.250x-1250)\text{e}^{-0.125x}$.
|
||||
|
||||
\item Démontrer que $f$ admet un point d'inflexion dont on donnera son abscisse.
|
||||
\end{enumerate}
|
||||
|
||||
\item Aire sous la courbe
|
||||
\begin{enumerate}
|
||||
\item On souhaite approximer la fonction $f$ sur l'intervalle $I$ par la droite $D$ qui relie les points $(0;f(0))$ et $(32, f(32))$. Tracer cette droite sur le graphique.
|
||||
\item On note $g$ la fonction affine qui décrit cette droite $D$. Détermine l'expression de $g$
|
||||
\item Calculer $\displaystyle \int_0^{32} g(x)\; dx$
|
||||
\item Avec la calculatrice, calculer une valeur approchée de $\displaystyle\int_{0}^{32} f(x) \; dx$
|
||||
\item Comparer les valeurs trouvées aux deux questions précédentes. Comment s'explique l'écart entre ces deux valeurs?
|
||||
\end{enumerate}
|
||||
\end{enumerate}
|
||||
|
||||
\medskip
|
||||
|
||||
\textbf{Partie C - Application économique}
|
||||
|
||||
Une entreprise a pris la décision de fermer son usine de production de smartphones en 32 mois.
|
||||
|
||||
La fonction capacité de production de cette usine est modélisée sur l'intervalle $\intFF{0}{32}$ par la fonction $f$ étudiée dans les parties A et B.
|
||||
|
||||
Le nombre $x$ représente le temps en mois après la décision de la fermeture du site et le nombre $f(x)$ représente capacité production de smartphone au moment $x$.
|
||||
|
||||
Utiliser les résultats de la partie B afin de répondre aux questions suivantes:
|
||||
|
||||
\begin{enumerate}
|
||||
\setcounter{enumi}{7}
|
||||
\item Combien de smartphones pouvaient être produit à la fermeture de l'usine?
|
||||
\item Pendant combien de temps la capacité de production de l'usine a réussi à se maintenir au dessus de \np{40000}?
|
||||
\item Combien de smartphones ont pu être produit entre la prise de décision et la fermeture de l'usine?
|
||||
\end{enumerate}
|
||||
|
||||
\end{exercise}
|
||||
|
||||
\begin{solution}
|
||||
\begin{enumerate}
|
||||
\item
|
||||
\begin{tikzpicture}[yscale=1, xscale=0.46875]
|
||||
\tkzInit[xmin=0,xmax=33,xstep=1,
|
||||
ymin=0,ymax=90000,ystep=9000]
|
||||
\tkzGrid
|
||||
\tkzGrid[sub, subxstep=0.2, subystep=1800.0]
|
||||
\tkzAxeXY[up space=0.5,right space=.2]
|
||||
\tkzFct[domain = 0:32, line width=1pt]{10000*(x+8)*exp(-0.125*x)}
|
||||
%M\tkzFct[domain = 0:32, line width=1pt, blue]{40000}
|
||||
\end{tikzpicture}
|
||||
\item Tracer la droite $y=40000$. C'est l'abscisse de l'intersection entre cette droite et la courbe
|
||||
\item
|
||||
\item
|
||||
|
||||
\end{enumerate}
|
||||
|
||||
\end{solution}
|
||||
|
||||
\end{document}
|
||||
|
||||
%%% Local Variables:
|
||||
%%% mode: latex
|
||||
%%% TeX-master: "master"
|
||||
%%% End:
|
||||
BIN
TES/DM/DM_20_02/all_DM_20_02.pdf
Normal file
BIN
TES/DM/DM_20_02/all_DM_20_02.pdf
Normal file
Binary file not shown.
121
TES/DM/DM_20_02/corr_01_DM_20_02.tex
Normal file
121
TES/DM/DM_20_02/corr_01_DM_20_02.tex
Normal file
@@ -0,0 +1,121 @@
|
||||
\documentclass[a4paper,10pt]{article}
|
||||
\usepackage{tasks}
|
||||
\usepackage{myXsim}
|
||||
|
||||
\title{DM 2 -- AIT BEN SAID Loubna}
|
||||
\tribe{Terminale ES-L}
|
||||
\date{9 mars 2020}
|
||||
|
||||
\xsimsetup{
|
||||
solution/print = true
|
||||
}
|
||||
|
||||
\begin{document}
|
||||
\maketitle
|
||||
|
||||
\begin{exercise}[subtitle={Étude de fonction}]
|
||||
On considère la fonction dérivable $f$ définie sur $I = [0~;~64]$ par:
|
||||
|
||||
\[f(x) = \np{3000}(x + 16)\text{e}^{- 0.0625x}.\]
|
||||
|
||||
\textbf{Partie A - Étude graphique}
|
||||
|
||||
On a représenté sur le graphique en annexe, la courbe représentative de la fonction $f$.
|
||||
|
||||
\begin{enumerate}
|
||||
\item Avec un tableur tracer et imprimer la courbe représentative de $f$ sur $I$
|
||||
\end{enumerate}
|
||||
\emph{Répondre aux questions suivantes par lecture graphique.}
|
||||
\begin{enumerate}
|
||||
\setcounter{enumi}{1}
|
||||
\item Résoudre graphiquement et de façon approchée l'équation $f(x) = \np{24000}$.
|
||||
\item Donner un encadrement de la quantité
|
||||
\[
|
||||
\int_{2}^{32} f(x) \; dx
|
||||
\]
|
||||
Vous expliquerez votre démarche en utilisant le graphique.
|
||||
\end{enumerate}
|
||||
|
||||
\medskip
|
||||
|
||||
\textbf{Partie B - Étude théorique}
|
||||
|
||||
\begin{enumerate}
|
||||
\setcounter{enumi}{4}
|
||||
\item Étude des variations.
|
||||
\begin{enumerate}
|
||||
\item On note $f'$ la dérivée de la fonction $f$ sur $\intFF{0}{64}$.
|
||||
|
||||
Démontrer que pour tout $x$ de $\intFF{0}{20}$ , $f'(x) = - 187.5000x\text{e}^{-0.0625x}$.
|
||||
|
||||
\item En déduire le sens de variation de $f$ et dresser son tableau des variations sur l'intervalle $\intFF{0}{64}$. Si nécessaire, arrondir à l'unité les valeurs présentes dans le tableau.
|
||||
|
||||
\item Démontrer que l'équation $f(x) = \np{24000}$ admet une unique solution $\alpha$ sur $\intFF{0}{64}$, puis donner une valeur approchée de $\alpha$ à $10^{-2}$ près à l'aide de la calculatrice.
|
||||
\end{enumerate}
|
||||
|
||||
\item Étude de la convexité
|
||||
\begin{enumerate}
|
||||
\item On note $f''$ la dérivée seconde de $f$ sur $\intFF{0}{64}$.
|
||||
|
||||
Démontrer que pour tout $x$ de $\intFF{0}{20}$ , $f''(x) = (11.71875000x-187.5000)\text{e}^{-0.0625x}$.
|
||||
|
||||
\item Démontrer que $f$ admet un point d'inflexion dont on donnera son abscisse.
|
||||
\end{enumerate}
|
||||
|
||||
\item Aire sous la courbe
|
||||
\begin{enumerate}
|
||||
\item On souhaite approximer la fonction $f$ sur l'intervalle $I$ par la droite $D$ qui relie les points $(0;f(0))$ et $(64, f(64))$. Tracer cette droite sur le graphique.
|
||||
\item On note $g$ la fonction affine qui décrit cette droite $D$. Détermine l'expression de $g$
|
||||
\item Calculer $\displaystyle \int_0^{64} g(x)\; dx$
|
||||
\item Avec la calculatrice, calculer une valeur approchée de $\displaystyle\int_{0}^{64} f(x) \; dx$
|
||||
\item Comparer les valeurs trouvées aux deux questions précédentes. Comment s'explique l'écart entre ces deux valeurs?
|
||||
\end{enumerate}
|
||||
\end{enumerate}
|
||||
|
||||
\medskip
|
||||
|
||||
\textbf{Partie C - Application économique}
|
||||
|
||||
Une entreprise a pris la décision de fermer son usine de production de smartphones en 64 mois.
|
||||
|
||||
La fonction capacité de production de cette usine est modélisée sur l'intervalle $\intFF{0}{64}$ par la fonction $f$ étudiée dans les parties A et B.
|
||||
|
||||
Le nombre $x$ représente le temps en mois après la décision de la fermeture du site et le nombre $f(x)$ représente capacité production de smartphone au moment $x$.
|
||||
|
||||
Utiliser les résultats de la partie B afin de répondre aux questions suivantes:
|
||||
|
||||
\begin{enumerate}
|
||||
\setcounter{enumi}{7}
|
||||
\item Combien de smartphones pouvaient être produit à la fermeture de l'usine?
|
||||
\item Pendant combien de temps la capacité de production de l'usine a réussi à se maintenir au dessus de \np{24000}?
|
||||
\item Combien de smartphones ont pu être produit entre la prise de décision et la fermeture de l'usine?
|
||||
\end{enumerate}
|
||||
|
||||
\end{exercise}
|
||||
|
||||
\begin{solution}
|
||||
\begin{enumerate}
|
||||
\item
|
||||
\begin{tikzpicture}[yscale=1, xscale=0.234375]
|
||||
\tkzInit[xmin=0,xmax=65,xstep=1,
|
||||
ymin=0,ymax=51000,ystep=5000]
|
||||
\tkzGrid
|
||||
\tkzGrid[sub, subxstep=0.2, subystep=1000.0]
|
||||
\tkzAxeXY[up space=0.5,right space=.2]
|
||||
\tkzFct[domain = 0:64, line width=1pt]{3000*(x+16)*exp(-0.0625*x)}
|
||||
%M\tkzFct[domain = 0:64, line width=1pt, blue]{24000}
|
||||
\end{tikzpicture}
|
||||
\item Tracer la droite $y=24000$. C'est l'abscisse de l'intersection entre cette droite et la courbe
|
||||
\item
|
||||
\item
|
||||
|
||||
\end{enumerate}
|
||||
|
||||
\end{solution}
|
||||
|
||||
\end{document}
|
||||
|
||||
%%% Local Variables:
|
||||
%%% mode: latex
|
||||
%%% TeX-master: "master"
|
||||
%%% End:
|
||||
121
TES/DM/DM_20_02/corr_02_DM_20_02.tex
Normal file
121
TES/DM/DM_20_02/corr_02_DM_20_02.tex
Normal file
@@ -0,0 +1,121 @@
|
||||
\documentclass[a4paper,10pt]{article}
|
||||
\usepackage{tasks}
|
||||
\usepackage{myXsim}
|
||||
|
||||
\title{DM 2 -- BATEMAN Amélie}
|
||||
\tribe{Terminale ES-L}
|
||||
\date{9 mars 2020}
|
||||
|
||||
\xsimsetup{
|
||||
solution/print = true
|
||||
}
|
||||
|
||||
\begin{document}
|
||||
\maketitle
|
||||
|
||||
\begin{exercise}[subtitle={Étude de fonction}]
|
||||
On considère la fonction dérivable $f$ définie sur $I = [0~;~32]$ par:
|
||||
|
||||
\[f(x) = \np{10000}(x + 8)\text{e}^{- 0.125x}.\]
|
||||
|
||||
\textbf{Partie A - Étude graphique}
|
||||
|
||||
On a représenté sur le graphique en annexe, la courbe représentative de la fonction $f$.
|
||||
|
||||
\begin{enumerate}
|
||||
\item Avec un tableur tracer et imprimer la courbe représentative de $f$ sur $I$
|
||||
\end{enumerate}
|
||||
\emph{Répondre aux questions suivantes par lecture graphique.}
|
||||
\begin{enumerate}
|
||||
\setcounter{enumi}{1}
|
||||
\item Résoudre graphiquement et de façon approchée l'équation $f(x) = \np{40000}$.
|
||||
\item Donner un encadrement de la quantité
|
||||
\[
|
||||
\int_{2}^{16} f(x) \; dx
|
||||
\]
|
||||
Vous expliquerez votre démarche en utilisant le graphique.
|
||||
\end{enumerate}
|
||||
|
||||
\medskip
|
||||
|
||||
\textbf{Partie B - Étude théorique}
|
||||
|
||||
\begin{enumerate}
|
||||
\setcounter{enumi}{4}
|
||||
\item Étude des variations.
|
||||
\begin{enumerate}
|
||||
\item On note $f'$ la dérivée de la fonction $f$ sur $\intFF{0}{32}$.
|
||||
|
||||
Démontrer que pour tout $x$ de $\intFF{0}{20}$ , $f'(x) = - 1250x\text{e}^{-0.125x}$.
|
||||
|
||||
\item En déduire le sens de variation de $f$ et dresser son tableau des variations sur l'intervalle $\intFF{0}{32}$. Si nécessaire, arrondir à l'unité les valeurs présentes dans le tableau.
|
||||
|
||||
\item Démontrer que l'équation $f(x) = \np{40000}$ admet une unique solution $\alpha$ sur $\intFF{0}{32}$, puis donner une valeur approchée de $\alpha$ à $10^{-2}$ près à l'aide de la calculatrice.
|
||||
\end{enumerate}
|
||||
|
||||
\item Étude de la convexité
|
||||
\begin{enumerate}
|
||||
\item On note $f''$ la dérivée seconde de $f$ sur $\intFF{0}{32}$.
|
||||
|
||||
Démontrer que pour tout $x$ de $\intFF{0}{20}$ , $f''(x) = (156.250x-1250)\text{e}^{-0.125x}$.
|
||||
|
||||
\item Démontrer que $f$ admet un point d'inflexion dont on donnera son abscisse.
|
||||
\end{enumerate}
|
||||
|
||||
\item Aire sous la courbe
|
||||
\begin{enumerate}
|
||||
\item On souhaite approximer la fonction $f$ sur l'intervalle $I$ par la droite $D$ qui relie les points $(0;f(0))$ et $(32, f(32))$. Tracer cette droite sur le graphique.
|
||||
\item On note $g$ la fonction affine qui décrit cette droite $D$. Détermine l'expression de $g$
|
||||
\item Calculer $\displaystyle \int_0^{32} g(x)\; dx$
|
||||
\item Avec la calculatrice, calculer une valeur approchée de $\displaystyle\int_{0}^{32} f(x) \; dx$
|
||||
\item Comparer les valeurs trouvées aux deux questions précédentes. Comment s'explique l'écart entre ces deux valeurs?
|
||||
\end{enumerate}
|
||||
\end{enumerate}
|
||||
|
||||
\medskip
|
||||
|
||||
\textbf{Partie C - Application économique}
|
||||
|
||||
Une entreprise a pris la décision de fermer son usine de production de smartphones en 32 mois.
|
||||
|
||||
La fonction capacité de production de cette usine est modélisée sur l'intervalle $\intFF{0}{32}$ par la fonction $f$ étudiée dans les parties A et B.
|
||||
|
||||
Le nombre $x$ représente le temps en mois après la décision de la fermeture du site et le nombre $f(x)$ représente capacité production de smartphone au moment $x$.
|
||||
|
||||
Utiliser les résultats de la partie B afin de répondre aux questions suivantes:
|
||||
|
||||
\begin{enumerate}
|
||||
\setcounter{enumi}{7}
|
||||
\item Combien de smartphones pouvaient être produit à la fermeture de l'usine?
|
||||
\item Pendant combien de temps la capacité de production de l'usine a réussi à se maintenir au dessus de \np{40000}?
|
||||
\item Combien de smartphones ont pu être produit entre la prise de décision et la fermeture de l'usine?
|
||||
\end{enumerate}
|
||||
|
||||
\end{exercise}
|
||||
|
||||
\begin{solution}
|
||||
\begin{enumerate}
|
||||
\item
|
||||
\begin{tikzpicture}[yscale=1, xscale=0.46875]
|
||||
\tkzInit[xmin=0,xmax=33,xstep=1,
|
||||
ymin=0,ymax=90000,ystep=9000]
|
||||
\tkzGrid
|
||||
\tkzGrid[sub, subxstep=0.2, subystep=1800.0]
|
||||
\tkzAxeXY[up space=0.5,right space=.2]
|
||||
\tkzFct[domain = 0:32, line width=1pt]{10000*(x+8)*exp(-0.125*x)}
|
||||
%M\tkzFct[domain = 0:32, line width=1pt, blue]{40000}
|
||||
\end{tikzpicture}
|
||||
\item Tracer la droite $y=40000$. C'est l'abscisse de l'intersection entre cette droite et la courbe
|
||||
\item
|
||||
\item
|
||||
|
||||
\end{enumerate}
|
||||
|
||||
\end{solution}
|
||||
|
||||
\end{document}
|
||||
|
||||
%%% Local Variables:
|
||||
%%% mode: latex
|
||||
%%% TeX-master: "master"
|
||||
%%% End:
|
||||
121
TES/DM/DM_20_02/corr_03_DM_20_02.tex
Normal file
121
TES/DM/DM_20_02/corr_03_DM_20_02.tex
Normal file
@@ -0,0 +1,121 @@
|
||||
\documentclass[a4paper,10pt]{article}
|
||||
\usepackage{tasks}
|
||||
\usepackage{myXsim}
|
||||
|
||||
\title{DM 2 -- BOUNOUS Matthieu}
|
||||
\tribe{Terminale ES-L}
|
||||
\date{9 mars 2020}
|
||||
|
||||
\xsimsetup{
|
||||
solution/print = true
|
||||
}
|
||||
|
||||
\begin{document}
|
||||
\maketitle
|
||||
|
||||
\begin{exercise}[subtitle={Étude de fonction}]
|
||||
On considère la fonction dérivable $f$ définie sur $I = [0~;~64]$ par:
|
||||
|
||||
\[f(x) = \np{10000}(x + 16)\text{e}^{- 0.0625x}.\]
|
||||
|
||||
\textbf{Partie A - Étude graphique}
|
||||
|
||||
On a représenté sur le graphique en annexe, la courbe représentative de la fonction $f$.
|
||||
|
||||
\begin{enumerate}
|
||||
\item Avec un tableur tracer et imprimer la courbe représentative de $f$ sur $I$
|
||||
\end{enumerate}
|
||||
\emph{Répondre aux questions suivantes par lecture graphique.}
|
||||
\begin{enumerate}
|
||||
\setcounter{enumi}{1}
|
||||
\item Résoudre graphiquement et de façon approchée l'équation $f(x) = \np{80000}$.
|
||||
\item Donner un encadrement de la quantité
|
||||
\[
|
||||
\int_{2}^{32} f(x) \; dx
|
||||
\]
|
||||
Vous expliquerez votre démarche en utilisant le graphique.
|
||||
\end{enumerate}
|
||||
|
||||
\medskip
|
||||
|
||||
\textbf{Partie B - Étude théorique}
|
||||
|
||||
\begin{enumerate}
|
||||
\setcounter{enumi}{4}
|
||||
\item Étude des variations.
|
||||
\begin{enumerate}
|
||||
\item On note $f'$ la dérivée de la fonction $f$ sur $\intFF{0}{64}$.
|
||||
|
||||
Démontrer que pour tout $x$ de $\intFF{0}{20}$ , $f'(x) = - 625x\text{e}^{-0.0625x}$.
|
||||
|
||||
\item En déduire le sens de variation de $f$ et dresser son tableau des variations sur l'intervalle $\intFF{0}{64}$. Si nécessaire, arrondir à l'unité les valeurs présentes dans le tableau.
|
||||
|
||||
\item Démontrer que l'équation $f(x) = \np{80000}$ admet une unique solution $\alpha$ sur $\intFF{0}{64}$, puis donner une valeur approchée de $\alpha$ à $10^{-2}$ près à l'aide de la calculatrice.
|
||||
\end{enumerate}
|
||||
|
||||
\item Étude de la convexité
|
||||
\begin{enumerate}
|
||||
\item On note $f''$ la dérivée seconde de $f$ sur $\intFF{0}{64}$.
|
||||
|
||||
Démontrer que pour tout $x$ de $\intFF{0}{20}$ , $f''(x) = (39.0625x-625)\text{e}^{-0.0625x}$.
|
||||
|
||||
\item Démontrer que $f$ admet un point d'inflexion dont on donnera son abscisse.
|
||||
\end{enumerate}
|
||||
|
||||
\item Aire sous la courbe
|
||||
\begin{enumerate}
|
||||
\item On souhaite approximer la fonction $f$ sur l'intervalle $I$ par la droite $D$ qui relie les points $(0;f(0))$ et $(64, f(64))$. Tracer cette droite sur le graphique.
|
||||
\item On note $g$ la fonction affine qui décrit cette droite $D$. Détermine l'expression de $g$
|
||||
\item Calculer $\displaystyle \int_0^{64} g(x)\; dx$
|
||||
\item Avec la calculatrice, calculer une valeur approchée de $\displaystyle\int_{0}^{64} f(x) \; dx$
|
||||
\item Comparer les valeurs trouvées aux deux questions précédentes. Comment s'explique l'écart entre ces deux valeurs?
|
||||
\end{enumerate}
|
||||
\end{enumerate}
|
||||
|
||||
\medskip
|
||||
|
||||
\textbf{Partie C - Application économique}
|
||||
|
||||
Une entreprise a pris la décision de fermer son usine de production de smartphones en 64 mois.
|
||||
|
||||
La fonction capacité de production de cette usine est modélisée sur l'intervalle $\intFF{0}{64}$ par la fonction $f$ étudiée dans les parties A et B.
|
||||
|
||||
Le nombre $x$ représente le temps en mois après la décision de la fermeture du site et le nombre $f(x)$ représente capacité production de smartphone au moment $x$.
|
||||
|
||||
Utiliser les résultats de la partie B afin de répondre aux questions suivantes:
|
||||
|
||||
\begin{enumerate}
|
||||
\setcounter{enumi}{7}
|
||||
\item Combien de smartphones pouvaient être produit à la fermeture de l'usine?
|
||||
\item Pendant combien de temps la capacité de production de l'usine a réussi à se maintenir au dessus de \np{80000}?
|
||||
\item Combien de smartphones ont pu être produit entre la prise de décision et la fermeture de l'usine?
|
||||
\end{enumerate}
|
||||
|
||||
\end{exercise}
|
||||
|
||||
\begin{solution}
|
||||
\begin{enumerate}
|
||||
\item
|
||||
\begin{tikzpicture}[yscale=1, xscale=0.234375]
|
||||
\tkzInit[xmin=0,xmax=65,xstep=1,
|
||||
ymin=0,ymax=170000,ystep=17000]
|
||||
\tkzGrid
|
||||
\tkzGrid[sub, subxstep=0.2, subystep=3400.0]
|
||||
\tkzAxeXY[up space=0.5,right space=.2]
|
||||
\tkzFct[domain = 0:64, line width=1pt]{10000*(x+16)*exp(-0.0625*x)}
|
||||
%M\tkzFct[domain = 0:64, line width=1pt, blue]{80000}
|
||||
\end{tikzpicture}
|
||||
\item Tracer la droite $y=80000$. C'est l'abscisse de l'intersection entre cette droite et la courbe
|
||||
\item
|
||||
\item
|
||||
|
||||
\end{enumerate}
|
||||
|
||||
\end{solution}
|
||||
|
||||
\end{document}
|
||||
|
||||
%%% Local Variables:
|
||||
%%% mode: latex
|
||||
%%% TeX-master: "master"
|
||||
%%% End:
|
||||
121
TES/DM/DM_20_02/corr_04_DM_20_02.tex
Normal file
121
TES/DM/DM_20_02/corr_04_DM_20_02.tex
Normal file
@@ -0,0 +1,121 @@
|
||||
\documentclass[a4paper,10pt]{article}
|
||||
\usepackage{tasks}
|
||||
\usepackage{myXsim}
|
||||
|
||||
\title{DM 2 -- CRETIN Marie}
|
||||
\tribe{Terminale ES-L}
|
||||
\date{9 mars 2020}
|
||||
|
||||
\xsimsetup{
|
||||
solution/print = true
|
||||
}
|
||||
|
||||
\begin{document}
|
||||
\maketitle
|
||||
|
||||
\begin{exercise}[subtitle={Étude de fonction}]
|
||||
On considère la fonction dérivable $f$ définie sur $I = [0~;~80]$ par:
|
||||
|
||||
\[f(x) = \np{10000}(x + 20)\text{e}^{- 0.05x}.\]
|
||||
|
||||
\textbf{Partie A - Étude graphique}
|
||||
|
||||
On a représenté sur le graphique en annexe, la courbe représentative de la fonction $f$.
|
||||
|
||||
\begin{enumerate}
|
||||
\item Avec un tableur tracer et imprimer la courbe représentative de $f$ sur $I$
|
||||
\end{enumerate}
|
||||
\emph{Répondre aux questions suivantes par lecture graphique.}
|
||||
\begin{enumerate}
|
||||
\setcounter{enumi}{1}
|
||||
\item Résoudre graphiquement et de façon approchée l'équation $f(x) = \np{100000}$.
|
||||
\item Donner un encadrement de la quantité
|
||||
\[
|
||||
\int_{2}^{40} f(x) \; dx
|
||||
\]
|
||||
Vous expliquerez votre démarche en utilisant le graphique.
|
||||
\end{enumerate}
|
||||
|
||||
\medskip
|
||||
|
||||
\textbf{Partie B - Étude théorique}
|
||||
|
||||
\begin{enumerate}
|
||||
\setcounter{enumi}{4}
|
||||
\item Étude des variations.
|
||||
\begin{enumerate}
|
||||
\item On note $f'$ la dérivée de la fonction $f$ sur $\intFF{0}{80}$.
|
||||
|
||||
Démontrer que pour tout $x$ de $\intFF{0}{20}$ , $f'(x) = - 500x\text{e}^{-0.05x}$.
|
||||
|
||||
\item En déduire le sens de variation de $f$ et dresser son tableau des variations sur l'intervalle $\intFF{0}{80}$. Si nécessaire, arrondir à l'unité les valeurs présentes dans le tableau.
|
||||
|
||||
\item Démontrer que l'équation $f(x) = \np{100000}$ admet une unique solution $\alpha$ sur $\intFF{0}{80}$, puis donner une valeur approchée de $\alpha$ à $10^{-2}$ près à l'aide de la calculatrice.
|
||||
\end{enumerate}
|
||||
|
||||
\item Étude de la convexité
|
||||
\begin{enumerate}
|
||||
\item On note $f''$ la dérivée seconde de $f$ sur $\intFF{0}{80}$.
|
||||
|
||||
Démontrer que pour tout $x$ de $\intFF{0}{20}$ , $f''(x) = (25x-500)\text{e}^{-0.05x}$.
|
||||
|
||||
\item Démontrer que $f$ admet un point d'inflexion dont on donnera son abscisse.
|
||||
\end{enumerate}
|
||||
|
||||
\item Aire sous la courbe
|
||||
\begin{enumerate}
|
||||
\item On souhaite approximer la fonction $f$ sur l'intervalle $I$ par la droite $D$ qui relie les points $(0;f(0))$ et $(80, f(80))$. Tracer cette droite sur le graphique.
|
||||
\item On note $g$ la fonction affine qui décrit cette droite $D$. Détermine l'expression de $g$
|
||||
\item Calculer $\displaystyle \int_0^{80} g(x)\; dx$
|
||||
\item Avec la calculatrice, calculer une valeur approchée de $\displaystyle\int_{0}^{80} f(x) \; dx$
|
||||
\item Comparer les valeurs trouvées aux deux questions précédentes. Comment s'explique l'écart entre ces deux valeurs?
|
||||
\end{enumerate}
|
||||
\end{enumerate}
|
||||
|
||||
\medskip
|
||||
|
||||
\textbf{Partie C - Application économique}
|
||||
|
||||
Une entreprise a pris la décision de fermer son usine de production de smartphones en 80 mois.
|
||||
|
||||
La fonction capacité de production de cette usine est modélisée sur l'intervalle $\intFF{0}{80}$ par la fonction $f$ étudiée dans les parties A et B.
|
||||
|
||||
Le nombre $x$ représente le temps en mois après la décision de la fermeture du site et le nombre $f(x)$ représente capacité production de smartphone au moment $x$.
|
||||
|
||||
Utiliser les résultats de la partie B afin de répondre aux questions suivantes:
|
||||
|
||||
\begin{enumerate}
|
||||
\setcounter{enumi}{7}
|
||||
\item Combien de smartphones pouvaient être produit à la fermeture de l'usine?
|
||||
\item Pendant combien de temps la capacité de production de l'usine a réussi à se maintenir au dessus de \np{100000}?
|
||||
\item Combien de smartphones ont pu être produit entre la prise de décision et la fermeture de l'usine?
|
||||
\end{enumerate}
|
||||
|
||||
\end{exercise}
|
||||
|
||||
\begin{solution}
|
||||
\begin{enumerate}
|
||||
\item
|
||||
\begin{tikzpicture}[yscale=1, xscale=0.1875]
|
||||
\tkzInit[xmin=0,xmax=81,xstep=1,
|
||||
ymin=0,ymax=210000,ystep=21000]
|
||||
\tkzGrid
|
||||
\tkzGrid[sub, subxstep=0.2, subystep=4200.0]
|
||||
\tkzAxeXY[up space=0.5,right space=.2]
|
||||
\tkzFct[domain = 0:80, line width=1pt]{10000*(x+20)*exp(-0.05*x)}
|
||||
%M\tkzFct[domain = 0:80, line width=1pt, blue]{100000}
|
||||
\end{tikzpicture}
|
||||
\item Tracer la droite $y=100000$. C'est l'abscisse de l'intersection entre cette droite et la courbe
|
||||
\item
|
||||
\item
|
||||
|
||||
\end{enumerate}
|
||||
|
||||
\end{solution}
|
||||
|
||||
\end{document}
|
||||
|
||||
%%% Local Variables:
|
||||
%%% mode: latex
|
||||
%%% TeX-master: "master"
|
||||
%%% End:
|
||||
121
TES/DM/DM_20_02/corr_05_DM_20_02.tex
Normal file
121
TES/DM/DM_20_02/corr_05_DM_20_02.tex
Normal file
@@ -0,0 +1,121 @@
|
||||
\documentclass[a4paper,10pt]{article}
|
||||
\usepackage{tasks}
|
||||
\usepackage{myXsim}
|
||||
|
||||
\title{DM 2 -- DENIS Clarisse}
|
||||
\tribe{Terminale ES-L}
|
||||
\date{9 mars 2020}
|
||||
|
||||
\xsimsetup{
|
||||
solution/print = true
|
||||
}
|
||||
|
||||
\begin{document}
|
||||
\maketitle
|
||||
|
||||
\begin{exercise}[subtitle={Étude de fonction}]
|
||||
On considère la fonction dérivable $f$ définie sur $I = [0~;~64]$ par:
|
||||
|
||||
\[f(x) = \np{9000}(x + 16)\text{e}^{- 0.0625x}.\]
|
||||
|
||||
\textbf{Partie A - Étude graphique}
|
||||
|
||||
On a représenté sur le graphique en annexe, la courbe représentative de la fonction $f$.
|
||||
|
||||
\begin{enumerate}
|
||||
\item Avec un tableur tracer et imprimer la courbe représentative de $f$ sur $I$
|
||||
\end{enumerate}
|
||||
\emph{Répondre aux questions suivantes par lecture graphique.}
|
||||
\begin{enumerate}
|
||||
\setcounter{enumi}{1}
|
||||
\item Résoudre graphiquement et de façon approchée l'équation $f(x) = \np{72000}$.
|
||||
\item Donner un encadrement de la quantité
|
||||
\[
|
||||
\int_{2}^{32} f(x) \; dx
|
||||
\]
|
||||
Vous expliquerez votre démarche en utilisant le graphique.
|
||||
\end{enumerate}
|
||||
|
||||
\medskip
|
||||
|
||||
\textbf{Partie B - Étude théorique}
|
||||
|
||||
\begin{enumerate}
|
||||
\setcounter{enumi}{4}
|
||||
\item Étude des variations.
|
||||
\begin{enumerate}
|
||||
\item On note $f'$ la dérivée de la fonction $f$ sur $\intFF{0}{64}$.
|
||||
|
||||
Démontrer que pour tout $x$ de $\intFF{0}{20}$ , $f'(x) = - 562.5000x\text{e}^{-0.0625x}$.
|
||||
|
||||
\item En déduire le sens de variation de $f$ et dresser son tableau des variations sur l'intervalle $\intFF{0}{64}$. Si nécessaire, arrondir à l'unité les valeurs présentes dans le tableau.
|
||||
|
||||
\item Démontrer que l'équation $f(x) = \np{72000}$ admet une unique solution $\alpha$ sur $\intFF{0}{64}$, puis donner une valeur approchée de $\alpha$ à $10^{-2}$ près à l'aide de la calculatrice.
|
||||
\end{enumerate}
|
||||
|
||||
\item Étude de la convexité
|
||||
\begin{enumerate}
|
||||
\item On note $f''$ la dérivée seconde de $f$ sur $\intFF{0}{64}$.
|
||||
|
||||
Démontrer que pour tout $x$ de $\intFF{0}{20}$ , $f''(x) = (35.15625000x-562.5000)\text{e}^{-0.0625x}$.
|
||||
|
||||
\item Démontrer que $f$ admet un point d'inflexion dont on donnera son abscisse.
|
||||
\end{enumerate}
|
||||
|
||||
\item Aire sous la courbe
|
||||
\begin{enumerate}
|
||||
\item On souhaite approximer la fonction $f$ sur l'intervalle $I$ par la droite $D$ qui relie les points $(0;f(0))$ et $(64, f(64))$. Tracer cette droite sur le graphique.
|
||||
\item On note $g$ la fonction affine qui décrit cette droite $D$. Détermine l'expression de $g$
|
||||
\item Calculer $\displaystyle \int_0^{64} g(x)\; dx$
|
||||
\item Avec la calculatrice, calculer une valeur approchée de $\displaystyle\int_{0}^{64} f(x) \; dx$
|
||||
\item Comparer les valeurs trouvées aux deux questions précédentes. Comment s'explique l'écart entre ces deux valeurs?
|
||||
\end{enumerate}
|
||||
\end{enumerate}
|
||||
|
||||
\medskip
|
||||
|
||||
\textbf{Partie C - Application économique}
|
||||
|
||||
Une entreprise a pris la décision de fermer son usine de production de smartphones en 64 mois.
|
||||
|
||||
La fonction capacité de production de cette usine est modélisée sur l'intervalle $\intFF{0}{64}$ par la fonction $f$ étudiée dans les parties A et B.
|
||||
|
||||
Le nombre $x$ représente le temps en mois après la décision de la fermeture du site et le nombre $f(x)$ représente capacité production de smartphone au moment $x$.
|
||||
|
||||
Utiliser les résultats de la partie B afin de répondre aux questions suivantes:
|
||||
|
||||
\begin{enumerate}
|
||||
\setcounter{enumi}{7}
|
||||
\item Combien de smartphones pouvaient être produit à la fermeture de l'usine?
|
||||
\item Pendant combien de temps la capacité de production de l'usine a réussi à se maintenir au dessus de \np{72000}?
|
||||
\item Combien de smartphones ont pu être produit entre la prise de décision et la fermeture de l'usine?
|
||||
\end{enumerate}
|
||||
|
||||
\end{exercise}
|
||||
|
||||
\begin{solution}
|
||||
\begin{enumerate}
|
||||
\item
|
||||
\begin{tikzpicture}[yscale=1, xscale=0.234375]
|
||||
\tkzInit[xmin=0,xmax=65,xstep=1,
|
||||
ymin=0,ymax=153000,ystep=15000]
|
||||
\tkzGrid
|
||||
\tkzGrid[sub, subxstep=0.2, subystep=3000.0]
|
||||
\tkzAxeXY[up space=0.5,right space=.2]
|
||||
\tkzFct[domain = 0:64, line width=1pt]{9000*(x+16)*exp(-0.0625*x)}
|
||||
%M\tkzFct[domain = 0:64, line width=1pt, blue]{72000}
|
||||
\end{tikzpicture}
|
||||
\item Tracer la droite $y=72000$. C'est l'abscisse de l'intersection entre cette droite et la courbe
|
||||
\item
|
||||
\item
|
||||
|
||||
\end{enumerate}
|
||||
|
||||
\end{solution}
|
||||
|
||||
\end{document}
|
||||
|
||||
%%% Local Variables:
|
||||
%%% mode: latex
|
||||
%%% TeX-master: "master"
|
||||
%%% End:
|
||||
121
TES/DM/DM_20_02/corr_06_DM_20_02.tex
Normal file
121
TES/DM/DM_20_02/corr_06_DM_20_02.tex
Normal file
@@ -0,0 +1,121 @@
|
||||
\documentclass[a4paper,10pt]{article}
|
||||
\usepackage{tasks}
|
||||
\usepackage{myXsim}
|
||||
|
||||
\title{DM 2 -- DOS SANTOS Théo}
|
||||
\tribe{Terminale ES-L}
|
||||
\date{9 mars 2020}
|
||||
|
||||
\xsimsetup{
|
||||
solution/print = true
|
||||
}
|
||||
|
||||
\begin{document}
|
||||
\maketitle
|
||||
|
||||
\begin{exercise}[subtitle={Étude de fonction}]
|
||||
On considère la fonction dérivable $f$ définie sur $I = [0~;~64]$ par:
|
||||
|
||||
\[f(x) = \np{2000}(x + 16)\text{e}^{- 0.0625x}.\]
|
||||
|
||||
\textbf{Partie A - Étude graphique}
|
||||
|
||||
On a représenté sur le graphique en annexe, la courbe représentative de la fonction $f$.
|
||||
|
||||
\begin{enumerate}
|
||||
\item Avec un tableur tracer et imprimer la courbe représentative de $f$ sur $I$
|
||||
\end{enumerate}
|
||||
\emph{Répondre aux questions suivantes par lecture graphique.}
|
||||
\begin{enumerate}
|
||||
\setcounter{enumi}{1}
|
||||
\item Résoudre graphiquement et de façon approchée l'équation $f(x) = \np{16000}$.
|
||||
\item Donner un encadrement de la quantité
|
||||
\[
|
||||
\int_{2}^{32} f(x) \; dx
|
||||
\]
|
||||
Vous expliquerez votre démarche en utilisant le graphique.
|
||||
\end{enumerate}
|
||||
|
||||
\medskip
|
||||
|
||||
\textbf{Partie B - Étude théorique}
|
||||
|
||||
\begin{enumerate}
|
||||
\setcounter{enumi}{4}
|
||||
\item Étude des variations.
|
||||
\begin{enumerate}
|
||||
\item On note $f'$ la dérivée de la fonction $f$ sur $\intFF{0}{64}$.
|
||||
|
||||
Démontrer que pour tout $x$ de $\intFF{0}{20}$ , $f'(x) = - 125x\text{e}^{-0.0625x}$.
|
||||
|
||||
\item En déduire le sens de variation de $f$ et dresser son tableau des variations sur l'intervalle $\intFF{0}{64}$. Si nécessaire, arrondir à l'unité les valeurs présentes dans le tableau.
|
||||
|
||||
\item Démontrer que l'équation $f(x) = \np{16000}$ admet une unique solution $\alpha$ sur $\intFF{0}{64}$, puis donner une valeur approchée de $\alpha$ à $10^{-2}$ près à l'aide de la calculatrice.
|
||||
\end{enumerate}
|
||||
|
||||
\item Étude de la convexité
|
||||
\begin{enumerate}
|
||||
\item On note $f''$ la dérivée seconde de $f$ sur $\intFF{0}{64}$.
|
||||
|
||||
Démontrer que pour tout $x$ de $\intFF{0}{20}$ , $f''(x) = (7.8125x-125)\text{e}^{-0.0625x}$.
|
||||
|
||||
\item Démontrer que $f$ admet un point d'inflexion dont on donnera son abscisse.
|
||||
\end{enumerate}
|
||||
|
||||
\item Aire sous la courbe
|
||||
\begin{enumerate}
|
||||
\item On souhaite approximer la fonction $f$ sur l'intervalle $I$ par la droite $D$ qui relie les points $(0;f(0))$ et $(64, f(64))$. Tracer cette droite sur le graphique.
|
||||
\item On note $g$ la fonction affine qui décrit cette droite $D$. Détermine l'expression de $g$
|
||||
\item Calculer $\displaystyle \int_0^{64} g(x)\; dx$
|
||||
\item Avec la calculatrice, calculer une valeur approchée de $\displaystyle\int_{0}^{64} f(x) \; dx$
|
||||
\item Comparer les valeurs trouvées aux deux questions précédentes. Comment s'explique l'écart entre ces deux valeurs?
|
||||
\end{enumerate}
|
||||
\end{enumerate}
|
||||
|
||||
\medskip
|
||||
|
||||
\textbf{Partie C - Application économique}
|
||||
|
||||
Une entreprise a pris la décision de fermer son usine de production de smartphones en 64 mois.
|
||||
|
||||
La fonction capacité de production de cette usine est modélisée sur l'intervalle $\intFF{0}{64}$ par la fonction $f$ étudiée dans les parties A et B.
|
||||
|
||||
Le nombre $x$ représente le temps en mois après la décision de la fermeture du site et le nombre $f(x)$ représente capacité production de smartphone au moment $x$.
|
||||
|
||||
Utiliser les résultats de la partie B afin de répondre aux questions suivantes:
|
||||
|
||||
\begin{enumerate}
|
||||
\setcounter{enumi}{7}
|
||||
\item Combien de smartphones pouvaient être produit à la fermeture de l'usine?
|
||||
\item Pendant combien de temps la capacité de production de l'usine a réussi à se maintenir au dessus de \np{16000}?
|
||||
\item Combien de smartphones ont pu être produit entre la prise de décision et la fermeture de l'usine?
|
||||
\end{enumerate}
|
||||
|
||||
\end{exercise}
|
||||
|
||||
\begin{solution}
|
||||
\begin{enumerate}
|
||||
\item
|
||||
\begin{tikzpicture}[yscale=1, xscale=0.234375]
|
||||
\tkzInit[xmin=0,xmax=65,xstep=1,
|
||||
ymin=0,ymax=34000,ystep=3000]
|
||||
\tkzGrid
|
||||
\tkzGrid[sub, subxstep=0.2, subystep=600.0]
|
||||
\tkzAxeXY[up space=0.5,right space=.2]
|
||||
\tkzFct[domain = 0:64, line width=1pt]{2000*(x+16)*exp(-0.0625*x)}
|
||||
%M\tkzFct[domain = 0:64, line width=1pt, blue]{16000}
|
||||
\end{tikzpicture}
|
||||
\item Tracer la droite $y=16000$. C'est l'abscisse de l'intersection entre cette droite et la courbe
|
||||
\item
|
||||
\item
|
||||
|
||||
\end{enumerate}
|
||||
|
||||
\end{solution}
|
||||
|
||||
\end{document}
|
||||
|
||||
%%% Local Variables:
|
||||
%%% mode: latex
|
||||
%%% TeX-master: "master"
|
||||
%%% End:
|
||||
121
TES/DM/DM_20_02/corr_07_DM_20_02.tex
Normal file
121
TES/DM/DM_20_02/corr_07_DM_20_02.tex
Normal file
@@ -0,0 +1,121 @@
|
||||
\documentclass[a4paper,10pt]{article}
|
||||
\usepackage{tasks}
|
||||
\usepackage{myXsim}
|
||||
|
||||
\title{DM 2 -- FERREIRA Tina}
|
||||
\tribe{Terminale ES-L}
|
||||
\date{9 mars 2020}
|
||||
|
||||
\xsimsetup{
|
||||
solution/print = true
|
||||
}
|
||||
|
||||
\begin{document}
|
||||
\maketitle
|
||||
|
||||
\begin{exercise}[subtitle={Étude de fonction}]
|
||||
On considère la fonction dérivable $f$ définie sur $I = [0~;~80]$ par:
|
||||
|
||||
\[f(x) = \np{8000}(x + 20)\text{e}^{- 0.05x}.\]
|
||||
|
||||
\textbf{Partie A - Étude graphique}
|
||||
|
||||
On a représenté sur le graphique en annexe, la courbe représentative de la fonction $f$.
|
||||
|
||||
\begin{enumerate}
|
||||
\item Avec un tableur tracer et imprimer la courbe représentative de $f$ sur $I$
|
||||
\end{enumerate}
|
||||
\emph{Répondre aux questions suivantes par lecture graphique.}
|
||||
\begin{enumerate}
|
||||
\setcounter{enumi}{1}
|
||||
\item Résoudre graphiquement et de façon approchée l'équation $f(x) = \np{80000}$.
|
||||
\item Donner un encadrement de la quantité
|
||||
\[
|
||||
\int_{2}^{40} f(x) \; dx
|
||||
\]
|
||||
Vous expliquerez votre démarche en utilisant le graphique.
|
||||
\end{enumerate}
|
||||
|
||||
\medskip
|
||||
|
||||
\textbf{Partie B - Étude théorique}
|
||||
|
||||
\begin{enumerate}
|
||||
\setcounter{enumi}{4}
|
||||
\item Étude des variations.
|
||||
\begin{enumerate}
|
||||
\item On note $f'$ la dérivée de la fonction $f$ sur $\intFF{0}{80}$.
|
||||
|
||||
Démontrer que pour tout $x$ de $\intFF{0}{20}$ , $f'(x) = - 400x\text{e}^{-0.05x}$.
|
||||
|
||||
\item En déduire le sens de variation de $f$ et dresser son tableau des variations sur l'intervalle $\intFF{0}{80}$. Si nécessaire, arrondir à l'unité les valeurs présentes dans le tableau.
|
||||
|
||||
\item Démontrer que l'équation $f(x) = \np{80000}$ admet une unique solution $\alpha$ sur $\intFF{0}{80}$, puis donner une valeur approchée de $\alpha$ à $10^{-2}$ près à l'aide de la calculatrice.
|
||||
\end{enumerate}
|
||||
|
||||
\item Étude de la convexité
|
||||
\begin{enumerate}
|
||||
\item On note $f''$ la dérivée seconde de $f$ sur $\intFF{0}{80}$.
|
||||
|
||||
Démontrer que pour tout $x$ de $\intFF{0}{20}$ , $f''(x) = (20x-400)\text{e}^{-0.05x}$.
|
||||
|
||||
\item Démontrer que $f$ admet un point d'inflexion dont on donnera son abscisse.
|
||||
\end{enumerate}
|
||||
|
||||
\item Aire sous la courbe
|
||||
\begin{enumerate}
|
||||
\item On souhaite approximer la fonction $f$ sur l'intervalle $I$ par la droite $D$ qui relie les points $(0;f(0))$ et $(80, f(80))$. Tracer cette droite sur le graphique.
|
||||
\item On note $g$ la fonction affine qui décrit cette droite $D$. Détermine l'expression de $g$
|
||||
\item Calculer $\displaystyle \int_0^{80} g(x)\; dx$
|
||||
\item Avec la calculatrice, calculer une valeur approchée de $\displaystyle\int_{0}^{80} f(x) \; dx$
|
||||
\item Comparer les valeurs trouvées aux deux questions précédentes. Comment s'explique l'écart entre ces deux valeurs?
|
||||
\end{enumerate}
|
||||
\end{enumerate}
|
||||
|
||||
\medskip
|
||||
|
||||
\textbf{Partie C - Application économique}
|
||||
|
||||
Une entreprise a pris la décision de fermer son usine de production de smartphones en 80 mois.
|
||||
|
||||
La fonction capacité de production de cette usine est modélisée sur l'intervalle $\intFF{0}{80}$ par la fonction $f$ étudiée dans les parties A et B.
|
||||
|
||||
Le nombre $x$ représente le temps en mois après la décision de la fermeture du site et le nombre $f(x)$ représente capacité production de smartphone au moment $x$.
|
||||
|
||||
Utiliser les résultats de la partie B afin de répondre aux questions suivantes:
|
||||
|
||||
\begin{enumerate}
|
||||
\setcounter{enumi}{7}
|
||||
\item Combien de smartphones pouvaient être produit à la fermeture de l'usine?
|
||||
\item Pendant combien de temps la capacité de production de l'usine a réussi à se maintenir au dessus de \np{80000}?
|
||||
\item Combien de smartphones ont pu être produit entre la prise de décision et la fermeture de l'usine?
|
||||
\end{enumerate}
|
||||
|
||||
\end{exercise}
|
||||
|
||||
\begin{solution}
|
||||
\begin{enumerate}
|
||||
\item
|
||||
\begin{tikzpicture}[yscale=1, xscale=0.1875]
|
||||
\tkzInit[xmin=0,xmax=81,xstep=1,
|
||||
ymin=0,ymax=168000,ystep=16000]
|
||||
\tkzGrid
|
||||
\tkzGrid[sub, subxstep=0.2, subystep=3200.0]
|
||||
\tkzAxeXY[up space=0.5,right space=.2]
|
||||
\tkzFct[domain = 0:80, line width=1pt]{8000*(x+20)*exp(-0.05*x)}
|
||||
%M\tkzFct[domain = 0:80, line width=1pt, blue]{80000}
|
||||
\end{tikzpicture}
|
||||
\item Tracer la droite $y=80000$. C'est l'abscisse de l'intersection entre cette droite et la courbe
|
||||
\item
|
||||
\item
|
||||
|
||||
\end{enumerate}
|
||||
|
||||
\end{solution}
|
||||
|
||||
\end{document}
|
||||
|
||||
%%% Local Variables:
|
||||
%%% mode: latex
|
||||
%%% TeX-master: "master"
|
||||
%%% End:
|
||||
121
TES/DM/DM_20_02/corr_08_DM_20_02.tex
Normal file
121
TES/DM/DM_20_02/corr_08_DM_20_02.tex
Normal file
@@ -0,0 +1,121 @@
|
||||
\documentclass[a4paper,10pt]{article}
|
||||
\usepackage{tasks}
|
||||
\usepackage{myXsim}
|
||||
|
||||
\title{DM 2 -- GAUDARD Camille}
|
||||
\tribe{Terminale ES-L}
|
||||
\date{9 mars 2020}
|
||||
|
||||
\xsimsetup{
|
||||
solution/print = true
|
||||
}
|
||||
|
||||
\begin{document}
|
||||
\maketitle
|
||||
|
||||
\begin{exercise}[subtitle={Étude de fonction}]
|
||||
On considère la fonction dérivable $f$ définie sur $I = [0~;~80]$ par:
|
||||
|
||||
\[f(x) = \np{7000}(x + 20)\text{e}^{- 0.05x}.\]
|
||||
|
||||
\textbf{Partie A - Étude graphique}
|
||||
|
||||
On a représenté sur le graphique en annexe, la courbe représentative de la fonction $f$.
|
||||
|
||||
\begin{enumerate}
|
||||
\item Avec un tableur tracer et imprimer la courbe représentative de $f$ sur $I$
|
||||
\end{enumerate}
|
||||
\emph{Répondre aux questions suivantes par lecture graphique.}
|
||||
\begin{enumerate}
|
||||
\setcounter{enumi}{1}
|
||||
\item Résoudre graphiquement et de façon approchée l'équation $f(x) = \np{70000}$.
|
||||
\item Donner un encadrement de la quantité
|
||||
\[
|
||||
\int_{2}^{40} f(x) \; dx
|
||||
\]
|
||||
Vous expliquerez votre démarche en utilisant le graphique.
|
||||
\end{enumerate}
|
||||
|
||||
\medskip
|
||||
|
||||
\textbf{Partie B - Étude théorique}
|
||||
|
||||
\begin{enumerate}
|
||||
\setcounter{enumi}{4}
|
||||
\item Étude des variations.
|
||||
\begin{enumerate}
|
||||
\item On note $f'$ la dérivée de la fonction $f$ sur $\intFF{0}{80}$.
|
||||
|
||||
Démontrer que pour tout $x$ de $\intFF{0}{20}$ , $f'(x) = - 350x\text{e}^{-0.05x}$.
|
||||
|
||||
\item En déduire le sens de variation de $f$ et dresser son tableau des variations sur l'intervalle $\intFF{0}{80}$. Si nécessaire, arrondir à l'unité les valeurs présentes dans le tableau.
|
||||
|
||||
\item Démontrer que l'équation $f(x) = \np{70000}$ admet une unique solution $\alpha$ sur $\intFF{0}{80}$, puis donner une valeur approchée de $\alpha$ à $10^{-2}$ près à l'aide de la calculatrice.
|
||||
\end{enumerate}
|
||||
|
||||
\item Étude de la convexité
|
||||
\begin{enumerate}
|
||||
\item On note $f''$ la dérivée seconde de $f$ sur $\intFF{0}{80}$.
|
||||
|
||||
Démontrer que pour tout $x$ de $\intFF{0}{20}$ , $f''(x) = (17.50x-350)\text{e}^{-0.05x}$.
|
||||
|
||||
\item Démontrer que $f$ admet un point d'inflexion dont on donnera son abscisse.
|
||||
\end{enumerate}
|
||||
|
||||
\item Aire sous la courbe
|
||||
\begin{enumerate}
|
||||
\item On souhaite approximer la fonction $f$ sur l'intervalle $I$ par la droite $D$ qui relie les points $(0;f(0))$ et $(80, f(80))$. Tracer cette droite sur le graphique.
|
||||
\item On note $g$ la fonction affine qui décrit cette droite $D$. Détermine l'expression de $g$
|
||||
\item Calculer $\displaystyle \int_0^{80} g(x)\; dx$
|
||||
\item Avec la calculatrice, calculer une valeur approchée de $\displaystyle\int_{0}^{80} f(x) \; dx$
|
||||
\item Comparer les valeurs trouvées aux deux questions précédentes. Comment s'explique l'écart entre ces deux valeurs?
|
||||
\end{enumerate}
|
||||
\end{enumerate}
|
||||
|
||||
\medskip
|
||||
|
||||
\textbf{Partie C - Application économique}
|
||||
|
||||
Une entreprise a pris la décision de fermer son usine de production de smartphones en 80 mois.
|
||||
|
||||
La fonction capacité de production de cette usine est modélisée sur l'intervalle $\intFF{0}{80}$ par la fonction $f$ étudiée dans les parties A et B.
|
||||
|
||||
Le nombre $x$ représente le temps en mois après la décision de la fermeture du site et le nombre $f(x)$ représente capacité production de smartphone au moment $x$.
|
||||
|
||||
Utiliser les résultats de la partie B afin de répondre aux questions suivantes:
|
||||
|
||||
\begin{enumerate}
|
||||
\setcounter{enumi}{7}
|
||||
\item Combien de smartphones pouvaient être produit à la fermeture de l'usine?
|
||||
\item Pendant combien de temps la capacité de production de l'usine a réussi à se maintenir au dessus de \np{70000}?
|
||||
\item Combien de smartphones ont pu être produit entre la prise de décision et la fermeture de l'usine?
|
||||
\end{enumerate}
|
||||
|
||||
\end{exercise}
|
||||
|
||||
\begin{solution}
|
||||
\begin{enumerate}
|
||||
\item
|
||||
\begin{tikzpicture}[yscale=1, xscale=0.1875]
|
||||
\tkzInit[xmin=0,xmax=81,xstep=1,
|
||||
ymin=0,ymax=147000,ystep=14000]
|
||||
\tkzGrid
|
||||
\tkzGrid[sub, subxstep=0.2, subystep=2800.0]
|
||||
\tkzAxeXY[up space=0.5,right space=.2]
|
||||
\tkzFct[domain = 0:80, line width=1pt]{7000*(x+20)*exp(-0.05*x)}
|
||||
%M\tkzFct[domain = 0:80, line width=1pt, blue]{70000}
|
||||
\end{tikzpicture}
|
||||
\item Tracer la droite $y=70000$. C'est l'abscisse de l'intersection entre cette droite et la courbe
|
||||
\item
|
||||
\item
|
||||
|
||||
\end{enumerate}
|
||||
|
||||
\end{solution}
|
||||
|
||||
\end{document}
|
||||
|
||||
%%% Local Variables:
|
||||
%%% mode: latex
|
||||
%%% TeX-master: "master"
|
||||
%%% End:
|
||||
121
TES/DM/DM_20_02/corr_09_DM_20_02.tex
Normal file
121
TES/DM/DM_20_02/corr_09_DM_20_02.tex
Normal file
@@ -0,0 +1,121 @@
|
||||
\documentclass[a4paper,10pt]{article}
|
||||
\usepackage{tasks}
|
||||
\usepackage{myXsim}
|
||||
|
||||
\title{DM 2 -- GUVERCIN Dilara Melisa}
|
||||
\tribe{Terminale ES-L}
|
||||
\date{9 mars 2020}
|
||||
|
||||
\xsimsetup{
|
||||
solution/print = true
|
||||
}
|
||||
|
||||
\begin{document}
|
||||
\maketitle
|
||||
|
||||
\begin{exercise}[subtitle={Étude de fonction}]
|
||||
On considère la fonction dérivable $f$ définie sur $I = [0~;~80]$ par:
|
||||
|
||||
\[f(x) = \np{4000}(x + 20)\text{e}^{- 0.05x}.\]
|
||||
|
||||
\textbf{Partie A - Étude graphique}
|
||||
|
||||
On a représenté sur le graphique en annexe, la courbe représentative de la fonction $f$.
|
||||
|
||||
\begin{enumerate}
|
||||
\item Avec un tableur tracer et imprimer la courbe représentative de $f$ sur $I$
|
||||
\end{enumerate}
|
||||
\emph{Répondre aux questions suivantes par lecture graphique.}
|
||||
\begin{enumerate}
|
||||
\setcounter{enumi}{1}
|
||||
\item Résoudre graphiquement et de façon approchée l'équation $f(x) = \np{40000}$.
|
||||
\item Donner un encadrement de la quantité
|
||||
\[
|
||||
\int_{2}^{40} f(x) \; dx
|
||||
\]
|
||||
Vous expliquerez votre démarche en utilisant le graphique.
|
||||
\end{enumerate}
|
||||
|
||||
\medskip
|
||||
|
||||
\textbf{Partie B - Étude théorique}
|
||||
|
||||
\begin{enumerate}
|
||||
\setcounter{enumi}{4}
|
||||
\item Étude des variations.
|
||||
\begin{enumerate}
|
||||
\item On note $f'$ la dérivée de la fonction $f$ sur $\intFF{0}{80}$.
|
||||
|
||||
Démontrer que pour tout $x$ de $\intFF{0}{20}$ , $f'(x) = - 200x\text{e}^{-0.05x}$.
|
||||
|
||||
\item En déduire le sens de variation de $f$ et dresser son tableau des variations sur l'intervalle $\intFF{0}{80}$. Si nécessaire, arrondir à l'unité les valeurs présentes dans le tableau.
|
||||
|
||||
\item Démontrer que l'équation $f(x) = \np{40000}$ admet une unique solution $\alpha$ sur $\intFF{0}{80}$, puis donner une valeur approchée de $\alpha$ à $10^{-2}$ près à l'aide de la calculatrice.
|
||||
\end{enumerate}
|
||||
|
||||
\item Étude de la convexité
|
||||
\begin{enumerate}
|
||||
\item On note $f''$ la dérivée seconde de $f$ sur $\intFF{0}{80}$.
|
||||
|
||||
Démontrer que pour tout $x$ de $\intFF{0}{20}$ , $f''(x) = (10x-200)\text{e}^{-0.05x}$.
|
||||
|
||||
\item Démontrer que $f$ admet un point d'inflexion dont on donnera son abscisse.
|
||||
\end{enumerate}
|
||||
|
||||
\item Aire sous la courbe
|
||||
\begin{enumerate}
|
||||
\item On souhaite approximer la fonction $f$ sur l'intervalle $I$ par la droite $D$ qui relie les points $(0;f(0))$ et $(80, f(80))$. Tracer cette droite sur le graphique.
|
||||
\item On note $g$ la fonction affine qui décrit cette droite $D$. Détermine l'expression de $g$
|
||||
\item Calculer $\displaystyle \int_0^{80} g(x)\; dx$
|
||||
\item Avec la calculatrice, calculer une valeur approchée de $\displaystyle\int_{0}^{80} f(x) \; dx$
|
||||
\item Comparer les valeurs trouvées aux deux questions précédentes. Comment s'explique l'écart entre ces deux valeurs?
|
||||
\end{enumerate}
|
||||
\end{enumerate}
|
||||
|
||||
\medskip
|
||||
|
||||
\textbf{Partie C - Application économique}
|
||||
|
||||
Une entreprise a pris la décision de fermer son usine de production de smartphones en 80 mois.
|
||||
|
||||
La fonction capacité de production de cette usine est modélisée sur l'intervalle $\intFF{0}{80}$ par la fonction $f$ étudiée dans les parties A et B.
|
||||
|
||||
Le nombre $x$ représente le temps en mois après la décision de la fermeture du site et le nombre $f(x)$ représente capacité production de smartphone au moment $x$.
|
||||
|
||||
Utiliser les résultats de la partie B afin de répondre aux questions suivantes:
|
||||
|
||||
\begin{enumerate}
|
||||
\setcounter{enumi}{7}
|
||||
\item Combien de smartphones pouvaient être produit à la fermeture de l'usine?
|
||||
\item Pendant combien de temps la capacité de production de l'usine a réussi à se maintenir au dessus de \np{40000}?
|
||||
\item Combien de smartphones ont pu être produit entre la prise de décision et la fermeture de l'usine?
|
||||
\end{enumerate}
|
||||
|
||||
\end{exercise}
|
||||
|
||||
\begin{solution}
|
||||
\begin{enumerate}
|
||||
\item
|
||||
\begin{tikzpicture}[yscale=1, xscale=0.1875]
|
||||
\tkzInit[xmin=0,xmax=81,xstep=1,
|
||||
ymin=0,ymax=84000,ystep=8000]
|
||||
\tkzGrid
|
||||
\tkzGrid[sub, subxstep=0.2, subystep=1600.0]
|
||||
\tkzAxeXY[up space=0.5,right space=.2]
|
||||
\tkzFct[domain = 0:80, line width=1pt]{4000*(x+20)*exp(-0.05*x)}
|
||||
%M\tkzFct[domain = 0:80, line width=1pt, blue]{40000}
|
||||
\end{tikzpicture}
|
||||
\item Tracer la droite $y=40000$. C'est l'abscisse de l'intersection entre cette droite et la courbe
|
||||
\item
|
||||
\item
|
||||
|
||||
\end{enumerate}
|
||||
|
||||
\end{solution}
|
||||
|
||||
\end{document}
|
||||
|
||||
%%% Local Variables:
|
||||
%%% mode: latex
|
||||
%%% TeX-master: "master"
|
||||
%%% End:
|
||||
121
TES/DM/DM_20_02/corr_10_DM_20_02.tex
Normal file
121
TES/DM/DM_20_02/corr_10_DM_20_02.tex
Normal file
@@ -0,0 +1,121 @@
|
||||
\documentclass[a4paper,10pt]{article}
|
||||
\usepackage{tasks}
|
||||
\usepackage{myXsim}
|
||||
|
||||
\title{DM 2 -- HALEGOI Agathe}
|
||||
\tribe{Terminale ES-L}
|
||||
\date{9 mars 2020}
|
||||
|
||||
\xsimsetup{
|
||||
solution/print = true
|
||||
}
|
||||
|
||||
\begin{document}
|
||||
\maketitle
|
||||
|
||||
\begin{exercise}[subtitle={Étude de fonction}]
|
||||
On considère la fonction dérivable $f$ définie sur $I = [0~;~64]$ par:
|
||||
|
||||
\[f(x) = \np{6000}(x + 16)\text{e}^{- 0.0625x}.\]
|
||||
|
||||
\textbf{Partie A - Étude graphique}
|
||||
|
||||
On a représenté sur le graphique en annexe, la courbe représentative de la fonction $f$.
|
||||
|
||||
\begin{enumerate}
|
||||
\item Avec un tableur tracer et imprimer la courbe représentative de $f$ sur $I$
|
||||
\end{enumerate}
|
||||
\emph{Répondre aux questions suivantes par lecture graphique.}
|
||||
\begin{enumerate}
|
||||
\setcounter{enumi}{1}
|
||||
\item Résoudre graphiquement et de façon approchée l'équation $f(x) = \np{48000}$.
|
||||
\item Donner un encadrement de la quantité
|
||||
\[
|
||||
\int_{2}^{32} f(x) \; dx
|
||||
\]
|
||||
Vous expliquerez votre démarche en utilisant le graphique.
|
||||
\end{enumerate}
|
||||
|
||||
\medskip
|
||||
|
||||
\textbf{Partie B - Étude théorique}
|
||||
|
||||
\begin{enumerate}
|
||||
\setcounter{enumi}{4}
|
||||
\item Étude des variations.
|
||||
\begin{enumerate}
|
||||
\item On note $f'$ la dérivée de la fonction $f$ sur $\intFF{0}{64}$.
|
||||
|
||||
Démontrer que pour tout $x$ de $\intFF{0}{20}$ , $f'(x) = - 375x\text{e}^{-0.0625x}$.
|
||||
|
||||
\item En déduire le sens de variation de $f$ et dresser son tableau des variations sur l'intervalle $\intFF{0}{64}$. Si nécessaire, arrondir à l'unité les valeurs présentes dans le tableau.
|
||||
|
||||
\item Démontrer que l'équation $f(x) = \np{48000}$ admet une unique solution $\alpha$ sur $\intFF{0}{64}$, puis donner une valeur approchée de $\alpha$ à $10^{-2}$ près à l'aide de la calculatrice.
|
||||
\end{enumerate}
|
||||
|
||||
\item Étude de la convexité
|
||||
\begin{enumerate}
|
||||
\item On note $f''$ la dérivée seconde de $f$ sur $\intFF{0}{64}$.
|
||||
|
||||
Démontrer que pour tout $x$ de $\intFF{0}{20}$ , $f''(x) = (23.4375x-375)\text{e}^{-0.0625x}$.
|
||||
|
||||
\item Démontrer que $f$ admet un point d'inflexion dont on donnera son abscisse.
|
||||
\end{enumerate}
|
||||
|
||||
\item Aire sous la courbe
|
||||
\begin{enumerate}
|
||||
\item On souhaite approximer la fonction $f$ sur l'intervalle $I$ par la droite $D$ qui relie les points $(0;f(0))$ et $(64, f(64))$. Tracer cette droite sur le graphique.
|
||||
\item On note $g$ la fonction affine qui décrit cette droite $D$. Détermine l'expression de $g$
|
||||
\item Calculer $\displaystyle \int_0^{64} g(x)\; dx$
|
||||
\item Avec la calculatrice, calculer une valeur approchée de $\displaystyle\int_{0}^{64} f(x) \; dx$
|
||||
\item Comparer les valeurs trouvées aux deux questions précédentes. Comment s'explique l'écart entre ces deux valeurs?
|
||||
\end{enumerate}
|
||||
\end{enumerate}
|
||||
|
||||
\medskip
|
||||
|
||||
\textbf{Partie C - Application économique}
|
||||
|
||||
Une entreprise a pris la décision de fermer son usine de production de smartphones en 64 mois.
|
||||
|
||||
La fonction capacité de production de cette usine est modélisée sur l'intervalle $\intFF{0}{64}$ par la fonction $f$ étudiée dans les parties A et B.
|
||||
|
||||
Le nombre $x$ représente le temps en mois après la décision de la fermeture du site et le nombre $f(x)$ représente capacité production de smartphone au moment $x$.
|
||||
|
||||
Utiliser les résultats de la partie B afin de répondre aux questions suivantes:
|
||||
|
||||
\begin{enumerate}
|
||||
\setcounter{enumi}{7}
|
||||
\item Combien de smartphones pouvaient être produit à la fermeture de l'usine?
|
||||
\item Pendant combien de temps la capacité de production de l'usine a réussi à se maintenir au dessus de \np{48000}?
|
||||
\item Combien de smartphones ont pu être produit entre la prise de décision et la fermeture de l'usine?
|
||||
\end{enumerate}
|
||||
|
||||
\end{exercise}
|
||||
|
||||
\begin{solution}
|
||||
\begin{enumerate}
|
||||
\item
|
||||
\begin{tikzpicture}[yscale=1, xscale=0.234375]
|
||||
\tkzInit[xmin=0,xmax=65,xstep=1,
|
||||
ymin=0,ymax=102000,ystep=10000]
|
||||
\tkzGrid
|
||||
\tkzGrid[sub, subxstep=0.2, subystep=2000.0]
|
||||
\tkzAxeXY[up space=0.5,right space=.2]
|
||||
\tkzFct[domain = 0:64, line width=1pt]{6000*(x+16)*exp(-0.0625*x)}
|
||||
%M\tkzFct[domain = 0:64, line width=1pt, blue]{48000}
|
||||
\end{tikzpicture}
|
||||
\item Tracer la droite $y=48000$. C'est l'abscisse de l'intersection entre cette droite et la courbe
|
||||
\item
|
||||
\item
|
||||
|
||||
\end{enumerate}
|
||||
|
||||
\end{solution}
|
||||
|
||||
\end{document}
|
||||
|
||||
%%% Local Variables:
|
||||
%%% mode: latex
|
||||
%%% TeX-master: "master"
|
||||
%%% End:
|
||||
121
TES/DM/DM_20_02/corr_11_DM_20_02.tex
Normal file
121
TES/DM/DM_20_02/corr_11_DM_20_02.tex
Normal file
@@ -0,0 +1,121 @@
|
||||
\documentclass[a4paper,10pt]{article}
|
||||
\usepackage{tasks}
|
||||
\usepackage{myXsim}
|
||||
|
||||
\title{DM 2 -- JOURDAN Alice}
|
||||
\tribe{Terminale ES-L}
|
||||
\date{9 mars 2020}
|
||||
|
||||
\xsimsetup{
|
||||
solution/print = true
|
||||
}
|
||||
|
||||
\begin{document}
|
||||
\maketitle
|
||||
|
||||
\begin{exercise}[subtitle={Étude de fonction}]
|
||||
On considère la fonction dérivable $f$ définie sur $I = [0~;~32]$ par:
|
||||
|
||||
\[f(x) = \np{3000}(x + 8)\text{e}^{- 0.125x}.\]
|
||||
|
||||
\textbf{Partie A - Étude graphique}
|
||||
|
||||
On a représenté sur le graphique en annexe, la courbe représentative de la fonction $f$.
|
||||
|
||||
\begin{enumerate}
|
||||
\item Avec un tableur tracer et imprimer la courbe représentative de $f$ sur $I$
|
||||
\end{enumerate}
|
||||
\emph{Répondre aux questions suivantes par lecture graphique.}
|
||||
\begin{enumerate}
|
||||
\setcounter{enumi}{1}
|
||||
\item Résoudre graphiquement et de façon approchée l'équation $f(x) = \np{12000}$.
|
||||
\item Donner un encadrement de la quantité
|
||||
\[
|
||||
\int_{2}^{16} f(x) \; dx
|
||||
\]
|
||||
Vous expliquerez votre démarche en utilisant le graphique.
|
||||
\end{enumerate}
|
||||
|
||||
\medskip
|
||||
|
||||
\textbf{Partie B - Étude théorique}
|
||||
|
||||
\begin{enumerate}
|
||||
\setcounter{enumi}{4}
|
||||
\item Étude des variations.
|
||||
\begin{enumerate}
|
||||
\item On note $f'$ la dérivée de la fonction $f$ sur $\intFF{0}{32}$.
|
||||
|
||||
Démontrer que pour tout $x$ de $\intFF{0}{20}$ , $f'(x) = - 375x\text{e}^{-0.125x}$.
|
||||
|
||||
\item En déduire le sens de variation de $f$ et dresser son tableau des variations sur l'intervalle $\intFF{0}{32}$. Si nécessaire, arrondir à l'unité les valeurs présentes dans le tableau.
|
||||
|
||||
\item Démontrer que l'équation $f(x) = \np{12000}$ admet une unique solution $\alpha$ sur $\intFF{0}{32}$, puis donner une valeur approchée de $\alpha$ à $10^{-2}$ près à l'aide de la calculatrice.
|
||||
\end{enumerate}
|
||||
|
||||
\item Étude de la convexité
|
||||
\begin{enumerate}
|
||||
\item On note $f''$ la dérivée seconde de $f$ sur $\intFF{0}{32}$.
|
||||
|
||||
Démontrer que pour tout $x$ de $\intFF{0}{20}$ , $f''(x) = (46.875x-375)\text{e}^{-0.125x}$.
|
||||
|
||||
\item Démontrer que $f$ admet un point d'inflexion dont on donnera son abscisse.
|
||||
\end{enumerate}
|
||||
|
||||
\item Aire sous la courbe
|
||||
\begin{enumerate}
|
||||
\item On souhaite approximer la fonction $f$ sur l'intervalle $I$ par la droite $D$ qui relie les points $(0;f(0))$ et $(32, f(32))$. Tracer cette droite sur le graphique.
|
||||
\item On note $g$ la fonction affine qui décrit cette droite $D$. Détermine l'expression de $g$
|
||||
\item Calculer $\displaystyle \int_0^{32} g(x)\; dx$
|
||||
\item Avec la calculatrice, calculer une valeur approchée de $\displaystyle\int_{0}^{32} f(x) \; dx$
|
||||
\item Comparer les valeurs trouvées aux deux questions précédentes. Comment s'explique l'écart entre ces deux valeurs?
|
||||
\end{enumerate}
|
||||
\end{enumerate}
|
||||
|
||||
\medskip
|
||||
|
||||
\textbf{Partie C - Application économique}
|
||||
|
||||
Une entreprise a pris la décision de fermer son usine de production de smartphones en 32 mois.
|
||||
|
||||
La fonction capacité de production de cette usine est modélisée sur l'intervalle $\intFF{0}{32}$ par la fonction $f$ étudiée dans les parties A et B.
|
||||
|
||||
Le nombre $x$ représente le temps en mois après la décision de la fermeture du site et le nombre $f(x)$ représente capacité production de smartphone au moment $x$.
|
||||
|
||||
Utiliser les résultats de la partie B afin de répondre aux questions suivantes:
|
||||
|
||||
\begin{enumerate}
|
||||
\setcounter{enumi}{7}
|
||||
\item Combien de smartphones pouvaient être produit à la fermeture de l'usine?
|
||||
\item Pendant combien de temps la capacité de production de l'usine a réussi à se maintenir au dessus de \np{12000}?
|
||||
\item Combien de smartphones ont pu être produit entre la prise de décision et la fermeture de l'usine?
|
||||
\end{enumerate}
|
||||
|
||||
\end{exercise}
|
||||
|
||||
\begin{solution}
|
||||
\begin{enumerate}
|
||||
\item
|
||||
\begin{tikzpicture}[yscale=1, xscale=0.46875]
|
||||
\tkzInit[xmin=0,xmax=33,xstep=1,
|
||||
ymin=0,ymax=27000,ystep=2000]
|
||||
\tkzGrid
|
||||
\tkzGrid[sub, subxstep=0.2, subystep=400.0]
|
||||
\tkzAxeXY[up space=0.5,right space=.2]
|
||||
\tkzFct[domain = 0:32, line width=1pt]{3000*(x+8)*exp(-0.125*x)}
|
||||
%M\tkzFct[domain = 0:32, line width=1pt, blue]{12000}
|
||||
\end{tikzpicture}
|
||||
\item Tracer la droite $y=12000$. C'est l'abscisse de l'intersection entre cette droite et la courbe
|
||||
\item
|
||||
\item
|
||||
|
||||
\end{enumerate}
|
||||
|
||||
\end{solution}
|
||||
|
||||
\end{document}
|
||||
|
||||
%%% Local Variables:
|
||||
%%% mode: latex
|
||||
%%% TeX-master: "master"
|
||||
%%% End:
|
||||
121
TES/DM/DM_20_02/corr_12_DM_20_02.tex
Normal file
121
TES/DM/DM_20_02/corr_12_DM_20_02.tex
Normal file
@@ -0,0 +1,121 @@
|
||||
\documentclass[a4paper,10pt]{article}
|
||||
\usepackage{tasks}
|
||||
\usepackage{myXsim}
|
||||
|
||||
\title{DM 2 -- LIANDRAT Léa}
|
||||
\tribe{Terminale ES-L}
|
||||
\date{9 mars 2020}
|
||||
|
||||
\xsimsetup{
|
||||
solution/print = true
|
||||
}
|
||||
|
||||
\begin{document}
|
||||
\maketitle
|
||||
|
||||
\begin{exercise}[subtitle={Étude de fonction}]
|
||||
On considère la fonction dérivable $f$ définie sur $I = [0~;~32]$ par:
|
||||
|
||||
\[f(x) = \np{3000}(x + 8)\text{e}^{- 0.125x}.\]
|
||||
|
||||
\textbf{Partie A - Étude graphique}
|
||||
|
||||
On a représenté sur le graphique en annexe, la courbe représentative de la fonction $f$.
|
||||
|
||||
\begin{enumerate}
|
||||
\item Avec un tableur tracer et imprimer la courbe représentative de $f$ sur $I$
|
||||
\end{enumerate}
|
||||
\emph{Répondre aux questions suivantes par lecture graphique.}
|
||||
\begin{enumerate}
|
||||
\setcounter{enumi}{1}
|
||||
\item Résoudre graphiquement et de façon approchée l'équation $f(x) = \np{12000}$.
|
||||
\item Donner un encadrement de la quantité
|
||||
\[
|
||||
\int_{2}^{16} f(x) \; dx
|
||||
\]
|
||||
Vous expliquerez votre démarche en utilisant le graphique.
|
||||
\end{enumerate}
|
||||
|
||||
\medskip
|
||||
|
||||
\textbf{Partie B - Étude théorique}
|
||||
|
||||
\begin{enumerate}
|
||||
\setcounter{enumi}{4}
|
||||
\item Étude des variations.
|
||||
\begin{enumerate}
|
||||
\item On note $f'$ la dérivée de la fonction $f$ sur $\intFF{0}{32}$.
|
||||
|
||||
Démontrer que pour tout $x$ de $\intFF{0}{20}$ , $f'(x) = - 375x\text{e}^{-0.125x}$.
|
||||
|
||||
\item En déduire le sens de variation de $f$ et dresser son tableau des variations sur l'intervalle $\intFF{0}{32}$. Si nécessaire, arrondir à l'unité les valeurs présentes dans le tableau.
|
||||
|
||||
\item Démontrer que l'équation $f(x) = \np{12000}$ admet une unique solution $\alpha$ sur $\intFF{0}{32}$, puis donner une valeur approchée de $\alpha$ à $10^{-2}$ près à l'aide de la calculatrice.
|
||||
\end{enumerate}
|
||||
|
||||
\item Étude de la convexité
|
||||
\begin{enumerate}
|
||||
\item On note $f''$ la dérivée seconde de $f$ sur $\intFF{0}{32}$.
|
||||
|
||||
Démontrer que pour tout $x$ de $\intFF{0}{20}$ , $f''(x) = (46.875x-375)\text{e}^{-0.125x}$.
|
||||
|
||||
\item Démontrer que $f$ admet un point d'inflexion dont on donnera son abscisse.
|
||||
\end{enumerate}
|
||||
|
||||
\item Aire sous la courbe
|
||||
\begin{enumerate}
|
||||
\item On souhaite approximer la fonction $f$ sur l'intervalle $I$ par la droite $D$ qui relie les points $(0;f(0))$ et $(32, f(32))$. Tracer cette droite sur le graphique.
|
||||
\item On note $g$ la fonction affine qui décrit cette droite $D$. Détermine l'expression de $g$
|
||||
\item Calculer $\displaystyle \int_0^{32} g(x)\; dx$
|
||||
\item Avec la calculatrice, calculer une valeur approchée de $\displaystyle\int_{0}^{32} f(x) \; dx$
|
||||
\item Comparer les valeurs trouvées aux deux questions précédentes. Comment s'explique l'écart entre ces deux valeurs?
|
||||
\end{enumerate}
|
||||
\end{enumerate}
|
||||
|
||||
\medskip
|
||||
|
||||
\textbf{Partie C - Application économique}
|
||||
|
||||
Une entreprise a pris la décision de fermer son usine de production de smartphones en 32 mois.
|
||||
|
||||
La fonction capacité de production de cette usine est modélisée sur l'intervalle $\intFF{0}{32}$ par la fonction $f$ étudiée dans les parties A et B.
|
||||
|
||||
Le nombre $x$ représente le temps en mois après la décision de la fermeture du site et le nombre $f(x)$ représente capacité production de smartphone au moment $x$.
|
||||
|
||||
Utiliser les résultats de la partie B afin de répondre aux questions suivantes:
|
||||
|
||||
\begin{enumerate}
|
||||
\setcounter{enumi}{7}
|
||||
\item Combien de smartphones pouvaient être produit à la fermeture de l'usine?
|
||||
\item Pendant combien de temps la capacité de production de l'usine a réussi à se maintenir au dessus de \np{12000}?
|
||||
\item Combien de smartphones ont pu être produit entre la prise de décision et la fermeture de l'usine?
|
||||
\end{enumerate}
|
||||
|
||||
\end{exercise}
|
||||
|
||||
\begin{solution}
|
||||
\begin{enumerate}
|
||||
\item
|
||||
\begin{tikzpicture}[yscale=1, xscale=0.46875]
|
||||
\tkzInit[xmin=0,xmax=33,xstep=1,
|
||||
ymin=0,ymax=27000,ystep=2000]
|
||||
\tkzGrid
|
||||
\tkzGrid[sub, subxstep=0.2, subystep=400.0]
|
||||
\tkzAxeXY[up space=0.5,right space=.2]
|
||||
\tkzFct[domain = 0:32, line width=1pt]{3000*(x+8)*exp(-0.125*x)}
|
||||
%M\tkzFct[domain = 0:32, line width=1pt, blue]{12000}
|
||||
\end{tikzpicture}
|
||||
\item Tracer la droite $y=12000$. C'est l'abscisse de l'intersection entre cette droite et la courbe
|
||||
\item
|
||||
\item
|
||||
|
||||
\end{enumerate}
|
||||
|
||||
\end{solution}
|
||||
|
||||
\end{document}
|
||||
|
||||
%%% Local Variables:
|
||||
%%% mode: latex
|
||||
%%% TeX-master: "master"
|
||||
%%% End:
|
||||
121
TES/DM/DM_20_02/corr_13_DM_20_02.tex
Normal file
121
TES/DM/DM_20_02/corr_13_DM_20_02.tex
Normal file
@@ -0,0 +1,121 @@
|
||||
\documentclass[a4paper,10pt]{article}
|
||||
\usepackage{tasks}
|
||||
\usepackage{myXsim}
|
||||
|
||||
\title{DM 2 -- LOULID Manar}
|
||||
\tribe{Terminale ES-L}
|
||||
\date{9 mars 2020}
|
||||
|
||||
\xsimsetup{
|
||||
solution/print = true
|
||||
}
|
||||
|
||||
\begin{document}
|
||||
\maketitle
|
||||
|
||||
\begin{exercise}[subtitle={Étude de fonction}]
|
||||
On considère la fonction dérivable $f$ définie sur $I = [0~;~16]$ par:
|
||||
|
||||
\[f(x) = \np{7000}(x + 4)\text{e}^{- 0.25x}.\]
|
||||
|
||||
\textbf{Partie A - Étude graphique}
|
||||
|
||||
On a représenté sur le graphique en annexe, la courbe représentative de la fonction $f$.
|
||||
|
||||
\begin{enumerate}
|
||||
\item Avec un tableur tracer et imprimer la courbe représentative de $f$ sur $I$
|
||||
\end{enumerate}
|
||||
\emph{Répondre aux questions suivantes par lecture graphique.}
|
||||
\begin{enumerate}
|
||||
\setcounter{enumi}{1}
|
||||
\item Résoudre graphiquement et de façon approchée l'équation $f(x) = \np{14000}$.
|
||||
\item Donner un encadrement de la quantité
|
||||
\[
|
||||
\int_{2}^{8} f(x) \; dx
|
||||
\]
|
||||
Vous expliquerez votre démarche en utilisant le graphique.
|
||||
\end{enumerate}
|
||||
|
||||
\medskip
|
||||
|
||||
\textbf{Partie B - Étude théorique}
|
||||
|
||||
\begin{enumerate}
|
||||
\setcounter{enumi}{4}
|
||||
\item Étude des variations.
|
||||
\begin{enumerate}
|
||||
\item On note $f'$ la dérivée de la fonction $f$ sur $\intFF{0}{16}$.
|
||||
|
||||
Démontrer que pour tout $x$ de $\intFF{0}{20}$ , $f'(x) = - 1750x\text{e}^{-0.25x}$.
|
||||
|
||||
\item En déduire le sens de variation de $f$ et dresser son tableau des variations sur l'intervalle $\intFF{0}{16}$. Si nécessaire, arrondir à l'unité les valeurs présentes dans le tableau.
|
||||
|
||||
\item Démontrer que l'équation $f(x) = \np{14000}$ admet une unique solution $\alpha$ sur $\intFF{0}{16}$, puis donner une valeur approchée de $\alpha$ à $10^{-2}$ près à l'aide de la calculatrice.
|
||||
\end{enumerate}
|
||||
|
||||
\item Étude de la convexité
|
||||
\begin{enumerate}
|
||||
\item On note $f''$ la dérivée seconde de $f$ sur $\intFF{0}{16}$.
|
||||
|
||||
Démontrer que pour tout $x$ de $\intFF{0}{20}$ , $f''(x) = (437.50x-1750)\text{e}^{-0.25x}$.
|
||||
|
||||
\item Démontrer que $f$ admet un point d'inflexion dont on donnera son abscisse.
|
||||
\end{enumerate}
|
||||
|
||||
\item Aire sous la courbe
|
||||
\begin{enumerate}
|
||||
\item On souhaite approximer la fonction $f$ sur l'intervalle $I$ par la droite $D$ qui relie les points $(0;f(0))$ et $(16, f(16))$. Tracer cette droite sur le graphique.
|
||||
\item On note $g$ la fonction affine qui décrit cette droite $D$. Détermine l'expression de $g$
|
||||
\item Calculer $\displaystyle \int_0^{16} g(x)\; dx$
|
||||
\item Avec la calculatrice, calculer une valeur approchée de $\displaystyle\int_{0}^{16} f(x) \; dx$
|
||||
\item Comparer les valeurs trouvées aux deux questions précédentes. Comment s'explique l'écart entre ces deux valeurs?
|
||||
\end{enumerate}
|
||||
\end{enumerate}
|
||||
|
||||
\medskip
|
||||
|
||||
\textbf{Partie C - Application économique}
|
||||
|
||||
Une entreprise a pris la décision de fermer son usine de production de smartphones en 16 mois.
|
||||
|
||||
La fonction capacité de production de cette usine est modélisée sur l'intervalle $\intFF{0}{16}$ par la fonction $f$ étudiée dans les parties A et B.
|
||||
|
||||
Le nombre $x$ représente le temps en mois après la décision de la fermeture du site et le nombre $f(x)$ représente capacité production de smartphone au moment $x$.
|
||||
|
||||
Utiliser les résultats de la partie B afin de répondre aux questions suivantes:
|
||||
|
||||
\begin{enumerate}
|
||||
\setcounter{enumi}{7}
|
||||
\item Combien de smartphones pouvaient être produit à la fermeture de l'usine?
|
||||
\item Pendant combien de temps la capacité de production de l'usine a réussi à se maintenir au dessus de \np{14000}?
|
||||
\item Combien de smartphones ont pu être produit entre la prise de décision et la fermeture de l'usine?
|
||||
\end{enumerate}
|
||||
|
||||
\end{exercise}
|
||||
|
||||
\begin{solution}
|
||||
\begin{enumerate}
|
||||
\item
|
||||
\begin{tikzpicture}[yscale=1, xscale=0.9375]
|
||||
\tkzInit[xmin=0,xmax=17,xstep=1,
|
||||
ymin=0,ymax=35000,ystep=3000]
|
||||
\tkzGrid
|
||||
\tkzGrid[sub, subxstep=0.2, subystep=600.0]
|
||||
\tkzAxeXY[up space=0.5,right space=.2]
|
||||
\tkzFct[domain = 0:16, line width=1pt]{7000*(x+4)*exp(-0.25*x)}
|
||||
%M\tkzFct[domain = 0:16, line width=1pt, blue]{14000}
|
||||
\end{tikzpicture}
|
||||
\item Tracer la droite $y=14000$. C'est l'abscisse de l'intersection entre cette droite et la courbe
|
||||
\item
|
||||
\item
|
||||
|
||||
\end{enumerate}
|
||||
|
||||
\end{solution}
|
||||
|
||||
\end{document}
|
||||
|
||||
%%% Local Variables:
|
||||
%%% mode: latex
|
||||
%%% TeX-master: "master"
|
||||
%%% End:
|
||||
121
TES/DM/DM_20_02/corr_14_DM_20_02.tex
Normal file
121
TES/DM/DM_20_02/corr_14_DM_20_02.tex
Normal file
@@ -0,0 +1,121 @@
|
||||
\documentclass[a4paper,10pt]{article}
|
||||
\usepackage{tasks}
|
||||
\usepackage{myXsim}
|
||||
|
||||
\title{DM 2 -- MARQUET Elisa}
|
||||
\tribe{Terminale ES-L}
|
||||
\date{9 mars 2020}
|
||||
|
||||
\xsimsetup{
|
||||
solution/print = true
|
||||
}
|
||||
|
||||
\begin{document}
|
||||
\maketitle
|
||||
|
||||
\begin{exercise}[subtitle={Étude de fonction}]
|
||||
On considère la fonction dérivable $f$ définie sur $I = [0~;~32]$ par:
|
||||
|
||||
\[f(x) = \np{8000}(x + 8)\text{e}^{- 0.125x}.\]
|
||||
|
||||
\textbf{Partie A - Étude graphique}
|
||||
|
||||
On a représenté sur le graphique en annexe, la courbe représentative de la fonction $f$.
|
||||
|
||||
\begin{enumerate}
|
||||
\item Avec un tableur tracer et imprimer la courbe représentative de $f$ sur $I$
|
||||
\end{enumerate}
|
||||
\emph{Répondre aux questions suivantes par lecture graphique.}
|
||||
\begin{enumerate}
|
||||
\setcounter{enumi}{1}
|
||||
\item Résoudre graphiquement et de façon approchée l'équation $f(x) = \np{32000}$.
|
||||
\item Donner un encadrement de la quantité
|
||||
\[
|
||||
\int_{2}^{16} f(x) \; dx
|
||||
\]
|
||||
Vous expliquerez votre démarche en utilisant le graphique.
|
||||
\end{enumerate}
|
||||
|
||||
\medskip
|
||||
|
||||
\textbf{Partie B - Étude théorique}
|
||||
|
||||
\begin{enumerate}
|
||||
\setcounter{enumi}{4}
|
||||
\item Étude des variations.
|
||||
\begin{enumerate}
|
||||
\item On note $f'$ la dérivée de la fonction $f$ sur $\intFF{0}{32}$.
|
||||
|
||||
Démontrer que pour tout $x$ de $\intFF{0}{20}$ , $f'(x) = - 1000x\text{e}^{-0.125x}$.
|
||||
|
||||
\item En déduire le sens de variation de $f$ et dresser son tableau des variations sur l'intervalle $\intFF{0}{32}$. Si nécessaire, arrondir à l'unité les valeurs présentes dans le tableau.
|
||||
|
||||
\item Démontrer que l'équation $f(x) = \np{32000}$ admet une unique solution $\alpha$ sur $\intFF{0}{32}$, puis donner une valeur approchée de $\alpha$ à $10^{-2}$ près à l'aide de la calculatrice.
|
||||
\end{enumerate}
|
||||
|
||||
\item Étude de la convexité
|
||||
\begin{enumerate}
|
||||
\item On note $f''$ la dérivée seconde de $f$ sur $\intFF{0}{32}$.
|
||||
|
||||
Démontrer que pour tout $x$ de $\intFF{0}{20}$ , $f''(x) = (125x-1000)\text{e}^{-0.125x}$.
|
||||
|
||||
\item Démontrer que $f$ admet un point d'inflexion dont on donnera son abscisse.
|
||||
\end{enumerate}
|
||||
|
||||
\item Aire sous la courbe
|
||||
\begin{enumerate}
|
||||
\item On souhaite approximer la fonction $f$ sur l'intervalle $I$ par la droite $D$ qui relie les points $(0;f(0))$ et $(32, f(32))$. Tracer cette droite sur le graphique.
|
||||
\item On note $g$ la fonction affine qui décrit cette droite $D$. Détermine l'expression de $g$
|
||||
\item Calculer $\displaystyle \int_0^{32} g(x)\; dx$
|
||||
\item Avec la calculatrice, calculer une valeur approchée de $\displaystyle\int_{0}^{32} f(x) \; dx$
|
||||
\item Comparer les valeurs trouvées aux deux questions précédentes. Comment s'explique l'écart entre ces deux valeurs?
|
||||
\end{enumerate}
|
||||
\end{enumerate}
|
||||
|
||||
\medskip
|
||||
|
||||
\textbf{Partie C - Application économique}
|
||||
|
||||
Une entreprise a pris la décision de fermer son usine de production de smartphones en 32 mois.
|
||||
|
||||
La fonction capacité de production de cette usine est modélisée sur l'intervalle $\intFF{0}{32}$ par la fonction $f$ étudiée dans les parties A et B.
|
||||
|
||||
Le nombre $x$ représente le temps en mois après la décision de la fermeture du site et le nombre $f(x)$ représente capacité production de smartphone au moment $x$.
|
||||
|
||||
Utiliser les résultats de la partie B afin de répondre aux questions suivantes:
|
||||
|
||||
\begin{enumerate}
|
||||
\setcounter{enumi}{7}
|
||||
\item Combien de smartphones pouvaient être produit à la fermeture de l'usine?
|
||||
\item Pendant combien de temps la capacité de production de l'usine a réussi à se maintenir au dessus de \np{32000}?
|
||||
\item Combien de smartphones ont pu être produit entre la prise de décision et la fermeture de l'usine?
|
||||
\end{enumerate}
|
||||
|
||||
\end{exercise}
|
||||
|
||||
\begin{solution}
|
||||
\begin{enumerate}
|
||||
\item
|
||||
\begin{tikzpicture}[yscale=1, xscale=0.46875]
|
||||
\tkzInit[xmin=0,xmax=33,xstep=1,
|
||||
ymin=0,ymax=72000,ystep=7000]
|
||||
\tkzGrid
|
||||
\tkzGrid[sub, subxstep=0.2, subystep=1400.0]
|
||||
\tkzAxeXY[up space=0.5,right space=.2]
|
||||
\tkzFct[domain = 0:32, line width=1pt]{8000*(x+8)*exp(-0.125*x)}
|
||||
%M\tkzFct[domain = 0:32, line width=1pt, blue]{32000}
|
||||
\end{tikzpicture}
|
||||
\item Tracer la droite $y=32000$. C'est l'abscisse de l'intersection entre cette droite et la courbe
|
||||
\item
|
||||
\item
|
||||
|
||||
\end{enumerate}
|
||||
|
||||
\end{solution}
|
||||
|
||||
\end{document}
|
||||
|
||||
%%% Local Variables:
|
||||
%%% mode: latex
|
||||
%%% TeX-master: "master"
|
||||
%%% End:
|
||||
121
TES/DM/DM_20_02/corr_15_DM_20_02.tex
Normal file
121
TES/DM/DM_20_02/corr_15_DM_20_02.tex
Normal file
@@ -0,0 +1,121 @@
|
||||
\documentclass[a4paper,10pt]{article}
|
||||
\usepackage{tasks}
|
||||
\usepackage{myXsim}
|
||||
|
||||
\title{DM 2 -- MENARD Cassandre}
|
||||
\tribe{Terminale ES-L}
|
||||
\date{9 mars 2020}
|
||||
|
||||
\xsimsetup{
|
||||
solution/print = true
|
||||
}
|
||||
|
||||
\begin{document}
|
||||
\maketitle
|
||||
|
||||
\begin{exercise}[subtitle={Étude de fonction}]
|
||||
On considère la fonction dérivable $f$ définie sur $I = [0~;~80]$ par:
|
||||
|
||||
\[f(x) = \np{6000}(x + 20)\text{e}^{- 0.05x}.\]
|
||||
|
||||
\textbf{Partie A - Étude graphique}
|
||||
|
||||
On a représenté sur le graphique en annexe, la courbe représentative de la fonction $f$.
|
||||
|
||||
\begin{enumerate}
|
||||
\item Avec un tableur tracer et imprimer la courbe représentative de $f$ sur $I$
|
||||
\end{enumerate}
|
||||
\emph{Répondre aux questions suivantes par lecture graphique.}
|
||||
\begin{enumerate}
|
||||
\setcounter{enumi}{1}
|
||||
\item Résoudre graphiquement et de façon approchée l'équation $f(x) = \np{60000}$.
|
||||
\item Donner un encadrement de la quantité
|
||||
\[
|
||||
\int_{2}^{40} f(x) \; dx
|
||||
\]
|
||||
Vous expliquerez votre démarche en utilisant le graphique.
|
||||
\end{enumerate}
|
||||
|
||||
\medskip
|
||||
|
||||
\textbf{Partie B - Étude théorique}
|
||||
|
||||
\begin{enumerate}
|
||||
\setcounter{enumi}{4}
|
||||
\item Étude des variations.
|
||||
\begin{enumerate}
|
||||
\item On note $f'$ la dérivée de la fonction $f$ sur $\intFF{0}{80}$.
|
||||
|
||||
Démontrer que pour tout $x$ de $\intFF{0}{20}$ , $f'(x) = - 300x\text{e}^{-0.05x}$.
|
||||
|
||||
\item En déduire le sens de variation de $f$ et dresser son tableau des variations sur l'intervalle $\intFF{0}{80}$. Si nécessaire, arrondir à l'unité les valeurs présentes dans le tableau.
|
||||
|
||||
\item Démontrer que l'équation $f(x) = \np{60000}$ admet une unique solution $\alpha$ sur $\intFF{0}{80}$, puis donner une valeur approchée de $\alpha$ à $10^{-2}$ près à l'aide de la calculatrice.
|
||||
\end{enumerate}
|
||||
|
||||
\item Étude de la convexité
|
||||
\begin{enumerate}
|
||||
\item On note $f''$ la dérivée seconde de $f$ sur $\intFF{0}{80}$.
|
||||
|
||||
Démontrer que pour tout $x$ de $\intFF{0}{20}$ , $f''(x) = (15x-300)\text{e}^{-0.05x}$.
|
||||
|
||||
\item Démontrer que $f$ admet un point d'inflexion dont on donnera son abscisse.
|
||||
\end{enumerate}
|
||||
|
||||
\item Aire sous la courbe
|
||||
\begin{enumerate}
|
||||
\item On souhaite approximer la fonction $f$ sur l'intervalle $I$ par la droite $D$ qui relie les points $(0;f(0))$ et $(80, f(80))$. Tracer cette droite sur le graphique.
|
||||
\item On note $g$ la fonction affine qui décrit cette droite $D$. Détermine l'expression de $g$
|
||||
\item Calculer $\displaystyle \int_0^{80} g(x)\; dx$
|
||||
\item Avec la calculatrice, calculer une valeur approchée de $\displaystyle\int_{0}^{80} f(x) \; dx$
|
||||
\item Comparer les valeurs trouvées aux deux questions précédentes. Comment s'explique l'écart entre ces deux valeurs?
|
||||
\end{enumerate}
|
||||
\end{enumerate}
|
||||
|
||||
\medskip
|
||||
|
||||
\textbf{Partie C - Application économique}
|
||||
|
||||
Une entreprise a pris la décision de fermer son usine de production de smartphones en 80 mois.
|
||||
|
||||
La fonction capacité de production de cette usine est modélisée sur l'intervalle $\intFF{0}{80}$ par la fonction $f$ étudiée dans les parties A et B.
|
||||
|
||||
Le nombre $x$ représente le temps en mois après la décision de la fermeture du site et le nombre $f(x)$ représente capacité production de smartphone au moment $x$.
|
||||
|
||||
Utiliser les résultats de la partie B afin de répondre aux questions suivantes:
|
||||
|
||||
\begin{enumerate}
|
||||
\setcounter{enumi}{7}
|
||||
\item Combien de smartphones pouvaient être produit à la fermeture de l'usine?
|
||||
\item Pendant combien de temps la capacité de production de l'usine a réussi à se maintenir au dessus de \np{60000}?
|
||||
\item Combien de smartphones ont pu être produit entre la prise de décision et la fermeture de l'usine?
|
||||
\end{enumerate}
|
||||
|
||||
\end{exercise}
|
||||
|
||||
\begin{solution}
|
||||
\begin{enumerate}
|
||||
\item
|
||||
\begin{tikzpicture}[yscale=1, xscale=0.1875]
|
||||
\tkzInit[xmin=0,xmax=81,xstep=1,
|
||||
ymin=0,ymax=126000,ystep=12000]
|
||||
\tkzGrid
|
||||
\tkzGrid[sub, subxstep=0.2, subystep=2400.0]
|
||||
\tkzAxeXY[up space=0.5,right space=.2]
|
||||
\tkzFct[domain = 0:80, line width=1pt]{6000*(x+20)*exp(-0.05*x)}
|
||||
%M\tkzFct[domain = 0:80, line width=1pt, blue]{60000}
|
||||
\end{tikzpicture}
|
||||
\item Tracer la droite $y=60000$. C'est l'abscisse de l'intersection entre cette droite et la courbe
|
||||
\item
|
||||
\item
|
||||
|
||||
\end{enumerate}
|
||||
|
||||
\end{solution}
|
||||
|
||||
\end{document}
|
||||
|
||||
%%% Local Variables:
|
||||
%%% mode: latex
|
||||
%%% TeX-master: "master"
|
||||
%%% End:
|
||||
121
TES/DM/DM_20_02/corr_16_DM_20_02.tex
Normal file
121
TES/DM/DM_20_02/corr_16_DM_20_02.tex
Normal file
@@ -0,0 +1,121 @@
|
||||
\documentclass[a4paper,10pt]{article}
|
||||
\usepackage{tasks}
|
||||
\usepackage{myXsim}
|
||||
|
||||
\title{DM 2 -- MICHEL-PROST Lauryne}
|
||||
\tribe{Terminale ES-L}
|
||||
\date{9 mars 2020}
|
||||
|
||||
\xsimsetup{
|
||||
solution/print = true
|
||||
}
|
||||
|
||||
\begin{document}
|
||||
\maketitle
|
||||
|
||||
\begin{exercise}[subtitle={Étude de fonction}]
|
||||
On considère la fonction dérivable $f$ définie sur $I = [0~;~80]$ par:
|
||||
|
||||
\[f(x) = \np{10000}(x + 20)\text{e}^{- 0.05x}.\]
|
||||
|
||||
\textbf{Partie A - Étude graphique}
|
||||
|
||||
On a représenté sur le graphique en annexe, la courbe représentative de la fonction $f$.
|
||||
|
||||
\begin{enumerate}
|
||||
\item Avec un tableur tracer et imprimer la courbe représentative de $f$ sur $I$
|
||||
\end{enumerate}
|
||||
\emph{Répondre aux questions suivantes par lecture graphique.}
|
||||
\begin{enumerate}
|
||||
\setcounter{enumi}{1}
|
||||
\item Résoudre graphiquement et de façon approchée l'équation $f(x) = \np{100000}$.
|
||||
\item Donner un encadrement de la quantité
|
||||
\[
|
||||
\int_{2}^{40} f(x) \; dx
|
||||
\]
|
||||
Vous expliquerez votre démarche en utilisant le graphique.
|
||||
\end{enumerate}
|
||||
|
||||
\medskip
|
||||
|
||||
\textbf{Partie B - Étude théorique}
|
||||
|
||||
\begin{enumerate}
|
||||
\setcounter{enumi}{4}
|
||||
\item Étude des variations.
|
||||
\begin{enumerate}
|
||||
\item On note $f'$ la dérivée de la fonction $f$ sur $\intFF{0}{80}$.
|
||||
|
||||
Démontrer que pour tout $x$ de $\intFF{0}{20}$ , $f'(x) = - 500x\text{e}^{-0.05x}$.
|
||||
|
||||
\item En déduire le sens de variation de $f$ et dresser son tableau des variations sur l'intervalle $\intFF{0}{80}$. Si nécessaire, arrondir à l'unité les valeurs présentes dans le tableau.
|
||||
|
||||
\item Démontrer que l'équation $f(x) = \np{100000}$ admet une unique solution $\alpha$ sur $\intFF{0}{80}$, puis donner une valeur approchée de $\alpha$ à $10^{-2}$ près à l'aide de la calculatrice.
|
||||
\end{enumerate}
|
||||
|
||||
\item Étude de la convexité
|
||||
\begin{enumerate}
|
||||
\item On note $f''$ la dérivée seconde de $f$ sur $\intFF{0}{80}$.
|
||||
|
||||
Démontrer que pour tout $x$ de $\intFF{0}{20}$ , $f''(x) = (25x-500)\text{e}^{-0.05x}$.
|
||||
|
||||
\item Démontrer que $f$ admet un point d'inflexion dont on donnera son abscisse.
|
||||
\end{enumerate}
|
||||
|
||||
\item Aire sous la courbe
|
||||
\begin{enumerate}
|
||||
\item On souhaite approximer la fonction $f$ sur l'intervalle $I$ par la droite $D$ qui relie les points $(0;f(0))$ et $(80, f(80))$. Tracer cette droite sur le graphique.
|
||||
\item On note $g$ la fonction affine qui décrit cette droite $D$. Détermine l'expression de $g$
|
||||
\item Calculer $\displaystyle \int_0^{80} g(x)\; dx$
|
||||
\item Avec la calculatrice, calculer une valeur approchée de $\displaystyle\int_{0}^{80} f(x) \; dx$
|
||||
\item Comparer les valeurs trouvées aux deux questions précédentes. Comment s'explique l'écart entre ces deux valeurs?
|
||||
\end{enumerate}
|
||||
\end{enumerate}
|
||||
|
||||
\medskip
|
||||
|
||||
\textbf{Partie C - Application économique}
|
||||
|
||||
Une entreprise a pris la décision de fermer son usine de production de smartphones en 80 mois.
|
||||
|
||||
La fonction capacité de production de cette usine est modélisée sur l'intervalle $\intFF{0}{80}$ par la fonction $f$ étudiée dans les parties A et B.
|
||||
|
||||
Le nombre $x$ représente le temps en mois après la décision de la fermeture du site et le nombre $f(x)$ représente capacité production de smartphone au moment $x$.
|
||||
|
||||
Utiliser les résultats de la partie B afin de répondre aux questions suivantes:
|
||||
|
||||
\begin{enumerate}
|
||||
\setcounter{enumi}{7}
|
||||
\item Combien de smartphones pouvaient être produit à la fermeture de l'usine?
|
||||
\item Pendant combien de temps la capacité de production de l'usine a réussi à se maintenir au dessus de \np{100000}?
|
||||
\item Combien de smartphones ont pu être produit entre la prise de décision et la fermeture de l'usine?
|
||||
\end{enumerate}
|
||||
|
||||
\end{exercise}
|
||||
|
||||
\begin{solution}
|
||||
\begin{enumerate}
|
||||
\item
|
||||
\begin{tikzpicture}[yscale=1, xscale=0.1875]
|
||||
\tkzInit[xmin=0,xmax=81,xstep=1,
|
||||
ymin=0,ymax=210000,ystep=21000]
|
||||
\tkzGrid
|
||||
\tkzGrid[sub, subxstep=0.2, subystep=4200.0]
|
||||
\tkzAxeXY[up space=0.5,right space=.2]
|
||||
\tkzFct[domain = 0:80, line width=1pt]{10000*(x+20)*exp(-0.05*x)}
|
||||
%M\tkzFct[domain = 0:80, line width=1pt, blue]{100000}
|
||||
\end{tikzpicture}
|
||||
\item Tracer la droite $y=100000$. C'est l'abscisse de l'intersection entre cette droite et la courbe
|
||||
\item
|
||||
\item
|
||||
|
||||
\end{enumerate}
|
||||
|
||||
\end{solution}
|
||||
|
||||
\end{document}
|
||||
|
||||
%%% Local Variables:
|
||||
%%% mode: latex
|
||||
%%% TeX-master: "master"
|
||||
%%% End:
|
||||
121
TES/DM/DM_20_02/corr_17_DM_20_02.tex
Normal file
121
TES/DM/DM_20_02/corr_17_DM_20_02.tex
Normal file
@@ -0,0 +1,121 @@
|
||||
\documentclass[a4paper,10pt]{article}
|
||||
\usepackage{tasks}
|
||||
\usepackage{myXsim}
|
||||
|
||||
\title{DM 2 -- MOUBARIK Ines}
|
||||
\tribe{Terminale ES-L}
|
||||
\date{9 mars 2020}
|
||||
|
||||
\xsimsetup{
|
||||
solution/print = true
|
||||
}
|
||||
|
||||
\begin{document}
|
||||
\maketitle
|
||||
|
||||
\begin{exercise}[subtitle={Étude de fonction}]
|
||||
On considère la fonction dérivable $f$ définie sur $I = [0~;~32]$ par:
|
||||
|
||||
\[f(x) = \np{3000}(x + 8)\text{e}^{- 0.125x}.\]
|
||||
|
||||
\textbf{Partie A - Étude graphique}
|
||||
|
||||
On a représenté sur le graphique en annexe, la courbe représentative de la fonction $f$.
|
||||
|
||||
\begin{enumerate}
|
||||
\item Avec un tableur tracer et imprimer la courbe représentative de $f$ sur $I$
|
||||
\end{enumerate}
|
||||
\emph{Répondre aux questions suivantes par lecture graphique.}
|
||||
\begin{enumerate}
|
||||
\setcounter{enumi}{1}
|
||||
\item Résoudre graphiquement et de façon approchée l'équation $f(x) = \np{12000}$.
|
||||
\item Donner un encadrement de la quantité
|
||||
\[
|
||||
\int_{2}^{16} f(x) \; dx
|
||||
\]
|
||||
Vous expliquerez votre démarche en utilisant le graphique.
|
||||
\end{enumerate}
|
||||
|
||||
\medskip
|
||||
|
||||
\textbf{Partie B - Étude théorique}
|
||||
|
||||
\begin{enumerate}
|
||||
\setcounter{enumi}{4}
|
||||
\item Étude des variations.
|
||||
\begin{enumerate}
|
||||
\item On note $f'$ la dérivée de la fonction $f$ sur $\intFF{0}{32}$.
|
||||
|
||||
Démontrer que pour tout $x$ de $\intFF{0}{20}$ , $f'(x) = - 375x\text{e}^{-0.125x}$.
|
||||
|
||||
\item En déduire le sens de variation de $f$ et dresser son tableau des variations sur l'intervalle $\intFF{0}{32}$. Si nécessaire, arrondir à l'unité les valeurs présentes dans le tableau.
|
||||
|
||||
\item Démontrer que l'équation $f(x) = \np{12000}$ admet une unique solution $\alpha$ sur $\intFF{0}{32}$, puis donner une valeur approchée de $\alpha$ à $10^{-2}$ près à l'aide de la calculatrice.
|
||||
\end{enumerate}
|
||||
|
||||
\item Étude de la convexité
|
||||
\begin{enumerate}
|
||||
\item On note $f''$ la dérivée seconde de $f$ sur $\intFF{0}{32}$.
|
||||
|
||||
Démontrer que pour tout $x$ de $\intFF{0}{20}$ , $f''(x) = (46.875x-375)\text{e}^{-0.125x}$.
|
||||
|
||||
\item Démontrer que $f$ admet un point d'inflexion dont on donnera son abscisse.
|
||||
\end{enumerate}
|
||||
|
||||
\item Aire sous la courbe
|
||||
\begin{enumerate}
|
||||
\item On souhaite approximer la fonction $f$ sur l'intervalle $I$ par la droite $D$ qui relie les points $(0;f(0))$ et $(32, f(32))$. Tracer cette droite sur le graphique.
|
||||
\item On note $g$ la fonction affine qui décrit cette droite $D$. Détermine l'expression de $g$
|
||||
\item Calculer $\displaystyle \int_0^{32} g(x)\; dx$
|
||||
\item Avec la calculatrice, calculer une valeur approchée de $\displaystyle\int_{0}^{32} f(x) \; dx$
|
||||
\item Comparer les valeurs trouvées aux deux questions précédentes. Comment s'explique l'écart entre ces deux valeurs?
|
||||
\end{enumerate}
|
||||
\end{enumerate}
|
||||
|
||||
\medskip
|
||||
|
||||
\textbf{Partie C - Application économique}
|
||||
|
||||
Une entreprise a pris la décision de fermer son usine de production de smartphones en 32 mois.
|
||||
|
||||
La fonction capacité de production de cette usine est modélisée sur l'intervalle $\intFF{0}{32}$ par la fonction $f$ étudiée dans les parties A et B.
|
||||
|
||||
Le nombre $x$ représente le temps en mois après la décision de la fermeture du site et le nombre $f(x)$ représente capacité production de smartphone au moment $x$.
|
||||
|
||||
Utiliser les résultats de la partie B afin de répondre aux questions suivantes:
|
||||
|
||||
\begin{enumerate}
|
||||
\setcounter{enumi}{7}
|
||||
\item Combien de smartphones pouvaient être produit à la fermeture de l'usine?
|
||||
\item Pendant combien de temps la capacité de production de l'usine a réussi à se maintenir au dessus de \np{12000}?
|
||||
\item Combien de smartphones ont pu être produit entre la prise de décision et la fermeture de l'usine?
|
||||
\end{enumerate}
|
||||
|
||||
\end{exercise}
|
||||
|
||||
\begin{solution}
|
||||
\begin{enumerate}
|
||||
\item
|
||||
\begin{tikzpicture}[yscale=1, xscale=0.46875]
|
||||
\tkzInit[xmin=0,xmax=33,xstep=1,
|
||||
ymin=0,ymax=27000,ystep=2000]
|
||||
\tkzGrid
|
||||
\tkzGrid[sub, subxstep=0.2, subystep=400.0]
|
||||
\tkzAxeXY[up space=0.5,right space=.2]
|
||||
\tkzFct[domain = 0:32, line width=1pt]{3000*(x+8)*exp(-0.125*x)}
|
||||
%M\tkzFct[domain = 0:32, line width=1pt, blue]{12000}
|
||||
\end{tikzpicture}
|
||||
\item Tracer la droite $y=12000$. C'est l'abscisse de l'intersection entre cette droite et la courbe
|
||||
\item
|
||||
\item
|
||||
|
||||
\end{enumerate}
|
||||
|
||||
\end{solution}
|
||||
|
||||
\end{document}
|
||||
|
||||
%%% Local Variables:
|
||||
%%% mode: latex
|
||||
%%% TeX-master: "master"
|
||||
%%% End:
|
||||
121
TES/DM/DM_20_02/corr_18_DM_20_02.tex
Normal file
121
TES/DM/DM_20_02/corr_18_DM_20_02.tex
Normal file
@@ -0,0 +1,121 @@
|
||||
\documentclass[a4paper,10pt]{article}
|
||||
\usepackage{tasks}
|
||||
\usepackage{myXsim}
|
||||
|
||||
\title{DM 2 -- MOUBARIK Sarah}
|
||||
\tribe{Terminale ES-L}
|
||||
\date{9 mars 2020}
|
||||
|
||||
\xsimsetup{
|
||||
solution/print = true
|
||||
}
|
||||
|
||||
\begin{document}
|
||||
\maketitle
|
||||
|
||||
\begin{exercise}[subtitle={Étude de fonction}]
|
||||
On considère la fonction dérivable $f$ définie sur $I = [0~;~32]$ par:
|
||||
|
||||
\[f(x) = \np{2000}(x + 8)\text{e}^{- 0.125x}.\]
|
||||
|
||||
\textbf{Partie A - Étude graphique}
|
||||
|
||||
On a représenté sur le graphique en annexe, la courbe représentative de la fonction $f$.
|
||||
|
||||
\begin{enumerate}
|
||||
\item Avec un tableur tracer et imprimer la courbe représentative de $f$ sur $I$
|
||||
\end{enumerate}
|
||||
\emph{Répondre aux questions suivantes par lecture graphique.}
|
||||
\begin{enumerate}
|
||||
\setcounter{enumi}{1}
|
||||
\item Résoudre graphiquement et de façon approchée l'équation $f(x) = \np{8000}$.
|
||||
\item Donner un encadrement de la quantité
|
||||
\[
|
||||
\int_{2}^{16} f(x) \; dx
|
||||
\]
|
||||
Vous expliquerez votre démarche en utilisant le graphique.
|
||||
\end{enumerate}
|
||||
|
||||
\medskip
|
||||
|
||||
\textbf{Partie B - Étude théorique}
|
||||
|
||||
\begin{enumerate}
|
||||
\setcounter{enumi}{4}
|
||||
\item Étude des variations.
|
||||
\begin{enumerate}
|
||||
\item On note $f'$ la dérivée de la fonction $f$ sur $\intFF{0}{32}$.
|
||||
|
||||
Démontrer que pour tout $x$ de $\intFF{0}{20}$ , $f'(x) = - 250x\text{e}^{-0.125x}$.
|
||||
|
||||
\item En déduire le sens de variation de $f$ et dresser son tableau des variations sur l'intervalle $\intFF{0}{32}$. Si nécessaire, arrondir à l'unité les valeurs présentes dans le tableau.
|
||||
|
||||
\item Démontrer que l'équation $f(x) = \np{8000}$ admet une unique solution $\alpha$ sur $\intFF{0}{32}$, puis donner une valeur approchée de $\alpha$ à $10^{-2}$ près à l'aide de la calculatrice.
|
||||
\end{enumerate}
|
||||
|
||||
\item Étude de la convexité
|
||||
\begin{enumerate}
|
||||
\item On note $f''$ la dérivée seconde de $f$ sur $\intFF{0}{32}$.
|
||||
|
||||
Démontrer que pour tout $x$ de $\intFF{0}{20}$ , $f''(x) = (31.250x-250)\text{e}^{-0.125x}$.
|
||||
|
||||
\item Démontrer que $f$ admet un point d'inflexion dont on donnera son abscisse.
|
||||
\end{enumerate}
|
||||
|
||||
\item Aire sous la courbe
|
||||
\begin{enumerate}
|
||||
\item On souhaite approximer la fonction $f$ sur l'intervalle $I$ par la droite $D$ qui relie les points $(0;f(0))$ et $(32, f(32))$. Tracer cette droite sur le graphique.
|
||||
\item On note $g$ la fonction affine qui décrit cette droite $D$. Détermine l'expression de $g$
|
||||
\item Calculer $\displaystyle \int_0^{32} g(x)\; dx$
|
||||
\item Avec la calculatrice, calculer une valeur approchée de $\displaystyle\int_{0}^{32} f(x) \; dx$
|
||||
\item Comparer les valeurs trouvées aux deux questions précédentes. Comment s'explique l'écart entre ces deux valeurs?
|
||||
\end{enumerate}
|
||||
\end{enumerate}
|
||||
|
||||
\medskip
|
||||
|
||||
\textbf{Partie C - Application économique}
|
||||
|
||||
Une entreprise a pris la décision de fermer son usine de production de smartphones en 32 mois.
|
||||
|
||||
La fonction capacité de production de cette usine est modélisée sur l'intervalle $\intFF{0}{32}$ par la fonction $f$ étudiée dans les parties A et B.
|
||||
|
||||
Le nombre $x$ représente le temps en mois après la décision de la fermeture du site et le nombre $f(x)$ représente capacité production de smartphone au moment $x$.
|
||||
|
||||
Utiliser les résultats de la partie B afin de répondre aux questions suivantes:
|
||||
|
||||
\begin{enumerate}
|
||||
\setcounter{enumi}{7}
|
||||
\item Combien de smartphones pouvaient être produit à la fermeture de l'usine?
|
||||
\item Pendant combien de temps la capacité de production de l'usine a réussi à se maintenir au dessus de \np{8000}?
|
||||
\item Combien de smartphones ont pu être produit entre la prise de décision et la fermeture de l'usine?
|
||||
\end{enumerate}
|
||||
|
||||
\end{exercise}
|
||||
|
||||
\begin{solution}
|
||||
\begin{enumerate}
|
||||
\item
|
||||
\begin{tikzpicture}[yscale=1, xscale=0.46875]
|
||||
\tkzInit[xmin=0,xmax=33,xstep=1,
|
||||
ymin=0,ymax=18000,ystep=1000]
|
||||
\tkzGrid
|
||||
\tkzGrid[sub, subxstep=0.2, subystep=200.0]
|
||||
\tkzAxeXY[up space=0.5,right space=.2]
|
||||
\tkzFct[domain = 0:32, line width=1pt]{2000*(x+8)*exp(-0.125*x)}
|
||||
%M\tkzFct[domain = 0:32, line width=1pt, blue]{8000}
|
||||
\end{tikzpicture}
|
||||
\item Tracer la droite $y=8000$. C'est l'abscisse de l'intersection entre cette droite et la courbe
|
||||
\item
|
||||
\item
|
||||
|
||||
\end{enumerate}
|
||||
|
||||
\end{solution}
|
||||
|
||||
\end{document}
|
||||
|
||||
%%% Local Variables:
|
||||
%%% mode: latex
|
||||
%%% TeX-master: "master"
|
||||
%%% End:
|
||||
121
TES/DM/DM_20_02/corr_19_DM_20_02.tex
Normal file
121
TES/DM/DM_20_02/corr_19_DM_20_02.tex
Normal file
@@ -0,0 +1,121 @@
|
||||
\documentclass[a4paper,10pt]{article}
|
||||
\usepackage{tasks}
|
||||
\usepackage{myXsim}
|
||||
|
||||
\title{DM 2 -- PERREARD Noémie}
|
||||
\tribe{Terminale ES-L}
|
||||
\date{9 mars 2020}
|
||||
|
||||
\xsimsetup{
|
||||
solution/print = true
|
||||
}
|
||||
|
||||
\begin{document}
|
||||
\maketitle
|
||||
|
||||
\begin{exercise}[subtitle={Étude de fonction}]
|
||||
On considère la fonction dérivable $f$ définie sur $I = [0~;~32]$ par:
|
||||
|
||||
\[f(x) = \np{10000}(x + 8)\text{e}^{- 0.125x}.\]
|
||||
|
||||
\textbf{Partie A - Étude graphique}
|
||||
|
||||
On a représenté sur le graphique en annexe, la courbe représentative de la fonction $f$.
|
||||
|
||||
\begin{enumerate}
|
||||
\item Avec un tableur tracer et imprimer la courbe représentative de $f$ sur $I$
|
||||
\end{enumerate}
|
||||
\emph{Répondre aux questions suivantes par lecture graphique.}
|
||||
\begin{enumerate}
|
||||
\setcounter{enumi}{1}
|
||||
\item Résoudre graphiquement et de façon approchée l'équation $f(x) = \np{40000}$.
|
||||
\item Donner un encadrement de la quantité
|
||||
\[
|
||||
\int_{2}^{16} f(x) \; dx
|
||||
\]
|
||||
Vous expliquerez votre démarche en utilisant le graphique.
|
||||
\end{enumerate}
|
||||
|
||||
\medskip
|
||||
|
||||
\textbf{Partie B - Étude théorique}
|
||||
|
||||
\begin{enumerate}
|
||||
\setcounter{enumi}{4}
|
||||
\item Étude des variations.
|
||||
\begin{enumerate}
|
||||
\item On note $f'$ la dérivée de la fonction $f$ sur $\intFF{0}{32}$.
|
||||
|
||||
Démontrer que pour tout $x$ de $\intFF{0}{20}$ , $f'(x) = - 1250x\text{e}^{-0.125x}$.
|
||||
|
||||
\item En déduire le sens de variation de $f$ et dresser son tableau des variations sur l'intervalle $\intFF{0}{32}$. Si nécessaire, arrondir à l'unité les valeurs présentes dans le tableau.
|
||||
|
||||
\item Démontrer que l'équation $f(x) = \np{40000}$ admet une unique solution $\alpha$ sur $\intFF{0}{32}$, puis donner une valeur approchée de $\alpha$ à $10^{-2}$ près à l'aide de la calculatrice.
|
||||
\end{enumerate}
|
||||
|
||||
\item Étude de la convexité
|
||||
\begin{enumerate}
|
||||
\item On note $f''$ la dérivée seconde de $f$ sur $\intFF{0}{32}$.
|
||||
|
||||
Démontrer que pour tout $x$ de $\intFF{0}{20}$ , $f''(x) = (156.250x-1250)\text{e}^{-0.125x}$.
|
||||
|
||||
\item Démontrer que $f$ admet un point d'inflexion dont on donnera son abscisse.
|
||||
\end{enumerate}
|
||||
|
||||
\item Aire sous la courbe
|
||||
\begin{enumerate}
|
||||
\item On souhaite approximer la fonction $f$ sur l'intervalle $I$ par la droite $D$ qui relie les points $(0;f(0))$ et $(32, f(32))$. Tracer cette droite sur le graphique.
|
||||
\item On note $g$ la fonction affine qui décrit cette droite $D$. Détermine l'expression de $g$
|
||||
\item Calculer $\displaystyle \int_0^{32} g(x)\; dx$
|
||||
\item Avec la calculatrice, calculer une valeur approchée de $\displaystyle\int_{0}^{32} f(x) \; dx$
|
||||
\item Comparer les valeurs trouvées aux deux questions précédentes. Comment s'explique l'écart entre ces deux valeurs?
|
||||
\end{enumerate}
|
||||
\end{enumerate}
|
||||
|
||||
\medskip
|
||||
|
||||
\textbf{Partie C - Application économique}
|
||||
|
||||
Une entreprise a pris la décision de fermer son usine de production de smartphones en 32 mois.
|
||||
|
||||
La fonction capacité de production de cette usine est modélisée sur l'intervalle $\intFF{0}{32}$ par la fonction $f$ étudiée dans les parties A et B.
|
||||
|
||||
Le nombre $x$ représente le temps en mois après la décision de la fermeture du site et le nombre $f(x)$ représente capacité production de smartphone au moment $x$.
|
||||
|
||||
Utiliser les résultats de la partie B afin de répondre aux questions suivantes:
|
||||
|
||||
\begin{enumerate}
|
||||
\setcounter{enumi}{7}
|
||||
\item Combien de smartphones pouvaient être produit à la fermeture de l'usine?
|
||||
\item Pendant combien de temps la capacité de production de l'usine a réussi à se maintenir au dessus de \np{40000}?
|
||||
\item Combien de smartphones ont pu être produit entre la prise de décision et la fermeture de l'usine?
|
||||
\end{enumerate}
|
||||
|
||||
\end{exercise}
|
||||
|
||||
\begin{solution}
|
||||
\begin{enumerate}
|
||||
\item
|
||||
\begin{tikzpicture}[yscale=1, xscale=0.46875]
|
||||
\tkzInit[xmin=0,xmax=33,xstep=1,
|
||||
ymin=0,ymax=90000,ystep=9000]
|
||||
\tkzGrid
|
||||
\tkzGrid[sub, subxstep=0.2, subystep=1800.0]
|
||||
\tkzAxeXY[up space=0.5,right space=.2]
|
||||
\tkzFct[domain = 0:32, line width=1pt]{10000*(x+8)*exp(-0.125*x)}
|
||||
%M\tkzFct[domain = 0:32, line width=1pt, blue]{40000}
|
||||
\end{tikzpicture}
|
||||
\item Tracer la droite $y=40000$. C'est l'abscisse de l'intersection entre cette droite et la courbe
|
||||
\item
|
||||
\item
|
||||
|
||||
\end{enumerate}
|
||||
|
||||
\end{solution}
|
||||
|
||||
\end{document}
|
||||
|
||||
%%% Local Variables:
|
||||
%%% mode: latex
|
||||
%%% TeX-master: "master"
|
||||
%%% End:
|
||||
121
TES/DM/DM_20_02/corr_20_DM_20_02.tex
Normal file
121
TES/DM/DM_20_02/corr_20_DM_20_02.tex
Normal file
@@ -0,0 +1,121 @@
|
||||
\documentclass[a4paper,10pt]{article}
|
||||
\usepackage{tasks}
|
||||
\usepackage{myXsim}
|
||||
|
||||
\title{DM 2 -- URPIN Flora}
|
||||
\tribe{Terminale ES-L}
|
||||
\date{9 mars 2020}
|
||||
|
||||
\xsimsetup{
|
||||
solution/print = true
|
||||
}
|
||||
|
||||
\begin{document}
|
||||
\maketitle
|
||||
|
||||
\begin{exercise}[subtitle={Étude de fonction}]
|
||||
On considère la fonction dérivable $f$ définie sur $I = [0~;~16]$ par:
|
||||
|
||||
\[f(x) = \np{3000}(x + 4)\text{e}^{- 0.25x}.\]
|
||||
|
||||
\textbf{Partie A - Étude graphique}
|
||||
|
||||
On a représenté sur le graphique en annexe, la courbe représentative de la fonction $f$.
|
||||
|
||||
\begin{enumerate}
|
||||
\item Avec un tableur tracer et imprimer la courbe représentative de $f$ sur $I$
|
||||
\end{enumerate}
|
||||
\emph{Répondre aux questions suivantes par lecture graphique.}
|
||||
\begin{enumerate}
|
||||
\setcounter{enumi}{1}
|
||||
\item Résoudre graphiquement et de façon approchée l'équation $f(x) = \np{6000}$.
|
||||
\item Donner un encadrement de la quantité
|
||||
\[
|
||||
\int_{2}^{8} f(x) \; dx
|
||||
\]
|
||||
Vous expliquerez votre démarche en utilisant le graphique.
|
||||
\end{enumerate}
|
||||
|
||||
\medskip
|
||||
|
||||
\textbf{Partie B - Étude théorique}
|
||||
|
||||
\begin{enumerate}
|
||||
\setcounter{enumi}{4}
|
||||
\item Étude des variations.
|
||||
\begin{enumerate}
|
||||
\item On note $f'$ la dérivée de la fonction $f$ sur $\intFF{0}{16}$.
|
||||
|
||||
Démontrer que pour tout $x$ de $\intFF{0}{20}$ , $f'(x) = - 750x\text{e}^{-0.25x}$.
|
||||
|
||||
\item En déduire le sens de variation de $f$ et dresser son tableau des variations sur l'intervalle $\intFF{0}{16}$. Si nécessaire, arrondir à l'unité les valeurs présentes dans le tableau.
|
||||
|
||||
\item Démontrer que l'équation $f(x) = \np{6000}$ admet une unique solution $\alpha$ sur $\intFF{0}{16}$, puis donner une valeur approchée de $\alpha$ à $10^{-2}$ près à l'aide de la calculatrice.
|
||||
\end{enumerate}
|
||||
|
||||
\item Étude de la convexité
|
||||
\begin{enumerate}
|
||||
\item On note $f''$ la dérivée seconde de $f$ sur $\intFF{0}{16}$.
|
||||
|
||||
Démontrer que pour tout $x$ de $\intFF{0}{20}$ , $f''(x) = (187.50x-750)\text{e}^{-0.25x}$.
|
||||
|
||||
\item Démontrer que $f$ admet un point d'inflexion dont on donnera son abscisse.
|
||||
\end{enumerate}
|
||||
|
||||
\item Aire sous la courbe
|
||||
\begin{enumerate}
|
||||
\item On souhaite approximer la fonction $f$ sur l'intervalle $I$ par la droite $D$ qui relie les points $(0;f(0))$ et $(16, f(16))$. Tracer cette droite sur le graphique.
|
||||
\item On note $g$ la fonction affine qui décrit cette droite $D$. Détermine l'expression de $g$
|
||||
\item Calculer $\displaystyle \int_0^{16} g(x)\; dx$
|
||||
\item Avec la calculatrice, calculer une valeur approchée de $\displaystyle\int_{0}^{16} f(x) \; dx$
|
||||
\item Comparer les valeurs trouvées aux deux questions précédentes. Comment s'explique l'écart entre ces deux valeurs?
|
||||
\end{enumerate}
|
||||
\end{enumerate}
|
||||
|
||||
\medskip
|
||||
|
||||
\textbf{Partie C - Application économique}
|
||||
|
||||
Une entreprise a pris la décision de fermer son usine de production de smartphones en 16 mois.
|
||||
|
||||
La fonction capacité de production de cette usine est modélisée sur l'intervalle $\intFF{0}{16}$ par la fonction $f$ étudiée dans les parties A et B.
|
||||
|
||||
Le nombre $x$ représente le temps en mois après la décision de la fermeture du site et le nombre $f(x)$ représente capacité production de smartphone au moment $x$.
|
||||
|
||||
Utiliser les résultats de la partie B afin de répondre aux questions suivantes:
|
||||
|
||||
\begin{enumerate}
|
||||
\setcounter{enumi}{7}
|
||||
\item Combien de smartphones pouvaient être produit à la fermeture de l'usine?
|
||||
\item Pendant combien de temps la capacité de production de l'usine a réussi à se maintenir au dessus de \np{6000}?
|
||||
\item Combien de smartphones ont pu être produit entre la prise de décision et la fermeture de l'usine?
|
||||
\end{enumerate}
|
||||
|
||||
\end{exercise}
|
||||
|
||||
\begin{solution}
|
||||
\begin{enumerate}
|
||||
\item
|
||||
\begin{tikzpicture}[yscale=1, xscale=0.9375]
|
||||
\tkzInit[xmin=0,xmax=17,xstep=1,
|
||||
ymin=0,ymax=15000,ystep=1000]
|
||||
\tkzGrid
|
||||
\tkzGrid[sub, subxstep=0.2, subystep=200.0]
|
||||
\tkzAxeXY[up space=0.5,right space=.2]
|
||||
\tkzFct[domain = 0:16, line width=1pt]{3000*(x+4)*exp(-0.25*x)}
|
||||
%M\tkzFct[domain = 0:16, line width=1pt, blue]{6000}
|
||||
\end{tikzpicture}
|
||||
\item Tracer la droite $y=6000$. C'est l'abscisse de l'intersection entre cette droite et la courbe
|
||||
\item
|
||||
\item
|
||||
|
||||
\end{enumerate}
|
||||
|
||||
\end{solution}
|
||||
|
||||
\end{document}
|
||||
|
||||
%%% Local Variables:
|
||||
%%% mode: latex
|
||||
%%% TeX-master: "master"
|
||||
%%% End:
|
||||
121
TES/DM/DM_20_02/corr_21_DM_20_02.tex
Normal file
121
TES/DM/DM_20_02/corr_21_DM_20_02.tex
Normal file
@@ -0,0 +1,121 @@
|
||||
\documentclass[a4paper,10pt]{article}
|
||||
\usepackage{tasks}
|
||||
\usepackage{myXsim}
|
||||
|
||||
\title{DM 2 -- VISENTIN Aurélie}
|
||||
\tribe{Terminale ES-L}
|
||||
\date{9 mars 2020}
|
||||
|
||||
\xsimsetup{
|
||||
solution/print = true
|
||||
}
|
||||
|
||||
\begin{document}
|
||||
\maketitle
|
||||
|
||||
\begin{exercise}[subtitle={Étude de fonction}]
|
||||
On considère la fonction dérivable $f$ définie sur $I = [0~;~32]$ par:
|
||||
|
||||
\[f(x) = \np{10000}(x + 8)\text{e}^{- 0.125x}.\]
|
||||
|
||||
\textbf{Partie A - Étude graphique}
|
||||
|
||||
On a représenté sur le graphique en annexe, la courbe représentative de la fonction $f$.
|
||||
|
||||
\begin{enumerate}
|
||||
\item Avec un tableur tracer et imprimer la courbe représentative de $f$ sur $I$
|
||||
\end{enumerate}
|
||||
\emph{Répondre aux questions suivantes par lecture graphique.}
|
||||
\begin{enumerate}
|
||||
\setcounter{enumi}{1}
|
||||
\item Résoudre graphiquement et de façon approchée l'équation $f(x) = \np{40000}$.
|
||||
\item Donner un encadrement de la quantité
|
||||
\[
|
||||
\int_{2}^{16} f(x) \; dx
|
||||
\]
|
||||
Vous expliquerez votre démarche en utilisant le graphique.
|
||||
\end{enumerate}
|
||||
|
||||
\medskip
|
||||
|
||||
\textbf{Partie B - Étude théorique}
|
||||
|
||||
\begin{enumerate}
|
||||
\setcounter{enumi}{4}
|
||||
\item Étude des variations.
|
||||
\begin{enumerate}
|
||||
\item On note $f'$ la dérivée de la fonction $f$ sur $\intFF{0}{32}$.
|
||||
|
||||
Démontrer que pour tout $x$ de $\intFF{0}{20}$ , $f'(x) = - 1250x\text{e}^{-0.125x}$.
|
||||
|
||||
\item En déduire le sens de variation de $f$ et dresser son tableau des variations sur l'intervalle $\intFF{0}{32}$. Si nécessaire, arrondir à l'unité les valeurs présentes dans le tableau.
|
||||
|
||||
\item Démontrer que l'équation $f(x) = \np{40000}$ admet une unique solution $\alpha$ sur $\intFF{0}{32}$, puis donner une valeur approchée de $\alpha$ à $10^{-2}$ près à l'aide de la calculatrice.
|
||||
\end{enumerate}
|
||||
|
||||
\item Étude de la convexité
|
||||
\begin{enumerate}
|
||||
\item On note $f''$ la dérivée seconde de $f$ sur $\intFF{0}{32}$.
|
||||
|
||||
Démontrer que pour tout $x$ de $\intFF{0}{20}$ , $f''(x) = (156.250x-1250)\text{e}^{-0.125x}$.
|
||||
|
||||
\item Démontrer que $f$ admet un point d'inflexion dont on donnera son abscisse.
|
||||
\end{enumerate}
|
||||
|
||||
\item Aire sous la courbe
|
||||
\begin{enumerate}
|
||||
\item On souhaite approximer la fonction $f$ sur l'intervalle $I$ par la droite $D$ qui relie les points $(0;f(0))$ et $(32, f(32))$. Tracer cette droite sur le graphique.
|
||||
\item On note $g$ la fonction affine qui décrit cette droite $D$. Détermine l'expression de $g$
|
||||
\item Calculer $\displaystyle \int_0^{32} g(x)\; dx$
|
||||
\item Avec la calculatrice, calculer une valeur approchée de $\displaystyle\int_{0}^{32} f(x) \; dx$
|
||||
\item Comparer les valeurs trouvées aux deux questions précédentes. Comment s'explique l'écart entre ces deux valeurs?
|
||||
\end{enumerate}
|
||||
\end{enumerate}
|
||||
|
||||
\medskip
|
||||
|
||||
\textbf{Partie C - Application économique}
|
||||
|
||||
Une entreprise a pris la décision de fermer son usine de production de smartphones en 32 mois.
|
||||
|
||||
La fonction capacité de production de cette usine est modélisée sur l'intervalle $\intFF{0}{32}$ par la fonction $f$ étudiée dans les parties A et B.
|
||||
|
||||
Le nombre $x$ représente le temps en mois après la décision de la fermeture du site et le nombre $f(x)$ représente capacité production de smartphone au moment $x$.
|
||||
|
||||
Utiliser les résultats de la partie B afin de répondre aux questions suivantes:
|
||||
|
||||
\begin{enumerate}
|
||||
\setcounter{enumi}{7}
|
||||
\item Combien de smartphones pouvaient être produit à la fermeture de l'usine?
|
||||
\item Pendant combien de temps la capacité de production de l'usine a réussi à se maintenir au dessus de \np{40000}?
|
||||
\item Combien de smartphones ont pu être produit entre la prise de décision et la fermeture de l'usine?
|
||||
\end{enumerate}
|
||||
|
||||
\end{exercise}
|
||||
|
||||
\begin{solution}
|
||||
\begin{enumerate}
|
||||
\item
|
||||
\begin{tikzpicture}[yscale=1, xscale=0.46875]
|
||||
\tkzInit[xmin=0,xmax=33,xstep=1,
|
||||
ymin=0,ymax=90000,ystep=9000]
|
||||
\tkzGrid
|
||||
\tkzGrid[sub, subxstep=0.2, subystep=1800.0]
|
||||
\tkzAxeXY[up space=0.5,right space=.2]
|
||||
\tkzFct[domain = 0:32, line width=1pt]{10000*(x+8)*exp(-0.125*x)}
|
||||
%M\tkzFct[domain = 0:32, line width=1pt, blue]{40000}
|
||||
\end{tikzpicture}
|
||||
\item Tracer la droite $y=40000$. C'est l'abscisse de l'intersection entre cette droite et la courbe
|
||||
\item
|
||||
\item
|
||||
|
||||
\end{enumerate}
|
||||
|
||||
\end{solution}
|
||||
|
||||
\end{document}
|
||||
|
||||
%%% Local Variables:
|
||||
%%% mode: latex
|
||||
%%% TeX-master: "master"
|
||||
%%% End:
|
||||
BIN
TES/DM/DM_20_02/corr_all_DM_20_02.pdf
Normal file
BIN
TES/DM/DM_20_02/corr_all_DM_20_02.pdf
Normal file
Binary file not shown.
24
TES/DM/DM_20_02/tpl_DM_20_02.tex
Normal file
24
TES/DM/DM_20_02/tpl_DM_20_02.tex
Normal file
@@ -0,0 +1,24 @@
|
||||
\documentclass[a4paper,10pt]{article}
|
||||
\usepackage{tasks}
|
||||
\usepackage{myXsim}
|
||||
|
||||
\title{DM 2 -- \Var{Nom}}
|
||||
\tribe{Terminale ES-L}
|
||||
\date{9 mars 2020}
|
||||
|
||||
\xsimsetup{
|
||||
solution/print = false
|
||||
}
|
||||
|
||||
\begin{document}
|
||||
\maketitle
|
||||
|
||||
\Block{include "./tpl_fonction.tex"}
|
||||
|
||||
\end{document}
|
||||
|
||||
%%% Local Variables:
|
||||
%%% mode: latex
|
||||
%%% TeX-master: "master"
|
||||
%%% End:
|
||||
|
||||
106
TES/DM/DM_20_02/tpl_fonction.tex
Normal file
106
TES/DM/DM_20_02/tpl_fonction.tex
Normal file
@@ -0,0 +1,106 @@
|
||||
%- set a = Integer.random(min_value=1,max_value=10) * 1000
|
||||
%- set b = Integer.random(min_value=1, max_value=5, rejected = [3]) * 4
|
||||
%- set c = (1 / b).decimal
|
||||
%- set borne = 4*b
|
||||
%- set valeq = (a*b/2).decimal
|
||||
\begin{exercise}[subtitle={Étude de fonction}]
|
||||
On considère la fonction dérivable $f$ définie sur $I = [0~;~\Var{borne}]$ par:
|
||||
|
||||
\[f(x) = \np{\Var{a}}(x + \Var{b})\text{e}^{- \Var{c}x}.\]
|
||||
|
||||
\textbf{Partie A - Étude graphique}
|
||||
|
||||
On a représenté sur le graphique en annexe, la courbe représentative de la fonction $f$.
|
||||
|
||||
\begin{enumerate}
|
||||
\item Avec un tableur tracer et imprimer la courbe représentative de $f$ sur $I$
|
||||
\end{enumerate}
|
||||
\emph{Répondre aux questions suivantes par lecture graphique.}
|
||||
\begin{enumerate}
|
||||
\setcounter{enumi}{1}
|
||||
\item Résoudre graphiquement et de façon approchée l'équation $f(x) = \np{\Var{valeq}}$.
|
||||
\item Donner un encadrement de la quantité
|
||||
\[
|
||||
\int_{2}^{\Var{(borne/2).decimal}} f(x) \; dx
|
||||
\]
|
||||
Vous expliquerez votre démarche en utilisant le graphique.
|
||||
\end{enumerate}
|
||||
|
||||
\medskip
|
||||
|
||||
\textbf{Partie B - Étude théorique}
|
||||
|
||||
\begin{enumerate}
|
||||
\setcounter{enumi}{4}
|
||||
\item Étude des variations.
|
||||
\begin{enumerate}
|
||||
\item On note $f'$ la dérivée de la fonction $f$ sur $\intFF{0}{\Var{borne}}$.
|
||||
|
||||
Démontrer que pour tout $x$ de $\intFF{0}{20}$ , $f'(x) = - \Var{a*c}x\text{e}^{-\Var{c}x}$.
|
||||
|
||||
\item En déduire le sens de variation de $f$ et dresser son tableau des variations sur l'intervalle $\intFF{0}{\Var{borne}}$. Si nécessaire, arrondir à l'unité les valeurs présentes dans le tableau.
|
||||
|
||||
\item Démontrer que l'équation $f(x) = \np{\Var{valeq}}$ admet une unique solution $\alpha$ sur $\intFF{0}{\Var{borne}}$, puis donner une valeur approchée de $\alpha$ à $10^{-2}$ près à l'aide de la calculatrice.
|
||||
\end{enumerate}
|
||||
|
||||
\item Étude de la convexité
|
||||
\begin{enumerate}
|
||||
\item On note $f''$ la dérivée seconde de $f$ sur $\intFF{0}{\Var{borne}}$.
|
||||
|
||||
Démontrer que pour tout $x$ de $\intFF{0}{20}$ , $f''(x) = (\Var{a*c*c}x-\Var{a*c})\text{e}^{-\Var{c}x}$.
|
||||
|
||||
\item Démontrer que $f$ admet un point d'inflexion dont on donnera son abscisse.
|
||||
\end{enumerate}
|
||||
|
||||
\item Aire sous la courbe
|
||||
\begin{enumerate}
|
||||
\item On souhaite approximer la fonction $f$ sur l'intervalle $I$ par la droite $D$ qui relie les points $(0;f(0))$ et $(\Var{borne}, f(\Var{borne}))$. Tracer cette droite sur le graphique.
|
||||
\item On note $g$ la fonction affine qui décrit cette droite $D$. Détermine l'expression de $g$
|
||||
\item Calculer $\displaystyle \int_0^{\Var{borne}} g(x)\; dx$
|
||||
\item Avec la calculatrice, calculer une valeur approchée de $\displaystyle\int_{0}^{\Var{borne}} f(x) \; dx$
|
||||
\item Comparer les valeurs trouvées aux deux questions précédentes. Comment s'explique l'écart entre ces deux valeurs?
|
||||
\end{enumerate}
|
||||
\end{enumerate}
|
||||
|
||||
\medskip
|
||||
|
||||
\textbf{Partie C - Application économique}
|
||||
|
||||
Une entreprise a pris la décision de fermer son usine de production de smartphones en \Var{borne} mois.
|
||||
|
||||
La fonction capacité de production de cette usine est modélisée sur l'intervalle $\intFF{0}{\Var{borne}}$ par la fonction $f$ étudiée dans les parties A et B.
|
||||
|
||||
Le nombre $x$ représente le temps en mois après la décision de la fermeture du site et le nombre $f(x)$ représente capacité production de smartphone au moment $x$.
|
||||
|
||||
Utiliser les résultats de la partie B afin de répondre aux questions suivantes:
|
||||
|
||||
\begin{enumerate}
|
||||
\setcounter{enumi}{7}
|
||||
\item Combien de smartphones pouvaient être produit à la fermeture de l'usine?
|
||||
\item Pendant combien de temps la capacité de production de l'usine a réussi à se maintenir au dessus de \np{\Var{valeq}}?
|
||||
\item Combien de smartphones ont pu être produit entre la prise de décision et la fermeture de l'usine?
|
||||
\end{enumerate}
|
||||
|
||||
\end{exercise}
|
||||
|
||||
\begin{solution}
|
||||
\begin{enumerate}
|
||||
\item
|
||||
%- set ymax = int((a*(b+1)/10).decimal._mo.value/1000)*1000
|
||||
%- set xscale = (15/borne).decimal
|
||||
\begin{tikzpicture}[yscale=1, xscale=\Var{xscale}]
|
||||
\tkzInit[xmin=0,xmax=\Var{borne+1},xstep=1,
|
||||
ymin=0,ymax=\Var{a*(b+1)},ystep=\Var{ymax}]
|
||||
\tkzGrid
|
||||
\tkzGrid[sub, subxstep=0.2, subystep=\Var{ymax/5}]
|
||||
\tkzAxeXY[up space=0.5,right space=.2]
|
||||
\tkzFct[domain = 0:\Var{borne}, line width=1pt]{\Var{a}*(x+\Var{b})*exp(-\Var{c}*x)}
|
||||
%M\tkzFct[domain = 0:\Var{borne}, line width=1pt, blue]{\Var{valeq}}
|
||||
\end{tikzpicture}
|
||||
\item Tracer la droite $y=\Var{valeq}$. C'est l'abscisse de l'intersection entre cette droite et la courbe
|
||||
\item
|
||||
\item
|
||||
|
||||
\end{enumerate}
|
||||
|
||||
\end{solution}
|
||||
BIN
TES/DS/BacBlanc/BacBlanc.pdf
Normal file
BIN
TES/DS/BacBlanc/BacBlanc.pdf
Normal file
Binary file not shown.
437
TES/DS/BacBlanc/BacBlanc.tex
Normal file
437
TES/DS/BacBlanc/BacBlanc.tex
Normal file
@@ -0,0 +1,437 @@
|
||||
\documentclass[a4paper,12pt]{article}
|
||||
\usepackage{myXsim}
|
||||
%\usepackage[inline]{enumitem}
|
||||
%\usepackage{tasks}
|
||||
|
||||
\title{Bac Blanc}
|
||||
\tribe{Terminale L-ES}
|
||||
\date{19 mars 2020}
|
||||
\duree{3h}
|
||||
|
||||
\xsimsetup{
|
||||
solution/print = false
|
||||
}
|
||||
|
||||
\begin{document}
|
||||
\titlepage
|
||||
|
||||
\begin{exercise}[subtitle={QCM}, points=4]
|
||||
\textbf{Commun à tous les candidats}
|
||||
\emph{Pour chaque question, une seule des propositions est exacte. Une réponse exacte rapporte un point. Une réponse fausse, plusieurs réponses ou l'absence de réponse n'ajoutent ni ne retirent aucun point.\\
|
||||
Inscrire sur la copie la référence de la question et la lettre de la réponse choisie.\\
|
||||
Aucune justification n'est demandée.}
|
||||
\begin{enumerate}
|
||||
\item On donne ci-dessous la courbe $\mathcal{C}$ représentative d'une fonction $f$ définie et dérivable sur $\intFO{0}{+\infty}$. On pose
|
||||
|
||||
\begin{minipage}{0.5\textwidth}
|
||||
On pose
|
||||
\[
|
||||
I = \int_{1}^{3} f(x)dx
|
||||
\]
|
||||
Un encadrement de $I$ est
|
||||
\begin{enumerate}
|
||||
\item $1 \leq I \leq 3$
|
||||
\item $2 \leq I \leq 4$
|
||||
\item $5 \leq I \leq 7$
|
||||
\end{enumerate}
|
||||
\end{minipage}
|
||||
\begin{minipage}{0.5\textwidth}
|
||||
\begin{tikzpicture}[scale=1]
|
||||
\tkzInit[xmin=-0.1,xmax=5,ymax=5]
|
||||
\tkzGrid
|
||||
\tkzAxeXY
|
||||
\tkzFct[color=red, line width=1pt]{4*x**2/(x**2+1)}
|
||||
\end{tikzpicture}
|
||||
\end{minipage}
|
||||
\item On considère la suite géométrique de premier terme 1 et de raison 2.
|
||||
La somme des 13 premiers termes de cette suite vaut :
|
||||
\begin{multicols}{3}
|
||||
\begin{enumerate}
|
||||
\item $\np{4095}$
|
||||
\item $\np{8191}$
|
||||
\item $\dfrac{1 -2^{14}}{1 - 2}$
|
||||
\end{enumerate}
|
||||
\end{multicols}
|
||||
\item On a tracé ci-dessous la représentation graphique de la dérivée seconde $k''$ d'une fonction $k$
|
||||
définie sur $[0~;~+ \infty[$.
|
||||
|
||||
\begin{center}
|
||||
%\def\f{x^2*(x^2-4)/3}
|
||||
\begin{tikzpicture}[yscale=1, xscale=3]
|
||||
\tkzInit[xmin=0,xmax=3,xstep=1,
|
||||
ymin=-2,ymax=3,ystep=1]
|
||||
\tkzGrid
|
||||
\tkzAxeXY[up space=0.5,right space=.2]
|
||||
\tkzFct[domain=0:3, line width=1pt]{x*x*(x*x-4)/3}
|
||||
%\draw (2,3) node [above left] {$\Delta$};
|
||||
\end{tikzpicture}
|
||||
\end{center}
|
||||
|
||||
\begin{multicols}{2}
|
||||
\begin{enumerate}
|
||||
\item $k$ est concave sur l'intervalle [1~;~2]
|
||||
\item $k$ est convexe sur l'intervalle [0~;~2]
|
||||
\item $k$ est convexe sur $[0~;~+ \infty[$
|
||||
\item $k$ est concave sur $[0~;~+ \infty[$
|
||||
\end{enumerate}
|
||||
\end{multicols}
|
||||
\item Le prix d’un produit est passé de 200\euro\; à 100 \euro.
|
||||
Cette évolution correspond à deux baisses successives et identiques d’environ :
|
||||
\begin{multicols}{4}
|
||||
\begin{enumerate}
|
||||
\item 50\%
|
||||
\item 25\%
|
||||
\item 29\%
|
||||
\item 71\%
|
||||
\end{enumerate}
|
||||
\end{multicols}
|
||||
\end{enumerate}
|
||||
\end{exercise}
|
||||
|
||||
\begin{exercise}[subtitle={Location de voitures}, points=5]
|
||||
\textbf{Candidats de la série ES n’ayant pas suivi l’enseignement de spécialité et candidats de la série L}
|
||||
|
||||
\medskip
|
||||
|
||||
Un loueur de voitures dispose au 1\ier{} mars 2015 d'un total de \np{10000} voitures pour l'Europe.
|
||||
|
||||
Afin d'entretenir son parc, il décide de revendre, au 1\ier{} mars de chaque année, 25\,\% de son parc automobile et d'acheter \np{3000} voitures neuves.
|
||||
|
||||
\smallskip
|
||||
|
||||
On modélise le nombre de voitures de l'agence à l'aide d'une suite:
|
||||
|
||||
Pour tout entier naturel $n$, on note $u_n$ le nombre de voitures présentes dans le parc automobile au 1\ier{} mars de l'année $2015+n$.
|
||||
|
||||
On a donc $u_0=\np{10000}$.
|
||||
|
||||
\begin{enumerate}
|
||||
\item Calculer le nombre de voitures présentes dans le parc automobile au 1er mars 2016.
|
||||
\item Expliquer pourquoi pour tout entier naturel $n$, $u_{n+1}=0,75 u_n+\np{3000}$.
|
||||
|
||||
\item Pour tout entier naturel $n$, on considère la suite $\left(v_n\right)$ définie par
|
||||
|
||||
\hfill$v_n=u_n-\np{12000}$.\hfill{}
|
||||
|
||||
\begin{enumerate}
|
||||
\item
|
||||
Montrer que la suite $\left(v_n\right)$ est une suite géométrique de raison 0,75. Préciser son premier terme.
|
||||
|
||||
\item Exprimer $v_n$ en fonction de $n$.
|
||||
|
||||
\item Déterminer la limite de la suite $\left(v_n\right)$.
|
||||
|
||||
\item Justifier que, pour tout entier naturel $n$, $u_n=\np{12000} - \np{2000} \times 0,75^n$.
|
||||
|
||||
\item En vous appuyant sur les réponses données aux deux questions précédentes, que pouvez-vous conjecturer sur le nombre de voitures que comptera le parc automobile de ce loueur au bout d'un grand nombre d'années?
|
||||
|
||||
\end{enumerate}
|
||||
|
||||
\item On admet dans cette question que la suite $\left(u_n\right)$ est croissante.
|
||||
|
||||
On aimerait déterminer l'année à partir de laquelle le parc automobile comptera au moins \np{11950} voitures.
|
||||
|
||||
\begin{enumerate}
|
||||
\item Recopier l'algorithme suivant et compléter les pointillés afin qu'il permette de répondre au problème posé.
|
||||
|
||||
\begin{center}
|
||||
\begin{tabular}{|l @{\hspace*{0.5cm}}l|}
|
||||
\hline
|
||||
Initialisation & U prend la valeur \np{10000}\\
|
||||
& N prend la valeur 0\\
|
||||
Traitement & Tant que \dots\\
|
||||
& \hspace*{0.3cm} N prend la valeur \dots \hspace*{1cm}\ \\
|
||||
& \hspace*{0.3cm} U prend la valeur \dots\\
|
||||
& Fin Tant que\\
|
||||
Sortie & Afficher \dots\\
|
||||
\hline
|
||||
\end{tabular}
|
||||
\end{center}
|
||||
|
||||
\item À l'aide de la calculatrice, déterminer l'année recherchée.
|
||||
|
||||
%\item Retrouver ce résultat en résolvant l'inéquation
|
||||
|
||||
% \hfill$\np{12000} - \np{2000}\times 0,75^n \geqslant \np{11950}$.\hfill{}
|
||||
|
||||
\end{enumerate}
|
||||
\end{enumerate}
|
||||
\end{exercise}
|
||||
|
||||
\pagebreak
|
||||
|
||||
\begin{exercise}[subtitle={Chasse-neige et automates}, points=5]
|
||||
\textbf{Candidats de ES ayant suivi l’enseignement de spécialité}
|
||||
|
||||
\textbf{Les deux parties de cet exercice sont indépendantes.}
|
||||
|
||||
\medskip
|
||||
|
||||
\textbf{Partie 1}
|
||||
|
||||
Suite à des intempéries, un chasse-neige doit déblayer toutes les routes reliant les stations de son secteur. On modélise ce secteur par le graphe ci-dessous dont les sommets représentent les différentes stations désignées par des lettres. Les poids des arêtes sont les durées moyennes de parcours, en minute, du chasse-neige entre deux stations.
|
||||
|
||||
\begin{center}
|
||||
\includegraphics[scale=0.2]{./fig/graph_chasse_neige}
|
||||
\end{center}
|
||||
|
||||
\begin{enumerate}
|
||||
\item Ce graphe est-il connexe? Justifier.
|
||||
\item Le chasse-neige part de la station G. Peut-il partir de cette station et y revenir en parcourant une et une seule fois chacune des routes, matérialisées par les arêtes de ce graphe ?
|
||||
\item Une saleuse doit de même parcourir l'ensemble des routes du secteur après déblaiement de la neige. Elle est garée à la station A et, après son travail, peut se garer dans n'importe quelle station.
|
||||
|
||||
Peut-elle parcourir une et une seule fois chacune des routes
|
||||
pour traiter l'ensemble du secteur ?
|
||||
\item On appelle $M$ la matrice d'adjacence associée au graphe, les sommets étant rangés dans \underline{l'ordre alphabétique}
|
||||
|
||||
Déterminer $M$.
|
||||
|
||||
\item On donne :
|
||||
\[
|
||||
M^4 = \begin{pmatrix}
|
||||
61 &48 &52 &28 &45 &55 &24\\
|
||||
48 &44 &41 &21 &42 &45 &20\\
|
||||
52 &41 &50 &25 &41 &52 &25\\
|
||||
28 &21 &25 &15 &20 &24 &10\\
|
||||
45 &42 &41 &20 &44 &48 &21\\
|
||||
55 &45 &52 &24 &48 &61 &28\\
|
||||
24 &20 &25 &\textbf{10} & 21 &28 &15
|
||||
\end{pmatrix}
|
||||
\]
|
||||
|
||||
Interpréter dans le contexte de l'exercice le nombre $10$ figurant en caractère gras dans la matrice.
|
||||
\end{enumerate}
|
||||
|
||||
\medskip
|
||||
|
||||
\textbf{Partie 2}
|
||||
|
||||
Pour accéder à un local d'une petite entreprise, les employés doivent choisir un code reconnu par l'automate suivant :
|
||||
|
||||
\begin{center}
|
||||
\includegraphics[scale=0.2]{./fig/graph_automate}
|
||||
\end{center}
|
||||
|
||||
Une succession de lettres constitue un code possible si ces lettres se succèdent sur un chemin du graphe orienté ci-dessus, en partant du sommet $\pscirclebox{1}$ et en sortant au sommet $\pscirclebox{4}$.
|
||||
|
||||
% \begin{list}{\textbullet}{Par exemple:}
|
||||
% \item le mot $bcbab$ est un mot reconnu par cet automate, et correspond au chemin 121334 ;
|
||||
% \item le mot $abac$ n'est pas reconnu par cet automate.
|
||||
% \end{list}
|
||||
|
||||
\begin{enumerate}
|
||||
\item Parmi les mots suivants, quels sont ceux qui sont reconnus par cet automate ?
|
||||
|
||||
\hfill{}$abab$, $abc$, $abbcbb$.\hfill{}
|
||||
|
||||
\item Recopier et compléter la matrice d'adjacence
|
||||
$M=
|
||||
\begin{pmatrix}
|
||||
0 & 2 & 1 & 0\\
|
||||
\cdots & \cdots & \cdots & \cdots\\
|
||||
\cdots & \cdots & \cdots & \cdots\\
|
||||
\cdots & \cdots & \cdots & \cdots
|
||||
\end{pmatrix}$
|
||||
associée au graphe orienté dans laquelle les sommets sont rangés dans l'ordre croissant.
|
||||
|
||||
\item
|
||||
%Un logiciel de calcul formel donne
|
||||
|
||||
%\[M^4=
|
||||
% \begin{pmatrix}
|
||||
% 5 & 3 & 10 & 5\\
|
||||
% 1 & 6 & 7 & 4 \\
|
||||
% 1 & 3 & 4 & 2 \\
|
||||
% 2 & 1 & 4 & 2
|
||||
% \end{pmatrix}
|
||||
% \qquad\text{et}\qquad
|
||||
% M^5=
|
||||
% \begin{pmatrix}
|
||||
% 3 & 15 & 18 & 10\\
|
||||
% 6 & 6 & 14 & 7\\
|
||||
% 3 & 4 & 8 & 4\\
|
||||
% 1 & 6 & 7 & 4
|
||||
% \end{pmatrix}
|
||||
%\]
|
||||
Combien de mots de 4 lettres sont-ils reconnus par l'automate ? Justifier. Quels sont-ils ?
|
||||
\end{enumerate}
|
||||
\end{exercise}
|
||||
|
||||
\pagebreak
|
||||
|
||||
\begin{exercise}[subtitle={Bois et energie}, points=6]
|
||||
\textbf{Commun à tous les candidats}
|
||||
\medskip
|
||||
|
||||
\textbf{La partie A peut être traitée indépendamment des parties B et C.}
|
||||
|
||||
\medskip
|
||||
|
||||
L'entreprise \emph{BBE (Bio Bois Énergie)} fabrique et vend des granulés de bois pour alimenter des
|
||||
chaudières et des poêles chez des particuliers ou dans des collectivités.
|
||||
|
||||
L'entreprise produit entre 1 et 15 tonnes de granulés par jour.
|
||||
|
||||
\setlength\parindent{1cm}
|
||||
\begin{itemize}
|
||||
\item[$\bullet~~$] Les coûts de fabrication quotidiens sont modélisés par la fonction $C$ définie sur l'intervalle [1~;~15] par :
|
||||
|
||||
\[C(x) = 0,3x^2 - x + \text{e}^{- x + 5}\]
|
||||
|
||||
où $x$ désigne la quantité de granulés en tonnes et $C(x)$ le coût de fabrication quotidien
|
||||
correspondant en centaines d'euros.
|
||||
\item[$\bullet~~$]Dans l'entreprise \emph{BBE} le prix de vente d'une tonne de granulés de bois est de $300$~euros.
|
||||
|
||||
La recette quotidienne de l'entreprise est donc donnée par la fonction $R$ définie sur l'intervalle
|
||||
[1~;~15] par:
|
||||
|
||||
\[R(x) = 3x\]
|
||||
|
||||
où $x$ désigne la quantité de granulés en tonnes et $R(x)$ la recette quotidienne correspondante
|
||||
en centaines d'euros.
|
||||
\item[$\bullet~~$]On définit par $D(x)$ le résultat net quotidien de l'entreprise en centaines d'euros, c'est-à-dire la différence entre la recette $R(x)$ et le coût $C(x)$, où $x$ désigne la quantité de granulés en tonnes.
|
||||
\end{itemize}
|
||||
\setlength\parindent{0cm}
|
||||
|
||||
\medskip
|
||||
\textbf{Partie A : Étude graphique}
|
||||
|
||||
Sur le graphique ci-dessous, on donne $\mathcal{C}$ et $\Delta$ les représentations graphiques respectives des fonctions $C$ et $R$ dans un repère d'origine O.
|
||||
|
||||
\begin{center}
|
||||
\begin{tikzpicture}[yscale=0.35, xscale=0.8]
|
||||
\tkzInit[xmin=0,xmax=15,xstep=1,
|
||||
ymin=0,ymax=54,ystep=2]
|
||||
\tkzGrid
|
||||
%\tkzGrid[sub, subxstep=0.2, subystep=200]
|
||||
\tkzAxeXY%[up space=0.5,right space=.2]
|
||||
\tkzFct[domain = 0:15, line width=1pt]{3*x}
|
||||
\draw (2,23) node [left] {$\mathcal{C}$};
|
||||
\tkzFct[domain = 0:15, line width=1pt]{0.3*x**2-x+exp(-x+5)}
|
||||
\draw (2,3) node [above left] {$\Delta$};
|
||||
\end{tikzpicture}
|
||||
\end{center}
|
||||
|
||||
\textbf{Dans cette partie A, répondre aux questions suivantes à l'aide du graphique, et avec la précision permise par celui-ci. Aucune justification n'est demandée.}
|
||||
|
||||
\medskip
|
||||
|
||||
\begin{enumerate}
|
||||
\item Déterminer la quantité de granulés en tonnes pour laquelle le coût quotidien de l'entreprise est
|
||||
minimal.
|
||||
\item
|
||||
\begin{enumerate}
|
||||
\item Déterminer les valeurs $C(6)$ et $R(6)$ puis en déduire une estimation du résultat net
|
||||
quotidien en euros dégagé par l'entreprise pour 6~tonnes de granulés fabriqués et vendus.
|
||||
\item Déterminer les quantités possibles de granulés en tonnes que l'entreprise doit produire et vendre quotidiennement pour dégager un résultat net positif, c'est-à-dire un bénéfice.
|
||||
\end{enumerate}
|
||||
\end{enumerate}
|
||||
|
||||
\medskip
|
||||
\textbf{Partie B : Étude d'une fonction}
|
||||
|
||||
On considère la fonction $g$ définie sur l'intervalle [1~;~15] par :
|
||||
|
||||
\[g(x) = - 0,6x + 4 + \text{e}^{- x + 5}\]
|
||||
|
||||
On admet que la fonction $g$ est dérivable sur l'intervalle [1~;~15] et on note $g'$ sa fonction dérivée.
|
||||
|
||||
\medskip
|
||||
|
||||
\begin{enumerate}
|
||||
\item
|
||||
\begin{enumerate}
|
||||
\item Calculer $g'(x)$ pour tout réel $x$ de l'intervalle [1~;~15].
|
||||
\item En déduire que la fonction $g$ est décroissante sur l'intervalle [1~;~15].
|
||||
\end{enumerate}
|
||||
\item
|
||||
\begin{enumerate}
|
||||
\item Dresser le tableau de variation de la fonction $g$ sur l'intervalle [1~;~15], en précisant les valeurs $g(1)$ et $g(15)$ arrondies à l'unité.
|
||||
\item Démontrer que l'équation g(x) = 0 admet une unique solution $\alpha$ et donner une valeur approchée à 0,1 de cette solution
|
||||
|
||||
\item Déduire des questions précédentes le tableau de signe de $g(x)$ sur l'intervalle [1~;~15].
|
||||
\end{enumerate}
|
||||
\end{enumerate}
|
||||
|
||||
\medskip
|
||||
\textbf{Partie C : Application économique}
|
||||
|
||||
\begin{enumerate}
|
||||
\item Démontrer que pour tout réel $x$ de l'intervalle [1~;~15], on a :
|
||||
|
||||
\[D (x) = - 0,3x^2 + 4x - \text{e}^{- x + 5}\]
|
||||
|
||||
\item On admet que la fonction $D$ est dérivable sur l'intervalle [1~;~15] et on note $D'$ sa fonction dérivée.
|
||||
|
||||
Démontrer que pour tout réel $x$ de l'intervalle [1~;~15], on a $D'(x) = g(x)$, où $g$ est la fonction étudiée dans la partie B.
|
||||
\item En déduire les variations de la fonction $D$ sur l'intervalle [1~;~15].
|
||||
\item
|
||||
\begin{enumerate}
|
||||
\item Pour quelle quantité de granulés l'entreprise va-t-elle rendre son bénéfice maximal ?
|
||||
|
||||
On donnera une valeur approchée du résultat à $0,1$ tonne près.
|
||||
\item Calculer alors le bénéfice maximal à l'euro près.
|
||||
\end{enumerate}
|
||||
\end{enumerate}
|
||||
\end{exercise}
|
||||
|
||||
\pagebreak
|
||||
|
||||
\begin{exercise}[subtitle={Enquête client}, points=5]
|
||||
\textbf{Commun à tous les candidats}
|
||||
\medskip
|
||||
|
||||
Dans cet exercice, les résultats approchés sont à arrondir au millième.
|
||||
|
||||
À partir d'une étude statistique dans une chaîne de restaurants, on a modélisé le comportement des clients par :
|
||||
|
||||
\begin{itemize}
|
||||
\item[$\bullet~~$] 60\,\% des clients sont des hommes ;
|
||||
\item[$\bullet~~$] 80\,\% des hommes mangent un dessert alors que seulement 45\,\% des femmes en mangent un.
|
||||
\end{itemize}
|
||||
|
||||
On interroge au hasard un client de cette chaîne. On note :
|
||||
\begin{itemize}
|
||||
\item[$\bullet~~$] $H$ l'évènement \og le client interrogé est un homme \fg{};
|
||||
\item[$\bullet~~$] $D$ l'évènement \og le client interrogé a mangé un dessert \fg.
|
||||
\end{itemize}
|
||||
|
||||
On note également :
|
||||
\begin{itemize}
|
||||
\item[$\bullet~~$] $\overline{A}$ l'évènement contraire d'un évènement $A$ ;
|
||||
\item[$\bullet~~$] $p(A)$ la probabilité d'un évènement $A$.
|
||||
\end{itemize}
|
||||
|
||||
\medskip
|
||||
|
||||
\textbf{Partie A}
|
||||
|
||||
\begin{enumerate}
|
||||
\item Représenter la situation par un arbre pondéré.
|
||||
\item Calculer la probabilité que le client interrogé soit un homme et ait mangé un dessert.
|
||||
\item Montrer que $p(D) = 0,66$.
|
||||
\item Le client interrogé affirme avoir pris un dessert. Quelle est la probabilité que ce soit une femme ?
|
||||
\end{enumerate}
|
||||
|
||||
\medskip
|
||||
|
||||
\textbf{Partie B}
|
||||
|
||||
On rappelle que d'après cette enquête, la probabilité pour qu'un client ait mangé un dessert est de 0,66.
|
||||
|
||||
La chaine de restaurant souhaite estimer le nombre de dessert à préparer à l'avance pour une salle de 60 personnes. Elle suppose alors que le choix des clients se fait de façon identique et indépendantes de celui de autres.
|
||||
|
||||
On note $X$ la variable aléatoire qui compte le nombre de clients qui veulent un dessert.
|
||||
\begin{enumerate}
|
||||
\item Quelle loi suit la variable aléatoire $X$?
|
||||
\item Calculer la probabilité que 50 personnes souhaitent un dessert.
|
||||
\item Quelle est la probabilité que moins de 10 clients souhaitent un dessert?
|
||||
\item À l'aide de la calculatrice, déterminer la valeur minimale de $a$ telle que $P(X \leq a) > 0,95$. Interpréter le résultat.
|
||||
\end{enumerate}
|
||||
\end{exercise}
|
||||
|
||||
\end{document}
|
||||
%%% Local Variables:
|
||||
%%% mode: latex
|
||||
%%% TeX-master: "master"
|
||||
%%% End:
|
||||
BIN
TES/DS/BacBlanc/Correction_Jacques.pdf
Normal file
BIN
TES/DS/BacBlanc/Correction_Jacques.pdf
Normal file
Binary file not shown.
BIN
TES/DS/BacBlanc/fig/graph_automate.png
Normal file
BIN
TES/DS/BacBlanc/fig/graph_automate.png
Normal file
Binary file not shown.
|
After Width: | Height: | Size: 60 KiB |
BIN
TES/DS/BacBlanc/fig/graph_chasse_neige.png
Normal file
BIN
TES/DS/BacBlanc/fig/graph_chasse_neige.png
Normal file
Binary file not shown.
|
After Width: | Height: | Size: 56 KiB |
BIN
TES/DS/DS_19_09_25/DS_19_09_25.pdf
Normal file
BIN
TES/DS/DS_19_09_25/DS_19_09_25.pdf
Normal file
Binary file not shown.
259
TES/DS/DS_19_09_25/DS_19_09_25.tex
Normal file
259
TES/DS/DS_19_09_25/DS_19_09_25.tex
Normal file
@@ -0,0 +1,259 @@
|
||||
\documentclass[a4paper,12pt]{article}
|
||||
\usepackage{myXsim}
|
||||
\usepackage[inline]{enumitem}
|
||||
\usepackage{tasks}
|
||||
|
||||
% Title Page
|
||||
\title{DS 1}
|
||||
\tribe{Terminale ES-L}
|
||||
\date{25 septembre 2019}
|
||||
|
||||
\begin{document}
|
||||
\maketitle
|
||||
|
||||
Le barème est donné à titre indicatif, il pourra être modifié.
|
||||
|
||||
Une part importante de la note sera dédiée à la rédaction, aux explications et à l'utilisation des notations mathématiques.
|
||||
|
||||
\begin{exercise}[subtitle={QCM},points=3]
|
||||
\textit{Cet exercice est un QCM (questionnaire à choix multiples).}
|
||||
|
||||
\textit{Pour chacune des questions posées, une seule des trois réponses est exacte. Recopier le numéro de la
|
||||
question et la réponse exacte. Aucune justification n’est demandée. Une réponse exacte rapporte 1 point,
|
||||
une réponse fausse ou l’absence de réponse ne rapporte ni n’enlève de point. Une réponse multiple ne
|
||||
rapporte aucun point.}
|
||||
|
||||
\begin{enumerate}
|
||||
\item Quelle est la meilleure remise?
|
||||
\begin{tasks}(3)
|
||||
\task Une remise de 20\%
|
||||
\task Trois remises de 7\%
|
||||
\task Quatre remises de 6\%
|
||||
\end{tasks}
|
||||
\item La suite $(u_n)$ définie par $u_0 = 2$ et par la formule de récurrence $u_{n+1} = 3u_n + 2u_n$ est
|
||||
\begin{tasks}(3)
|
||||
\task Arithmétique
|
||||
\task Géométrique
|
||||
\task Ni géométrique ni Arithmétique
|
||||
\end{tasks}
|
||||
\item Soit $X$ une variable aléatoire qui suit la loi $\mathcal{U}([2, 11])$. Alors $P(X>5,5)$ est égale à
|
||||
\begin{tasks}(3)
|
||||
\task 0,5
|
||||
\task 5,5
|
||||
\task 8
|
||||
\end{tasks}
|
||||
\end{enumerate}
|
||||
\end{exercise}
|
||||
|
||||
\begin{solution}
|
||||
\textit{Les réponses suivantes sont justifiées ce qui n'est pas demandé dans ce QCM.}
|
||||
\begin{enumerate}
|
||||
\item Réponse c) Quatre remises de 6\%.
|
||||
|
||||
\begin{enumerate}
|
||||
\item Une remise de 20\% revient à multiplier par $0.8$
|
||||
\item Trois remises de 7\% revient à multiplier par $(1-0.07)^3 = 0.93^3 \approx 0.804$
|
||||
\item Quatre remises de 6\% revient à multiplier par $(1 - 0.06)^4 = 0.94^4 \approx 0.78$
|
||||
\end{enumerate}
|
||||
\item Réponse b) Géométrique
|
||||
|
||||
En effet, $u_{n+1} = 3u_n+2u_n = 5u_n$, pour passer d'un terme au suivant on multiplie par 5.
|
||||
|
||||
\item Réponse a) 0.5
|
||||
|
||||
Les deux autres réponses ne sont pas des probabilités car 5,5 et 8 ne sont pas des nombres compris entre 0 et 1.
|
||||
\end{enumerate}
|
||||
\end{solution}
|
||||
|
||||
\begin{exercise}[subtitle={Un peu de hasard},points=5]
|
||||
\textit{Les questions suivants sont indépendantes}
|
||||
\begin{enumerate}
|
||||
\item Après une tétée, un bébé dort 30min puis peut demander à manger à n'importe quel moment dans les 3 heures qui suivent. Lucas a terminé de téter à 9h.
|
||||
|
||||
On note $X$ la variable aléatoire décrivant l'heure où Lucas va demander à nouveau à manger.
|
||||
\begin{enumerate}
|
||||
\item Avec quelle loi peut-on modéliser $X$?
|
||||
\item Calculer $P(X<11)$.
|
||||
\item Sa maman veut partir faire des courses entre 10h30 et 11h45. Quelle est la probabilité que Lucas réclame pendant son absence?
|
||||
\end{enumerate}
|
||||
\item La masse en gramme des melons d'un maraîcher est modélisée par une variable aléatoire M qui suit une loi uniforme sur l'intervalle [850;x] avec x>1200. On constate que 75\% des melons du maraîcher ont une masse comprise entre 900 g et 1200 g. Déterminer x.
|
||||
\end{enumerate}
|
||||
\end{exercise}
|
||||
|
||||
\begin{solution}
|
||||
\begin{enumerate}
|
||||
\item
|
||||
\begin{enumerate}
|
||||
\item $X$ peut suivre une loi uniforme sur $\intFF{9,5}{12,5}$ notée $\mathcal{U}(\intFF{9.5}{12.5})$.
|
||||
\item $P(X>11) = \dfrac{12,5 - 11}{12,5 - 9,5} = \dfrac{1,5}{3} = 0,5$
|
||||
\item Cette probabilité revient à calculer $P(10,5 < X < 11,75) = \dfrac{1,25}{3} \approx 0.42$
|
||||
\end{enumerate}
|
||||
\item On cherche à déterminer $x$ tel que
|
||||
\begin{eqnarray*}
|
||||
0,75 &=& P(900 < M < 1200) = \dfrac{1200 - 900}{x - 850} \\
|
||||
0.75 (x -850) &=& 300 \\
|
||||
0.75 x - 637.5 &=& 300 \\
|
||||
0.75 x &=& 937.5 \\
|
||||
x &=& \dfrac{937.5}{0.75} = 1250
|
||||
\end{eqnarray*}
|
||||
Donc $M$ suit la loi $\mathcal{U}(\intFF{850}{1250})$.
|
||||
\end{enumerate}
|
||||
\end{solution}
|
||||
|
||||
\pagebreak
|
||||
|
||||
\begin{exercise}[subtitle={Pollution de l'air},points=5]
|
||||
\textit{Dans cette question l'utilisation des outils et des notations mathématiques sera valorisée même si elle n'est pas obligatoire.}
|
||||
|
||||
Afin de respecter l'accord signé sur la pollution de l'air, certaines entreprises, dès l'année 2014, ont été contraintes de diminuer chaque année la quantité de CO$_2$ qu'elles produisent.
|
||||
|
||||
Une de ces entreprises émettait $15$ milliers de tonnes de CO$_2$ en 2014 et $14,7$ milliers de
|
||||
tonnes en 2015.
|
||||
|
||||
On suppose que le taux de diminution annuel de CO$_2$ émis restera constant pendant les années suivantes.
|
||||
|
||||
\medskip
|
||||
|
||||
\begin{enumerate}
|
||||
\item Calculer le taux d'évolution de l'émission de CO$_2$ par cette entreprise entre 2014 et 2015.
|
||||
\item L'accord prévoit que cette entreprise devra produire moins de $12$ milliers de tonnes de CO$_2$ par an. En détaillant la méthode employée, déterminer à partir de quelle année la
|
||||
quantité de CO$_2$ émise par cette entreprise passera en dessous de ce seuil de $12$~milliers
|
||||
de tonnes.
|
||||
\end{enumerate}
|
||||
\end{exercise}
|
||||
|
||||
\begin{solution}
|
||||
\begin{enumerate}
|
||||
\item Taux d'évolution entre 2014 et 2015.
|
||||
\[
|
||||
\dfrac{v_a - v_d}{v_d} = \dfrac{14.7 - 15}{15} = -0.02 = -2\%
|
||||
\]
|
||||
\item On peut modéliser la quantité émise par l'entreprise avec une suite $(u_n)$. Comme le taux de diminution est supposé constant et égale à -2\%, elle sera géométrique, de raison $1 - 0.02 = 0.98$ et aura pour premier terme $u_0 = 15$.
|
||||
\begin{eqnarray*}
|
||||
u_0 &=& 15 \\
|
||||
u_1 &=& u_0\times 0.98 = 15\times 0.98 = 14.7 \\
|
||||
u_2 &=& 14.70 \times 0.98 = 14.41 \\
|
||||
u_3 &=& 14.41 \times 0.98 = 14.12 \\
|
||||
u_4 &=& 14.12 \times 0.98 = 13.84 \\
|
||||
u_5 &=& 13.84 \times 0.98 = 13.56 \\
|
||||
u_6 &=& 13.56 \times 0.98 = 13.29 \\
|
||||
u_7 &=& 13.29 \times 0.98 = 13.02 \\
|
||||
u_8 &=& 13.02 \times 0.98 = 12.76 \\
|
||||
u_9 &=& 12.76 \times 0.98 = 12.51 \\
|
||||
u_{10} &=& 12.51 \times 0.98 = 12.26 \\
|
||||
u_{11} &=& 12.26 \times 0.98 = 12.01 \\
|
||||
u_{12} &=& 12.01 \times 0.98 = 11.77 \\
|
||||
\end{eqnarray*}
|
||||
Il faudra attendre 12 années soit 2026.
|
||||
\end{enumerate}
|
||||
|
||||
\end{solution}
|
||||
|
||||
|
||||
\begin{exercise}[subtitle={Pyrale du buis},points=7]
|
||||
% Ex 2 - 2019 Liban
|
||||
La Pyrale du buis est une espèce de lépidoptères de la famille des Crambid\ae, originaire d'Extrême-Orient.
|
||||
Introduite accidentellement en Europe dans les années 2000, elle y est devenue invasive. Une étude décomptant le nombre de chenilles de Pyrale dans un camping d'Ardèche donne les estimations suivantes:
|
||||
|
||||
\begin{center}
|
||||
\begin{tabular}{|c|c|c|c|}
|
||||
\hline
|
||||
Date & 01/06/18 & 02/06/18 & 03/06/18\\
|
||||
\hline
|
||||
$n$ & 0 & 1 & 2 \\
|
||||
\hline
|
||||
Nombre de chenilles en centaines & 97 & 181 & 258\\
|
||||
\hline
|
||||
\end{tabular}
|
||||
\end{center}
|
||||
|
||||
L'exercice étudie et compare deux modélisations de l'évolution du nombre de chenilles.
|
||||
|
||||
\begin{flushleft}
|
||||
\textbf{Partie 1:} \emph{Modèle 1}
|
||||
\end{flushleft}
|
||||
|
||||
Dans cette partie, on modélise le nombre de chenilles le $n$-ième jour après le 1\ier{} juin 2018 (nombre exprimé en centaines) par une suite géométrique $(u_n)$ de raison $q=1,63$. Ainsi $u_0=97$.
|
||||
|
||||
\begin{enumerate}
|
||||
\item Calculer $u_2$. Arrondir à l'unité.
|
||||
\item Exprimer $u_n$ en fonction de $n$, pour tout entier naturel $n$.
|
||||
\item Justifier que la suite $(u_n)$ est croissante.
|
||||
\item Selon ce modèle, quel sera le nombre de chenilles le 13 juin 2018? Arrondir à la centaine.
|
||||
\end{enumerate}
|
||||
|
||||
\begin{flushleft}
|
||||
\textbf{Partie 2:} \emph{Modèle 2}
|
||||
\end{flushleft}
|
||||
|
||||
Dans cette partie, on modélise le nombre de chenilles le $n$-ième jour après le 1\ier{} juin 2018 (nombre exprimé en centaines) par une suite $(v_n)$ telle que:
|
||||
|
||||
\hfill{}$v_0=97$ et, pour tout entier naturel $n$, $v_{n+1} = 0,91 v_n + 93$.\hfill{}
|
||||
|
||||
\begin{enumerate}
|
||||
\setcounter{enumi}{4}
|
||||
\item On admet que, pour tout entier naturel $n$: $v_n=\dfrac{1}{3} \left ( \np{-2809} \times 0,91^n + \np{3100}\strut\right )$.
|
||||
|
||||
Selon ce modèle, quel sera le nombre de chenilles le 13 juin 2018? Arrondir à la centaine.
|
||||
|
||||
\item En étudiant le signe de $v_{n+1} - v_n$, montrer que la suite $(v_n)$ est croissante.
|
||||
\end{enumerate}
|
||||
|
||||
\begin{flushleft}
|
||||
\textbf{Partie 3:} \emph{Comparaison des différents modèles}
|
||||
\end{flushleft}
|
||||
|
||||
Les valeurs relevées dans le camping sur le mois de juin n'ont jamais dépassé 1000 centaines de chenilles.
|
||||
|
||||
\begin{enumerate}
|
||||
\setcounter{enumi}{6}
|
||||
\item Quel modèle paraît le plus adapté?
|
||||
\end{enumerate}
|
||||
\end{exercise}
|
||||
|
||||
\begin{solution}
|
||||
\begin{enumerate}
|
||||
\item
|
||||
\begin{eqnarray*}
|
||||
u_1 &=& u_0 \times 1.63 = 97 \times 1.63 = 158 \\
|
||||
u_2 &=& u_1 \times 1.63 = 158 \times 1.63 = 257
|
||||
\end{eqnarray*}
|
||||
\item Forme explicite de la suite $u_n = 97 \times 1.63^n$
|
||||
\item $(u_n)$ est croissante car la raison $q = 1.63$ est supérieur à 1.
|
||||
\item Le nombre de chenilles le 13 juin 2018 correspond à
|
||||
\[
|
||||
u_{12} = 97 \times 1.63^{12} = \np{34121}
|
||||
\]
|
||||
\item Comme précédemment le 13 juin 2018 correspond au 12e terme de la suite
|
||||
\[
|
||||
v_{12}=\dfrac{1}{3} \left ( \np{-2809} \times 0,91^{12} + \np{3100}\strut\right ) = 731
|
||||
\]
|
||||
\item Variation de la suite $(v_n)$
|
||||
\[
|
||||
v_{n+1} = \dfrac{1}{3} \left ( \np{-2809} \times 0,91^{n+1} + \np{3100}\strut\right )
|
||||
\]
|
||||
Donc
|
||||
\begin{eqnarray*}
|
||||
v_{n+1} - v_n &=& \dfrac{1}{3} \left ( \np{-2809} \times 0,91^{n+1} + \np{3100}\strut\right ) - \dfrac{1}{3} \left ( \np{-2809} \times 0,91^{n} + \np{3100}\strut\right )\\
|
||||
v_{n+1} - v_n &=& \dfrac{1}{3} \left ( \np{-2809} \times 0,91^{n+1} + \np{3100} + \np{2809} \times 0,91^{n} - \np{3100}\strut\right ) \\
|
||||
v_{n+1} - v_n &=& \dfrac{1}{3} \left ( \np{-2809} \times 0,91^{n+1} + \np{2809} \times 0,91^{n}) \\
|
||||
v_{n+1} - v_n &=& \dfrac{1}{3} \times 2809 ( - 0,91^{n+1} + 0,91^{n}) \\
|
||||
v_{n+1} - v_n &=& \dfrac{1}{3} \times 2809 ( - 0,91^{n} \times 0.91 + 0,91^{n}) \\
|
||||
v_{n+1} - v_n &=& \dfrac{1}{3} \times 2809 \times 0.91^n \times ( -0.91 + 1)
|
||||
\end{eqnarray*}
|
||||
Les nombres en dehors des parenthèses sont positifs.\\
|
||||
$-0.91 + 1 = 0.09$ est aussi positif.\\
|
||||
Donc $v_{n+1} - v_n$ est positif et donc la suite est croissante.
|
||||
\item D'après la question 4., $v_{12} = \np{34121}$ ce qui est beaucoup plus grand que les 1000 observées. La raison étant plus grande au 1, ce nombre va continuer d'augmenter. Le premier modèle ne semble donc pas adapté.
|
||||
|
||||
Le deuxième modèle donne des quantités plus faible (731 au 13 juin). Il semble donc plus convenir.
|
||||
\end{enumerate}
|
||||
\end{solution}
|
||||
|
||||
\end{document}
|
||||
|
||||
%%% Local Variables:
|
||||
%%% mode: latex
|
||||
%%% TeX-master: "master"
|
||||
%%% End:
|
||||
|
||||
BIN
TES/DS/DS_19_09_25/DS_19_09_25_corr.pdf
Normal file
BIN
TES/DS/DS_19_09_25/DS_19_09_25_corr.pdf
Normal file
Binary file not shown.
BIN
TES/DS/DS_19_10_14/DS_19_10_14.pdf
Normal file
BIN
TES/DS/DS_19_10_14/DS_19_10_14.pdf
Normal file
Binary file not shown.
191
TES/DS/DS_19_10_14/DS_19_10_14.tex
Normal file
191
TES/DS/DS_19_10_14/DS_19_10_14.tex
Normal file
@@ -0,0 +1,191 @@
|
||||
\documentclass[a4paper,10pt]{article}
|
||||
\usepackage{myXsim}
|
||||
%\usepackage[inline]{enumitem}
|
||||
\usepackage{tasks}
|
||||
|
||||
\title{DS 2}
|
||||
\tribe{Terminale ES-L}
|
||||
\date{14 octobre 2019}
|
||||
|
||||
\begin{document}
|
||||
\maketitle
|
||||
|
||||
Le barème est donné à titre indicatif, il pourra être modifié.
|
||||
|
||||
Une part importante de la note sera dédiée à la rédaction, aux explications et à l'utilisation des notations mathématiques.
|
||||
|
||||
\begin{exercise}[subtitle={QCM},points=4]
|
||||
\textit{Cet exercice est un QCM (questionnaire à choix multiples).}
|
||||
|
||||
\textit{Pour chacune des questions posées, une seule des trois ou quatre réponses est exacte. Recopier le numéro de la
|
||||
question et la réponse exacte. Aucune justification n’est demandée. Une réponse exacte rapporte 1 point,
|
||||
une réponse fausse ou l’absence de réponse ne rapporte ni n’enlève de point. Une réponse multiple ne
|
||||
rapporte aucun point.}
|
||||
|
||||
\begin{enumerate}
|
||||
\item Dans une ville de la région, sur une population de \np{4200}~ habitants, $36$\,\% ont pris connaissance de la publicité lors de la première semaine de la campagne. Le nombre d'habitants de cette ville ayant pris connaissance de la publicité lors de la première semaine de la campagne est:
|
||||
|
||||
\begin{tasks}(4)
|
||||
\task \np{2688}
|
||||
\task \np{1512}
|
||||
\task \np{1167}
|
||||
\task \np{4164}
|
||||
\end{tasks}
|
||||
\item Dans une concession automobile de la région, le temps d'attente, exprimé en minutes, avant d'être reçu par un conseiller commercial peut être modélisé par une variable aléatoire qui suit la loi uniforme sur l'intervalle [1~;~10].
|
||||
|
||||
Un visiteur se présente. Quelle est la probabilité qu'il attende au moins 5 minutes avant d'être reçu par un conseiller commercial ?
|
||||
|
||||
\begin{tasks}(4)
|
||||
\task 0,4
|
||||
\task 0,5
|
||||
\task $\dfrac{4}{9}$
|
||||
\task $\dfrac{5}{9}$
|
||||
\end{tasks}
|
||||
\item Entre 2006 et 2018, dans un restaurant universitaire, le prix d'un repas est passé de 2 euros à 3,50~euros en augmentant chaque année de x\,\%. Parmi ces valeurs, la valeur la plus proche de $x$ est:
|
||||
|
||||
\begin{tasks}(4)
|
||||
\task 6,25
|
||||
\task 4,77
|
||||
\task 14,58
|
||||
\task0,85
|
||||
\end{tasks}
|
||||
\item On donne $u_{n+1} = 0.9u_n+\np{24000}$ et $v_n = u_n-\np{240000}$. Alors
|
||||
|
||||
\begin{multicols}{2}
|
||||
\begin{enumerate}
|
||||
\item $(v_n)$ est arithmétiques de raison \np{24000}
|
||||
\item $(v_n)$ est arithmétiques de raison \np{-240000}
|
||||
\item $(v_n)$ est géométrique de raison \np{0.9}
|
||||
\item $(v_n)$ est ni géométrique ni arithmétiques.
|
||||
\end{enumerate}
|
||||
\end{multicols}
|
||||
\end{enumerate}
|
||||
\end{exercise}
|
||||
|
||||
\begin{exercise}[subtitle={Population}, points=4]
|
||||
Une ville a vu sa population augmenter de \np{124000} habitants début 2014 à \np{128460} habitants début 2015.
|
||||
\begin{enumerate}
|
||||
\item Calculer le taux d'évolution de la population de cette ville entre 2014 et 2014.
|
||||
\item Proposer un modèle pour calculer le nombre d'habitants de cette ville si l'on suppose que ce taux d'évolution reste constant.
|
||||
\item En comparant avec l'évolution de villes analogues, les démographes pendent que la ville comptera \np{198600} habitants début 2030. Cette estimation est-elle compatible avec le modèle construit à la question 2?
|
||||
\end{enumerate}
|
||||
\end{exercise}
|
||||
|
||||
\begin{exercise}[subtitle={Grand seigneur}, points=4]
|
||||
Dans le restaurant qu'il fréquente chaque midi, Max dépense entre 10 et 35euros, "tout compris" selon ses choix. On modélise le prix d'un repas de Maw par la variable aléatoire $X$ dont la loi est uniforme entre 10 et 35\euro.
|
||||
\begin{enumerate}
|
||||
\item Calculer la probabilité que Max dépense moins de 17,50\euro.
|
||||
\item Calculer $P(19 < X < 27)$
|
||||
\item Combien va-t-il payer en moyenne son repas?
|
||||
\end{enumerate}
|
||||
Max, qui paye en espèces et se s'embarrasse pas avec la monnaie, a pour habitude de donner au serveur un somme arrondie au multiple de 5 supérieur.
|
||||
\begin{enumerate}
|
||||
\setcounter{enumi}{3}
|
||||
\item Quel montant donne-t-il quand son repas coûte 19\euro?
|
||||
\end{enumerate}
|
||||
On modélise la somme donnée par Max pour payer son repas par la variable aléatoire $Y$.
|
||||
\begin{enumerate}
|
||||
\setcounter{enumi}{4}
|
||||
\item Quelles valeurs peut prendre $Y$?
|
||||
\item Est-ce que $Y$ suit une loi uniforme? Si oui, préciser les paramètres. Si non, quelle est sa loi?
|
||||
\end{enumerate}
|
||||
\end{exercise}
|
||||
|
||||
\begin{exercise}[subtitle={Occupation des lits}, points=8]
|
||||
On donne ci-dessous la courbe $\mathcal{C}$ représentative d'une fonction $f$ définie et dérivable sur l'intervalle $\intFF{0}{4}$. On note $f'$ la fonction dérivée de $f$.
|
||||
|
||||
La droite $\mathcal{D}$ est la tangente à la courbe $\mathcal{C}$ au point d'abscisse $0$ ; elle passe par le point A de coordonnées (0,5~;~1).
|
||||
|
||||
La tangente $T$ à la courbe $\mathcal{C}$ au point d'abscisse 1.45 est parallèle à l'axe des abscisses.
|
||||
|
||||
\begin{minipage}{0.5\textwidth}
|
||||
\def\tkzRatioLineGrid{0.75}
|
||||
\begin{tikzpicture}[yscale=3, xscale=2, baseline=(a.north)]
|
||||
\tkzInit[xmin=0,xmax=4,xstep=1,
|
||||
ymin=0,ymax=1.5,ystep=1]
|
||||
\tkzGrid[sub, ligne width=1.5]
|
||||
\tkzAxeXY[up space=0.2,right space=0.2]
|
||||
|
||||
\tkzFct[domain=0:4,color=black,very thick]%
|
||||
{0.1*x*(x-4)*(x-5)}
|
||||
\draw (3.6,0.4) node {$\mathcal{C}$};
|
||||
|
||||
\tkzFct[domain=0:4,color=black,thick]%
|
||||
{2*x}
|
||||
\draw (0.5,1) node {$\times$} node[above left] {A};
|
||||
|
||||
\draw [<->, thick] (1,1.31) -- (1.46,1.31) node {$\times$} node[above left] {$T$} -- (2, 1.31);
|
||||
|
||||
\tkzFct[domain=2.3:3.7,color=black,thick]%
|
||||
{-0.7*x+2.7}
|
||||
|
||||
\draw (3,0.6) node[very thick] {\large +};
|
||||
\end{tikzpicture}
|
||||
\end{minipage}
|
||||
\begin{minipage}{0.5\textwidth}
|
||||
\textbf{Partie A}
|
||||
|
||||
\medskip
|
||||
|
||||
Dans cette partie les réponses seront obtenues par lecture graphique.
|
||||
|
||||
\medskip
|
||||
|
||||
\begin{enumerate}
|
||||
\item Donner la valeur de $f'(1.45)$.
|
||||
\item Déterminer une équation de la droite $\mathcal{D}$.
|
||||
\item Proposer un intervalle sur lequel la fonction semble concave.
|
||||
\end{enumerate}
|
||||
|
||||
\end{minipage}
|
||||
|
||||
\bigskip
|
||||
|
||||
\textbf{Partie B}
|
||||
|
||||
\medskip
|
||||
|
||||
La fonction $f$ est définie sur l'intervalle [0~;~4] par $f(x) = 0.3x^3-0.9x^2+2x$
|
||||
|
||||
On admet que son tableau de variation est
|
||||
|
||||
\begin{center}
|
||||
\begin{tikzpicture}[baseline=(a.north)]
|
||||
\tkzTabInit[lgt=1,espcl=2]{$x$/1,$f(x)$/1}{0, 1.46, 4}
|
||||
\tkzTabVar{-/ 0, +/ 1.31, -/ 0}
|
||||
\end{tikzpicture}
|
||||
\end{center}
|
||||
|
||||
\begin{enumerate}
|
||||
\item Démontrer que l'équation $f(x) = 1.1$ admet une unique solution sur $\intFF{0}{1}$.
|
||||
\item Donner une valeur approchée à $10^{-2}$ de cette solution.
|
||||
\end{enumerate}
|
||||
|
||||
\bigskip
|
||||
|
||||
\textbf{Partie C}
|
||||
|
||||
\medskip
|
||||
|
||||
En Europe, les observateurs d'une maladie nécessitant une hospitalisation considèrent qu'ils peuvent modéliser par cette fonction $f$ l'évolution du nombre de lits occupés par des malades pendant les quatre mois d'hiver.
|
||||
|
||||
Pour tout $x$ appartenant à l'intervalle [0~;~4], $f(x)$ représente le nombre de lits occupés, exprimé en million, à l'instant $x$, exprimé en mois.
|
||||
|
||||
Un journal affirme que cet hiver:
|
||||
|
||||
{\setlength\parindent{10mm}
|
||||
\begin{itemize}
|
||||
\item le nombre de lits occupés a dépassé le million pendant plus d'un mois.
|
||||
\item le maximum de lits occupés a été atteint autour du milieu du deuxième moi.
|
||||
\item après 3 mois, la diminution du nombre de lit a ralentit.
|
||||
\end{itemize}}
|
||||
|
||||
Que dire de ces trois affirmations ?
|
||||
\end{exercise}
|
||||
|
||||
\end{document}
|
||||
|
||||
%%% Local Variables:
|
||||
%%% mode: latex
|
||||
%%% TeX-master: "master"
|
||||
%%% End:
|
||||
Some files were not shown because too many files have changed in this diff Show More
Reference in New Issue
Block a user